You are on page 1of 291

M.K.

GUPTA
CA CLASSES
PH: 9212011367/ 9811429230 www.mkguptacaclasses.com
e-mail id: mkguptataxclasses@rediffmail.com

VALUE ADDED TAX/ SERVICE TAX


50 MARKS

CA
IPCC
NOV-2012
F.Y. 2011-12

F. A. 2011 FINANCIAL YEAR

2011-12
`200

SYLLABUS
IPCC
Paper 4: Taxation
(One paper Three hours 100 Marks) Level of Knowledge: Working knowledge Objectives: (a) To gain knowledge of the provisions of Income-tax law relating to the topics mentioned in the contents below and (b) To gain ability to solve simple problems concerning assessees with the status of Individual; and covering the areas mentioned in the contents below.

Contents:
Part I: Income-tax (50 Marks) 1. Important definitions in the Income-tax Act, 1961 2. Basis of charge; Rates of taxes applicable for different types of assessees 3. Concepts of previous year and assessment year 4. Residential status and scope of total income; Income deemed to be received / deemed to accrue or arise in India 5. Incomes which do not form part of total income 6. Heads of income and the provisions governing computation of income under different heads 7. Income of other persons included in assessees total income 8. Aggregation of income; Set-off or carry forward and set-off of losses 9. Deductions from gross total income 10. Computation of total income and tax payable; Rebates and reliefs 11. Provisions concerning advance tax and tax deducted at source 12. Provisions for filing of return of income. Part II: Service Tax (25 Marks) and VAT (25 Marks) Objective: To gain knowledge of the provisions of service tax as mentioned below and basic concepts of Value added tax (VAT) in India. Service Tax (25 Marks) Contents: 1. Service tax Concepts and general principles 2. Charge of service tax and taxable services 3. Valuation of taxable services 4. Payment of service tax and filing of returns Individual services covered in the syllabus (i) Consulting Engineer Section 65(105)(g) (ii) Mandap Keeper Section 65(105)(m) (iii) Practicing Chartered Accountants Services Section 65(105)(s) (iv) Scientific and Technical Consultancy Section 65(105)(za) (v) Commercial Training or Coaching Section 65(105)(zzc) (vi) Technical Testing and Analysis Section 65(105)(zzh) (vii) Business Exhibition Section 65(105)(zzo) (viii) Information Technology Software Section 65(105)(zzzze) VAT (25 Marks) 5. VAT Concepts and general Principles

3 6. Calculation of VAT Liability including input Tax Credits 7. Small Dealers and Composition Scheme 8. VAT Procedures. Note: If new legislations are enacted in place of the existing legislations the syllabus will accordingly include the corresponding provisions of such new legislations in the place of the existing legislations with effect from the date to be notified by the Institute. Students shall not be examined with reference to any particular State VAT Law.

PART 1 PART 2

VALUE ADDED TAX SERVICE TAX

19-158 159-287

VALUE ADDED TAX


INDEX
Q1 (V. Imp.): Explain the concept of Value Added Tax / Explain Taxonomy (process of making groups) of VAT/ Different stages of VAT. Q2 (V. Imp.): Explain concept of input tax credit. Q3 (V. Imp.) : Explain Central Sales Tax including rates under Central Sales Tax. Q4: Explain scope of input tax credit. Q5 (V. Imp.): Explain purchases not eligible for input tax credit. Q6 (V. Imp.): Explain provisions of Stock Transfer. Q7 (V. Imp.): Write a note on registration under State VAT Act and Central Sales Tax Act. Q8: Write a note on amendment in registration. Q9: Write a note on cancellation of registration certificate. Q10: Explain accounting treatment of VAT as suggested by ICAI. (not covered in syllabus rather it is only for self reading) Q11 (V. Imp.): What are the Variants (different types) of VAT. Q12: What are the rates under VAT. Q13: Explain non- creditable/non-vatable goods. Q14 (V. Imp.): Explain composition scheme for small traders. Q15 (V. Imp): What are methods for computation of Value Added Tax. Q16 (V. Imp.): Explain VAT Invoice / Tax Invoice Q17 (V. Imp.): Explain concept of excise duty (Central Value Added Tax). Q18 (V. Imp): Write a note on tax credit in case of manufacturer. Q19 (V. Imp): What is the common procedure for availing and adjusting cenvat credit for Excise Duty, Service Tax as per Cenvat Credit Rules, 2004. Q20 (Imp.): Write a note on filing of return under state VAT. Q21 (Imp.): Write a note on assessment under State VAT. Q22: Write a note on System of Cross Checking. Q23 (Imp.): Write a note on maintaining of books of accounts and records under State VAT. Q24 (Imp.): Explain the provisions of audit under State VAT. Q25 (V. Imp.): Explain merits of VAT. Q26 (V. Imp): Explain demerits of VAT. Q27: Explain role of ICAI in VAT. Q28 (Imp.): Explain role of Chartered Accountant in VAT.

1. VIJENDER AGGARWAL NOV-2010 (Roll No.174639) All India Topper in CA-IPCC A person who possesses such vast knowledge in the field of taxation, that we people can only dream of, is none other than M. K. Gupta Sir. He possesses the rare ability to teach this procedural subject with utmost ease, enabling his students to grasp all the provisions without any confusion. The quality of study material provided is such that a good study of it helped me score 92 marks. The variety and complexity of practical problems covered in the books are not available anywhere else. One can find many places where taxation is being taught but it is hardly possible to find a better place where tax laws are combined with their practical applicability to ensure that all concepts are crystal clear. Sir is extremely generous. Money-making doesnt appear to be his priority and it is clearly reflected in his classes, where the infrastructure and administration stands second to none and students are awarded handsome cash-prizes not only in classes but also in tests, which are regularly conducted. Thanking Sir for all what he has done would be an insult since it was only his efforts that helped me reach this position. Sir, its your success. The relationship between us started in CPT only and continued in IPCC and I hope it will continue forever. 2. PRACHI JAIN MAY-2010 (Roll No.66312) All India Topper in CA-PCC M. K. Gupta Sir is an outstanding teacher. He is not only a good teacher but a good person by heart. His way of teaching is excellent. There are many provisions in tax but Sir repeats every provision atleast two times. This helps in understanding those provisions easily. His books are very good. Everything from theory to PRACTICAL ILLUSTRATION, EXAMINATION QUESTIONS and BARE ACT is covered in his books. Sirs staff and management is also very good. Everything is handled in a systematic manner and on time. Overall it was a good experience. Thanks Sir !! :3. SAHIL AGGARWAL NOV-2011 (Roll No.271741) 4th RANK The material provided by the Sir in the class as well as in books is self sufficient for the exams. M.K. Gupta Sirs knowledge about the subject is excellent. Moreover, his down-to-earth nature make him the best tax teacher one could get. 4. MAYANK AGARWAL JUNE-2009 (Roll No. 35586) 4th RANK Sir has always been very motivating and his guidance has helped me to excel in the exams. Sir has an immense knowledge of the subject and has always been available to answer our queries. Thank you Sir! 5. SAHIL ARORA NOV-2011(Roll No.215712) 6th RANK My opinion towards M.K. Gupta Sir is very good. His enthusiasm in class has amazed me a lot. There are many provisions in the subject but Sir repeats them atleast twice which is of immense help. Also, the explanation of topic with a relevant practical example clears all the queries. M.K. Gupta Sirs books along with notes cover easily the entire syllabus and the notes prove really helpful one day before the exam. OVERALL A GREAT EXPERIENCE! 6. NIKETA SABOO MAY-2011(Roll No.52328) 6th RANK I think M.K. Gupta Sir is an outstanding Teacher. He has excellent knowledge of the subject. With his skills and knowledge, he has made Tax subject very easy to understand and really interesting. I would recommend every aspiring CA, to take guidance from Sir. It will really be helpful. Thank you Sir, for guiding us throughout. It was a wonderful experience. 7. ANU GOEL NOV-2011 (Roll No.288966) 7th RANK The experience with M.K. Gupta Tax Classes was really amazing!! Sir is full of knowledge and experience. He repeats every provision and concept behind it in detail. I have personally never seen him in a hurry to

5 finish the course. His very motive is to make sure the student understand the provisions fully! The book has large number of practical illustrations, which further helps to understand the subject! Thank you Sir!!! 8. MADHAV BANKA NOV-2011(Roll No.233313) 8th RANK M.K. Gupta Sir is one of the best teacher in my life and he is the best teacher of taxation in whole India. He has a vast knowledge in the field of taxation which helped me a lot to get such good marks and he inspired me to get rank. Because of him, I developed great interest in tax due to which I am going to choose my career in field of taxation. For this I am very thankful to the Sir. 9. RAJAT GUPTA NOV-2011(Roll No.408691) 9th RANK I got 9th rank by scoring great marks in all six subject but the biggest credit for my achievement goes to M.K. Gupta Sir. His way of appreciating the students is excellent. Books given by M.K. Gupta Sir is so easy and understandable. In CA market M.K. Gupta Sir is the only name which helps the students in all areas whether in studies or in motivating the students. Thank you Sir. 10. NEVAID AGGARWAL NOV-2010(Roll No.178788) 10th RANK My opinion towards Sir is very good. His enthusiasm in class amazed me a lot. His indepth knowledge of the subject along with detailed books was of immense help. He is by far the best tax teacher one can go for guidance. 11. RUCHIKA ARORA MAY-2008 (Roll No. 62449) 13th RANK M.K. Gupta Sir has vast knowledge about the subject and is every ready to help their students to touch the heights of their career. I owe a great deal of my success to Sir and this institute and considers myself to be fortunate of being the part of this institute. M.K. GUPTA SIR ROCKS !! 12. ASHISH SABOO JUNE-2009 (Roll No. 47977) 13th RANK As a teacher, M.K. Gupta Sir is just like a sea of knowledge and you get each and every thing from very beginning to end from him. Sir is really a nice person. He gives each and everything to his student. Coaching is excellent and the book is complete. If a student read Sirs notebook + book and get Sirs knowledge he will sureshot get very good marks. 13. GARIMA MADAN MAY-2010 (Roll No.57619) 13th RANK No one can replace M. K. Gupta Sir. His way of teaching is unique in this world. The environment of classrooms is well maintained. The facilities provided were upto the mark. There is no need to refer any other book if you are studying M. K. Gupta Sirs notes and study material. Be honest towards your studies and Sir will show you the way of success. 14. KESHAV GOEL NOV-2011 (Roll No.234597) 14th RANK A complete package. A good teacher with good teaching skills making you learn and practice a lot in the class itself. His notes are excellent both for the exams and future. He also provides a lot of knowledge for the other issues as well. Thank you Sir! Thanks a lot for your help and support. 15. ADITI AGARWAL NOV-2011 (Roll No.298278) 15th RANK M.K. Gupta Sir is a great teacher having an exceptionally well knowledge about the subject. He teaches every concept in very detailed manners and the notes provided are sufficient for preparation during exam. 16. HARSH KUMAR GARG NOV-2011 (Roll No.207614) 15th RANK The notes of M.K. Gupta Sir was very useful for my preparation. I give the credit of my success to Sir. Also, the study material and the motivational words of Sir was very useful. 17. SANDIP AGARWAL MAY-2010 (Roll No. 53610) 15th RANK A man who has achieved all the levels of satisfaction and is here just to serve the society with his knowledge and aura. He is the man with the ability to produce rank holders with most ease. His book is more than enough for scoring heavily. 18. REEMA NOV-2010 (Roll No.174999) 15th RANK I think, M. K. Gupta Sir is one of the best teacher in Taxation. For me, taxation becomes so much easy just because I have studied from M.K. Gupta Sir. Its a life time achievement for me that I have studied from M.K. Gupta Sir. Thanks a lot to Sir, you are great. 19. SACHIN GUPTA MAY-2008 (Roll No. 29304) 16th RANK

6 EXCELLENT COACHING ! EXCELLENT KNOWLEDGE ! EXCELLENT BOOK ! EXCELLENT WAY OF TAKING UP QUERIES ! The way of teaching of Sir is very nice. He makes each and every thing so easy that we can learn it in the class room itself. The notes and books provided by Sir are excellent. There is no need to consult any book and in my opinion M.K. Gupta Sir is the best teacher of income tax in world. 20. SUMAN RANI NOV-2008 (Roll No. 21180) 16th RANK He is the best teacher and having excellent practical knowledge. He is very cooperative and boosts the confidence level of the students. No teacher can be compared with him because he is the best. He is also a superb person as a human being and understands the problems of students and solves them. I am very Thankful to him and even words are not sufficient to express his gratitude. 21. RIDHI NARULA NOV-2009 (Roll No. 60965) 16th RANK Coaching provided by M. K. Gupta Sir is very good and knowledge of teacher is very good. Excellent Teacher, Fabulous personality and Marvellous Guide these are the apt words to describe M.K. Gupta Sir. He taught me how beautiful the studying is. His knowledge of the subject is outstanding. 22. ANIL KUMAR MALI NOV-2010 (Roll No. 53101) 16th RANK Broadly Speaking, M. K. Gupta Sir knowledge, experience and CA oriented approach is far beyond other teachers. He is very much excellent in his concept. Thanking you and my colleagues and seniors namely Amit, Ankit, Manoj, My C.A. & Akshay Sir. Thanking you again. If you want to get Rank, do what your teacher and your mind vis--vis your heart guides. Really! 23. RAM DAS SONAWALA MAY-2011 (Roll No.45919) 16th RANK First of all Thank You Very Much Sir for your support, it helped me to take a rank. You are a good teacher. Your method of teaching and your never ending knowledge, become very useful for me to get the good marks in taxation. Overall I am fully satisfied with your teaching. 24. CHARU SINGHAL JUNE-2009 (Roll No. 46941) 17th RANK M.K. Gupta Sir is the best teacher for Tax. He is a very nice person. He is always there to help the students. His class notes and book are self sufficient to score good marks in Tax. I admire him not only as a teacher but also as a guide and as a person. Thank you Sir. 25. VARUN LAKRA NOV-2008 (Roll No. 69351) 18th RANK In my view, M.K. Gupta Sir is the best teacher for Income Tax for PCC. He is a brilliant teacher. The study material provided was excellent with perfect examples. Lastly, it is hard work and support of our teacher which has helped me to get 18th Rank in PCC. THANK YOU SIR!! REGARDS 26. AMAN JAIN NOV-2011 (Roll No.233247) 19th RANK The mediocre teacher tells. The Good Teacher explains. The superior teacher demonstrates & The Great teacher inspires. M.K. Gupta Sir is really a great teacher. His unmatchable style of teaching coupled with Sirs peace & calm nature of dealing with students & handling their queries is really excellent. The reason behind my excellence in taxation entirely goes to M.K. Gupta Sir. The notes & books provided is more than enough & exhaustive. It was really a great experience & memorable period of my life. Thanks a lot Sir! 27. ARPIT BANSAL NOV-2011 (Roll No.222713) 20th RANK You know you are in the right place when you are taking coaching from M.K. Gupta Sir. M.K. Gupta Sirs vast knowledge and amazing conceptual clarity goes a long way in helping you understanding Taxation in great detail. Thank You Sir for teaching me!! 28. ANSHIKA GUPTA NOV-2010 (Roll No.175016) 20th RANK M. K. Gupta Sir is very good teacher and he has a Treasure of Knowledge. His teaching style is excellent and I will pray to God for giving successful life/future to M.K. Gupta Sir. Books contain all the necessary materials. The Institute is also very nice and the staff is very co-operative in nature and finally I wish to pay Thanks to Sir and his staff for their excellent teaching and co-operative behaviour. Thanks!!! 29. ANKIT GOLASH JUNE-2009 (Roll No. 47738) 21st RANK Guidance given by Sir was excellent. Notes are excellent, your concepts are very good. As far as your book is concerned, it is a complete package in itself, covering all the peculiar questions!

7 Thanks for all your support and guidance! Thanks a lot ! 30. RAHUL JAIN JUNE-2009 (Roll No. 47597) 22nd RANK M.K. Gupta is the best teacher for Taxation. M.K. Gupta Sir has vast knowledge about the subject and is ready to help their students to touch the heights of their Career. He has always been encouraging and without his support, I would not have been able to score a rank. Thank you Sir. 31. MANOJ NARWAL MAY-2011 (Roll No.53612) 23rd RANK First of all I would like to say THANKS thousand times to M. K. Gupta Sir. I owe my success to him. He is a complete package for every CA aspirant for Taxation. I think Sir should be called as Tax Guru. I have joined this coaching in order to achieve a rank and Sir made it possible. Sir boosted our morale from time to time, gave us the confidence needed to aim the Rank. Sir handled our queries both, in and after the class, so well that we do not need to worry about it at home. Teaching style of Sir is fabulous and Sir himself is a very great personality. In last, please bless me Sir. Thanking you. 32. M. SRINIDHI NOV-2011 (Roll No.220746) 25th RANK M.K. Gupta Sir teaches tax to all his students in the easiest possible way so that the concepts are clear to every student in the class. He not only teaches us but also makes us learn in the class itself. So attending the class completes the preparation then & there. The material provided is another important plus. Thank You Sir. Your teaching has indeed made TAXATION a favourite subject of mine. 33. ROUNAK AGARWAL NOV-2011 (Roll No.236372) 25th RANK No words can describe our teacher (M.K. Gupta Sir), if anyone wants to be under good hands of taxation, you are where you should be. The teacher, the centre were all amazing. The performance of students itself speaks about the teacher and the coaching. What was taught in the classes was more than enough for a rank. 34. CHHAVI MEHTA MAY-2010 (Roll No.56911) 25th RANK Income tax coaching were very helpful in my preparation for the exams. Study material is very well organized. The practice questions clarified the concept beautifully. I thank M. K. Gupta Sir for helping me score so well in taxation which helped my overall score in the PCC examination. My warm regards and a heart full Thank You to you Sir. 35. NISHANT GUPTA NOV-2010 (Roll No.173038) 25th RANK Excellent coaching for income tax with detailed notes, with supporting examples on each point. The excellent query handling procedure undertaken here helps immensely. 36. DHRUV AGGARWAL NOV-2008 (Roll No. 66185) 26th RANK Well what should I say about M.K. GUPTA Sir, he is a mobile encyclopedia in Taxation. He has vast and exhaustive knowledge about Taxation which he always tries to pass on to the students. You cant forget Sirs sense of humor. He is one of the most dedicated teachers I have ever seen. Extremely punctual unlike us!! I truly dedicate my achievements to Sir. 37. MADHUR AGGARWAL NOV-2009 (Roll No. 53009) 26th RANK Sir made Tax look a simple subject. Classes were full of fun and the book is very comprehensive yet easy to study. Feel lucky to have chosen M. K. Gupta Sir as my Tax teacher!! His coaching is very good, knowledge is excellent and queries taken are excellent. 38. BRIJESH AGGARWAL MAY-2010 (Roll No. 59234) 26th RANK The best tax faculty in India. Thanks to M.K. Gupta Sir, and his staff for providing me such environment which help me get such marks. 39. HIMANSHU GUPTA NOV-2009 (Roll No. 55960) 27th RANK In my knowledge till date I have not seen such a brilliant teacher full with enthusiasm, energy & vast pool of knowledge & experience. His nature is very good. The way Sir attends our queries is just fantastic. Material given by Sir is more than enough. U dont need to refer any other book. He is the best teacher for Taxation. 40. SAMEER ROUSHAN MAY-2011 (Roll No.192754) 27th RANK

8 I was simply enthralled by M.K. Gupta Sirs way of teaching. The conceptual clarity is awesome. The study material is comprehensive and the management is excellent. Overall, it was a life changing experience and the dedication of Sir cannot be ever forgotten. Thank you very much Sir!! 41. NITIN KUMAR JAIN JUNE-2009 (Roll No. 47826) 28th RANK There is no match of M.K. Gupta Sir his knowledge is so vast that is beyond our thinking. The topic wise modules provided by Sir covers each & everything that one requires to get good marks in taxation. The atmosphere of the class is so great that I was proud to be a student of M.K. Gupta Sir, also Sirs personality always inspire the students and motivate them. Thank you Sir. 42. KUSHAL ARORA NOV-2010(Roll No.36718) 29th RANK M. K. Gupta Sir is a very knowledgeable person and he has a very unique style of teaching which keeps the students glued to the explanations. The coaching staff is friendly and co-operative. The book covers all the aspects required for the exams. It is detailed and exhaustive. 43. SANTOSH GIRI NOV-2011 (Roll No.227904) 30th RANK M.K. Gupta Sir is the best teacher in taxation. Best faculty and good study material. 44. ABHISHEK JAIN MAY-2011 (Roll No.184085) 30th RANK M.K. Gupta Sir is not only a brilliant teacher, but also extremely helpful person. His simplicity and style of teaching makes him different from other teachers. Thank you Sir for all your efforts, that you have given for ordinary students like me to make us shine like stars. 45. VARUN V KALE MAY-2011 (Roll No.182349) 31st RANK My experience as an student of M.K. Gupta Sir has been wonderful. Sir not only explains the provisions of the Act but also the reason behind those provisions. Sir has amazing ways of taking queries from his students where students can ask their doubts during the class or after the class or even by message service. I feel that the way Sir motivates his students in the form of incentives is amazing. Sir is very particular regarding time which I feel adds to his greatness as a teacher. 46. ROHIT MURMURIA NOV-2011 (Roll No. 216642) 31st RANK M.K. Gupta Sir is wizard of taxation. He is really brilliant. He makes conceptual clearance of every provision to provide us insightful knowledge of tax. 47. ABHISHEK BATRA NOV2011 (Roll No.407692) 32nd RANK Sir is very motivating and the class notes given by him in classes are very good and knowledgeable for the exam purposes. It is advised for every student that just go through the books and class notes of Sir. They are much sufficient for scoring better in exams. 48. ARUN SETIA NOV-2008 (Roll No. 21201) 32nd RANK I think M.K. Gupta Sir is best faculty I have ever seen. He is having a tremendous knowledge & command in taxation. The way he carries his class is unbelievable. After coming to his class, I always get encouraged from him. Thanks to Sir, for being so nice to us. 49. HELINA MALANI MAY-2011 (Roll No.47662) 32nd RANK I am grateful to the institute and M.K. Gupta Sir for his excellent teaching and knowledge which he has shared with students like me. Thank You Sir!! 50. SAHIL CHADHA MAY-2011 (Roll No.183569) 32nd RANK M.K. Gupta Sir is one of the best teachers that we have in this CA profession. His calm & composed explanations make the student understand the concept in depth. His query resolving techniques are beyond comparison and also he try to exhibit positive energy among the students which is commendable. Their study material and notes are more than sufficient. I would like to thank and congratulate Sir for my 32nd Rank. Thank You Sir. 51. KUNAL MENDIRATTA MAY-2011 (Roll No.48976) 32nd RANK I am extremely grateful to M.K. Gupta Sir for helping me in getting exemption in tax and for raising my hopes of my passing CA-PCC Exam.

9 Further to add more, the books that Sir offered to us contained all the relevant questions that we are asked in CA examination. 51. MONISHA PRUTHI JUNE-2009 (Roll No. 48005) 33rd RANK The motivation provided by our dear Sir led me into the path where I never felt down. There was always a hand on us who guided towards the correct path. Moreover, the knowledge provided by M.K. Gupta Sir is unbeatable and more than sufficient. I really respect him for the whole life and thanks him for whatever I am today. 53. VASUDHA ARORA MAY-2010 (Roll No.57857) 33rd RANK M. K. Gupta Sir is a very good teacher. His way of teaching every single topic in a detailed way is one thing that I like the most. With this it builds up a lot of confidence & clarity in mind relating to concept of the topic concerned. 54. NUPUR JAIN NOV-2009 (Roll No. 63265) 34th RANK M. K. Gupta Sir is a brilliant teacher. Not only that, he is a very good human being and is always there to help the students. He has always motivated me and has been a source of inspiration. The coaching classes suited me really well. The way Sir handle the queries is worth appreciation. I truly owe my success to M. K. Gupta Sir! Thank you So much Sir for your continuous guidance and support. Thank you! 55. NEHA AGGARWAL NOV-2010 (Roll No.151847) 34th RANK In IPCC, tax seems to be the toughest but when you are trained under M. K. Gupta Sir, the fact becomes a myth. The classes are made so spontaneously interesting that no point can be missed out. Incentives given by the Sir are a boost to study hard and brings in the sense of competitiveness among students. The study material and the notes given by Sir are enough to score excellent marks in exams. Thank you Sir for your blessings. 56. MOHIT SABOO NOV-2011 (Roll No.233293) 35th RANK M.K. Gupta Sir has excellent teaching skills. I sincerely thank Sir for his guidance. M.K. Gupta Sirs simple and easy approach towards every topic makes the subject easier to understand. The study material that is provided has all the concepts covered. Overall it was a very good experience. 57. QURICA AGARWAL NOV-2011 (Roll No.200078) 35th RANK Mere words cannot explain my gratitude towards our honourable teacher M.K. Gupta Sir. The study material is the ultimate one. The way of his teaching is really commendable. He enhanced our practical knowledge immensely. Thank You Sir. 58. PITAM GOEL NOV-2008 (Roll No. 66077) 35th RANK The book given by M.K. GUPTA Sir is more than enough to score good marks. It was due to the hard work and due efforts of M.K. GUPTA Sir that I am able to score good marks in PCE. Thanks a lot Sir for your cooperation and sincerity. KEEP ROCKING!! 59. HIMANSHU KUMAR MAY-2011 (Roll No.47698) 35th RANK He is very good teacher. He is very helpful. He is always motivating students by providing gifts. 60. NIHARIKA GUPTA NOV-2011 (Roll No.222611) 36th RANK Every aspect was taken care by M.K. Gupta Sir from books to class interaction. The style of teaching was very good and easily understandable. Queries were always solved on the spot without delay. I was very satisfied by the coaching as it helped me to improve my capability and the classes provided a very nurturing environment. Coaching was excellent. 61. ANAND GUTGUTIA MAY-2010(Roll No.65223) 36th RANK It was great overall experience and evoked interest in tax. I always looked forward to my tax classes. 62. PRIYA SINGHAL NOV-2009 (Roll No. 39736) 37th RANK In my view, M. K. Gupta Sir is a fabulous teacher. His knowledge regarding the subject is ultimate. He is very devoted & frank to the student. We can ask any number of doubts in the class or after the class without any hesitation. Most important things I liked about him (1) He calls the student on stage to solve the question that help to boost the confidence in student (2) His view of encouraging students by providing various incentives/rewards.

10 About books, I can say his books are very good covering whole syllabus & written in very easy language. 63. RAHUL YADAV NOV-2009 (Roll No. 52981) 37th RANK M.K. Gupta Sir is simply a Tax Guru. His knowledge about his subject is complete. The way he teach us is very simple & short. You need not read any other books. Even the classroom notes are enough to score good marks. His coaching is excellent, knowledge is excellent and queries taken are excellent. 64. AARUSHI GARG MAY-2010 (Roll No. 66027) 37th RANK M. K. Gupta Sir is a vast reservoir of knowledge. His style of teaching is good with a combination of excellent and very helpful notes. Books provided by M. K. Gupta Sir are sufficient in themselves to help any student achieve a good rank. Heartiest Thanks to M. K. Gupta Sir !!! 65. LOKESH AGGARWAL NOV-2010 (Roll No.189271) 37th RANK Sir, is excellent in each & every area of taxation. His knowledge and teaching style is superb. When I started my tax classes from him, I find this subject easiest among all subjects. Really Sir, U r awesome. All I can say is east or west, M.K. Gupta Sir is the best. 66. DHWANI SEHGAL NOV-2010 (Roll No.174543) 37th RANK M. K. Gupta Sir is intelligent, jolly and knows the subject very well. The classes were enriching since he linked the law with our day-to-day lives explaining the reason exactly why the law was enacted. This gave us a deep knowledge of the subject and clear understanding. The book has a good variety of questions, easy language and case laws. 67. VIRENDRA DAYAMA MAY-2011 (Roll No.45423) 37th RANK Best thing about M.K. Gupta Sir is that, he repeats every provision at least two times, which helps a lot. He is very knowledgeable person and keeps the class environment, students friendly. He has sound knowledge of the subject. The Books are very good and student does not require anything else for preparation. Thank you Sir, for your motivation and being an inspiration to us. Thank You Sir. 68. PRABIN BARAL NOV-2011(Roll No.22849) 38th RANK Conceptual clarity is the most remarkable feat of M.K. Gupta Sir. His notes are magnificent and books are self-sufficient. 69. KIRTI ANEJA NOV-2009 (Roll No. 51444) 38th RANK Excellent teacher with excellent knowledge. His way of teaching is very good. What I liked about M. K. Gupta Sir is that he thinks from the point of view of students. His book is just great and was sufficient for getting good marks in taxation. His book has a variety of questions. Thanx a lot Sir. 70. AASHISH GUPTA NOV-2009 (Roll No. 53088) 38th RANK Its been an honor to be a student of M. K. Gupta Sir. He is a superb teacher and has a very vast knowledge of the subject. He is one of the best teacher I have come across my life till date. Sir Thank you very much and I owe a part of my success to you. His coaching is excellent, knowledge is excellent and queries taken are excellent. 71. BABITA MITTAL NOV-2009 (Roll No. 52776) 38th RANK M. K. Gupta Sir is the best teacher in tax, very helping and give due care to every student individually. He has the answer for every query. I respect him a lot. His coaching is very good, knowledge is very good and queries taken are excellent. 72. RISHABH KUMAR GUPTA NOV-2010 (Roll No.174748) 38th RANK M. K. Gupta Sir is a very brilliant teacher. He has got a very positive attitude and he keeps motivating his students to perform well and clearing their queries in a timely manner as well. The books provided by Sir are also very good containing very good questions as well. Wish he could teach me in CA finals also. Thank you Sir for all your guidance to achieve such a result. 73. AAYUSHI KATYAL NOV-2011 (Roll No.271628) 39th RANK M.K. Gupta Sir is the best tax teacher and taking his classes has always been an exciting affair. His knowledge about tax and especially the way things are handled practically has been quite enlightening. He has made the most feared subject(usually), the most fun filled and easy. Thank You so much Sir and I wish

11 everyone has an opportunity to sit in your class. The book, notes and all the management is done very properly and in a smooth manner. All in all, the best way to study tax. 74. SUMEDHA RANI AGGARWAL NOV-2011(Roll No.222716) 39th RANK M.K. Gupta Sir made Tax look a simple subject, despite of the fact that Tax is the most difficult & complicated subject in IPCC having number of provisions. He made all the concepts crystal clear by explaining each concept atleast twice. Even the CAs employed by Sir for solving the doubts and guiding the students were very helpful. All I could say is that it was an amazing experience to study at M.K. Gupta Classes. The relationship between us started in CPT with Vaibhav Sir & Neha Mam, it continued in IPCC with M.K. Gupta Sir & I hope it will last long forever. 75. PIYUSH GUPTA NOV-2008 (Roll No. 65203) 39th RANK Sir is excellent in his way of teaching and revision, 100% coverage of his syllabus, personal attention, flexible timings student friendly attitude are some of the attributes of these classes. 76. VANDANA KAUSHIK JUNE-2009 (Roll No. 48100) 39th RANK M. K. Gupta Sir is a best teacher. His way of teaching is very good & makes us to learn the subject very easily. His knowledge about the subject is ultimate. 77. SUMANT GUPTA NOV-2009 (Roll No. 50053) 39th RANK I think he is the best person in the job. His excellence in Taxation and his friendly relations with students and his coolness and calmness and further encouragement to his students, just lifts off the burden of the course. I thank Sir for being there and helping me to achieve this feat in my life. 78. MEGHA ARORA NOV-2009 (Roll No. 52763) 39th RANK M. K. Gupta Sir has always been a support to all the students while their teaching sessions. He has always boosted our morale to achieve great heights and great score not only in taxation but also in every other subject. He laid equal importance to both theory and practical which helps the students to apply whatever they are studying in their practical training. At the end I would like to thank him for being such a good teacher and a friend. 79. SUNNY GIRDHAR NOV-2010 (Roll No.60524) 39th RANK Coaching by Sir gave me an insight into practical aspects of taxation with the help of which I was able to co-relate my studies with practical training i.e. articleship. Also study material provided was good and compact enough to be revised in exams. I thanks Sir for providing me such guidance. 80. VAIBHAV AGGARWAL NOV-2010 (Roll No.56278) 39th RANK M. K. Gupta Sir is the lifeline of PCC Tax. A teacher with abundant knowledge. In serious terms, a perfect Tax Guru. 81. VARAD V KALE MAY-2011 (Roll No.182310) 39th RANK My experience of being a student here at M.K. Gupta Tax Classes has been an unforgettable one. I had attended the batch (Monday-Sunday) with little hopes of understanding the tax laws in such a small time frame. But the manner in which Sir taught gradually made me realize that I was wrong. Sir taught each and every provision in a manner that a person who listens carefully in the class would understand it 75%-80% then and there. Sirs technique of taking up doubts is also amazing. Cash incentives for answering classroom questions correctly quicker than anyone else improves speed, fosters sense of competition and also attracts one to listen to what is being taught so that he/she would get a chance to earn that incentive. Thank you Sir for everything. 82. JYOTI GOYAL JUNE-2009 (Roll No. 46348) 40th RANK In my opinion M. K. Gupta Sir is one of the best teacher. His method of teaching is excellent specially the repetition of sections again and again in the class. His way of taking up queries is unmatchable. He makes the Tax concept crystal clear among the students through well explained examples. All students of PCC are blessed with such a good teacher. Thank to you Sir because of you Tax has been very easy subject for me. 83. ANKIT GOEL NOV-2010 (Roll No.156437) 40th RANK M. K. Gupta Sir is an excellent faculty in tax and his books are very nice and way of taking queries is very nice. Every aspect of tax is covered in his books. Overall it was a very good experience.

12 84. SONIA AGGARWAL NOV-2008(Roll No. 69153) 41st RANK Sir is a pool of knowledge, the only thing you need is to learn us how to swim & gain as much as you can. The book given by Sir covers the whole module. The overall environment of class is very good for studies. 85. RISHU GOYAL MAY-2010(Roll No. 57632) 41st RANK Book is the Best, Teacher is the Best, queries are taken in the Best way by Sir. You appear before me as god and only because of you I was able to secure/got rank. Thank You Sir. 86. MOHIT GOEL NOV-2010 (Roll No.49010) 41st RANK M. K. Gupta Sir is the best teacher for tax according to me. The journey is more appropriate word for me rather than studying tax with him. What he gives in tax is commendable and uncomparable for me. Rather than making students preparing for studies, he tells them what it is and gives them a broad view of how the provisions come into effect and their applications. If you want to enrich yourself with tax knowledge M. K. Gupta Sir is best. Thank You Sir for giving me the pearls of wisdom from the sea of your knowledge. 87. KHUSHBOO TEKCHANDANI NOV-2011 (Roll No.270637) 42nd RANK I had an excellent time in the coaching. I learnt a lot even the things which are not covered in the course. M.K. Gupta Sir has a vast knowledge in all fields and specially taxation. The material provided by the center was extremely useful. The Bare Text helped in clearing all the concepts. I would like to thank Sir for giving me such an immense experience and knowledge. The tests which were hold regularly were also helpful in studying the subject. Thank you Sir!!! 88. PIYUSH SHARMA NOV-2010 (Roll No.174793) 42nd RANK The classes were excellently managed. Sir repeats all the sections atleast two times, which helps a lot. M.K. Gupta Sir is best for Tax. Thanks Sir. 89. MANISHA JINDAL NOV-2011 (Roll No.225454) 43rd RANK M.K. Gupta Sir teaches very well. Most importantly, he focuses on practical application of tax provisions which makes understanding of the provisions very easier. Also his books are very good and are in easy language. Thank You Sir!! 90. MEGHA AGARWAL NOV-2011 (ROLL NO.267518) 43rd RANK Going to Sirs coaching was the best part of the day. Sirs style of teaching is just fabuluos. Sir is very punctual and the staff is so cooperative and adjusting. The books covered all the provisions successfully. 91. MOHIT GARG NOV-2008 (Roll No. 65676) 43rd RANK M.K. Gupta Sirs approach is not short term but long term. He not only wants a child to clear but also wants him/her to excel in the field of taxation by providing his knowledge. His approach of calling students to solve questions on board has worked to a great extent. 92. KISHAN KUMAR NOV-2010(Roll No.179153) 43rd RANK Studying taxation with M. K. Gupta Sir is truly a fun and an enriching experience. His simplicity, manner of conducting class and query session is indeed gratifying and awesome. However, the best thing is the emphasis on practical questions which is indispensable for securing decent marks. The notes given during the class by Sir is excellent and covers entire syllabus in a precise & crisp manner which is very helpful especially during the examination time. Finally, a heartfull thanks to you Sir. You really rock!!! 93. TRIPTI GUPTA JUNE-2009 (Roll No. 47936) 44th RANK Sir is really a very good teacher. His knowledge about tax is great. The way he links practical life with theory life is really very good. The study modules provided by Sir is more than sufficient with full knowledge of course and easy language. Sir has really cooperated with us. I really want to thank him for his contribution in my life. Thanks Sir!! 94. KANHAIYA MITTAL MAY-2010(Roll No. 57773) 44th RANK M. K. Gupta Sir is a very good teacher. We did not require to refer any other book. Sirs notes were more than sufficient. He is a very qualified teacher and his teaching skills are unmatchable. 95. ABHISHEK SACHDEVA NOV-2011(Roll No.228089) 45th RANK M.K. Gupta Sir is one the most knowledgeable person I have ever met. Books provided by them are best and so is the coaching. M.K. Gupta Sir motivate each & every student by giving the rewards.

13 I would like to thank Sir from the bottom of my heart. THANK YOU SIR!!! 96. SAHIL MANGLA MAY-2010(Roll No.58598) 46th RANK The books given by Sir and the knowledge provided in the classes is uncomparable and more than sufficient. I thank M. K. Gupta Sir, whole heartedly for the knowledge he provided by his sincere efforts.

97. MAHESH AGGARWAL MAY-2011 (Roll No.52164) 46th RANK WELL, PERSONALLY I am a big admirer of M. K. Gupta Sir. He is truly the BEST TEACHER and his teaching is the best of all, I have ever experienced, His way of imparting knowledge and clarifying subject is so good that students gain complete knowledge without much personal efforts. I would call M. K. Gupta Sir classes not only teaching but a full training programme for students. Teaching is not a business for Sir, rather it is more of social welfare that he is doing by giving brilliant minds full of knowledge to the society. He motivates students by excellent means and enhance their knowledge to another level. I must say class notes are more than sufficient to get good marks. I heartily dedicate my achievement to Sir. Highly greatful to you Sir. Thank you so much. 98. AKHIL SEHGAL NOV-2011 (Roll No.211527) 46th RANK M.K. Gupta Sir teaches the course very brightly. Really Sir, you are the god in the industry of teaching. Sir you are the best. 99. AMAN MITTAL NOV-2011 (Roll No.274222) 47th RANK In my opinion, no teacher can teach the tax better than the M.K. Gupta Sir. He is very good by heart. He teaches in such a manner that no student can face problems. 100. ROUNAK SAWARIA NOV-2011 (Roll No.236371) 47th RANK M.K. Gupta Sir has a very deep knowledge about the subject and his practical approach towards the subject helps understand the concepts very easily. M.K. Gupta Sir repeats every section atleast twice and makes us learn them in the class itself. The notes provided in the class are more than enough for IPCC and the questions in his book ranges from simple problems to difficult ones. Last but not the least, M.K. Gupta Sir is very kind hearted and generous person and he is so hardworking which is highly motivational for all of us. 101. PANCHANAN AGGARWAL NOV-2009 (Roll No. 52360) 47th RANK In my personal opinion, he is a very good teacher and also very friendly in nature while teaching. He always give the right way to success. His coaching is excellent, knowledge is excellent and queries taken up are excellent. 102. DEVENDER PANDEY NOV-2009 (Roll No. 60647) 47th RANK The best thing about M. K. Gupta Sir is that he explains each and everything related to the topic. Class notes are sufficient for getting good marks. His coaching is excellent, knowledge is excellent and queries taken up are excellent. 103. ASHISH JAIN NOV-2009 (Roll No. 53180) 47th RANK I think M. K. Gupta Sir should be greeted with the title of Tax Guru. His knowledge about the subject is outstanding and matchless. The method of teaching is unique. The material provided by the Sir is a package in itself. The style of taking up the query is excellent. I suggest every CA aspirant to join M. K. Gupta Sir classes. Sir put enough efforts for the success of students. Once you join Tax classes of Sir leave everything on him and just concentrate in the class & have belief in him. 104. PANKAJ BANSAL NOV-2009 (Roll No. 52640) 47th RANK Knowledge coupled with experience make M. K. Gupta Sir a great teacher. He is down to earth man who always guided and encouraged us to achieve the success. Book is quite good involving plenty of questions with solutions which helped me out very much. His coaching is excellent, knowledge is excellent and queries taken up are excellent. 105. PIYUSH BORANIA NOV-2009 (Roll No.40193) 47th RANK M. K. Gupta Sir is the best teacher for availing tax coaching. Sir responds to queries in an excellent manner.

14 The incentive scheme of Sir for encouraging students is awesome. The study material of M. K. Gupta Sir is excellent and the problems in the books are very comprehensive. 106. NITIN KUMAR NOV-2010 (Roll No.184364) 47th RANK M. K. Gupta Sir is very knowledgeable person. He has vast experience of teaching. His book is very helpful and cover all concepts.

107. SIDDHARTH JAIN NOV-2010 (Roll No.183051) 47th RANK Sirs knowledge was excellent, his motivation in class did wonders, it was always interesting because Sir made class very interesting. I really enjoyed the practical questions in class, especially his notes that he made us write in class. The book was helpful, especially the service tax/vat book, I enjoyed the summarized portion of service tax. Overall an excellent experience! 108. CHETAN MALIK MAY-2011(Roll No.47892) 47th RANK The teacher is really good. It was a really good experience. Books are the best and so as the coaching provided by the Sir. 109. VIVEK SUGANDH NOV-2011 (Roll No.222907) 48th RANK M.K. Gupta Sir, in my opinion is a vast storehouse of knowledge in the field of taxation. He, apart from being a great teacher is also an awesome human being. His coaching classes provides a great amount of competitive environment and I feel myself blessed to be his student. 110. AMANDEEP SINGH BHATIA NOV-2008 (Roll No. 66060) 48th RANK M.K. Gupta Sir is a good teacher. He knows each and every aspect of TAXATION and also gives his precious knowledge to students. He is very helpful teacher. He is always ready to solve the problems of every student. I must say that I just got rank because of Sir. He motivates every students to study hard. 111. NIMISHA SARDA NOV-2011(Roll No.271629) 49th RANK M.K. Gupta Sir is an awesome teacher. The way he explains us the subject, it is not just understandable but becomes so interesting. Not just this, M.K. Gupta Sirs incentives also show how much he wants his students to excel. There comes a lot of confusion regarding completeness of syllabus, especially of indirect tax, but when it comes to Sirs notes and books combined- there remains not a single doubt. Thank You so much Sir. 112. SHIVAM MARWAH NOV-2011(Roll No. 406941) 49th RANK M.K. Gupta Sir is an amazing teacher having excellent knowledge of the subject. He is always ready to solve queries. His techniques of motivating students uncomparable. M.K. Gupta Sir is Best. Study material is excellent. Examples are very good in the books. Thank you. 113. KESHAV BHARDWAJ NOV-2011 (Roll No.421167) 50th RANK M.K. Gupta Tax Classes are a perfect blend of studies as well as innovation and skill development. One can not only gain in academic arena but studying grooms personality and professional attitude. Talking about M.K. Gupta Sir, he is not only a good teacher but also a good motivator. The apt and appropriate presentation of the relevant question and answer have helped a lot in reaching a new height of success. Great teacher, Great human being. Thanks a lot Sir. Thank You so much. 114. PANKAJ GULATI NOV-2009 (Roll No. 52267) 50th RANK The knowledge given by Sir is excellent. M.K. Gupta Sir motivate each and every student by giving them various rewards. He is available all time to solve the Queries of the students. The books provided by Sir are enough to score good marks. Thank you Sir. 115. HEENA GUPTA MAY-2010 (Roll No.53424) 50th RANK The M. K. Gupta Sir classes were fabulous. The way Sir mixes classroom knowledge with practical work is unexplainable. Knowledge of Sir is incomparable. The study environment of his classes was so comfortable that I was able to concentrate in classes. I would like to thank Sir from the bottom of my heart for his kind contribution in my studies. Thank you Sir.

15

A.R. Rahman
A winner all through the way A R Rahman, the famous music composer is always par excellence. Outstanding numbers and the latest awarding of the Oscar for the original music score in Slumdog millionaire only proves his credibility. A shy person, Rahman also exudes confidence. The biggest virtue he has is that he is multi talented in all aspects of music. He dares to explore and collaborate with new talents and inclusions.

A R Rahman has high regards for his mother. His awards and achievements only make him produce new music to all the music lovers. His composition in Roja, Slumdog millionaire, Taal and the various voice treatments he gives to his singers is unparalleled. Maintaining the distinct ethnic standards he incorporates the latest in music and instruments. He has several national and international awards to his credit including the prestigious Filmfare award. Going through hardships in life A R Rahman is blessed with all the humility which propels him to a niche carved by his own self made virtue. Bollywood Dreams was another music venture in the international award. Words seem less to write about this Lata Mangeshkar awardee who has done India proud with his contribution to music.

16

SRINIVASA RAMANUJAN Mathematician

Srinivasa Ramanujan was a great Mathematician, who became world famous at the age of twenty six. He was born at Erode in Tamil Nadu on 22 December 1887. Ramanujan could not complete his college education because of illness. He was so interested in mathematics that he learned on his own. He found out new formulas for solving mathematical problems and wrote articles about them. Professor Hardy a scientist in the Cambridge University saw one his article and impressed by his knowledge, took Ramanujan to England. Ramanujan was considered as the master of theory of numbers. The most outstanding of his contributions was his formula for p (n), the number of 'partitions' of 'n'. It was in 1914, while he was working in Trinity College he developed the 'Number Theory' and for his valuable contribution, was elected fellow of Trinity College on 18th October 1917. He returned to India in 1919 and began Research. He passed away on 26 April 1920. Government of India issued a commemorative stamp in his honour. Indian National Science academy and many other scientific institutions in India are giving various awards in memory of this brilliant mathematician.

17

J. C. BOSE
Founder of Bose Research Institute,

Biologist, Physicist
Jagadish Chandra Bose was one of the pioneers of modern science in India. His research was on the properties of electro-magnetic waves. His major achievement was to demonstrate the similarity of responses to stimulation among the living and the nonliving as well as the fundamental similarity of responses in plant and animal tissues. The British Government knighted him in 1917. He founded the 'Bose Research Institute' in Kolkatta in 1917.

18 Bose was not only a biologist, but also a physicist. Bose believed that by focusing on the boundaries between different physical and biological sciences, he would be able to demonstrate the underlying unity of all things. Bose's biological researches were founded initially by the discovery that an electric receiver seems to show science of fatigue after continued use. He can rightly be called the inventor of wireless telegraphy. Bose was the first in the world to fabricate and demonstrate in public (1985) the device that generated microwaves-radio waves of very short wave length. But his invention was not patented before Guglielmo Marconi (1896) who became internationally recognised as the inventor. After completing his studies in London, Bose return to Kolkatta and was appointed Professor at Presidency College in Kolkatta. Then he became the director of the institute he founded and remained in the post till his death on 23rd November 1937.

19

J.R.D. TATA Industrialist,

4th Chairman of TATA Industries


Jehangir Ratanji Dadabhoy Tata, was born on July 29, 1904 in Paris. He was the second child of Mr Ratanji Dadabhoy Tata. He spent much of his childhood in France since his mother was French. In 1922, Tata returned to India to join the family business. He inherited most of his grandfather Jamshedji's industrial empire but first became a pioneer in aviation. He had a great personality with a charm and style of his own. After his father's death in 1926, Tata became the director of the Board of Tata Sons Ltd. Eventually in 1938, he became the chairman of the company. Under his leadership Tata Sons expanded into one of the largest industrial empire in the country- from ironworks and steelworks into chemicals, hotels, engineering and lot of other industries. In 1945, Tata Steel promoted the Tata Engineering and Locomotive Company (TELCO) with an objective to produce locomotives for the Indian Railways. Today Telco has emerged as the countrys largest Commercial Vehicle producer. JRD Tata was the first Indian pilot to qualify for a British private license. He founded Tata Airlines in 1932 and by 1953, it developed and came to be known as Indian Airlines. Till 1978, Tata was the Chairman of the Indian Airlines and Air India. Tata was an early advocate of family planning and he created the Family Planning Foundation in 1971. His innovations in India's fledgling hotel and tourist industry as well as his contributions to scientific and technical research and corporate management gained public recognition from the Indian Government. He was honoured by India's highest civilian award, Bharat Ratna in 1991 and United Nations Population Award in 1992. In 1991, at the age of 87, Tata retired from Tata Sons. He passed away on November 29, 1993.

Value Added Tax

20

DIRECT TAXES: Income Tax / Wealth Tax INDIRECT TAXES: Central Excise Duty/
Service Tax/Custom Duty/VAT
DIRECT TAX / INDIRECT TAX If incidence of tax is borne by the person who is making payment of tax, such tax is called Direct Tax e.g. Income Tax or Wealth Tax but if incidence is borne by one person and payment is made by some other person, it is called Indirect Tax like Central Excise Duty, Service Tax, Custom Duty and Sales Tax. Income Tax and Wealth Tax are levied and collected by Central Government and are monitored by Central Board of Direct Taxes. Central Excise Duty, Custom Duty and Service Tax are levied and collected by Central Govt. and are regulated by Central Board of Excise and Custom. Sales Tax also called Value Added Tax is levied and collected by State Government and it is regulated by Acts of individual States. E.g. in case of Delhi the relevant Act is Delhi Value Added Tax Act, 2004 Central Board of Direct Taxes (CBDT) and Central Board of Excise and Custom (CBEC) work under Department of Revenue of Finance Ministry. Value Added Tax system is applicable only in Indirect Taxes. At IPCC level there will be one paper of Taxation and there will be 50 marks for Direct Tax and 50 Marks for Indirect Tax and at CA-FINAL level, there will be 2 papers of Taxation Direct Tax 100 Marks (Income Tax 90 Marks/Wealth Tax 10 Marks) Indirect Tax 100 Marks (Service Tax + VAT 40 Marks / Excise 40 Marks / Custom 20 Marks) Introduction Concept of Value Added Tax was introduced first of all in France in 1954. It was introduced in India in Central Excise Duty wef 1 st March 1986 and at that time it was called MODVAT CREDIT (Modified Value Added Tax Credit) but afterwards its name was changed as CENVAT CREDIT (Central Value Added Tax Credit). Central Excise Duty, Custom Duty and Service Tax is charged by Central Govt. and its tax credit is called Cenvat Credit and the rules for Cenvat Credit are given in Cenvat Credit Rules, 2004 (Applicable wef 10.09.2004). Concept of tax credit was introduced in Sales Tax in the year 2005 and Sales Tax is being charged by the State Govt. and accordingly every State Govt. has its own Value Added Tax Act (For example in Delhi Delhi Value Added Tax, 2004) Tax credit rules are the same for different tax but there is some difference because some of the tax are charged by the Central Govt. and some of the tax are charged by State Govt.

Value Added Tax

21

Question 1 (V. Imp.): Explain the concept of Value Added Tax / Explain Taxonomy (process of making groups) of VAT/ Different stages of VAT. Answer: Before VAT If a Manufacturer has sold goods to the Wholesaler and has charged sales tax and Wholesaler has further sold the same goods to the Retailer and has again charged sales tax, in this case there are two anomalies (defect): (i) Sales tax has been charged twice on the same item. (ii) There is sales tax on sales tax (it is called cascading effect) Example If a Manufacturer has manufactured certain goods and cost price is `1,00,000 and his expenses plus profit is `20,000 and sales tax is 10%, in this case goods shall be sold for ( `1,00,000 plus 20,000) + (10% of `1,20,000) = `1,32,000. If the Wholesaler is purchasing goods from Manufacturer for `1,32,000 and his expenses plus profit is `20,000 and is charging sales tax @ 10%, the goods shall be sold for ( `1,32,000 plus 20,000) + (10% of `1,52,000) = `1,67,200. The Retailer has added his expenses plus profit of 20,000 and is charging sales tax @ 10%, the goods shall be sold for (`1,67,200 plus 20,000) + (10% of `1,87,200) = `2,05,920. In this case sales tax has been charged three times on the original amount of `1,20,000 (once by manufacturer, once by the Wholesaler and once by retailer) accordingly a single item has suffered sales tax three times. Further the Wholesaler has charged sales tax on the amount of the sales tax paid by him i.e. there is sales tax on sales tax which is called Cascading Effect. After VAT The above anomalies can be rectified with the help of VAT. Under VAT, the Wholesaler is allowed to take tax credit of `12,000 being the sales tax paid by him and he will be selling the goods for `1,40,000 (`1,20,000 + `20,000), sales tax charged by him shall be `14,000. Since he has taken credit of `12,000, net sales tax payable by him shall be `2,000 and it will be called value added tax i.e. sales tax is only on the value addition of `20,000. Similarly, the Retailer shall be allowed to take credit for the tax paid by him and he will pay sales tax only on the value addition. The tax paid by the Wholesaler to the Manufacturer shall be called his Input tax ( `12,000) and tax charged by him from the Retailer shall be called Output tax ( `14,000) and tax payable by him shall be called Net tax (`2,000). Value Added Tax is a Multi Stage tax and is being charged at every stage of sale and these stages are called stages of VAT and are as given below: 1. Manufacturer to Distributor 2. Distributor to Wholesaler 3. Wholesaler to Retailer 4. Retailer to Consumer

Value Added Tax

22

Question 2 (V. Imp.): Explain concept of input tax credit. Answer: If any Registered Dealer is purchasing goods within a particular State and has paid value added tax and subsequently the goods were sold either in the same State or to some other State, in that case such Registered Dealer shall be allowed to take credit for input tax, however, tax credit is allowed only to the Registered Dealers and further registered dealer should purchase goods only from registered dealer i.e. if the goods have been purchased from unregistered dealer, no VAT credit is allowed. Registration is required if the sale turnover has exceeded the threshold limit (basic limit) of `10 lakh during the year. Voluntary registration is allowed to every dealer at any time. Example Mr. D is a dealer registered in Delhi and he purchased goods of `20,00,000 plus Delhi VAT @ 12.5% and sold the goods in Delhi for `30,00,000 plus Delhi VAT @ 12.5%, in this case, tax credit for input tax shall be allowed and computation of output tax, input tax and net tax shall be as given below: Sale 30,00,000 Output Tax @ 12.5% 3,75,000 Less: Input Tax credit (20,00,000 x 12.5%) 2,50,000 Net Delhi VAT Payable 1,25,000 If in the above case goods were sold to an unregistered dealer in UP and Central Sales Tax was charged @ 12.5%, in that case also tax credit is allowed and tax treatment shall be as given below: Sale 30,00,000 Output Tax Central Sales Tax @ 12.5% 3,75,000 Less: Input Tax credit (20,00,000 x 12.5%) 2,50,000 Net CST Payable 1,25,000 If the goods have been sold to some Registered Dealer in U.P. and Central Sales Tax was charged @ 2%, in that case also, tax credit is allowed and tax treatment shall be as given below: Sale 30,00,000 Output Tax Central Sales Tax @ 2% 60,000 Less: Input Tax credit (20,00,000 x 12.5%) 2,50,000 Balance VAT Credit (to be carried forward or refund can be taken) 1,90,000 If the goods have been purchased from outside the State and has paid Central Sales Tax to that particular State, tax credit for such INPUT TAX is not allowed. If in the above case goods were purchased from Punjab and CST was paid @ 2% and the goods were sold either in Delhi or in some others States, tax credit is not allowed and tax treatment shall be as given below: Example (a) Goods sold in Delhi Sale 30,00,000 Output Tax @ 12.5% 3,75,000 Input tax credit is not allowed Net Delhi VAT Payable 3,75,000 (b) Goods sold to Unregistered Dealer in U.P. Sale Output Tax Central Sales Tax @ 12.5% Input tax credit is not allowed Net CST Payable (c) Goods sold to Registered Dealer in U.P. against Form C Sale 30,00,000 3,75,000 3,75,000 30,00,000

Value Added Tax Output Tax Central Sales Tax @ 2% Input tax credit is not allowed Net CST Payable

23 60,000 60,000

Tax credit in case of goods exempt from output tax If raw material or other goods have been purchased and have been utilized in manufacturing, processing or packing of the product which is exempt from sales tax, no VAT credit shall be allowed for such sales tax. Tax credit in case of Export or supply to Special Economic Zone If the goods have been exported, in that case such export sales are exempt from excise duty or sales tax etc but even then as a special case tax credit is allowed and export goods are also called zero-rated goods. Similarly if the goods have been sold in Special Economic Zone, there will be same procedure as in case of export. Special Economic Zone means a specific area which has been declared to be SEZ by the Government and the persons having their units in the SEZ shall be required to export the goods manufactured by them and there will not be any output tax. The persons selling goods or providing services in the SEZ shall not charge any tax from the buyers in the SEZ but the seller shall be allowed tax credit as a special case. Example Mr. X is a manufacturer and is registered in DVAT ACT, 2004 and he has purchased raw material for `20,00,000 and paid DVAT @ 4% and manufactured certain goods which were sold in Delhi for `30,00,000 and the goods are exempt from DVAT, in this case, tax credit is not allowed and tax treatment shall be as given below: Sale 30,00,000 Output Tax Nil Input tax credit is not allowed Net Delhi VAT Payable Nil If in the above case goods have been exported or goods have been sold in Special Economic Zone (SEZ), tax treatment shall be as given below: Sale 30,00,000 Output Tax Nil Input tax credit is allowed (20,00,000 x 4%) 80,000 Net Delhi VAT Payable Nil Dealer can claim refund for such unutilized input tax credit. Stock Transfer If any dealer has purchased goods within the state and the goods were stock transferred to some other state, in such cases VAT credit is allowed after retaining 2% of the input tax. Example ABC Ltd. has purchased raw material for `20,00,000 in the state of Delhi and paid DVAT @ 12.5% and goods were stock transferred to there branch in U.P., in this case input tax credit allowed shall be 20,00,000 x 10.5% (12.5% - 2%) 2,10,000 Tax credit on capital goods If any person has purchased capital goods and such capital goods are to be used in the manufacturing of the goods which are taxable, in that case Tax credit should be allowed or not will depend in each case and there may be three situations (i) Tax credit is allowed in the same year i.e. Consumption Variant. (ii) Tax credit is allowed in instalments depend upon the life of the capital asset i.e. Income Variant.

Value Added Tax (iii) Tax credit should not be allowed i.e. Gross Product Variant.

24

If the capital goods are being used for manufacturing of product which is exempt from sales tax, no VAT credit shall be allowed. If capital goods are to be used for manufacturing of taxable goods as well as exempt goods, in that case only proportionate VAT credit is allowed. (In case of Central Excise, if capital goods are being used for manufacturing of taxable goods as well as exmept goods, in that case cenvat credit for excise duty is allowed for the capital goods) Illustration 1: Compute the invoice value to be charged and amount of tax payable under VAT by a Registered Dealer who had purchased goods for `1,50,000 (exclusive of VAT) from the same State and after adding for expenses of `12,000 and of profit `25,000 had sold in the same State. The rate of VAT on purchase and sales is 12.5%. Solution: ` Purchase price 1,50,000 Add: Expenses 12,000 Add: Profit 25,000 Amount to be billed 1,87,000 Add: VAT @ 12.5% - Output Tax 23,375 Total invoice value 2,10,375 VAT Payable VAT charged in the invoice - Output Tax Less: VAT credit on input 12.5% of `1,50,000 - Input Tax Net VAT Payable (b) Presume the goods were sold in some other States to Unregistered Dealer. Solution: Purchase price Add: Expenses Add: Profit Amount to be billed Add: CST @ 12.5% - Output Tax Total invoice value CST Payable CST charged in the invoice - Output Tax Less: VAT credit on input 12.5% of `1,50,000 - Input Tax Net CST Payable (c) Presume the goods were sold in some other States to Registered Dealer against Form C. Solution: Purchase price Add: Expenses Add: Profit Amount to be billed Add: CST @ 2% - Output Tax Total invoice value VAT Payable VAT charged in the invoice - Output Tax Less: VAT credit on input 12.5% of `1,50,000 - Input Tax Balance VAT Credit (to be carried forward or refund can be taken) 23,375 (18,750) 4,625

1,50,000 12,000 25,000 1,87,000 23,375 2,10,375 23,375 (18,750) 4,625

1,50,000 12,000 25,000 1,87,000 3,740 1,90,740 3,740 (18,750) 15,010

Value Added Tax

25

Illustration 2: Compute the VAT amount payable by Mr. A (Registered Dealer) who purchases goods from a manufacturer on payment of `4,50,000 (including VAT) and earns 15% profit on purchase. The goods have been sold to retailers and VAT rate on purchase and sale is 12.5%. Solution: ` Purchase price 4,50,000.00 Less: Input tax (4,50,000 x 12.5% / 112.5%) (50,000.00) 4,00,000.00 Add: Profit (4,00,000 x 15%) 60,000.00 Amount to be billed 4,60,000.00 Add: VAT @ 12.5% - Output Tax 57,500.00 Total invoice value 5,17,500.00 VAT Payable Output tax Less: Tax credit (12.5% of `4,00,000) - Input Tax Net VAT Payable (b) Presume the goods were sold in some other States to Unregistered Dealer. Solution: Purchase price Less: Input tax (4,50,000 x 12.5% / 112.5%) Add: Profit (4,00,000 x 15%) Amount to be billed Add: CST @ 12.5% - Output Tax Total invoice value CST Payable Output tax Less: Tax credit (12.5% of `4,00,000) - Input Tax Net CST Payable (c) Presume the goods were sold in some other States to Registered Dealer against Form C. Solution: Purchase price Less: Input tax (4,50,000 x 12.5% / 112.5%) Add: Profit (4,00,000 x 15%) Amount to be billed Add: CST @ 2% - Output Tax Total invoice value VAT Payable Output tax Less: Tax credit (12.5% of `4,00,000) - Input Tax Balance VAT Credit (to be carried forward or refund can be taken) 57,500.00 (50,000.00) 7,500.00 ` 4,50,000.00 (50,000.00) 4,00,000.00 60,000.00 4,60,000.00 57,500.00 5,17,500.00 57,500.00 (50,000.00) 7,500.00 ` 4,50,000.00 (50,000.00) 4,00,000.00 60,000.00 4,60,000.00 9,200.00 4,69,200.00 9,200.00 (50,000.00) 40,800.00

Illustration 3: Mr. X (Registered Dealer) is a trader in Delhi and he has purchased certain goods from Punjab for `4,00,000 and has paid central sales tax @ 2%. He has sold all the goods in the state of Delhi for `6,00,000 plus VAT @ 12.5%.

Value Added Tax

26

He has purchased certain goods in Delhi for `5,00,000 and paid VAT @ 12.5% and all the goods were sold by him under inter state sale to a registered dealer in U.P. for `7,00,000 plus central sales tax @ 2%. Show VAT calculation. Solution : Computation of VAT payable: Purchase from Punjab and sold in Delhi Purchase price Add: Central sales tax @ 2% Purchase Price Sale Add: VAT @ 12.5% - Output tax Selling Price Purchase from Delhi and sold in U.P. Purchase price Add: Sales tax @ 12.5% - VAT credit Purchase Price Sale Add: Central sales tax @ 2% - Output tax Selling Price State VAT Less: VAT credit Net State VAT payable CST Payable

` 4,00,000 8,000 4,08,000 6,00,000 75,000 6,75,000 ` 5,00,000 62,500 5,62,500 7,00,000 14,000 7,14,000 75,000 62,500 12,500 14,000

Illustration 4: Mr. X (Registered Dealer in Delhi) is a manufacturer and he has purchased raw material R1 from Punjab for `2,00,000 plus central sales tax @ 2%. He has purchased raw material R2 in Delhi for `3,30,000 inclusive of sales tax @ 10%. His processing charges is `1,00,000 and profit margin is `1,00,000. Half of the goods were sold in Delhi and VAT payable is @ 10% and remaining half were sold to a person in U.P. under inter state sale and has charged central sales tax @ 2%. Show working for VAT. Solution: Computation of VAT payable: Raw material R1 Purchase price Add: Central sales tax @ 2% Purchase Price Raw material R2 Purchase price Less: Input tax (3,30,000 x 10 / 110) Net purchase price exclusive of VAT Cost of final product Raw material R1 ` 2,00,000 4,000 2,04,000 3,30,000 30,000 3,00,000 2,04,000

Value Added Tax Raw material R2 Processing charges Profit margin Total Goods sold in Delhi Assessable value Add: VAT @ 10% - Output tax Sales value Goods sold in U.P. Assessable value Add: Central sales tax @ 2% - Output tax Sales value State VAT Less: VAT credit Net State VAT payable CST Payable

27 3,00,000 1,00,000 1,00,000 7,04,000 3,52,000 35,200 3,87,200 3,52,000 7,040 3,59,040 35,200 30,000 5,200 7,040

Illustration 5: Mr. X (Registered Dealer) is a manufacturer sells goods to Mr. B (Registered Dealer), a distributor for `2,000. Mr. B sells goods to Mr. K (Registered Dealer), a wholesaler for `2,400. The wholesaler sells the goods to a Retailer (Registered Dealer) for `3,000. The retailer sold the goods to consumers for `4,000. All the above amounts are exclusive of VAT. Compute input tax credit, output tax and net tax under invoice method for each of the person and VAT rate is @ 12.5%. (b) Presume all the above amounts are inclusive of VAT @ 12.5%. Solution (a): Manufacturer (Mr. X) Sale price Add: VAT @ 12.5% Total invoice value VAT A/C: Output tax Less: Tax credit Net Tax Payable Distributor (Mr. B) Purchase price Add: Profit Amount to be billed Add: VAT @ 12.5% Total invoice value VAT A/C: Output tax ` 2,000 250 2,250 250 Nil 250 2,000 400 2,400 300 2,700 300

Value Added Tax Less: Tax credit Net Tax Payable Wholesaler (Mr. K) Purchase price Add: Profit Amount to be billed Add: VAT @ 12.5% Total invoice value VAT A/C: Output tax Less: Tax credit Net Tax Payable Retailer Purchase price Add: Profit Amount to be billed Add: VAT @ 12.5% Total invoice value VAT A/C: Output tax Less: Tax credit Net Tax Payable Solution (b): Manufacturer (Mr. X) Sale price Output tax (2,000 x 12.5 % / 112.5%) VAT A/C: Output tax Less: Tax credit Net Tax Payable Distributor (Mr. B) Sale price Output tax (2,400 x 12.5% / 112.5%) VAT A/C: Output tax Less: Tax credit Net Tax Payable Wholesaler (Mr. K) Sale price Output tax (3,000 x 12.5% / 112.5%) VAT A/C: Output tax

28 250 50 2,400 600 3,000 375 3,375 375 300 75 3,000 1,000 4,000 500 4,500 500 375 125 ` 2,000 222 222 Nil 222 2,400 267 267 222 45 3,000 333 333

Value Added Tax Less: Tax credit Net Tax Payable Retailer Sale price Output tax (4,000 x 12.5% / 112.5%) VAT A/C: Output tax Less: Tax credit Net Tax Payable Question 3 (V. Imp.) : Explain Central Sales Tax including rates under Central Sales Tax. Answer: Central Sales Tax was introduced in 1956 through Central Sales Tax Act 1956.

29 267 66 4,000 444 444 333 111

Prior to levy of CST, State Governments use to charge sales tax even in case of inter-state sales and there was multiple taxation on the basis of nexus theory e.g. If seller was in Punjab and buyer was in U.P., both of the States collected sales tax because of nexus i.e. connection of the sales transaction with each of the state. In order to check the problem of multiple taxation, Central Sales Tax Act was enacted (to pass a law) in 1956 and in case of Inter State sale, Central Sales Tax was levied and it was to be collected by the State Government from where the sales has taken place. It is collected by the State Government on behalf of the Central Government but as per provisions of Central Sales Tax Act, the tax so collected shall be retained by the respective State Government. As the purpose of levying of CST was not to collect Central Sales Tax by the Central Government rather the purpose was to check multiple taxation. If Central Sales Tax has been paid in one particular State as input tax and output tax is to be paid in some other State, tax credit shall not be allowed for such input tax e.g. A registered dealer of Delhi Mr. D has purchased certain goods from Punjab for `1,00,000 and paid Central Sales Tax to the Punjab Government `2,000 @ 2% and subsequently the same goods were sold in Delhi for `1,50,000 and has charged local sales tax @ 12.5% amounting to `18,750, in this case VAT credit of `2,000 shall not be allowed and entire amount of `18,750 shall be paid to the Delhi Government. If the Dealer has purchased goods from Delhi and has sold the goods in U.P. and has collected Central Sales Tax from the buyer in U.P., in this case such CST has to be paid to Delhi Government hence VAT credit for input tax paid in Delhi shall be allowed. Rates under Central Sales Tax If the goods have been sold to a dealer registered under Central Sales Tax, rate of CST shall be equal to LST but subject to highest rate of CST at 2%. If goods have been sold to any other person, rate of CST shall be equal to LST and it can be shown as given below: Rate of LST 1% 2% 3% 10% 12% 12.5% Sale to Dealer Registered under CST 1% 2% 2% 2% 2% 2% Sale to any Other Person 1% 2% 3% 10% 12% 12.5%

Value Added Tax

30

The dealer selling the goods to the registered dealer shall obtain Form C from the purchasing dealer being the proof of his being a registered dealer. If the goods are sold to the Government department, it will not be considered to be registered dealer. Question 4: Explain scope of input tax credit. Answer: Scope of input tax credit For the purpose of claiming input tax credit, the taxable goods should be purchased for any one of the following purposes: 1. for sale/resale within the State; 2. for sale to other parts of India in the course of inter-state trade or commerce; 3. to be used as containers or packing materials or raw materials or consumable stores, required for the purpose of manufacture of taxable goods or in the packing of such manufactured goods intended for sale; 4. for being used in the execution of a works contract (rendering of services alongwith supply of goods e.g. construction contract); 5. to be used as capital goods required for the purpose of manufacture or resale of taxable goods; 6. to be used as raw materials or capital goods or consumable stores and packing materials/containers for manufacturing/packing goods to be sold in the course of export out of the territory of India. Question 5 (V. Imp.): Explain purchases not eligible for input tax credit. Answer: Purchases not eligible for input tax credit Input tax credit shall not be allowed in the following circumstances: 1. Purchases from unregistered dealers; 2. Purchases from registered dealer who opt for composition scheme; 3. Purchase of goods as may be notified by the State Government; 4. Purchase of goods, which are being utilized in the manufacture of, exempted goods; 5. Goods in stock, which have suffered tax under an earlier Act but under VAT Act they are covered under exempted items; 6. Goods imported from outside the territory of India; 7. Goods imported from other States. Illustration 6: Mr. X is a Dealer Registered in Delhi Value Added Tax Act, 2004 and also under Central Sales Tax Act, 1956 and he has submitted the informations as given below: (i) Purchased Goods A from Delhi `1,00,000 and paid VAT @ 4% and sold the goods in Delhi for `1,50,000 and charged VAT @ 4%.

Value Added Tax

31

(ii) Purchased goods B from U.P. for `2,00,000 and paid central sales tax @ 2% and sold goods in Delhi for `2,50,000 and charged VAT @ 12.5%. (iii) Purchased goods C from Delhi for `4,00,000 and paid VAT @ 12.5% and sold the goods to a registered dealer in Orissa for `4,75,000 and charged central sales tax @ 2% (iv) Purchased goods D for `5,00,000 in Delhi and paid VAT @ 12.5% and sold the goods for `5,50,000 to an unregistered dealer in Punjab and charged central sales tax @ 12.5%. (v) Purchased goods E from Madhya Pradesh for `3,00,000 and paid central sales tax @ 1% and sold goods in Maharashtra for `3,50,000 and charged central sales tax @ 1%. (vi) Purchased goods F from Delhi `7,00,000 and paid VAT @ 1% and the goods were sold to an unregistered dealer in Maharashtra for `7,50,000 and charged central sales tax @ 1%. (vii) Purchased goods G for `6,00,000 in Delhi and paid VAT @ 12.5% and goods were stock transferred to some other state. (viii) Purchased goods H for `8,00,000 in Delhi and paid VAT @ 4% and goods were exported for `8,50,000 and VAT was charged (because as per section 5 Central Sales Tax Act, 1956, VAT can not be charged in case of export sale.) (ix) Purchased goods I for `9,00,000 in Delhi and paid VAT @ 12.5% and sold the goods to a manufacturer in SEZ for `10,50,000 and no VAT was charged. Show the tax treatment for VAT and also compute his income tax liability for the assessment year 2012-13. Solution: (i) Purchased Goods A from Delhi Add: VAT @ 4% Purchase Price Input tax credit Goods sold in Delhi Add: VAT @ 4% Selling Price (ii) Purchased goods B from U.P. Add: Central sales tax @ 2% Purchase Price Input tax credit Goods sold in Delhi Add: VAT @ 12.5% Selling Price (iii) Purchased goods C from Delhi Add: VAT @ 12.5% Purchase Price Input tax credit Goods sold in Orissa ` 1,00,000 4,000 1,04,000 4,000 1,50,000 6,000 1,56,000 2,00,000 4,000 2,04,000 Nil 2,50,000 31,250 2,81,250 4,00,000 50,000 4,50,000 50,000 4,75,000

Value Added Tax Add: Central sales tax @ 2% Selling Price (iv) Purchased goods D from Delhi Add: VAT @ 12.5% Purchase Price Input tax credit Goods sold in Punjab to unregistered dealer Add: Central sales tax @ 12.5% Selling Price (v) Purchased goods E from Madhya Pradesh Add: Central sales tax @ 1% Purchase Price Input tax credit Goods sold in Maharashtra Add: Central sales tax @ 1% Selling Price (vi) Purchased goods F from Delhi Add: VAT @ 1% Purchase Price Input tax credit Goods sold in Maharashtra to unregistered dealer Add: Central sales tax @ 1% Selling Price (vii) Purchased goods G from Delhi Add: VAT @ 12.5% Purchase Price Goods Stock transferred VAT credit allowed in stock transfer (6,00,000 x 10.5%) (in case of stock transfer, VAT credit shall be allowed after retaining 2%) (viii) Purchased goods H from Delhi Add: VAT @ 4% Purchase Price Input tax credit Goods exported (ix) Purchased goods I from Delhi Add: VAT @ 12.5% Purchase Price Input tax credit Goods sold to manufacturer in SEZ

32 9,500 4,84,500 5,00,000 62,500 5,62,500 62,500 5,50,000 68,750 6,18,750 3,00,000 3,000 3,03,000 Nil 3,50,000 3,500 3,53,500 7,00,000 7,000 7,07,000 7,000 7,50,000 7,500 7,57,500 6,00,000 75,000 6,75,000 6,00,000 63,000

8,00,000 32,000 8,32,000 32,000 8,50,000 9,00,000 1,12,500 10,12,500 1,12,500 10,50,000

Value Added Tax VAT A/C Particulars OUTPUT TAX Goods A Goods B Goods C Goods D Goods E Goods F Goods G (Stock transfer) Goods H (Export) Goods I (Sale to SEZ) LESS: INPUT TAX CREDIT Goods A Goods B Goods C Goods D Goods E Goods F Goods G Goods H Goods I 4,000 Not allowed 50,000 62,500 Not allowed 7,000 63,000 32,000 1,12,500 3,31,000 ` VAT 6,000 31,250 --------Not applicable Nil Nil 37,250 ` CST ----9,500 68,750 3,500 7,500 ----89,250

33

After adjusting output VAT of `37,250 and CST of `89,250, there will be unutilised VAT credit of `2,04,500 and it can be set off from other output tax or it can be carried forward or refund can be claimed but procedure differs from State to State. At the year end it should be shown on the assets side of the balance sheet under the head CURRENT ASSETS, LOAN AND ADVANCES. Computation of income tax liability Particulars Purchases Goods A Goods B Goods C Goods D Goods E Goods F Goods H Goods I Net profit Amount ` 1,00,000 2,04,000 4,00,000 5,00,000 3,03,000 7,00,000 8,00,000 9,00,000 5,18,000 44,25,000 Particulars Sales Goods A Goods B Goods C Goods D Goods E Goods F Goods H Goods I Amount ` 1,50,000 2,50,000 4,75,000 5,50,000 3,50,000 7,50,000 8,50,000 10,50,000 44,25,000 5,18,000 5,18,000 Nil 5,18,000 35,600 712 356

Income under the head Business/Profession Gross Total Income Less: Deduction u/s 80C to 80U Total Income Tax on `5,18,000 at slab rate Add: Education cess @ 2% Add: SHEC @ 1%

Value Added Tax Tax Liability Rounded off u/s 288B

34 36,668 36,670

Income shall be computed exclusive of VAT because any VAT collected shall be paid to the Government and it will not be considered to be income. Similarly VAT paid by the dealer is collected from the customer hence it will not be considered to be expense. If any balance is left in VAT credit, it will be shown as asset in the Balance Sheet on the last day of the relevant financial year. Further, the stock transfer of goods G is having a netural effect and thus ignored for calculation of business/profession income. Illustration 7: Mr. X is a Dealer Registered in Delhi Value Added Tax Act, 2004 and also under Central Sales Tax Act, 1956 and he has submitted the informations as given below: (i) Purchased Goods A from Delhi `1,00,000 inclusive of VAT @ 4% and sold the goods in Delhi for `1,50,000 inclusive of VAT @ 4%. (ii) Purchased goods B from U.P. for `2,00,000 inclusive of central sales tax @ 2% and sold goods in Delhi for `2,50,000 inclusive of VAT @ 12.5%. (iii) Purchased goods C from Delhi for `4,00,000 inclusive of VAT @ 12.5% and sold the goods to a registered dealer in Orissa for `4,75,000 inclusive of central sales tax @ 2% (iv) Purchased goods D for `5,00,000 in Delhi inclusive of VAT @ 12.5% and sold the goods for `5,50,000 to an unregistered dealer in Punjab inclusive of central sales tax @ 12.5%. (v) Purchased goods E from Madhya Pradesh for `3,00,000 inclusive of central sales tax @ 1% and sold goods in Maharashtra for `3,50,000 inclusive of central sales tax @ 1%. (vi) Purchased goods F from Delhi `7,00,000 inclusive of VAT @ 1% and the goods were sold to an unregistered dealer in Maharashtra for `7,50,000 inclusive of central sales tax @ 1%. (vii) Purchased goods G for `6,00,000 in Delhi inclusive of VAT @ 12.5% and goods were stock transferred to some other state. (viii) Purchased goods H for `8,00,000 in Delhi inclusive of VAT @ 4% and goods were exported for `8,50,000 and VAT was charged (because as per section 5 Central Sales Tax Act, 1956, VAT cannot be charged in case of export sale.) (ix) Purchased goods I for `9,00,000 in Delhi inclusive of VAT @ 12.5% and sold the goods to a manufacturer in SEZ for `10,50,000 and no VAT was charged. Show the tax treatment for VAT and also compute his income tax liability for the assessment year 2012-13. Solution: ` (i) Purchased Goods A from Delhi Purchase Price 1,00,000 VAT (1,00,000 x 4% / 104%) 3,846 Purchase price net of Tax 96,154 Input tax credit 3,846 Goods sold in Delhi Selling Price 1,50,000 VAT (1,50,000 x 4% / 104%) 5,769

Value Added Tax Selling Price net of Tax (ii) Purchased goods B from U.P. Purchase Price Central sales tax (2,00,000 x 2% / 102%) Input tax credit Goods sold in Delhi Selling Price VAT (2,50,000 x 12.5% / 112.5%) Selling Price net of Tax (iii) Purchased goods C from Delhi Purchase Price VAT (4,00,000 x 12.5% / 112.5%) Purchase price net of Tax Input tax credit Goods sold in Orissa Selling Price Central sales tax (4,75,000 x 2% / 102%) Selling Price net of Tax (iv) Purchased goods D from Delhi Purchase Price VAT (5,00,000 x 12.5% / 112.5%) Purchase price net of Tax Input tax credit Goods sold in Punjab to unregistered dealer Selling Price Central sales tax (5,50,000 x 12.5% / 112.5%) Selling Price net of Tax (v) Purchased goods E from Madhya Pradesh Purchase Price Central sales tax (3,00,000 x 1% / 101%) Input tax credit Goods sold in Maharashtra Selling Price Central sales tax (3,50,000 x 1% / 101%) Selling Price net of Tax (vi) Purchased goods F from Delhi Purchase Price VAT (7,00,000 x 1% / 101%) Purchase price net of Tax Input tax credit Goods sold in Maharashtra to unregistered dealer Selling Price

35 1,44,231

2,00,000 3,922 Nil 2,50,000 27,778 2,22,222

4,00,000 44,444 3,55,556 44,444 4,75,000 9,314 4,65,686

5,00,000 55,556 4,44,444 55,556 5,50,000 61,111 4,88,889

3,00,000 2,970 Nil 3,50,000 3,465 3,46,535

7,00,000 6,931 6,93,069 6,931 7,50,000

Value Added Tax Central sales tax (7,50,000 x 1% / 101%) Selling Price net of Tax (vii) Purchased goods G from Delhi Purchase Price VAT (6,00,000 x 12.5% / 112.5%) Purchase price net of Tax Goods Stock transferred VAT credit allowed in stock transfer (6,00,000 x 10.5% / 112.5%) (in case of stock transfer, VAT credit shall be allowed after retaining 2%) (viii) Purchased goods H from Delhi Purchase Price VAT (8,00,000 x 4% / 104%) Purchase price net of Tax Input tax credit Goods exported Selling Price Output tax Selling Price net of Tax (ix) Purchased goods I from Delhi Purchase Price VAT (9,00,000 x 12.5% / 112.5%) Purchase price net of Tax Input tax credit Goods sold to manufacturer in SEZ Selling Price Output tax Selling Price net of Tax VAT A/C Particulars OUTPUT TAX Goods A Goods B Goods C Goods D Goods E Goods F Goods G (Stock transfer) Goods H (Export) Goods I (Sale to SEZ) LESS: INPUT TAX CREDIT Goods A Goods B Goods C 3,846 Not allowed 44,444 ` VAT 5,769 27,778 --------Not applicable Nil Nil 33,547 ` CST ----9,314 61,111 3,465 7,426 ------81,316

36 7,426 7,42,574

6,00,000 66,667 5,33,333 6,00,000 56,000

8,00,000 30,769 7,69,231 30,769 8,50,000 Nil 8,50,000

9,00,000 1,00,000 8,00,000 1,00,000 10,50,000 Nil 10,50,000

Value Added Tax Goods D Goods E Goods F Goods G Goods H Goods I 55,556 Not allowed 6,931 56,000 30,769 1,00,000 2,97,546

37

After adjusting output VAT of `33,547 and CST of `81,316, there will be unutilised VAT credit of `1,82,683 and it can be set off from other output tax or it can be carried forward or refund can be claimed but procedure differs from State to State. At the year end it should be shown on the assets side of the balance sheet under the head CURRENT ASSETS, LOAN AND ADVANCES. Computation of income tax liability Particulars Purchases Goods A Goods B Goods C Goods D Goods E Goods F Goods H Goods I Net profit Amount ` 96,154 2,00,000 3,55,556 4,44,444 3,00,000 6,93,069 7,69,231 8,00,000 6,51,683 43,10,137 Particulars Sales Goods A Goods B Goods C Goods D Goods E Goods F Goods H Goods I Amount ` 1,44,231 2,22,222 4,65,686 4,88,889 3,46,535 7,42,574 8,50,000 10,50,000 43,10,137 6,51,683.00 6,51,683.00 Nil 6,51,683.00 6,51,680.00 62,336.00 1,246.72 623.36 64,206.08 64,210.00

Income under the head Business/Profession Gross Total Income Less: Deduction u/s 80C to 80U Total Income Rounded off u/s 288A Tax on `6,51,680 at slab rate Add: Education cess @ 2% Add: SHEC @ 1% Tax Liability Rounded off u/s 288B

Income shall be computed exclusive of VAT because any VAT collected shall be paid to the Government and it will not be considered to be income. Similarly VAT paid by the dealer is collected from the customer hence it will not be considered to be expense. If any balance is left in VAT credit, it will be shown as asset in the Balance Sheet on the last day of the relevant financial year. Further, the stock transfer of goods G is having a neutral effect and thus ignored for calculation of business/profession income. Illustration 8: Mr. X is a Dealer Registered in Delhi Value Added Tax Act, 2004 and also under Central Sales Tax Act, 1956 and he has submitted the informations as given below: (i) Purchased Goods A from Delhi `1,00,000 and paid VAT @ 4% and sold the goods in Delhi at a profit of 50% on purchase price and charged VAT @ 4%.

Value Added Tax

38

(ii) Purchased goods B from U.P. for `2,00,000 and paid central sales tax @ 2% and sold goods in Delhi at a profit of 50% on purchase price and charged VAT @ 12.5%. (iii) Purchased goods C from Delhi for `4,00,000 and paid VAT @ 12.5% and sold the goods at a profit of 50% on purchase price to a registered dealer in Orissa and charged central sales tax @ 2% (iv) Purchased goods D for `5,00,000 in Delhi and paid VAT @ 12.5% and sold the goods at a profit of 50% on purchase price to an unregistered dealer in Punjab and charged central sales tax @ 12.5%. (v) Purchased goods E from Madhya Pradesh for `3,00,000 and paid central sales tax @ 1% and sold goods at a profit of 50% on purchase price in Maharashtra and charged central sales tax @ 1%. (vi) Purchased goods F from Delhi `7,00,000 and paid VAT @ 1% and the goods were sold at a profit of 50% on purchase price to an unregistered dealer in Maharashtra and charged central sales tax @ 1%. (vii) Purchased goods G for `6,00,000 in Delhi and paid VAT @ 12.5% and goods were stock transferred to some other state. (viii) Purchased goods H for `8,00,000 in Delhi and paid VAT @ 4% and goods were exported at a profit of 50% on purchase price and VAT was charged (because as per section 5 Central Sales Tax Act, 1956, VAT can not be charged in case of export sale.) (ix) Purchased goods I for `9,00,000 in Delhi and paid VAT @ 12.5% and sold the goods at a profit of 50% on purchase price to a manufacturer in SEZ and no VAT was charged. Show the tax treatment for VAT and also compute his income tax liability for the assessment year 2012-13. Solution: (i) Purchased Goods A from Delhi Add: VAT @ 4% Purchase Price Input tax credit Cost Add: Profit {1,04,000-4,000(as VAT credit is available)} x 50% Sale Price before VAT Goods sold in Delhi Add: VAT @ 4% Sale Price (ii) Purchased goods B from U.P. Add: Central sales tax @ 2% Purchase Price Add: Profit (2,04,000 x 50%) Sale Price before VAT Input tax credit Goods sold in Delhi Add: VAT @ 12.5% Sale Price ` 1,00,000 4,000 1,04,000 4,000 1,00,000 50,000 1,50,000 1,50,000 6,000 1,56,000 2,00,000 4,000 2,04,000 1,02,000 3,06,000 Nil 3,06,000 38,250 3,44,250

Value Added Tax (iii) Purchased goods C from Delhi Add: VAT @ 12.5% Purchase Price Cost Add: Profit {4,50,000-50,000(as VAT credit is available)} x 50% Sale Price before CST Input tax credit Goods sold in Orissa Add: Central sales tax @ 2% Sale Price (iv) Purchased goods D from Delhi Add: VAT @ 12.5% Purchase Price Cost Add: Profit {5,62,500-62,500(as VAT credit is available)} x 50% Sale Price before CST Input tax credit Goods sold in Punjab to unregistered dealer Add: Central sales tax @ 12.5% Sale Price (v) Purchased goods E from Madhya Pradesh Add: Central sales tax @ 1% Purchase Price Add: Profit (3,03,000 x 50%) Sale Price Input tax credit Goods sold in Maharashtra Add: Central sales tax @ 1% Sale Price (vi) Purchased goods F from Delhi Add: VAT @ 1% Purchase Price Cost Add: Profit {7,07,000-7,000(as VAT credit is available)} x 50% Sale Price Input tax credit Goods sold in Maharashtra to unregistered dealer Add: Central sales tax @ 1% Sale Price (vii) Purchased goods G from Delhi Add: VAT @ 12.5% Purchase Price Goods Stock transferred

39 4,00,000 50,000 4,50,000 4,00,000 2,00,000 6,00,000 50,000 6,00,000 12,000 6,12,000 5,00,000 62,500 5,62,500 5,00,000 2,50,000 7,50,000 62,500 7,50,000 93,750 8,43,750 3,00,000 3,000 3,03,000 1,51,500 4,54,500 Nil 4,54,500 4,545 4,59,045 7,00,000 7,000 7,07,000 7,00,000 3,50,000 10,50,000 7,000 10,50,000 10,500 10,60,500 6,00,000 75,000 6,75,000 6,00,000

Value Added Tax VAT credit allowed in stock transfer (6,00,000 x 10.5%) (in case of stock transfer, VAT credit shall be allowed after retaining 2%) (viii) Purchased goods H from Delhi Add: VAT @ 4% Purchase Price Cost Add: Profit {8,32,000-32,000(as VAT credit is available)} x 50% Sale Price Input tax credit Goods exported (ix) Purchased goods I from Delhi Add: VAT @ 12.5% Purchase Price Cost Add: Profit {10,12,500-1,12,500(as VAT credit is available)} x 50% Sale Price Input tax credit Goods sold to manufacturer in SEZ VAT A/C Particulars OUTPUT TAX Goods A Goods B Goods C Goods D Goods E Goods F Goods G (Stock transfer) Goods H (Export) Goods I (Sale to SEZ) LESS: INPUT TAX CREDIT Goods A Goods B Goods C Goods D Goods E Goods F Goods G Goods H Goods I 4,000 Not allowed 50,000 62,500 Not allowed 7,000 63,000 32,000 1,12,500 3,31,000

40 63,000

8,00,000 32,000 8,32,000 8,00,000 4,00,000 12,00,000 32,000 12,00,000 9,00,000 1,12,500 10,12,500 9,00,000 4,50,000 13,50,000 1,12,500 13,50,000 ` VAT 6,000 38,250 --------Not applicable Nil Nil 44,250 ` CST ----12,000 93,750 4,545 10,500 ------1,20,795

After adjusting output VAT of `44,250 and CST of `1,20,795, there will be unutilised VAT credit of `1,65,955 and it can be set off from other output tax or it can be carried forward or refund can be claimed but procedure differs from State to State. At the year end it should be shown on the assets side of the balance sheet under the head CURRENT ASSETS, LOAN AND ADVANCES.

Value Added Tax Computation of income tax liability Particulars Purchases Goods A Goods B Goods C Goods D Goods E Goods F Goods H Goods I Net profit

41

Amount ` 1,00,000 2,04,000 4,00,000 5,00,000 3,03,000 7,00,000 8,00,000 9,00,000 19,53,500 58,60,500

Particulars Sales Goods A Goods B Goods C Goods D Goods E Goods F Goods H Goods I

Amount ` 1,50,000 3,06,000 6,00,000 7,50,000 4,54,500 10,50,000 12,00,000 13,50,000 58,60,500 19,53,500.00 19,53,500.00 Nil 19,53,500.00 4,38,050.00 8,761.00 4,380.50 4,51,191.50 4,51,190.00

Income under the head Business/Profession Gross Total Income Less: Deduction u/s 80C to 80U Total Income Tax on `19,53,500 at slab rate Add: Education cess @ 2% Add: SHEC @ 1% Tax Liability Rounded off u/s 288B

Income shall be computed exclusive of VAT & CST because any VAT & CST collected shall be paid to the Government and it will not be considered to be income. Similarly VAT paid by the dealer is collected from the customer hence it will not be considered to be expense. Further, the stock transfer of goods G is having a neutral effect and thus ignored for calculation of business/profession income. Illustration 9: Mr. X is a Dealer Registered in Delhi Value Added Tax Act, 2004 and also under Central Sales Tax Act, 1956 and he has submitted the informations as given below: (i) Purchased Goods A from Delhi `1,00,000 and paid VAT @ 4% and sold the goods at a profit of 50% on sale price in Delhi and charged local sales tax @ 4%. (ii) Purchased goods B from U.P. for `2,00,000 and paid central sales tax @ 2% and sold goods at a profit of 50% on sale price in Delhi and charged VAT @ 12.5%. (iii) Purchased goods C from Delhi for `4,00,000 and paid VAT @ 12.5% and sold the goods at a profit of 50% on sale price to a registered dealer in Orissa and charged central sales tax @ 2% (iv) Purchased goods D for `5,00,000 in Delhi and paid VAT @ 12.5% and sold the goods at a profit of 50% on sale price to an unregistered dealer in Punjab and charged central sales tax @ 12.5%. (v) Purchased goods E from Madhya Pradesh for `3,00,000 and paid central sales tax @ 1% and sold goods at a profit of 50% on sale price in Maharashtra and charged central sales tax @ 1%. (vi) Purchased goods F from Delhi `7,00,000 and paid VAT @ 1% and the goods were sold at a profit of 50% on sale price to an unregistered dealer in Maharashtra and charged central sales tax @ 1%.

Value Added Tax

42

(vii) Purchased goods G for `6,00,000 in Delhi and paid VAT @ 12.5% and goods were stock transferred to some other state. (viii) Purchased goods H for `8,00,000 in Delhi and paid VAT @ 4% and goods were exported at a profit of 50% on sale price and VAT was charged (because as per section 5 Central Sales Tax Act, 1956, VAT can not be charged in case of export sale.) (ix) Purchased goods I for `9,00,000 in Delhi and paid VAT @ 12.5% and sold the goods at a profit of 50% on sale price to a manufacturer in SEZ and no VAT was charged. Show the tax treatment for VAT and also compute his income tax liability for the assessment year 2012-13. Solution: (i) Purchased Goods A from Delhi Add: VAT @ 4% Purchase Price Cost Input tax credit Since, Profit is 50% of sale price, Cost of `1,00,000 is 50% of Sale Price Hence, Sale Price shall be 1,00,000 x 100% / 50% = `2,00,000 Goods sold in Delhi Add: VAT @ 4% Selling Price (ii) Purchased goods B from U.P. Add: Central sales tax @ 2% Purchase Price / Cost Since, Profit is 50% of sale price, Cost of `2,04,000 is 50% of sale price Hence, Sale Price shall be 2,04,000 x 100% / 50% = `4,08,000 Input tax credit Goods sold in Delhi Add: VAT @ 12.5% Selling Price (iii) Purchased goods C from Delhi Add: VAT @ 12.5% Purchase Price Input tax credit Since, Profit is 50% of sale price, `4,00,000 is 50% of sale price Hence, sale price shall be 4,00,000 x 100% / 50% = `8,00,000 Goods sold in Orissa Add: Central sales tax @ 2% Selling Price (iv) Purchased goods D from Delhi ` 1,00,000 4,000 1,04,000 1,00,000 4,000

2,00,000 8,000 2,08,000 2,00,000 4,000 2,04,000

Nil 4,08,000 51,000 4,59,000 4,00,000 50,000 4,50,000 50,000 8,00,000 16,000 8,16,000 5,00,000

Value Added Tax Add: VAT @ 12.5% Purchase Price Input tax credit Since, Profit is 50% of sale price, `5,00,000 is 50% of sale price Hence, Sale Price shall be 5,00,000 x 100% / 50% = 10,00,000 Goods sold in Punjab to unregistered dealer Add: Central sales tax @ 12.5% Selling Price (v) Purchased goods E from Madhya Pradesh Add: Central sales tax @ 1% Purchase Price / Cost Input tax credit Since, Profit is 50% of sale price, `3,03,000 is 50% of sale price Hence, sale price shall be 3,03,000 x 100% / 50% = `6,06,000 Goods sold in Maharashtra Add: Central sales tax @ 1% Selling Price (vi) Purchased goods F from Delhi Add: VAT @ 1% Purchae Price Input tax credit Since, Profit is 50% of sale price, `7,00,000 is 50% of sale price Hence, Sale Price shall be 7,00,000 x 100% / 50% = `14,00,000 Goods sold in Maharashtra to unregistered dealer Add: Central sales tax @ 1% Selling Price (vii) Purchased goods G from Delhi Add: VAT @ 12.5% Purchase Price Goods Stock transferred VAT credit allowed in stock transfer (6,00,000 x 10.5%) (in case of stock transfer, VAT credit shall be allowed after retaining 2%) (viii) Purchased goods H from Delhi Add: VAT @ 4% Purchase Price Input tax credit Since, Profit is 50% of Sale Price, `8,00,000 is 50% of sale price Hence, Sale Price shall be 8,00,000 x 100% / 50% = `16,00,000 Goods exported (ix) Purchased goods I from Delhi Add: VAT @ 12.5% Purchase Price

43 62,500 5,62,500 62,500 10,00,000 1,25,000 11,25,000 3,00,000 3,000 3,03,000 Nil 6,06,000 6,060 6,12,060 7,00,000 7,000 7,07,000 7,000 14,00,000 14,000 14,14,000 6,00,000 75,000 6,75,000 6,00,000 63,000

8,00,000 32,000 8,32,000 32,000 16,00,000 9,00,000 1,12,500 10,12,500

Value Added Tax Input tax credit Since, Profit is 50% of sale price, `9,00,000 is 50% of sale price Hence, Sale Price shall be 9,00,000 x 100% / 50% = `18,00,000 Goods sold to manufacturer in SEZ VAT A/C Particulars OUTPUT TAX Goods A Goods B Goods C Goods D Goods E Goods F Goods G (Stock transfer) Goods H (Export) Goods I (Sale to SEZ) LESS: INPUT TAX CREDIT Goods A Goods B Goods C Goods D Goods E Goods F Goods G Goods H Goods I 4,000 Not allowed 50,000 62,500 Not allowed 7,000 63,000 32,000 1,12,500 3,31,000

44 1,12,500 18,00,000 ` VAT 8,000 51,000 --------Not applicable Nil Nil 59,000 ` CST ----16,000 1,25,000 6,060 14,000 ------1,61,060

After adjusting output VAT of `59,000 and CST of `1,61,060, there will be unutilised VAT credit of `1,10,940 and it can be set off from other output tax or it can be carried forward or refund can be claimed but procedure differs from State to State. At the year end it should be shown on the assets side of the balance sheet under the head CURRENT ASSETS, LOAN AND ADVANCES. Computation of income tax liability Particulars Purchases Goods A Goods B Goods C Goods D Goods E Goods F Goods H Goods I Net profit Amount ` 1,00,000 2,04,000 4,00,000 5,00,000 3,03,000 7,00,000 8,00,000 9,00,000 39,07,000 78,14,000 Particulars Sales Goods A Goods B Goods C Goods D Goods E Goods F Goods H Goods I Amount ` 2,00,000 4,08,000 8,00,000 10,00,000 6,06,000 14,00,000 16,00,000 18,00,000 78,14,000 39,07,000.00 39,07,000.00

Income under the head Business/Profession Gross Total Income

Value Added Tax Less: Deduction u/s 80C to 80U Total Income Tax on `39,07,000 at slab rate Add: Education cess @ 2% Add: SHEC @ 1% Tax Liability Rounded off u/s 288B

45 Nil 39,07,000.00 10,24,100.00 20,482.00 10,241.00 10,54,823.00 10,54,820.00

Income shall be computed exclusive of VAT because any VAT collected shall be paid to the Government and it will not be considered to be income. Similarly VAT paid by the dealer is collected from the customer hence it will not be considered to be expense. Further, the stock transfer of goods G is having a neutral effect and thus ignored for calculation of business/profession income. Question 6 (V. Imp.): Explain provisions of Stock Transfer. Answer: Inter-State branch transfers do not involve sale and, therefore they are not subjected to sales-tax. The same position continues under VAT. However, the tax paid on: (i) inputs used in the manufacture of finished goods which are stock transferred: or (ii) purchases of goods which are stock transferred will be available as input tax credit after retention of 2% of such tax by the State Government. e.g. ABC Ltd. has purchased goods for `10 lakhs and has paid VAT @ 12.5% in Delhi and subsequently the goods were stock transferred to a branch in some other State, in this case VAT credit allowed shall be `1,05,000 (`10,00,000 x (12.5% - 2%)). Illustration 10: A dealer purchased 16,500 kgs of inputs on which VAT paid @ 4% was `6,000. He manufactured 15,000 Kgs of finished products from the inputs. 1,500 Kgs was the process loss. The final product was sold at a price of `10 per Kg, as follows : - Goods sold within State 6,000 Kgs. - Finished products sold in inter-state sale against C form 3,750 Kgs. - Goods sent on stock transfer to consignment agents outside the state 3,000 Kgs. Goods sold to Government department outside the state 2,250 Kgs.

There was no opening or closing stock of inputs, WIP or finished product. The State VAT rate on the finished product of dealer is 12.5%. Discuss tax treatment. Solution: CST against C form is 2%. Sale to Government will be treated as sale to unregistered dealer and tax payable is 12.5%. Thus, the tax payable would be as follows-

Value Added Tax Output tax shall be as given below: Description Sale within State @ 12.5% Goods sent on stock transfer Goods sold against C form, tax rate 2% Goods sold to Government, tax rate 12.5% Total Input tax credit shall be = 6,000 600* = 5,400 (i) State VAT Less: VAT credit Net State VAT payable CST payable Total tax payable Quantity sold Kg 6,000 3,000 3,750 2,250 15,000 Value of goods sold ` 60,000 30,000 37,500 22,500 1,50,000 CST payable ` 750 2,813 3,563 State VAT payable ` 7,500

46

7,500 ` 7,500 5,400 2,100 3,563 5,663

*In the given case total finished product is 15,000 kgs and stock transferred 3,000 kgs and proportionate VAT credit is `1,200 (6,000 / 15,000 x 3,000) @ 4% but in case of stock transfer VAT credit shall be allowed after deducting 2% i.e. VAT credit shall be allowed for `600 instead of `1,200 i.e. VAT credit for `600 shall not be allowed and total VAT credit allowed shall be (`6,000 600 = `5,400) (if raw material is consumed in process, VAT credit is allowed even for such raw material i.e. VAT credit even for process loss is allowed) In aforesaid example, if 3,000 Kgs were exported (and not stock transferred), what would be the tax liability and credit available. If finished product is exported. There is no tax liability. Further, the credit of tax paid on raw material is available. ` State VAT payable 7,500 Less: VAT credit 6,000 Net State VAT payable 1,500 CST payable 3,563 Total tax payable 5,063 Question 7 (V. Imp.): Write a note on registration under State VAT Act and Central Sales Tax Act. Answer: Registration under State Value Added Tax Act Compulsory Registration A dealer must apply for registration in the following cases: (i) If the turnover has exceeded `10 lakhs at any time during the year. (ii) He is registered under Central Sales Tax Act, 1956. (iii) He is purchasing goods from outside the state of Delhi for sale in Delhi.

Value Added Tax

47

Example Mr. X is an unregistered dealer in Delhi,. His sales turnover is `5,00,000 but he is purchasing some of the goods from outside Delhi, in this case, he should apply for compulsory registration. If in the above case, he is not purchasing goods from other States but he is selling some of the goods to other States, in that case also registration is required. If he is not purchasing goods from other States and also not selling goods to other states, registration is not required but if turnover is exceeding `10,00,000, registration is required. Voluntary Registration Any dealer may apply for voluntary registration under State Value Added Tax Act at any time. Only dealer registered under State Value Added Tax Act can charge sales tax and can issue tax invoice and VAT Credit is only allowed on the basis of tax invoice. Registration under Central Sales Tax Act, 1956 Compulsory Registration If any dealer has effected any Sale from one state to the other, registration is required under Central Sales Tax Act. Voluntary Registration If a dealer is registered under State Value Added Tax Act, he may apply for registration under Central Sales Tax Act at any time. Registration under State Value Added Tax Act and Central Sales Tax Act shall be different. TIN (Tax Payers Identification Number) is a code to identify a tax payer. It is the registration number of the dealer. The taxpayers identification number will consist of 11 digit numerals throughout the country. First two characters will represent the State code as used by the Union Ministry of Home Affairs. The set-off of the next nine characters will be, however, different in different States. TIN will help cross-check information on tax payer compliance, for example, the selective cross-checking of sales and purchases among VAT taxpayers. Question 8: Write a note on amendment in registration. Answer: Amendment in Registration certificate Registration certificate can be amended in the following situations: 1. If there is change in the name of business. 2. If there is change in the place of business. 3. If there is change in the nature of business i.e. Trading is converted into Manufacturing or Manufacturing is converted into Trading. 4. If one or more new goods have been included or one or more goods have been deleted 5. If there is change in the Constitution e.g. Partnership Firm is converted into Proprietary or Proprietary is converted into Partnership Firm. 6. If there is any other similar change.

Value Added Tax Amendment can be made at the request of the dealer or the department can make amendment on its own. Question 9: Write a note on cancellation of registration certificate. Answer: Cancellation of registration VAT registration can be cancelled on: (i) Discontinuance of business; or (ii) Disposal of business; or (iii) Death of the dealer (iv) Annual turnover falling below the specified limit e.g. In case of Delhi VAT if turnover becomes less than 10 lakhs (v) Violation of the provisions of State VAT Act leading to cancellation of certificate

48

Question 10: Explain accounting treatment of VAT as suggested by ICAI. (not covered in syllabus rather it is only for self reading) Answer: VAT credit in case of inputs / supplies 1. A dealer purchases the following goods in a State during the month of March 2012: Particulars 4% VAT Goods 12.5% VAT Goods VAT Exempt Goods Total Net Amount (`) 10,00,000 8,00,000 2,00,000 20,00,000 Input Tax Paid (`) 40,000 1,00,000 1,40,000 Total Amount (`) 10,40,000 9,00,000 2,00,000 21,40,000

2. The input tax paid on purchase of goods is eligible for VAT credit. 3. Sales made by the dealer during the month are as below: Particulars 4% VAT Goods 12.5% VAT Goods VAT Exempt Goods Total Net Sales Consideration (`) 11,00,000 9,00,000 2,50,000 22,50,000 Output Tax Collected (`) 44,000 1,12,500 1,56,500 Gross Amount (`) 11,44,000 10,12,500 2,50,000 24,06,500

Suggested Accounting Treatment 1. The dealer passes the following entry to record the goods purchased and input tax paid thereon: 4% VAT Goods Purchase A/c Dr. `10,00,000 12.5% VAT Goods Purchase A/c Dr. ` 8,00,000 VAT Exempt Goods Purchase A/c Dr. ` 2,00,000 VAT Credit Receivable (Inputs) A/c Dr. ` 1,40,000 To Bank A/c ` 21,40,000

Value Added Tax 2. The dealer passes the following entry to record the goods sold and VAT collected thereon: Bank A/c Dr. `24,06,500 To 4% VAT Goods Sales A/c `11,00,000 To 12.5% VAT Goods Sales A/c ` 9,00,000 To VAT Exempt Goods Sales A/c ` 2,50,000 To VAT Payable A/c ` 1,56,500

49

3. The dealer passes the following entry to record the liability for VAT payable met by using the balance in the VAT Credit Receivable (Inputs) Account and balance by bank: VAT Payable A/c Dr. ` 1,56,500 To VAT Credit Receivable (Inputs) A/c ` 1,40,000 To Bank ` 16,500 VAT credit in case of capital goods Example On July 1, 2011 a dealer purchases one machine in a State for the total cost of `93,60,000 which includes input tax of `3,60,000. As per the State VAT laws, input tax paid on purchases of machinery is adjustable as VAT credit over 3 annual installments. Till the end of the year, the dealer has not utilized the VAT credit available on the machine. Suggested Accounting Treatment 1. The dealer passes the following entry to record the machinery purchased and input tax paid thereon: Machinery A/c Dr. ` 90,00,000 VAT Credit Receivable (Capital Goods) A/c Dr. ` 1,20,000 VAT Credit Deferred (Capital Goods) A/c Dr. ` 2,40,000 To Bank A/c ` 93,60,000 In the subsequent year the dealer will transfer the amount from VAT Credit Deferred A/c to VAT Credit Receivable A/c 2. The dealer charges depreciation on the cost of machinery excluding VAT credit (i.e. `93,60,000 `3,60,000 = `90,00,000). 3. Balances in VAT Credit Deferred (Capital Goods) A/c and VAT credit Receivable (Capital Goods) A/c are disclosed in the balances sheet as on March 31, 2012 as below: Extracts from the Balance Sheet Assets Current Assets Loans and Advances VAT Credit Deferred (Capital Goods) A/c VAT Credit Receivable (Capital Goods) A/c Amounts (`) 2,40,000 1,20,000

Question 11 (V. Imp.): What are the Variants (different types) of VAT. Answer: VAT has 3 variants: (a) Gross Product Variant (b) Income Variant (c) Consumption Variant

Value Added Tax

50

(a) Gross Product Variant Gross Product Variant allows VAT credit on the raw materials, but tax credit is not allowed on capital goods like plant and machinery etc. (b) Income Variant of VAT The Income Variant of VAT allows VAT credit on raw materials etc and also on capital goods but VAT credit on capital goods is allowed in instalments depending on the life of capital goods. (c) Consumption Variant It allows VAT credit on raw materials etc. and also on capital goods in the very first year. Among the three variants of VAT, the consumption variant is widely used. Several countries of Europe and other continents have adopted this variant, because there is no multiple taxation and also there is no cascading effect and it can be shown as given below: Mr. X manufactures product A out of raw material X. The cost of raw material X is `2 lakh. The labour and other manufacturing costs are `8 lakh. The manufacturing process requires a machinery of `20 lakh (subject to VAT @ 12.5%). The useful life of the plant is 4 years with no salvage and rate of depreciation is 25% on straight line method. The expected output of product A is 1,000 units p.a. Mr. X fixes a profit margin of `100 per unit. Compute the selling price of product A and its cost to consumer if (a) No credit is allowed on the capital goods (Gross product variant); (b) credit is allowed on the capital goods (Consumption variant). The VAT rate on final product is 12.5%. There is no VAT on raw material. Solution: (a) No credit on capital goods - Gross Product Variant Raw material cost Labour and other manufacturing costs Depreciation on machinery: (20 lakh + 12.5% VAT) 4 (as VAT credit is not available) Total cost Cost per unit Profit per unit Selling price per unit Add: VAT @ 12.5% Cost to consumer (b) Credit available on capital goods- Consumption Variant Raw material cost Labour and other manufacturing costs Depreciation on machinery: (20 lakh 4) (VAT credit available) Total cost Cost per unit Profit per unit Selling price per unit Add: VAT @ 12.5% Cost to consumer ` 2,00,000.00 8,00,000.00 5,62,500.00 15,62,500.00 1,562.50 100.00 1,662.50 207.81 1,870.31 ` 2,00,000.00 8,00,000.00 5,00,000.00 15,00,000.00 1,500.00 100.00 1,600.00 200.00 1,800.00

Value Added Tax

51

The availability of VAT credit on capital goods, reduces the cost and resultant selling price of the goods and, therefore, eliminates cascading effect. Illustration 11: A manufacturer has purchased raw material for `2,08,000 (inclusive of 4% VAT) and plant and machinery of `4,50,000 (inclusive of 12.5% VAT). The manufacturing and other expenses (excluding depreciation) are `6,00,000. He sells the resultant products at 50% above cost (VAT on sales is 4%). The plant and machinery is to be depreciated at 50% straight line. (a) Compute the amount of VAT payable, as per the Gross Product Variant of VAT. (b) Compute VAT payable as per Income Variant of VAT. (c) Compute VAT payable as per Consumption Variant of VAT. Solution (a): Gross Product Variant Raw material net of VAT (2,08,000 x 100 104) Depreciation of plant and machinery (50% of 4,50,000 VAT credit not allowed) Manufacturing and other expenses Total cost Add: Profit 50% Sale price VAT on sales (4% of 15,37,500) Less: Input tax credit on raw material (2,08,000 x 4 104) VAT payable ` 2,00,000 2,25,000 6,00,000 10,25,000 5,12,500 15,37,500 61,500 8,000 53,500

Solution (b): Income Variant ` Raw material net of VAT (2,08,000 x 100 104) 2,00,000 Depreciation on plant and machinery (50% of `4,00,000 VAT credit allowed) 2,00,000 Manufacturing and other expenses 6,00,000 Total cost 10,00,000 Add: Profit @ 50% 5,00,000 Sale price 15,00,000 VAT on sales (4% of 15,00,000) 60,000 Less: Input tax credit as follows: Input tax credit on raw material (2,08,000 x 4 104) 8,000 Input tax credit on plant (50% of 50,000) 25,000 33,000 VAT payable 27,000 Note: The VAT paid on plant and machinery has been allowed as credit only to the extent of depreciation i.e. 50%. The balance VAT credit of `25,000 can be set-off in subsequent year. Solution (c): Consumption Variant Raw material net of VAT (2,08,000 x 100 104) Depreciation on plant and machinery (50% of 4,00,000 VAT credit is allowed) Manufacturing and other expenses Total cost ` 2,00,000 2,00,000 6,00,000 10,00,000

Value Added Tax Add: Profit 50% Sale price VAT on sales (4% of 15,00,000) Less: Input tax credit as follows: Input tax credit on raw material and components (2,08,000 x 4 104) Input tax credit on plant (100% of 50,000) VAT payable 8,000 50,000

52 5,00,000 15,00,000 60,000 58,000 2,000

Question 12: What are the rates under VAT. Answer: Exempted Category Under exempted category, there are about 50 commodities comprising of natural and unprocessed products in unorganized sector, items which are legally barred from taxation and items which have social implications. Included in this exempted category is a set of maximum of 10 commodities flexibly chosen by individual States from a list of goods (finalized by the Empowered Committee) which are of local social importance for the individual States without having any inter-State implication. Example: (i) Books, periodicals and journals including maps, charts and globes.;(ii) Curd, Lussi, butter milk and separated milk.; (iii) Earthen pot.; (iv) Electricity energy; (v) Fresh plants, saplings and fresh flowers.; (vi) Fresh vegetables and fruits.; (vii) All bangles except those made of precious metals.; (viii) Kumkum, bindi alta and sindur.; (ix) Blood including blood components 1% Category The special rate of 1% is meant for precious stones, bullion, gold and silver ornaments etc. 4% VAT Category The goods declared as per section 14 of Central Sales Tax Act shall be taxable @ 4% List of some of the goods is: (i) Coal including coke in all its forms, but excluding charcoal, (ii) Cotton yarn, but not including cotton yarn waste; (iii) Oil seeds; (iv) Pulses; (v) Sugar; (vi) Iron and steel; (vii) Liquefied petroleum gas for domestic use 5% VAT Category Under 5% VAT rate category, there are largest number of goods, common for all the States, comprising of items of basic necessities such as medicines and drugs, all agricultural and industrial inputs, capital goods and declared goods. The schedule of commodities are attached to the VAT Acts of the States. List of some of the goods is: 1. Bicycles having MRP above `3,500, Tricycles, rickshaws and parts including tyres and tubes thereof ; 2. Drugs & medicines including vaccines, syringes and dressings, medicated ointments produced under a drugs licence; 3. Coffee beans and seeds, green tea leaf and chicory.; 4. Cotton and cotton waste; 5. Edible oils and oil cake. 20% Category Petrol, diesel, Aviation Turbine Fuel, other motor spirit, liquor and lottery tickets etc. will be subjected to 20% floor rate of tax. 12.5% Category The remaining commodities, common for all the States, fall under the general VAT rate of 12.5%.

Value Added Tax

53

Question 13: Explain non- creditable/non-vatable goods. Answer: Description of Non- Creditable/Non-Vatable goods No VAT credit is allowed in case of some of the commodities because these goods are either not meant for resale or the goods are allowed to be sold only once and such goods are called non-creditable or non-vatable goods and the list of such goods differs from State to State and some of such items are as given below: (i) All automobiles including commercial vehicles, and two and three wheelers, and spare parts for repairs and maintenance and tyres and tube thereof; (ii) Fuels in the form of petrol, diesel, kerosene, LPG, CNG, PNG and coal; (iii) Food for human consumption; (iv) Beverages for human consumption; (v) Air conditioners, air conditioning plants or units other than those used for manufacturing purposes; air coolers, fans and air circulators; (vi) Goods for personal consumption or for gifts Question 14 (V. Imp.): Explain composition scheme for small traders. Answer: Composition scheme for small traders (Similar provision in Section 16 of Delhi Value Added Tax Act, 2004) If any dealer is having turnover UPTO ` 50 LAKHS, he may apply for Composition Scheme. Composition Scheme is not allowed in the following cases: (i) If any dealer is procuring goods from outside the State or is selling or supplying goods to any place outside the State at any time during the year. (ii) If he is registered under Central Sales Tax Act. Salient features of the Scheme are asunder: (i) A dealer covered under Composition Scheme is not allowed to take VAT Credit on his purchases but he must retain all the tax invoices for the goods purchased by him. (ii) A dealer covered under composition scheme is not allowed to issue tax invoice and also not allowed to charge any tax from the buyer rather he himself has to pay tax on his sales turnover (@ 1% in Delhi). (iii) The benefit of composition scheme is that dealer is exempt from maintaining lengthy records required under VAT. (iv) If any dealer is purchasing goods from a dealer covered under composition scheme, no VAT credit is allowed to such a purchasing dealer. (v) If any dealer is covered under composition scheme, he must purchase goods only from registered dealer. (vi) If turnover has exceeded `50 lakhs, he has to shift immediately to the normal system. (vii) If any dealer is covered under composition scheme, he may reject the scheme and may opt for normal procedure but only from beginning of the year.

Value Added Tax

54

(viii) If any dealer is opting out of the composition scheme, he will be allowed VAT credit for the stock held by him on the date of opting out. A dealer has to apply in the prescribed form for opting the scheme and rejecting the scheme A dealer opting for composition scheme has to apply in Form No. DVAT 01 under DVAT Act, 2004 Illustration 12: Mr. A, a retailer, presents the following information for the year (1) Purchases of goods: `30 lakhs (VAT @ 4%)

(2) Sales (at fixed selling price inclusive of all taxes): `49 lakhs (VAT on sales @ 4%). Expenses of keeping detailed statutory records required under the VAT-law will be `1,00,000 p.a. Other expenses are `6,00,000 p.a. (b) Presume he has opted for composition scheme and goods were sold for `49,00,000 and composition tax @ 1% for the turnover was paid by the dealer. Expenditure on maintaining records was reduced to `40,000 Show the treatment for VAT in both of the options and also pass accounting entries for the composition scheme. Solution: The cost to the ultimate consumer under two schemes is as under Sales (inclusive of all taxes) Less: Tax (VAT = 49 lakhs x 4 104) Cost of goods sold Add: Cost of maintaining records Add: Normal Expenses Profit of the dealer Solution (b): Sales Less: Cost of goods sold (No credit under composition scheme, hence, cost of goods sold will be higher) Cost of maintaining records Normal Expenses Composition tax (49,00,000 x 1%) Profit of the dealer

49,00,000 1,88,462 30,00,000 1,00,000 6,00,000 10,11,538 49,00,000 31,20,000 40,000 6,00,000 49,000 10,91,000

The profit of the dealer is higher if the dealer opts for composition scheme. Hence, composition scheme should be opted. The accounting entries under composition scheme are as under (Accounting entries are not part of the syllabus rather it is given for better understanding) 1. Purchases made: Purchase A/c To Cash /Bank A/c 2. Sales Made: Cash/Bank A/c Dr. Dr. ` 31,20,000 49,00,000 ` 31,20,000

Value Added Tax To Sales A/c 3. Composite Tax paid @ 1% Composition Tax A/c To Cash/Bank A/c 49,00,000 Dr. 49,000 49,000

55

Illustration 13: A Registered Dealer Mr. A of Punjab extracted raw produce X and raw produce Y from mines at `30,000 and `40,000 respectively and sold the same at 150% margin to Manufacturer B of Punjab (VAT rate is 4% on produce X and 12.5% on produce Y). Manufacturer B is a dealer operating under composition scheme who is liable to composition tax @ 0.4% of turnover. Manufacturer B used X and Y as raw material, added 100% of cost of raw material towards manufacturing expenses and profits and sold the product to wholesaler C of Punjab. Wholesaler C sold the same to Retailer D of Punjab at 25% above cost (VAT rate is 4%). The retailer D sold the same to a consumer at 20% above cost (VAT rate is 4%). Show the amount of VAT payable by each person. Solution: Manufacturer A Cost of raw produce X Add: Profit margin @ 150% Total Add: VAT @ 4% Total Cost of raw produce Y Add: Profit margin @ 150% Total Add: VAT @ 12.5% Total Selling Price VAT payable by Dealer Mr. A (3,000 + 12,500) Manufacturer B Cost of manufacturer B (78,000 + 1,12,500) Add: Expenses + Profit - 100% Total Selling Price Composition tax @ 0.4% (to be paid by the seller) Wholesaler C Cost of wholesaler C (No VAT credit is allowed because of purchase from a dealer opting under composition Scheme.) Add: Profit Margin @ 25% Total Add: VAT @ 4% Total Selling Price Retailer D Cost of Retailer D ` 30,000 45,000 75,000 3,000 78,000 40,000 60,000 1,00,000 12,500 1,12,500 15,500 1,90,500 1,90,500 3,81,000 1,524 3,81,000 95,250 4,76,250 19,050 4,95,300 4,76,250

Value Added Tax (VAT credit is allowed for `19,050) Add: Profit margin @ 20% Total Add: VAT @ 4% Total Selling Price VAT Payable (22,860 19,050) Question 15 (V. Imp): What are methods for computation of Value Added Tax. Answer: There are three methods to calculate the Value Added Tax. 1. Invoice Method 2. Addition Method 3. Subtraction Method

56 95,250 5,71,500 22,860 5,94,360 3,810

1. Invoice Method Under this method, tax is imposed at each stage of sales on the entire sale value and tax paid at the earlier stage is allowed as set-off. In other words, out of tax so calculated, tax paid at the earlier stage i.e., at the stage of purchases, is allowed as set-off from the tax payable, and at every stage the differential tax is being paid. The most important aspect of this method is that at each stage, tax credit is available for Tax paid on the purchases made. This method is also called the Tax Credit Method or Voucher Method and can be shown in the manner given below: Product X Sale Price `200 Gross VAT `20 Net VAT `20 Retailer C Sale Price `1150 Gross VAT `115 Net VAT `5 {`115 `110}

Manufacturer A Sale Price `1000 Gross VAT `100 Net VAT `60 {`100 (20+20)}

Manufacturer B Sale Price `1100 Gross VAT `110 Net VAT `10 {`110 `100}

Product Y Sale Price `200 Gross VAT `20 Net VAT `20 Assumed Tax rate 10%. Under this method, tax credit cannot be claimed unless the purchase invoice is produced. As a result, in chain, if at any stage the transaction is kept out of the books, still there is no loss of revenue. The department will be in a position to recover the full tax at the next stage. Thus, the possibility of tax evasion, if not entirely ruled out, will be reduced to a minimum. It is said that in this method the beneficiary is the trade and industry because in the above example, the total tax collection at all the stages is `365 whereas tax received by the State is only `115. 2. Addition Method This method aggregates all the factor payments like wages, rent, interest including profits to arrive at the total value addition on which the rate is applied to calculate the tax. This is in line with the income method of calculating National Income. E.g. In the above case value addition of manufacturer A is `600 (1,000 400), net tax shall be `60 i.e. 10% of `600. A drawback of this method is that is does not facilitate matching of invoices for detecting tax evasion.

Value Added Tax

57

3. Subtraction Method Under this method, the tax is charged only on the value added at each stage of the sale of goods. Under this method for imposing tax, value added is simply taken as the difference between sales and purchases. E.g. In the above case difference between the sale price and purchase price of Manufacturer A and Manufacturer B is `100 (1,100 1,000) and net tax payable by the Manufacturer B shall be `10 i.e. 10% of `100. If the difference is taken exclusive of sales tax, it will be called Direct Subtraction. If the difference is inclusive of sales tax, it will be called Intermediate Subtraction. E.g. In the above case difference in sale price and purchase price of Manufacturer A and Manufacturer B inclusive of sales tax is `110 (1,210-1,100) and amount of net tax shall be 110 x 10% / 110% = `10 The drawback of this method is that is does not facilitate matching of invoices for detecting tax evasion. Illustration 14: Registered Dealer A extracted raw produce X and raw produce Y from mines and sold the same to Manufacturer B for `80,000 and `1,20,000 respectively. Manufacturer B used X and Y as raw material and sold the resultant product to wholesaler C for `6,00,000. Wholesaler C sold the same to Retailer D for `9,00,000. The retailer D sold the same to a consumer for `10,00,000. Compute the VAT payable at each stage under (a) Invoice Method; and (b) Subtraction Method (intermediate subtraction); assuming that all sales are liable to VAT @ 4% and all the prices given above are exclusive of VAT. Solution : (a) Invoice Method Particulars 1. 2. 3. 4. Registered Dealer A [VAT = 4% of (80,000 + 1,20,000)] Manufacturer B [VAT = 4% of 6,00,000] Wholesaler C [VAT = 4% of 9,00,000] Retailer D [VAT = 4% of 10,00,000] Output VAT on sales 8,000 24,000 36,000 40,000 VAT credit 8,000 24,000 36,000 Net VAT payable 8,000 16,000 12,000 4,000 VAT on value added 8,000 16,000 12,000 4,000

(b) Subtraction Method - Intermediate Subtraction Particulars

Sale price inclusive of VAT 2,08,000 6,24,000 9,36,000 10,40,000

1. 2. 3. 4.

Registered Dealer A Manufacturer B Wholesaler C Retailer D

Purchase Value price inclusive inclusive of VAT of VAT - 2,08,000 2,08,000 4,16,000 6,24,000 3,12,000 9,36,000 1,04,000

Value Added Tax Question 16 (V. Imp.): Explain VAT Invoice / Tax Invoice Answer: A dealer may issue two type of invoices: 1. VAT invoice or Tax invoice 2. Retail invoice

58

1. Tax Invoice or VAT Invoice Tax credit is allowed only on the basis of Tax Invoice or VAT Invoice. No VAT credit is allowed on the basis of retail invoice. VAT invoice can be issued only by a registered dealer and also it can be issued only to a registered dealer and it must contain TIN of buyer as well as seller. Tax invoice can not be issued by a Composition Dealer. Tax invoice can not be issued in case of sale/purchase in the course of inter state trade or commerce. Tax invoice shall be issued in duplicate and original shall be given to the buyer and duplicate shall be retained by the seller. Tax invoice shall contain the following particulars: a. the words Tax Invoice at a prominent place; b. the name, address and registration number of the selling registered dealer; c. the name and address of the purchaser and his registration number. d. an individual pre-printed serialised number Invoice and the date on which the tax invoice is issued; A dealer may maintain separate numerical series, with distinct codes either, as a prefix or suffix, for each place of business in case the dealer has more than one place of business in State or for each product in case he deals in more than one product or both; e. description, quantity, volume and value of goods sold and services provided and the amount of tax charged thereon indicated separately; f. the signature of the selling dealer or his employee, manager or agent, duly authorized by him; and g. the name and address of the printer and first and last serial number of tax invoices printed and supplied by him to the dealer. Issue of Duplicate Tax Invoice If a purchaser claims to have lost the original tax invoice, the selling dealer may, subject to such conditions and restrictions as may be prescribed, provide a copy clearly marked as a duplicate. Where a purchasing dealer claims to have lost the original tax invoice, the selling dealer may, upon a request made by the purchasing dealer accompanied by an undertaking cum indemnity (in Delhi Form DVAT-36), provide a copy of such tax invoice clearly marked as a duplicate and shall furnish a copy of such undertaking cum indemnity along with his return for the tax period in which such duplicate tax invoice has been issued. (Similar provision in Rule 44 of DVAT Rules, 2005) 2. Retail invoice No VAT credit is allowed on the basis of retail invoice. Retail invoice can be issued if VAT credit is not allowed to the buyer.

Value Added Tax Contents of retail invoice shall be as given below: a. the words Retail Invoice or cash memorandum or bill at a prominent place; b. the name, address and registration number of the selling dealer, if registered;

59

c. in case the sale is in the course of inter-state trade or commerce, the name, registration number and address of the purchasing dealer and type of statutory form, if any, against which the sale has been made; d. an individual pre-printed serialized number invoice and the date on which the retail invoice is issued; e. description, quantity, volume and value of goods sold and services provided, inclusive of amount of tax charged thereon; and f. the signature of the selling dealer or his employee, manager or agent, duly authorized by him. Retail invoice shall be issued in duplicate, the original shall be issued to the purchaser and the copy shall be retained by the selling dealer. Question 17 (V. Imp.): Explain concept of excise duty (Central Value Added Tax). Answer: Central Excise Duty also called Central Value Added Tax is regulated through the following Acts/ Rules. - Central Excise Act, 1944 : It is the principal Act governing Central Excise Duty - Central Excise Rules, 2002 : It contains procedure for collection of Central Excise Duty. - Central Excise Tariff Act, 1985 : It contains the rates of Central Excise Duty - Cenvat Credit Rules, 2004 : It Contains the Rules for tax Credit including Service Tax and Custom Duty. Every manufacturer has to pay excise duty on the goods manufacutured by him. If excise duty is payable, it is computed on the total of cost plus profit and it is called assessable value as per section 4 of Central Excise Act, 1944. EC and SHEC are also payable on excise duty. Sales tax is charged on the total of cost plus profit plus excise duty plus EC and SHEC of excise duty. Example If cost is `5 lakhs, profit 1 lakh and excise duty rate is 10% and VAT rate is 12.5%, output tax shall be computed in the manner given below: ` Cost 5,00,000.00 Add: Profit 1,00,000.00 Total (it is called assessable value as per section 4 of Central Excise Act, 1944) 6,00,000.00 Excise Duty @ 10% 60,000.00 Add: Education Cess @ 3% 1,800.00 Total 6,61,800.00 Add: VAT @ 12.5% 82,725.00 Certain concessions have been given under Notification No. 8/2003 dated 01.03.2003 and accordingly the unit having a turnover upto `400 lakhs in the immediately preceding year shall be exempt from excise duty but only upto the turnover of `150 lakhs and excess over it shall be chargeable to excise duty. If turnover in the immediately preceding year is exceeding `400 lakhs, the unit has to pay excise duty from the beginning.

Value Added Tax

60

A unit with turnover upto `400 lakhs is called Small Scale Industry SSI. Example ABC Ltd. has turnover in F.Y. 2010-11 `300 lakhs and turnover in F.Y. 2011-12 `350 lakhs and if the rate of excise duty is 10%. Excise duty payable in F.Y. 2011-12 shall be Lakhs Total Turnover `350.00 Less: Exemption under Notification No. 8/2003 `150.00 Amount chargeable to Excise Duty `200.00 Excise duty @ 10% ` 20.00 Add: Education cess @ 3% ` 0.60 Excise Duty + EC ` 20.60 If turnover in F.Y. 2010-11 is `450 lakhs, excise duty shall be charged in F.Y. 2011-12 on entire amount of `350 lakhs. If a unit is availing exemption upto `150 lakhs under Notification No. 8/2003, Cenvat credit shall not be allowed for the inputs used in the manufacturing of final product which is covered within the turnover of `150 lakhs and tax credit shall be allowed for the inputs which will be used in manufacturing of final product which will be covered in the turnover in excess of `150 lakhs. As a special case, Cenvat credit shall be allowed for the capital goods even if capital goods have been used in manufacturing of final products which is covered in the turnover of `150 lakhs. Question 18 (V. Imp): Write a note on tax credit in case of manufacturer. Answer: A Manufacturer shall be allowed tax credit for the following tax paid by him. 1. Excise duty on inputs (raw material etc) or capital goods (plant and machinery, furniture and fixtures etc) used in or in connection with the goods to be manufactured by him. 2. Service Tax paid on the input services taken by him in connection with the goods to be manufactured by him and such services may be insurance services, banking services, warehousing, goods transport, renting of immovable property or other similar services.

3. Education cess paid on Excise Duty or Service Tax shall also be eligible for tax credit. 4. VAT paid by him on raw materials or capital goods used in or in connection with the goods to be manufactured by him.

The manufacturer can utilize the tax credit in the manner given below: 1. Tax credit for Excise Duty or Service Tax can be utilized against output excise duty. 2. 3. 4. Tax credit for Education cess of 2% can be utilized for payment of education cess of 2% on output excise duty. Tax credit for SHEC of 1% can be utilized for payment of SHEC of 1% on output excise duty. Tax credit for VAT (Sales Tax) can be utilized for payment of output VAT on sale of manufactured goods.

Value Added Tax

61

Inter-adjustment of Tax credit of Excise Duty, Service Tax is allowed to the manufacturer because all these taxes are collected by Central Government but inter-adjustment with VAT is not allowed because VAT is collected by State Government. Tax credit for Excise Duty, Service Tax is called Cenvat Credit and is regulated through Cenvat Credit Rules, 2004. Question 19 (V. Imp): What is the common procedure for availing and adjusting cenvat credit for Excise Duty, Service Tax as per Cenvat Credit Rules, 2004. Answer: Tax credit for Excise Duty, Service Tax is called Cenvat Credit and is regulated through Cenvat Credit Rules, 2004. Inter-adjustment of Tax credit of Excise Duty, Service Tax is allowed to the manufacturer because all these taxes are collected by Central Government but inter-adjustment with VAT is not allowed because VAT is collected by State Government. The common rules are as given below: 1. The CENVAT credit in respect of inputs may be taken immediately on receipt of the inputs in the factory of the manufacturer or in the premises of the provider of output service: 2. The CENVAT credit in respect of capital goods received in a factory or in the premises of the provider of output service, at any point of time in a given financial year shall be taken only for an amount not exceeding fifty per cent of the duty paid on such capital goods in the same financial year: The balance of CENVAT credit may be taken in any financial year subsequent to the financial year in which the capital goods were received in the factory of the manufacturer, or in the premises of the provider of output service, if the capital goods, are in the possession of the manufacturer of final products, or provider of output service in such subsequent years. Illustration.A manufacturer received machinery on the 16th day of April, 2011 in his factory. CENVAT of two lakh rupees is paid on this machinery. The manufacturer can take credit upto a maximum of one lakh rupees in the financial year 2011-12, and the balance in subsequent years. 3. In case of SSI unit availing exemption upto `150 lakhs, CENVAT credit for the capital goods shall be allowed in the same Financial Year. 4. While paying duty of excise or service tax, as the case may be, the CENVAT credit shall be utilized only to the extent such credit is available on the last day of the month or quarter, as the case may be, for payment of duty or tax relating to that month or the quarter, as the case may be: 5. The CENVAT credit in respect of input service shall be allowed, on the day on which the invoice, bill is received. In case the payment of the value of input service and the service tax payable as indicated in the invoice, is not made within three months of the date of the invoice, the manufacturer or the service provider who has taken credit on such input service, shall pay an amount equal to the CENVAT credit availed on such input service.

Value Added Tax

62

6. If any payment or part thereof, made towards an input service is received back or a credit note is received by the manufacturer or the service provider who has taken credit on such input service, he shall pay an amount equal to the CENVAT credit availed in respect of the amount so refunded or credited: 7. In case of an input service where the service tax is paid under reverse charge by the recipient of the service, the CENVAT credit in respect of such input service shall be allowed on the day on which payment is made of the value of input service and the service tax. 8. As per Rule 5 of Cenvat Credit Rules, 2004, unadjusted cenvat credit balance shall be allowed to be carried forward for unlimited period but its refund is not allowed however, as a special case refund is allowed in case of export. 9. As per Rule 9 of Cenvat Credit Rules, 2004, the CENVAT credit shall be taken by the manufacturer or the provider of output service or input service distributor on the basis of the invoice or bill issued by the manufacturer or importer or provider of output service or by input service distributor or by other specified persons. Illustration 15: Mr. X is registered in Central Excise/Delhi VAT/CST and he is a manufacturer and he has purchased raw material R1 for `2,50,000 and has paid excise duty @ 7% plus education cess and secondary and higher education cess and Delhi VAT @ 10%. He purchased raw material R2 for `3,20,000 and paid excise duty @ 5% plus education cess and secondary and higher education cess and central sales tax @ 2% and raw material was purchased from other state. He has purchased raw material R3 for `5,50,000 and has paid excise duty @ 7% plus education cess and secondary and higher education cess and Delhi VAT @ 10%. Processing charges `4,00,000 plus profit `70,000. The manufacturer has taken input services in connection with manufacturing of the product and has paid `5,00,000 plus service tax of `50,000 plus education cess. Final product was sold and excise duty is 18% plus education cess and Delhi VAT @ 10%. Show the working for VAT/Cenvat credit and also show the working for payment of tax at the time of sale of final product. Solution : Raw material R1 Assessable value Excise duty @ 7% EC @ 2% SHEC @ 1% Total Delhi VAT @ 10% - Input Tax Purchase Price Raw material R2 Assessable value Excise duty @ 5% ` 2,50,000 17,500 350 175 2,68,025 26,803 2,94,828 3,20,000 16,000

Value Added Tax EC @ 2% SHEC @ 1% Total Central Sales tax @ 2% - Input Tax Purchase Price Raw material R3 Assessable value Excise duty @ 7% EC @ 2% SHEC @ 1% Total Delhi VAT @ 10% - Input Tax Purchase Price Cost of Final Product Raw material - R1 Raw material - R2 (Purchase Price minus Excise Duty including EC & SHEC) Raw material - R3 Processing charges Payment for services Profit Assessable value (as per section 4 of Central Excise Act, 1944) Excise duty @ 18% EC @ 2% SHEC @ 1% Total Delhi VAT @ 10% - Output Tax Selling Price CENVAT /VAT ACCOUNT Excise Duty / Service Tax EC @ 2% ` ` 17,500.00 350.00 16,000.00 320.00 38,500.00 770.00 50,000.00 1,000.00 1,22,000.00 2,440.00 3,77,411.00 (1,22,000.00) 2,55,411.00 7,548.00 (2,440.00) 5,108.00 SHEC @ 1% ` 175.00 160.00 385.00 500.00 1,220.00 3,774.00 (1,220.00) 2,554.00

63 320 160 3,36,480 6,730 3,43,210 5,50,000 38,500 770 385 5,89,655 58,966 6,48,621 2,50,000 3,26,730 5,50,000 4,00,000 5,00,000 70,000 20,96,730 3,77,411 7,548 3,774 24,85,463 2,48,546 27,34,009 Delhi VAT ` 26,803.00 58,966.00 85,769.00 2,48,546.00 (85,769.00) 1,62,777.00

(Invoice 1) Raw material R1 (Invoice 2) Raw material R2 (Invoice 3) Raw material R3 Service tax Total Final product Output tax Less: VAT/CENVAT Credit Net tax payable

Illustration 16: ABC Limited is a manufacturing concern and the company has submitted the particulars as given below:Purchased raw material, R1: `2,00,000. (+) Excise Duty @10% (+) Education cess @ 2% (+) SHEC @1% (+) DVAT @10% Purchased raw material, R2: `3,00,000.

Value Added Tax (+) Excise Duty @12% (+) Education cess @ 2% (+) SHEC @1% (+) CST @2%

64

The company purchased plant and machinery for `10 Lakhs and paid excise duty @10% plus EC 2% plus SHEC @1% plus DVAT @ 4%. Life of the plant and machinery is 5 years and depreciation is allowed @ 20% on SLM. The company has taken certain services in connection with manufacturing of goods and has paid `3,00,000 plus service tax @ 10% plus EC 2% plus SHEC 1%. Other processing expenditure incurred by the company is `5,00,000 and profit is `3,00,000 and final product was sold by the company and output excise duty is 12% plus EC 2% plus SHEC @ 1% and output VAT is 10%. Company is registered under Central Excise Act, DVAT Act and CST Act and the company is not eligible for SSI exemption. Compute Output Excise Duty, Output VAT / Net Excise Duty/ Net VAT under Consumption Variant. (b) Presume the goods were sold in some other states to Registered Dealer against Form C (c) Presume all the goods were exported by ABC Ltd. Solution (a): Raw material R1 Purchase price Add: Excise duty @ 10% Add: Education cess @ 2% Add: SHEC @ 1% Add: Delhi VAT @ 10% Raw material R2 Purchase price 3,00,000.00 Add: Excise duty @12% Add: Education cess @ 2% Add: SHEC @1% Add: CST @ 2% Capital goods Purchase price 10,00,000.00 Add: Excise duty @10% Add: Education cess @ 2% Add: SHEC @1% Add: Delhi VAT @ 4% 2,00,000.00 20,000.00 400.00 200.00 2,20,600.00 22,060.00 2,42,660.00

36,000.00 720.00 360.00 3,37,080.00 6,741.60 3,43,821.60

1,00,000.00 2,000.00 1,000.00 11,03,000.00 44,120.00

Value Added Tax

65 11,47,120.00

Services Service Tax @ 10% Add: Education cess @ 2% Add: SHEC @1% Cost of final product Raw material R1 Raw material R2 Capital goods (10,00,000 @ 20%) Services Other processing charges Profit Assessable Value Add: Excise duty @12% Add: Education cess @ 2% Add: SHEC @ 1% Add: Delhi VAT @ 10% CENVAT/VAT ACCOUNT Excise Duty / Service Tax EC @ 2% Raw material R1 Raw material R2 Plant and machinery Service tax Total Final product Output tax Less: VAT/CENVAT Credit Net tax payable Rounded Off 20,000 36,000 1,00,000 30,000 1,86,000 2,16,809.00 1,86,000.00 30809.00 30809.00 400 720 2,000 600 3,720 4,336.18 3,720.00 616.18 616.00 SHEC @ 1% 200 360 1,000 300 1,860 2,168.09 1,860.00 308.09 308.00

3,00,000.00 30,000.00 600.00 300.00 3,30,900.00 2,00,000.00 3,06,741.60 2,00,000.00 3,00,000.00 5,00,000.00 3,00,000.00 18,06,741.60 2,16,809.00 4,336.18 2,168.09 20,30,054.87 2,03,005.49 Delhi VAT 22060 44,120 66,180 2,03,005.49 66,180.00 1,36,825.49 1,36,825.00

Solution (b): In this case the manufacturer shall charge central sales tax on the sale instead of Delhi Value Added Tax Act. Since CST shall also be paid to the Delhi Government, VAT credit shall be allowed in the normal manner and it can be adjusted against output CST and tax treatment shall be as given below: Raw material R1 Purchase price 2,00,000.00 Add: Excise duty @10% Add: Education cess @ 2% Add: SHEC @1% Add: Delhi VAT @ 10% Raw material R2

20,000.00 400.00 200.00 2,20,600.00 22,060.00 2,42,660.00

Value Added Tax Purchase price 3,00,000.00 Add: Excise duty @12% Add: Education cess @ 2% Add: SHEC @1% Add: CST @ 2% Capital goods Purchase price 10,00,000.00 Add: Excise duty @10% Add: Education cess @ 2% Add: SHEC @1% Add: Delhi VAT @ 4% Services Service Tax @ 10% Add: Education cess @ 2% Add: SHEC @1% Cost of final product Raw material R1 Raw material R2 Capital goods (10,00,000 @ 20%) Services Other processing charges Profit Assessable Value Add: Excise duty @12% Add: Education cess @ 2% Add: SHEC @1% Add: CST @ 2%- output tax CENVAT/VAT ACCOUNT Excise Duty / Service Tax EC @ 2% 20,000 400 36,000 720 1,00,000 2,000 30,000 600 1,86,000 3,720 2,16,809.00 1,86,000.00 30,809.00 4,336.18 3,720.00 616.18 SHEC @ 1% 200 360 1,000 300 1,860 2,168.09 1,860.00 308.09 -

66

36,000.00 720.00 360.00 3,37,080.00 6,741.60 3,43,821.60

1,00,000.00 2,000.00 1,000.00 11,03,000.00 44,120.00 11,47,120.00 3,00,000.00 30,000.00 600.00 300.00 3,30,900.00 2,00,000.00 3,06,741.60 2,00,000.00 3,00,000.00 5,00,000.00 3,00,000.00 18,06,741.60 2,16,809.00 4,336.18 2,168.09 20,30,054.87 40,601.10 Delhi VAT/CST 22,060 44,120 66,180 40,601.10 66,180.00 25,578.90

Raw material R1 Raw material R2 Plant and machinery Service tax Total Final product Output tax Less: VAT/CENVAT Credit Net tax payable CENVAT/VAT credit balance

Value Added Tax Rounded Off 30,809 616 308

67 25,579

Solution (c): Since the goods have been exported, there will not be any output tax and CENVAT credit/ VAT credit shall be refunded. CENVAT/VAT ACCOUNT Excise Duty / Service Tax EC @ 2% SHEC @ 1% Delhi VAT/CST Raw material R1 20,000 400 200 22,060 Raw material R2 36,000 720 360 Plant and machinery 1,00,000 2,000 1,000 44,120 Service tax 30,000 600 300 Total 1,86,000 3,720 1,860 66,180 Output tax Nil Nil Nil Nil Question 20 (Imp.): Write a note on filing of return under state VAT. Answer: A dealer is required to file return on monthly / quarterly / half-yearly / annual basis depending upon the turnover. In case of very high turnover, return has to be filed on monthly basis and in case of low turnover, return has to be filed on yearly basis e.g. As per Rule 26 of Delhi Value Added Tax Rules, 2005, return has to be filed in the manner given below: (i) If the turnover is upto `10 lakhs, return should be filed on yearly basis. (ii) If turnover is more than 10 lakhs but upto `50 lakhs, return should be filed on half-yearly basis. (iii) If it is more than 50 lakhs but upto ` 5 crores, return should be filed on quarterly basis. (iv) If it is exceeding `5 crores, return should be filed on monthly basis. Such duration of filing the return is also called Tax Period. Amount of State VAT should be paid within 28 days from end of the tax period. (Similar provision in Rule 28 of Delhi Value Added Tax Rules, 2005) Return of State VAT has to be filed in the manner given below: (a) Seventy five days from the end of the dealers tax period by the dealers having tax period of one year; (b) Forty five days from the end of the dealers tax period by the dealers having tax period of six months; (c) Twenty eight days from the end of the dealers tax period by the dealers having tax period of a quarter or one month, as the case may be. Question 21 (Imp.): Write a note on assessment under State VAT. Answer: Assessment The basic simplification of VAT is with reference to assessment. Under VAT system, there is no compulsory assessment at the end of each year. The VAT liability is self-assessed by the dealer himself in terms of submission of returns. The other procedures are also simple in all the States. Deemed assessment concept is a major feature of the VAT. If no specific notice is issued proposing departmental audit of the books of accounts of the dealer within the time limit specified in the respective

Value Added Tax

68

State VAT Acts, the dealer will be deemed to have been self-assessed on the basis of the returns submitted by him. VAT pre-supposes that all the dealers are honest. Scrutiny may be done in cases where a doubt arises of under-reporting of transaction or evasion of tax. Honest dealers will be protected and fictitious or dishonest would be penalized heavily.

Question 22: Write a note on System of Cross Checking. Answer: System of Cross Checking In the VAT system more emphasis has been laid on self-assessment. Hence, a system of cross-checking is essential. Dealers may be asked to submit the list of sales or purchases above a certain monetary value or to give the dealer-wise list from whom or to whom the goods have been purchased/sold for values exceeding a prescribed monetary ceiling. A cross-checking computerized system is being worked out on the basis of coordination between the tax authorities of the State Government and the authorities of Central Excise and Income-tax to compare constantly the tax returns and set-off documents of VAT system of the States and those of Central Excise and Income-tax. This comprehensive cross-checking system will help reduce tax evasion and also lead to significant growth of tax revenue. At the same time, by protecting the interests of tax-complying dealers against the unfair practices of tax-evaders, the system will also bring in more equal competition in the sphere of trade and industry. Question 23 (Imp.): Write a note on maintaining of books of accounts and records under State VAT. Answer: Books of Accounts/ Records/ Documents The following records should be maintained under VAT system: 1. Purchase records, showing details of purchases on which tax has been paid, purchases made without payment of tax, purchases made from an exempted unit (Military Canteen) and purchases made from outside State. 2. Sales records, showing separately sales made at different tax rates, zero-rated taxable sales and tax-free sales. 3. VAT account - A monthly account specifying total output tax, total input tax and net tax payable or the excess tax credit due for carry forward. 4. Details of input tax calculations where the dealer is making both taxable and tax free sales. 5. Stock records showing stock receipts and deliveries and manufacturing records. 6. Stock records showing separately the particulars of goods stored in cold storage, warehouse, godown or any other place taken on hire. 7. Order records and delivery challans, wherever applicable. 8. Annual accounts including trading, profit and loss accounts and the balance sheet.

Value Added Tax 9. Bank records, including statements, cheque book counter foils and pay-in-slips. 10. Cash book, daybook and ledger. The following documents should be retained:

69

1. Original tax invoices for purchases on which tax has been paid and invoices for purchases made without payment of tax in numerical order. 2. Copies of tax invoices related to taxable sales and invoices related to exempt sales shall be retained date wise and in numerical order. Question 24 (Imp.): Explain the provisions of audit under State VAT. Answer: Audit In the VAT system considerable weightage is placed on audit work in place of routine assessment work. Correctness of self assessment will be checked through a system of Departmental Audit. A certain percentage of the dealers will be taken up for audit every year on a scientific basis. If, however, evasion is detected in the course of audit, the preview records of the concerned dealer may be taken up for audit. Authorized officers of the departmental will visit the business place of the dealer to conduct the audit. The auditors will examine the correctness of the returns vis-a-vis the books of accounts of the dealer or any other information available with them. They will be equipped with the information gathered from various agencies such as suppliers, income tax department, excise and customs department, banks etc. Officers of the higher rank will supervise to ensure that the audit work is done in a free, fearless and impartial manner. Under the sales-tax laws, tax evasion is considered to be on a large scale. The sales-tax departments of various States have not been able to effectively check the menace (harm) of tax avoidance and tax evasion. Therefore, apart from the departmental audit many States have also incorporated the concept of audit of accounts by Chartered Accountants. The State of Maharashtra has prescribed an elaborate list of particulars to be furnished by the dealers. These particulars have to be verified by the VAT auditor. However, auditing for all types of dealers may not be necessary. The selection of cases for auditing has to be made in accordance with the criteria of the size of dealers. In Maharashtra and Rajasthan, the dealer whose turnover exceeds `40 lakhs in any year is required to get his accounts audited in respect of such year. Question 25 (V. Imp.): Explain merits of VAT. Answer: Merits 1. No Tax Evasion Under VAT, unless proper records are kept in respect of duty or tax paid on various inputs, it is not possible to claim credit. Hence, suppression of purchases or production will be very difficult. 2. Transparency Under a VAT system, the buyer knows, out of the total consideration paid for purchase of material, what is tax component. Thus, the system ensures transparency. 3. Neutrality (not affecting) The greatest advantage of the tax credit system is that it does not interfere in the choice of decision for purchases. This is because the system has anti-cascading effect. How much value is added and at what stage

Value Added Tax

70

it is added in the system of production/distribution is of no consequence. The system is neutral with regard to choice of production technique, as well as business organisation. All other things remaining the same, the issue of tax liability does not vary the decision about the source of purchase. VAT facilitates precise identification and rebate of the tax on purchases and thus ensures that there is no cascading effect of tax. In short, the allocation of resources is left to be decided by the free play of market forces and competition. For example, a manufacturer may purchase raw material after paying tax @ 5% or at higher rate of tax @ 8%. In both the cases the price of final product will remain same, because he will get deduction of taxes on purchases while paying taxes on final product. 4. Certainty The VAT is a system based simply on transactions. Thus there is no need to go through complicated definitions like sales, sales price, turnover of purchases and turnover of sales. The tax is also broad-based and applicable to all sales in business leaving little room for different interpretations. Thus, this system brings certainty to a great extent. Also there are only a few rates under VAT in comparison to numerous rates in earlier sales tax. 5. Better Revenue Collection and Stability There will be a minimum possibility of revenue leakage, since the tax credit will be given only if the proof of tax paid at an earlier stage is produced. This means that if the tax is evaded at one stage, full tax will be recoverable from the person at the subsequent stage or from a person unable to produce proof of such tax payment. Thus, in particular, an invoice of VAT will be self enforcing and will induce business to demand invoices from the suppliers. 6. Better Accounting Systems Since the tax paid on an earlier stage is to be received back, the system will promote better accounting systems. 7. Effect on Retail Price/VAT is Inflationary A persistent criticism of the VAT has been that since the tax is payable on the final sale price, the VAT usually increases the prices of the goods. However, VAT does not have any inflationary impact as it merely replaces the existing equal sales tax. Question 26 (V. Imp): Explain demerits of VAT. Answer: Demerits 1. Central VAT/ State VAT So long as Central VAT is not integrated with the State VAT, it will be difficult, to put the purchases from other States at par with the State purchases. 2. Increase in Accounting Cost For complying with the VAT provisions, the accounting cost will increase. The burden of this increase may not be commensurate with the benefit to traders and small firms. 3. Increase in Working Capital Since the tax is to be imposed or paid at various stages and not on last stage, it would increase the working capital requirements and the interest burden on the same. In this way it is considered to be non-beneficial as compared to the single stage-last point taxation system. 4. VAT is Regressive VAT is a form of consumption tax. Since, the proportion of income spent on consumption is larger for the poor than for the rich, VAT tends to be regressive. However, this weakness is inherent in all the forms of

Value Added Tax

71

consumption tax. While it may be possible to moderate the distribution impact of VAT by taxing necessities at a lower rate, it is always advisable to moderate the distribution considerations through other programmes rather than concessions or exemptions, which create complications for administration. 5. Increase in Administrative Cost As a result of introduction of VAT, the administration cost of the State increase significantly as the number of dealers to be administered will go up significantly. Question 27: Explain role of ICAI in VAT. Answer: ICAIs role in VAT The ICAI has rendered pioneering service in evoling the necessary accounting guidelines both for CENVAT as well as State Level VAT. It has brought out Guidance Notes for accounting for CENVAT as well as State-Level VAT. These Guidance Notes address all the accounting issues in regard to CENVAT and State Level VAT. Further, the institute has brought out a comprehensive study on State Level VAT in India. It contains an elaborate discussion of the various general principles of VAT and State Level VAT. These general principles have been incorporated in the various State Level VAT legislation. However, there are special provisions contained in the respective State level regislation to cater to the specific needs of the States. Various State government have issued detailed clarification on different practical issues arising on implementation of the State-Level VAT. Question 28 (Imp.): Explain role of Chartered Accountant in VAT. Answer: Role of Chartered Accountant in VAT Chartered Accountant have a key role to play in proper implementation of VAT. (i) Record Keeping : VAT requires proper record keeping and accounting. Systematic records of input credit and its proper utilisation is necessary for the success of VAT. Chartered Accountants are well equipped to perform such tasks. (ii) Tax Planning : In order to establish an efficient plan for purchases and sales, a careful study of VAT is required. A Chartered Accoountant is competent to analyze the impact of various alternatives and choose the most optimum method of purchases and sales in order to minimize the tax impact. (iii) Handling the audit departmental officers: There will be audit wing in department and certain percentage of dealers will be taken up for audit every year on scientific basis. Chartered Accountant can ensure proper record keeping so as to satisfy the departmetnal auditors. The professional expertise of a Chartered Accountant will help him in effectively replying audit queries and sorting out audit objections. (iv) External audit of VAT records: Under VAT system, trust has been reposed on tax payers as there will be no regular assessment of all VAT returns but only few returns will be scrutinized. In other cases, return filed by dealer will be accepted. Thus, a check on compliance becomes necessary. Chartered Accountants can play a very vital role in ensuring tax compliance by audit of VAT accounts. Illustration 17: The particulars regarding sale, purchase etc. of ABC Ltd. for the year 2011-12 are as under: Purchases of raw material within the state (i) R1 for `60,00,000 + VAT @ 1% (ii) R2 for `90,00,000 + VAT @ 4% (iii) R3 for `15,00,000 + VAT @ 12.5%

Value Added Tax (1) Sale of raw material R1 within the state `66,00,000 + sales tax @ 1% (2) Sale of goods manufactured from raw material R2 (i) taxable sale within the state `30,00,000 + VAT @ 4% (ii) exempted sale within the state `15,00,000 (iii) sale in the course of inter-state trade or commerce `15,00,000 + CST @ 2% (3) Goods manufactured from the raw material R3 `18,00,000 + VAT @ 12.5% Show the tax treatment. Solution: Computation of Output Tax Raw Material R1 Sale of Raw material within the State i.e. 66,00,000 x 1% = `66,000 Raw Material R2 (i) Sale of Raw material within the state = 30,00,000 x 4% = `1,20,000 (ii) since goods are exempt, no VAT shall be charged and also no tax credit shall be allowed. (iii) Sale of Raw material in the course of interstate sale = 15,00,000 x 2% = `30,000 Raw Material R3 Goods sold in the state 18,00,000 x 12.5% = `2,25,000 Output Tax (i) State VAT = 66,000 + 1,20,000 + 2,25,000 = `4,11,000 (ii) CST = 30,000 Computation of Input Tax Credit Raw material R1 60,00,000 x 1% = `60,000

72

Raw material R2 Raw material R2 was purchased for `90,00,000 but it was sold for `60,00,000 i.e. at a loss but in this case VAT credit shall be allowed for full amount however proportionate VAT credit for the goods which are exempt from VAT shall not be allowed hence VAT credit shall be allowed only for 75% of `90,00,000 i.e. 90,00,000 x 75% x 4% = `2,70,000 Raw material R3 15,00,000 x 12.5% = `1,87,500 Total Input Tax Credit = 60,000 + 2,70,000 + 1,87,500 = `5,17,500 Output Tax (i) State VAT = Less: VAT credit VAT Payable (ii) CST Less: VAT credit CST payable Balance of unutilized VAT ` 4,11,000 4,11,000 Nil 30,000 30,000 Nil 76,500

Value Added Tax

73

Unutilized VAT shall be carry forward and in the balance sheet it will be shown as assets (advances) Ultimately, the dealer can claim refund of VAT (no such refund is allowed in case of excise duty or service tax except in case of export) Illustration 18: Compute the net VAT liability of Mr. X from the information as given below: (i) Raw material purchased from foreign market (including basic custom duty paid on imports @ 20% + EC @ 3%) `70,500 (ii) Raw material purchased from local market (including VAT charged on the material @ 1%) `15,150 (iii) Raw material purchased from another state (excluding CST @ 2%) `30,000 (iv) Storage, transportation cost and insurance `4,500 (v) Other manufacturing expenses incurred `900. Mr. X sold the goods to Mr. Y adding margin of profit @ 10% on the selling price. VAT rate on sale of such goods is 10%. Solution: Total cost = 70,500 + 15,000 + 30,600 + 4,500 + 900 = `1,21,500. Since profit is 10% of sale price, `1,21,500 is 90% of sale price hence sale price shall be 1,21,500 x 100% / 90% = `1,35,000 (it means the product was sold for 1,35,000 and there is a profit of 10% i.e. 13,500) VAT rate is 10% hence output tax shall be 1,35,000 x 10% = `13,500 Input tax credit shall be 15,150 x 1% / 101% = `150 Net tax = 13,500 150 = `13,350 Illustration 19: Calculate the VAT liability for the period Jan. 1, 2012 to Jan. 31, 2012 from the following particulars: Input worth `1,60,000 were purchased within the state plus VAT @ 12.5% VAT paid on procurement of capital goods worth `1,50,000 plus VAT @ 12.5%. `3,00,000 worth of finished goods were sold within the state plus VAT @ 4% `1,50,000 worth of goods were sold in the course of inter-state trade plus CST @ 2%. Show the total tax liability under the State VAT law and under the Central Sales Tax Act considering it to be consumption variant. Solution: Output Tax ` Sale within State `3,00,000 x 4% 12,000 Inter state sale `1,50,000 x 2% 3,000 Less: VAT credit On Inputs `1,60,000 x 12.5% 20,000 On capital goods `1,50,000 x 12.5% 18,750

Value Added Tax

74

Thus, out of VAT credit of `38,750, `15,000 can be utilized to pay tax and balance `23,750 will be carried forward. Illustration 20: ABC Ltd. of Rajasthan purchased raw material A from Rajasthan for `20,800 (inclusive of 4% VAT), raw material B from Rajasthan for `45,000 (inclusive of 12.5% VAT), raw material C from China for `67,980 (inclusive of 10% basic custom duty + EC @ 3%,) and raw material D from Maharashtra for `600 (inclusive of 2% CST). The plant and machinery required for manufacture was purchased for `4,16,000 (inclusive of 4% VAT). Credit of VAT on plant and machinery is allowed in the year of purchase The manufacturing and other expenses (excluding depreciation) were `1,51,420 The plant is to be depreciated at 100%. The manufacturers margin is 20% on cost. The VAT rate on the manufactured product is 4%. By way of necessary accounting entries, show the mode of operation of VAT system. Ignore the Central Excise implications, assuming that there is no excise duty on the manufactured product. Solution: ` Raw material A (net of VAT `800) 20,000 Raw material B (net of VAT `5,000) 40,000 Raw material C (Basic custom duty + EC will form part of cost, as it is not available as 67,980 credit) Raw material D (CST will form part of cost, as it is not available as credit) 600 Depreciation on plant and machinery (100% of 4,00,000) 4,00,000 Manufacturing and other expenses 1,51,420 Cost of the product 6,80,000 Add: 20% margin 1,36,000 Selling price 8,16,000 Add: VAT @ 4% of 8,16,000 32,640 VAT payable by the manufacturer = 32,640 800 5,000 16,000 = `10,840 Accounting Entries (Accounting entries are not part of the syllabus rather it is given for better understanding) Dr. ` Cr. ` 1 Purchase of Raw material A . Raw material A A/c Dr. 20,000 Input Tax Credit A/c Dr. 800 To Bank 20,800 2 Purchase of Raw material B . Raw material B A/c Dr. 40,000 Input Tax Credit A/c Dr. 5,000 To Bank 45,000 3 Purchase of Raw material C . Raw material C A/c Dr. 67,980 To Bank 67,980 4 Purchase of Raw material D .

Value Added Tax Raw material D A/c To Bank Purchase of plant and machinery i.e. capital goods Plant and machinery A/c Input Tax Credit A/c To Bank Manufacturing & other expenses Manufacturing and other expenses A/c To Bank Depreciation Depreciation A/c To plant and machinery A/c Sale of manufacturing product Bank 9 . To sales A/c To VAT payable A/c Payment of VAT VAT payable A/c To Input Tax credit A/c To Bank Dr. 32,640 Dr. 600

75 600 Dr. Dr. 4,00,000 16,000 4,16,000 Dr. 1,51,420 1,51,420 Dr. 4,00,000 4,00,000 Dr. 8,48,640 8,16,000 32,640

5 .

6 . 7 . 8 .

21,800 10,840

Illustration 21: A manufacturer has purchased raw material for `2,08,000 (inclusive of 4% VAT) and plant and machinery of `4,00,000 (VAT nil). The manufacturing and other expenses (including building, rent, wages etc.) are `6,00,000. He sells the resultant products at 50% above cost (VAT on sales is 4%). The plant and machinery is to be depreciated at 50% straight line. Compute the amount of VAT payable as per the addition method. Solution: Computation of Value added and VAT payable Depreciation on plant and machinery @ 50% of 4,00,000 Manufacturing and other expenses Total factor payments Profit @ 50% of total cost i.e. Material cost + Factor payments i.e. 2,00,000 + 8,00,000 Value Added VAT @ 4% on value added ` 2,00,000 6,00,000 8,00,000 5,00,000 13,00,000 52,000

Illustration 22: Manufacturer A of Punjab extracted raw produce X and raw produce Y from mines at `30,000 and `40,000 respectively and sold the same at 150% margin to Manufacturer B of Delhi (CST rate is 2%).

Value Added Tax

76

Manufacturer B of Delhi used X and Y as raw material; added 100% of cost of raw material towards manufacturing expenses and profits and sold the resultant product to wholesaler C of Delhi (VAT rate is 4%). Wholesaler C of Delhi sold the same to Retailer D of Delhi at 25% above cost (VAT rate is 4%) The retailer D sold the same to a consumer at 20% above cost (VAT rate is 4%). Show the amount of VAT payable by each person under the invoice method of computation of VAT. Solution: Manufacturer A Cost of raw material X and Y (30,000 + 40,000) Add: Profit @ 150% of `70,000 Total Add: CST @ 2% Total Selling Price CST of `3,500 shall be payable to the Government of Punjab Manufacturer B Cost of Manufacturer B (VAT credit of `3,500 is not allowed because payment has been made in Punjab) Add: Profit @ 100% of cost Total Add: VAT @ 4% Total Selling Price Wholesaler C Cost of Wholesale C (VAT credit shall be allowed for `14,280) Add: Profit @ 25% Total Add: VAT @ 4% Total Selling Price Net tax payable shall be (17,850 14,280) Retailer D Cost of Retailer D (VAT credit shall be allowed for `17,850) Add: Profit @ 20% Total Add: VAT @ 4% Total Selling Price Net tax payable shall be (21,420 17,850) ` 70,000 1,05,000 1,75,000 3,500 1,78,500

1,78,500 1,78,500 3,57,000 14,280 3,71,280 3,57,000 89,250 4,46,250 17,850 4,64,100 3,570 4,46,250 89,250 5,35,500 21,420 5,56,920 3,570

Illustration 23: Manufacturer A of Jaipur extracted raw produce X and raw produce Y from mines at `30,000 and `40,000 respectively and sold the same at 150% margin to Manufacturer B of Jaipur (VAT rate is 4% on produce X and 12.5% on produce Y).

Value Added Tax

77

Manufacturer B of Jaipur used X and Y as raw material; added 100% of cost of raw material towards manufacturing expenses and profits and sold the resultant product to wholesaler C of Delhi (CST rate is 2%). Wholesaler C of Delhi sold the same to Retailer D of Delhi at 20% above cost (VAT rate is 4%). The Retailer D sold the same to a consumer at 20% above cost (VAT rate is 4%). Show, by way of invoice method, the amount of VAT payable by each person. Solution: Manufacturer A Cost of raw material X Add: Profit @ 150% of `30,000 Total Add: VAT @ 4% Total Selling Price Cost of raw material Y Add: Profit @ 150% of `40,000 Total Add: VAT @ 12.5% Total Selling Price Manufacturer B Cost of Manufacturer B (`75,000 + `1,00,000) Add: Profit @ 100% of cost Total Add: CST @ 2% Total Selling Price Balance in the VAT receivable account (3,000 + 12,500) 7,000 Wholesaler C Cost of Wholesale C Add: Profit @ 20% Total Add: VAT @ 4% Total Selling Price Retailer D Cost of Retailer D (VAT credit shall be allowed for `17,136) Add: Profit @ 20% Total Add: VAT @ 4% Total Selling Price Net tax payable shall be (20,563 17,136) ` 30,000 45,000 75,000 3,000 78,000 40,000 60,000 1,00,000 12,500 1,12,500 1,75,000 1,75,000 3,50,000 7,000 3,57,000 8,500 3,57,000 71,400 4,28,400 17,136 4,45,536 4,28,400 85,680 5,14,080 20,563 5,34,643 3,427

Illustration 24: Mr. X of Rajasthan is registered with VAT authorities. He presents the following details for the month of January, 2012 Purchase from Rajasthan 20,00,000

Value Added Tax Purchases from Delhi Sales within Rajasthan out of purchases from Delhi Sales to dealer of Maharastra out of purchases within Rajasthan Sales to dealer of Maharastra out of purchases from Delhi Aforesaid amounts are exclusive of taxes. VAT rate is 4%. CST rate is 2%. Compute tax payable by Mr. X. Solution: The tax payable by Mr. X for the month of January, 2012 is computed hereinbelowSales inside Rajasthan Total sales Output VAT liability @ 4% Input VAT credit available on purchases from Rajasthan (20,00,000 x 4%) Net tax payable Excess VAT credit to be utilized against payment of CST Sales outside Rajasthan Total sales Output CST liability @ 2% Input VAT credit available on purchases from Rajasthan Balance VAT credit available Illustration 25: Mr. A , a registered dealer presents following details for March, 2012 1. Opening Balance of Input VAT credit as on 01.03.2012 ` 30,000. 2. Inputs purchased during the month of March: `30 lakh. 3. Within the state sales of manufactured goods: `40 lakh. 4. Inter State Sales: `8 lakh.

78 16,00,000 16,00,000 2,00,000 4,00,000

(VAT)` 16,00,000 64,000 80,000 NIL 16,000 (CST) 6,00,000 12,000 16,000 4,000

CST rate is 2%. There was no inventory as on 01.03.2012 or 31.03.2012 The VAT laws governing Mr. A provide for the refund of input VAT credit after the end of the first financial year itself. The above amount are exclusive of VAT/ CST. VAT rate is 12.5% on inputs and 4% on sales. Compute the amount of refund available to Mr. A. Solution: Computation of refund available to Mr. A (Amounts in `) Opening balance of input VAT credit 30,000 Add: VAT credit availed on inputs purchased during March (12.5% of 30 lakh) 3,75,000 Less: VAT payable on sales (4% on `40 lakh) 1,60,000 Less: CST payable on inter-state sales (2% on 8 lakh) 16,000 Balance lying as VAT credit as on 31.03.2012 eligible for refund 2,29,000

Value Added Tax

79

Illustration 26: Mr. X of Maharashtra has purchased a plant (capital goods) valuing `30 lakh (VAT thereon @ 4%) on 01.01.2012. On 31.03.2012 he sold such plant for `28 lakh (net) to another trader engaged in same business. Pass the journal entries to record these transactions, assuming that there is no other transaction during the intervening period. Ignore Depreciation. (In Maharastra, VAT credit on capital goods is allowed in one instalments but if capital goods have been sold within 3 years, in that case proportionate amount of VAT credit shall be reversed.) Solution: Journal entries in the books of Mr. X (Accounting entries are not part of the syllabus rather it is given for better understanding) 01/01/12 Plant A/c Dr. 30,00,000 Input VAT Credit A/c Dr. 1,20,000 To Bank 31,20,000 (Plant purchased and 100% credit of VAT paid thereon allowed in the State of Maharashtra) 31/03/12 Plant A/c Dr. 1,10,000 To Input VAT credit A/c 1,10,000 (Since the plant has been sold after 3 months i.e. within 36 months, hence, proportion input-credit of VAT to be reversed on capital goods sold = 1,20,000 x 33 36 = `1,10,000. This amount will increase the cost of the plant). 31/03/12 Bank A/c Dr. 28,00,000 Loss on sale of Plant A/c Dr. 3,10,000 To Plant A/c 31,10,000 (Plant sold and loss booked) Illustration 27: Mr. M a manufacturer of taxable as well as tax-free goods, furnishes the following information. (a) Sales of Product A (tax-free goods): `100 lakhs; (b) Sales of Product B (taxable goods): `200 lakhs (VAT @ 12.5%) (c) Purchases of Input X (used in manufacturer of Product A only): `60 lakhs (VAT @ 4%) (d) Purchases of Input Y (used in manufacturer of Product B only): `150 lakhs (VAT @ 4%) (e) Purchases of Input Z (used in manufacturer of Product A & B): `30 lakhs (VAT @ 20%) Compute the amount of VAT payable in cash by Mr. M assuming that input Z is used in product A and B in the ratio of 1:2 Ignore implications under other laws. Solution: ` Computation of VAT liability of Mr. M Since product A is tax free, inputs used in manufacturing of product A shall not be eligible for VAT credit. Inputs used in manufacturing of product B shall be eligible for VAT credit and is as given below: Input Y (150,00,000 x 4%) 6,00,000 Input Z (30,00,000 x 2/3 x 20%) 4,00,000 Total VAT credit 10,00,000

Value Added Tax Sale of product B Output Tax (200,00,000 x 12.5%) Less: VAT credit VAT payable

80 25,00,000 10,00,000 15,00,000

Illustration 28: Mr. X is a Registered in Central Excise, Central Sales Tax and Delhi VAT and he has purchased raw material for `20,00,000 in Delhi and paid excise duty @ 10% plus education cess and secondary and higher education cess @ 3% and Delhi VAT @ 4%. (i) 1/8th of raw material is stock transferred to some other state. (ii) 1/8th of raw material is used in manufacturing of final product which is exempt from VAT and processing charges and profit is `1,50,000. (iii) 1/8th of raw material is used in manufacturing of final product which is exempt from excise duty but VAT is payable and processing charges and profit is `1,50,000. (iv) 1/8th of raw material is used in manufacturing of final product which is exported from India. (v) 1/8th of raw material is used in manufacturing of final product which is sold in some other state and CST @ 2% and processing charges and profit is `1,50,000. (vi) 1/8th of raw material is used in manufacturing of final product which is sold in the same state. Processing charges and profit is `1,50,000. (vii) Remaining raw material is lying in the stock. Output excise duty rate is 10% plus EC/ output Delhi VAT @ 12.5%. Compute Output tax / Tax Credit / Net tax for Excise duty /VAT. Solution: Raw material Excise duty @ 10% EC @ 2% SHEC @ 1% Total Delhi VAT @ 4% ` 20,00,000.00 2,00,000.00 4,000.00 2,000.00 22,06,000.00 88,240.00 22,94,240.00

(i) Since 1/8th of the stock has been transferred to some other state, VAT credit allowed for such stock transfer shall be 2% (4% - 2%). Tax paid (88,240 x 1/8) 11,030.00 VAT credit (4% - 2%) 5,515.00 (In case of stock transfer VAT credit is allowed after retaining 2%) (ii) Raw material VAT (`88,240 x 1/8) Processing charges Assessable value 2,50,000.00 11,030.00 1,50,000.00 4,11,030.00

Value Added Tax

81

Excise duty @ 10% 41,103.00 EC @ 2% 822.06 SHEC@ 1% 411.03 Total 4,53,366.09 VAT Nil (Since output sales tax is exempt, hence VAT credit for input sales tax is not allowed and it will be added in the cost) (iii) Raw material Excise duty @ 10% EC @ 2% SHEC@ 1% Processing charges Total Delhi VAT @ 12.5% 2,50,000.00 25,000.00 500.00 250.00 1,50,000.00 4,25,750.00 53,218.75 4,78,968.75 (Since output excise duty is exempt, Cenvat credit for input excise duty is not allowed and it will be added in the cost) (iv) No tax is payable in case of export but VAT credit will be allowed. (v) Raw material Processing charges Assessable value Excise duty @ 10% EC @ 2% SHEC @ 1% Total Central sales tax @ 2% (vi) Raw material Processing charges Assessable value Excise duty @ 10% EC @ 2% SHEC @ 1% Total Delhi VAT @ 12.5% Excise Duty ` 41,103.00 EC @ 2% ` 822.06 SHEC @ 1% ` 411.03 DVAT ` 53,218.75 2,50,000 1,50,000 4,00,000 40,000 800 400 4,41,200 8,824 4,50,024 2,50,000 1,50,000 4,00,000 40,000 800 400 4,41,200 55,150 4,96,350 CST ` -

Particulars OUTPUT TAX 1/8th raw material (Stock transfer) 1/8th Final Product (Exempt from sales tax) 1/8th Final Product (Exempt from excise duty)

Value Added Tax 1/8th Final product Exported 1/8th Final product sold in some other State 1/8th Final product sold in same State Total 40,000.00 40,000.00 1,21,103.00 800.00 800.00 2,422.06 400.00 400.00 1,211.03 -

82 8,824.00

55,150.00 1,08,368.7 8,824.00 5

Less: INPUT TAX CREDIT 1/8th raw material (Stock transfer) 25,000.00 500.00 250.00 5,515.00 1/8th Final Product (Exempt from 25,000.00 500.00 250.00 sales tax) 1/8th Final Product (Exempt from 11,030.00 excise duty) 1/8th Final product Exported 25,000.00 500.00 250.00 11,030.00 th 1/8 Final product sold in some other 25,000.00 500.00 250.00 11,030.00 State 1/8th Final product sold in same State 25,000.00 500.00 250.00 11,030.00 Raw material lying in the stock 50,000.00 1,000.00 500.00 22,060.00 Total 1,75,000.00 3,500.00 1,750.00 71,695.00 Tax payable 36,673.75 8,824.00 VAT / CENVAT Credit Balance 53,897.00 1,077.94 538.97 Cenvat credit for excise duty and service tax shall be refundable only in case of exports, otherwise its carry forward is allowed.

PRACTICE PROBLEMS
TOTAL PROBLEMS 15
Problem 1. Compute the invoice value to be charged and amount of tax payable under VAT by a Registered Dealer who had purchased goods for `2,00,000 (exclusive of VAT) and after adding for expenses of `18,000 and of profit `35,000 had sold out the same in the same state. The rate of VAT on purchase and sales is 12.5%. (b) Goods were sold under inter-state sale to a Registered Dealer. Problem 2: Compute the VAT amount payable by Mr. A (Registered Dealer) who purchases goods from a manufacturer on payment of `5,62,500 (including VAT) and earns 20% profit on purchase. The goods have been sold to retailers and VAT rate on purchase and sale is 12.5%. (b) Goods were sold under inter-state sale to an Unregistered Dealer. Problem 3: Mr X (Registered Dealer) is trader and he has purchased certain goods from Punjab for `4,50,000 and has paid central sales tax @ 2%. He has sold all the goods in the state of Delhi for `6,50,000 plus VAT @ 12.5%. He has purchased certain goods in Delhi for `5,50,000 and paid VAT @ 12.5% and all the goods were sold by him under inter state sale to some person in M.P. for `7,50,000 plus central sales tax @ 2%. Show VAT calculation.

Value Added Tax

83

Problem 4: Mr. X (Registered Dealer in Delhi) is a manufacturer and he has purchased raw material R1 from Punjab for `3,00,000 plus central sales tax @ 2%. He has purchased raw material R2 in Delhi for `4,00,000 inclusive of VAT @ 10%. His processing charges is `5,00,000 and profit margin is `5,00,000. Half of the goods were sold in Delhi and VAT payable is @ 10% and remaining half were sold to a person in M.P. under inter state sale and has charge central sales tax @ 2%. Show working of VAT/CST Payable. Problem 5: Mr. X is a manufacturer sells goods to Mr. B, a distributor for `6,70,000. Mr. B sells goods to Mr. K, a wholesaler for `9,60,000. The wholesaler sells the goods to a retailer for `11,40,000. The retailer sold the goods to consumers for `13,00,000. All the above amounts are exclusive of VAT. All the above persons are registered under local VAT and also under Central Sales Tax Act 1956. Compute input tax credit, output tax and net tax under invoice method for each of the person and VAT rate is @ 12.5%. (b) Presume all the above amounts are inclusive of VAT @ 12.5%. (c) Presume manufacturer and distributor are in Punjab and wholesaler and retailer are in Delhi. (i) VAT is exclusive (ii) VAT is inclusive (d) Presume manufacturer is in Punjab, distributor in Haryana, wholesaler in U.P. and retailer in M.P. Amounts are exclusive of CST @ 2%. Problem 6: Mr. X is a Dealer Registered in Delhi Value Added Tax Act, 2004 and also under Central Sales Tax Act, 1956 and he has submitted the informations as given below: (i) Purchased Goods A from Delhi for `2,00,000 and paid VAT @ 4% and sold the goods in Delhi at a profit of 50% on purchase price and charged VAT @ 4%. (ii) Purchased goods B from U.P. for `4,00,000 and paid central sales tax @ 2% and sold goods in Delhi at a profit of 50% on purchase price and charged VAT @ 12.5%. (iii) Purchased goods C from Delhi for `8,00,000 and paid VAT @ 12.5% and sold the goods at a profit of 50% on purchase price to a registered dealer in Orissa and charged Central Sales Tax @ 2% (iv) Purchased goods D for `10,00,000 in Delhi and paid VAT @ 12.5% and sold the goods at a profit of 50% on purchase price to an unregistered dealer in Punjab and charged Central Sales Tax @ 12.5%.

Value Added Tax

84

(v) Purchased goods E from Madhya Pradesh for `6,00,000 and paid Central Sales Tax @ 1% and sold goods at a profit of 50% on purchase price in Maharashtra and charged central sales tax @ 1%. (vi) Purchased goods F from Delhi `14,00,000 and paid VAT @ 1% and the goods were sold at a profit of 50% on purchase price to an unregistered dealer in Maharashtra and charged central sales tax @ 1%. (vii) Purchased goods G for `12,00,000 in Delhi and paid VAT @ 12.5% and goods were stock transferred to some other state. (viii) Purchased goods H for `16,00,000 in Delhi and paid VAT @ 4% and goods were exported at a profit of 50% on purchase price and no VAT was charged (because as per section 5 Central Sales Tax Act, 1956, VAT can not be charged in case of export sale.) (ix) Purchased goods I for `18,00,000 in Delhi and paid VAT @ 12.5% and sold the goods at a profit of 50% on purchase price to a manufacturer in SEZ and no VAT was charged. Show the tax treatment for VAT and also compute his income tax liability for the assessment year 2012-13. Problem 7: Mr. X is registered in Central Excise/Delhi VAT/CST is a manufacturer and he has purchased raw material R1 for `1,50,000 and has paid excise duty @ 8% plus education cess and secondary and higher education cess and Delhi VAT @ 10%. He purchased raw material R2 for `2,20,000 and paid excise duty @ 4% plus education cess and secondary and higher education cess and central sales tax @ 2% and raw material was purchased from other state. He has purchased raw material R3 for `4,50,000 and has paid excise duty @ 8% plus education cess and secondary and higher education cess and Delhi VAT @ 10%. Processing charges `6,00,000 plus profit `50,000. The manufacturer has taken input services in connection with manufacturing of the product and has paid `2,00,000 plus service tax of `20,000 plus education cess and secondary and higher education cess. Final product was sold and excise duty is 16% plus education cess plus SHEC and Delhi VAT @ 10%. Show the working for VAT credit and also show the working for payment of tax at the time of sale of final product. Problem 8: ABC Limited is a manufacturing concern and the company has submitted the particulars as given below:Purchased raw material, R1: `3,00,000. (+) Excise Duty @ 10% (+) Education cess @ 2% (+) SHEC @ 1% (+) DVAT @ 10% Purchased raw material, R2: `4,00,000. (+) Excise Duty @ 12% (+) Education cess @ 2% (+) SHEC @ 1% (+) CST @ 2%

Value Added Tax

85

The company purchased plant and machinery for `12 Lakhs and paid excise duty @10% plus EC 2% plus SHEC @1% plus DVAT @ 4%. Life of the plant and machinery is 5 years and depreciation is allowed @ 20% on SLM. The company has taken certain services in connection with manufacturing of goods and has paid `4,00,000 plus service tax @ 10% plus EC 2% plus SHEC 1%. Other processing expenditure incurred by the company is `6,00,000 and profit is `4,00,000 and final product was sold by the company and output excise duty is 12% plus EC 2% plus SHEC @ 1% and output VAT is 10%. Company is registered under Central Excise Act, DVAT Act and CST Act and the company is not eligible for SSI exemption. Compute Output Excise Duty, Output VAT / Net Excise Duty/ Net VAT under Consumption Variant. (b) Presume the goods were sold in some other states to Registered Dealer against Form C (c) Presume all the goods were exported by ABC Ltd. Problem 9: Mr. X is a Registered in Central Excise, Central Sales Tax and Delhi VAT and he has purchased raw material for `12,00,000 in Delhi and paid excise duty @ 10% plus education cess and secondary and higher education cess @ 3% and Delhi VAT @ 4%. (i) 1/6th of raw material is stock transferred to some other state. (ii) 1/6th of raw material is used in manufacturing of final product which is exempt from VAT and processing charges and profit is `1,00,000. (iii) 1/6th of raw material is used in manufacturing of final product which is exempt from excise duty but VAT is payable and processing charges and profit is `1,00,000. (iv) 1/6th of raw material is used in manufacturing of final product which is exported from India. (v) 1/6th of raw material is used in manufacturing of final product which is sold in some other state and CST @ 2% and processing charges and profit is `1,00,000. (vi) 1/6th of raw material is used in manufacturing of final product which is sold in the same state. Processing charges and profit is `1,00,000. Output excise duty rate is 10% plus EC / output Delhi VAT @ 12.5%. Compute Output Excise Duty / VAT / Tax Credit / Net Excise duty /VAT. Problem 10: The following particulars are provided by Mr. Karan of Calcutta, who has purchased Raw materials for manufacturing product A and Product B from Mr. Piyush. The State VAT for Raw Materials and other materials was 12.5%. `

Value Added Tax 1. Cost of Raw materials purchased 2. VAT paid to Mr. Piyush 3. Cost of other materials - Local Purchases - Interstate Purchases 4. VAT paid on Local Materials Purchased-12.5% 5. CST Paid @ 2% 6. Manufacturing Expenses 7. Profit Margin (on Sale Value)

86 3,00,000 37,500 50,000 80,000 6,250 1,600 49,200 20%

Mr. Karan utilized and manufactured 75% of production as Product A and 25% of production as Product B. While Product A are subject to 12.5% VAT, product B are exempt. All materials were used in production and there was no closing stock of Raw materials and other materials. What would be the invoice value of Sales charged by Mr. Karan if all the manufactured goods were sold within the State? What would be his liability under VAT?

Problem 11: Bhim, a registered dealer under DVAT /CST Act submits the following information for the month of February, 2012. Particulars Details of purchase Raw material purchased from another State (CST @ 2%). Raw material X purchased within the State Raw material Y imported from Singapore (includes custom duty paid @ 10%) Raw material Z purchased within the State. Details of sales Sale of goods produced from raw material X. Sale of goods produced from inter-State purchase and imported raw materials. Sale of goods produced from raw material Z. Amount ` 12,00,000 18,00,000 13,00,000 8,00,000 30,00,000 34,00,000 12,00,000 Rate of VAT 1% 12.5% 4% 1% 12.5%

Note: The purchase and sales figures given above do not include VAT/CST. Assume that there was no opening or closing inventory. Compute the amount of Value Added Tax (VAT) payable by Bhim for the month of February, 2012. Problem 12: Vijay Co., furnishes you the following information: Raw material purchased `9,00,000 plus VAT @ 4%.

Value Added Tax Manufacturing expenses (revenue nature) `3,00,000. Sale price `15,00,000 plus VAT @ 4% Plant & machinery acquired `4,00,000 plus VAT @ 4%. Compute VAT liability under (i) Gross Product Variant. (ii) Consumption Variant. State which variant is beneficial to the dealer?

87

Problem 13: The following are details of purchases, sales, etc. effected by Kapil & Co., a registered dealer, for the year ended 31.03.2012: Particulars Amount (`) Purchase of raw materials within State, 1000 units, inclusive of VAT levy at 6% 7,42,000 Inter-State purchase of raw materials, inclusive of CST at 2% Import of raw materials, inclusive of basic customs duty plus education cess of `38,050 Capital goods purchased on 01.05.2011, inclusive of VAT levy at 10% (input credit to be spread over 2 financial years) Other manufacturing expenses Sale of taxable goods within State, inclusive of VAT levy at 4% Sale of goods within State, exempt from levy of VAT (Goods were manufactured from the Inter-State purchase of raw materials) Closing stock as on 31.03.2012 was 100 units of raw materials purchased within the State Input credit is allowed only on raw material used in manufacture of the taxable goods. Compute the VAT liability of the dealer for the year ended 31.03.2012. Problem 14: Compute net VAT liability of Rahul from the following information: Particulars Raw materials from foreign market (Including Basic Custom duty @ 20% plus EC) Raw material purchased from local market Cost of raw material Add: Excise duty @ 16% Add: VAT @ 4% Raw material purchased from neighbouring State (Includes CST @ 2%) Storage and transportation cost ` 3,50,000 56,000 4,06,000 16,240 ` 1,36,500 3,06,000 5,35,000 5,50,000 2,00,000 10,40,000 2,00,000

4,22,240 56,100 14,000

Value Added Tax

88

Manufacturing expenses 36,000 Rahul sold goods to Sohan and earned profit @ 12% on the cost of production. VAT rate on sale of such goods is 4%. Problem 15: Compute the VAT liability of Mr. Pankaj Ahuja for the month of October, 2011, using the Invoice method of computation of VAT. Purchases from the local market (Includes VAT 4%) Storage cost incurred Transportation cost Goods sold at a margin of 5% on the cost of such goods VAT rate on sales 12.5%. `81,120 ` 2,800 ` 3,200

SOLUTIONS
TO

PRACTICE PROBLEMS
Solution 1: Purchase price Add: Expenses Add: Profit Amount to be billed Add: VAT @ 12.5% - Output Tax Total invoice value VAT Payable VAT charged in the invoice Output Tax Less: VAT credit on input 12.5% of `2,00,000 Input Tax Net VAT Payable Solution 1(b): Purchase price Add: Expenses Add: Profit Amount to be billed Add: CST @ 2% - Output Tax Total invoice value ` 2,00,000 18,000 35,000 2,53,000 31,625 2,84,625 31,625 (25,000) 6,625 ` 2,00,000 18,000 35,000 2,53,000 5,060 2,58,060

Value Added Tax VAT Payable CST charged in the invoice Output Tax Less: VAT credit on input 12.5% of `2,00,000 Input Tax Balance VAT Credit (to be carried forward or refund can be taken) Solution 2: Purchase price Less: Input tax (5,62,500 x 12.5 / 112.5) Purchase price net of tax Add: Profit (5,00,000 x 20%) Amount to be billed Add: VAT @ 12.5% - Output Tax Total invoice value VAT Payable Output tax Less: Tax credit (12.5% of `5,00,000) Input Tax Net VAT Payable Solution 2(b): Purchase price Less: Input tax (5,62,500 x 12.5 / 112.5) Purchase price net of tax Add: Profit (5,00,000 x 20%) Amount to be billed Add: CST @ 12.5% - Output Tax Total invoice value CST Payable Output tax Less: Tax credit (12.5% of `5,00,000) Input Tax Net CST Payable Solution 3: Computation of VAT payable: Purchase from Punjab and sold in Delhi Cost Add: Central sales tax @ 2% Purchase price Sale Add: VAT @ 12.5% - Output tax Selling Price Purchase from Delhi and sold in M.P. Cost Add: VAT @ 12.5% - VAT credit Purchase price Sale Add: Central sales tax @ 2% - Output tax

89

5,060 (25,000) 19,940 ` 5,62,500 (62,500) 5,00,000 1,00,000 6,00,000 75,000 6,75,000 75,000 (62,500) 12,500 ` 5,62,500 (62,500) 5,00,000 1,00,000 6,00,000 75,000 6,75,000 75,000 (62,500) 12,500

` 4,50,000 9,000 4,59,000 6,50,000 81,250 7,31,250 ` 5,50,000 68,750 6,18,750 7,50,000 15,000

Value Added Tax Selling Price State VAT Less: VAT credit Net State VAT payable CST Payable Solution 4: Computation of VAT payable: Raw material R1 Cost Add: Central sales tax @ 2% Purchase price Raw material R2 Purchase price Less: Input tax (4,00,000 x 10 / 110) Cost of final product Raw material R1 Raw material R2 Processing charges Profit margin Total Goods sold in Delhi Assessable value Add: VAT @ 10% - Output tax Sales value Goods sold in M.P. Assessable value Add: Central sales tax @ 2% - Output tax Sales value State VAT Less: VAT credit Net State VAT payable CST Payable Solution 5(a): Manufacturer (Mr. X) Sale price Add: VAT @ 12.5% Total invoice value VAT A/C: Output tax Less: Tax credit Net tax payable Distributor (Mr. B)

90 7,65,000 81,250 68,750 12,500 15,000 ` 3,00,000 6,000 3,06,000 4,00,000 36,364 3,63,636 3,06,000 3,63,636 5,00,000 5,00,000 16,69,636 8,34,818 83,482 9,18,300 8,34,818 16,696 8,51,514 83,482 36,364 47,118 16,696 ` 6,70,000 83,750 7,53,750 83,750 Nil 83,750

Value Added Tax Purchase price Add: Profit Amount to be billed Add: VAT @ 12.5% Total invoice value VAT A/C: Output tax Less: Tax credit Net tax payable Wholesaler (Mr. K) Purchase price Add: Profit Amount to be billed Add: VAT @ 12.5% Total invoice value VAT A/C: Output tax Less: Tax credit Net tax payable Retailer Purchase price Add: Profit Amount to be billed Add: VAT @ 12.5% Total invoice value VAT A/C: Output tax Less: Tax credit Net tax payable Solution 5(b): Manufacturer (Mr. X) Sale price Output tax (6,70,000 x 12.5 % / 112.5%) VAT A/C: Output tax Less: Tax credit Net tax payable Distributor (Mr. B) Sale price Output tax (9,60,000 x 12.5% / 112.5%) VAT A/C: Output tax Less: Tax credit

91 6,70,000 2,90,000 9,60,000 1,20,000 10,80,000 1,20,000 83,750 36,250 9,60,000 1,80,000 11,40,000 1,42,500 12,82,500 1,42,500 1,20,000 22,500 11,40,000 1,60,000 13,00,000 1,62,500 14,62,500 1,62,500 1,42,500 20,000 ` 6,70,000 74,444 74,444 Nil 74,444 9,60,000 1,06,667 1,06,667 74,444

Value Added Tax Net tax payable Wholesaler (Mr. K) Sale price Output tax (11,40,000 x 12.5% / 112.5%) VAT A/C: Output tax Less: Tax credit Net tax payable Retailer Sale price Output tax (13,00,000 x 12.5% / 112.5%) VAT A/C: Output tax Less: Tax credit Net tax payable Solution 5(c): (i) VAT is exclusive Manufacturer (Mr. X) Sale price Add: VAT @ 12.5% Total invoice value VAT A/C: Output tax Less: Tax credit Net tax payable Distributor (Mr. B) Purchase price Add: Profit Amount to be billed Add: CST @ 2% Total invoice value VAT A/C: Output tax Less: VAT credit to the extent of tax payable Tax Payable Net VAT credit Wholesaler (Mr. K) Purchase price Add: Profit Amount to be billed Add: VAT @ 12.5% Total invoice value

92 32,223 11,40,000 1,26,667 1,26,667 1,06,667 20,000 13,00,000 1,44,444 1,44,444 1,26,667 17,777 ` 6,70,000 83,750 7,53,750 83,750 Nil 83,750 6,70,000 2,90,000 9,60,000 19,200 9,79,200 19,200 19,200 NIL 64,550 9,79,200 1,60,800 11,40,000 1,42,500 12,82,500

Value Added Tax VAT A/C: Output tax Less: Tax credit Net tax payable Retailer Purchase price Add: Profit Amount to be billed Add: VAT @ 12.5% Total invoice value VAT A/C: Output tax Less: Tax credit Net tax payable Solution: (ii) VAT is inclusive Manufacturer (Mr. X) Sale price Output tax (6,70,000 x 12.5 % / 112.5%) VAT A/C: Output tax Less: Tax credit Net tax payable Distributor (Mr. B) Sale price Output tax (9,60,000 x 2% / 102%) VAT A/C: Output tax Less: VAT credit to the extent of tax payable Tax Payable Net VAT credit Wholesaler (Mr. K) Sale price Output tax (11,40,000 x 12.5% / 112.5%) VAT A/C: Output tax Less: Tax credit Net tax payable Retailer Sale price Output tax (13,00,000 x 12.5% / 112.5%) VAT A/C: Output tax Less: Tax credit Net tax payable

93 1,42,500 Nil 1,42,500 11,40,000 1,60,000 13,00,000 1,62,500 14,62,500 1,62,500 1,42,500 20,000 ` 6,70,000 74,444 74,444 Nil 74,444 9,60,000 18,824 18,824 18,824 NIL 55,620 11,40,000 1,26,667 1,26,667 Nil 1,26,667 13,00,000 1,44,444 1,44,444 1,26,667 17,777

Value Added Tax

94

Solution 5(d): No VAT credit is allowed to manufacturer, distributor, wholesaler and retailer because goods purchased from other states. ` Manufacturer (Mr. X) Sale price Add: CST @ 2% Total invoice value VAT A/C: Output tax Less: Tax credit Net tax payable Distributor (Mr. B) Purchase price Add: Profit Amount to be billed Add: CST @ 2% Total invoice value VAT A/C: Output tax Less: Tax credit Net tax payable Wholesaler (Mr. K) Purchase price Add: Profit Amount to be billed Add: CST @ 2% Total invoice value VAT A/C: Output tax Less: Tax credit Net tax payable Retailer Purchase price Add: Profit Amount to be billed Add: VAT @ 12.5% Total invoice value VAT A/C: Output tax Less: Tax credit Net tax payable Solution 6: (i) Purchased Goods A from Delhi 6,70,000 13,400 6,83,400 13,400 Nil 13,400 6,83,400 2,76,600 9,60,000 19,200 9,79,200 19,200 Nil 19,200 9,79,200 1,60,800 11,40,000 22,800 11,62,800 22,800 Nil 22,800 11,62,800 1,37,200 13,00,000 1,62,500 14,62,500 1,62,500 Nil 1,62,500 ` 2,00,000

Value Added Tax Add: VAT @ 4% Purchase Price Cost Add: Profit {2,08,000 8,000(as VAT credit is available)} x 50% Sale Price before VAT Input tax credit Goods sold in Delhi Add: VAT @ 4% Sale Price (ii) Purchased goods B from U.P. Add: Central sales tax @ 2% Purchase Price Add: Profit (4,08,000 x 50%) Sale Price before VAT Input tax credit Goods sold in Delhi Add: VAT @ 12.5% Sale Price (iii) Purchased goods C from Delhi Add: VAT @ 12.5% Purchase Price Cost Add: Profit {9,00,000 1,00,000(as VAT credit is available)} x 50% Sale Price before CST Input tax credit Goods sold in Orissa Add: Central sales tax @ 2% Sale Price (iv) Purchased goods D from Delhi Add: VAT @ 12.5% Purchase Price Cost Add: Profit {11,25,000 1,25,000(as VAT credit is available)} x 50% Sale Price before CST Input tax credit Goods sold in Punjab to unregistered dealer Add: Central sales tax @ 12.5% Sale Price (v) Purchased goods E from Madhya Pradesh Add: Central sales tax @ 1% Purchase Price Add: Profit (6,06,000 x 50%) Sale Price

95 8,000 2,08,000 2,00,000 1,00,000 3,00,000 8,000 3,00,000 12,000 3,12,000 4,00,000 8,000 4,08,000 2,04,000 6,12,000 Nil 6,12,000 76,500 6,88,500

8,00,000 1,00,000 9,00,000 8,00,000 4,00,000 12,00,000 1,00,000 12,00,000 24,000 12,24,000 10,00,000 1,25,000 11,25,000 10,00,000 5,00,000 15,00,000 1,25,000 15,00,000 1,87,500 16,87,500 6,00,000 6,000 6,06,000 3,03,000 9,09,000

Value Added Tax Input tax credit Goods sold in Maharashtra Add: Central sales tax @ 1% Sale Price (vi) Purchased goods F from Delhi Add: VAT @ 1% Purchase Price Cost Add: Profit {14,14,000-14,000(as VAT credit is available)} x 50% Sale Price Input tax credit Goods sold in Maharashtra to unregistered dealer Add: Central sales tax @ 1% Sale Price (vii) Purchased goods G from Delhi Add: VAT @ 12.5% Purchase Price Goods Stock transferred VAT credit allowed in stock transfer (12,00,000 x 10.5%) (in case of stock transfer, VAT credit shall be allowed after retaining 2%) (viii) Purchased goods H from Delhi Add: VAT @ 4% Purchase Price Cost Add: Profit {16,64,000 64,000(as VAT credit is available)} x 50% Sale Price Input tax credit Goods exported (ix) Purchased goods I from Delhi Add: VAT @ 12.5% Purchase Price Cost Add: Profit {20,25,000-2,25,000(as VAT credit is available)} x 50% Sale Price Input tax credit Goods sold to manufacturer in SEZ VAT A/C Particulars OUTPUT TAX Goods A Goods B Goods C Goods D ` VAT 12,000 76,500 ----` CST ----24,000 1,87,500

96 Nil 9,09,000 9,090 9,18,090 14,00,000 14,000 14,14,000 14,00,000 7,00,000 21,00,000 14,000 21,00,000 21,000 21,21,000 12,00,000 1,50,000 13,50,000 12,00,000 1,26,000

16,00,000 64,000 16,64,000 16,00,000 8,00,000 24,00,000 64,000 24,00,000 18,00,000 2,25,000 20,25,000 18,00,000 9,00,000 27,00,000 2,25,000 27,00,000

Value Added Tax Goods E Goods F Goods G (Stock transfer) Goods H (Export) Goods I (Sale to SEZ) LESS: INPUT TAX CREDIT Goods A Goods B Goods C Goods D Goods E Goods F Goods G Goods H Goods I 8,000 Not allowed 1,00,000 1,25,000 Not allowed 14,000 1,26,000 64,000 2,25,000 6,62,000 ----Not applicable Nil Nil 88,500 9,090 21,000 ------2,41,590

97

After adjusting output VAT of `88,500 and CST of `2,41,590, there will be unutilised VAT credit of `3,31,910 and it can be set off from other output tax or it can be carried forward or refund can be claimed but procedure differs from State to State. At the year end it should be shown on the assets side of the balance sheet under the head CURRENT ASSETS, LOAN AND ADVANCES. Computation of Income Tax Liability Particulars Amount Purchases ` Goods A 2,00,000 Goods B 4,08,000 Goods C 8,00,000 Goods D 10,00,000 Goods E 6,06,000 Goods F 14,00,000 Goods H 16,00,000 Goods I 18,00,000 Net profit 39,07,000 1,17,21,000 Income under the head Business/Profession Gross Total Income Less: Deduction u/s 80C to 80U Total Income Tax on `39,07,000 at slab rate Add: Education cess @ 2% Add: SHEC @ 1% Tax Liability Rounded off u/s 288B Particulars Sales Goods A Goods B Goods C Goods D Goods E Goods F Goods H Goods I Amount ` 3,00,000 6,12,000 12,00,000 15,00,000 9,09,000 21,00,000 24,00,000 27,00,000 1,17,21,000 39,07,000.00 39,07,000.00 Nil 39,07,000.00 10,24,100.00 20,482.00 10,241.00 10,54,823.00 10,54,820.00

Income shall be computed exclusive of VAT & CST because any VAT & CST collected shall be paid to the Government and it will not be considered to be income. Similarly VAT paid by the dealer is collected from the customer hence it will not be considered to be expense. Further, the stock transfer of goods G is having a neutral effect and thus ignored for calculation of business/profession income.

Value Added Tax Solution 7: Raw material R1 Assessable value Excise duty @ 8% EC @ 2% SHEC@ 1% Total Delhi VAT @ 10% - Input Tax Purchase Price Raw material R2 Assessable value Excise duty @ 4% EC @ 2% SHEC@ 1% Total Central Sales tax @ 2% - Input Tax Purchase Price Raw material R3 Assessable value Excise duty @ 8% EC @ 2% SHEC@ 1% Total Delhi VAT @ 10% - Input Tax Purchase Price Cost of Final Product Raw material - R1 Raw material - R2 Raw material - R3 Processing charges Payment for Services Profit Assessable value (as per section 4 of Central Excise Act, 1944) Excise duty @ 16% EC @ 2% SHEC@ 1% Total Delhi VAT @ 10% - Output Tax

98

1,50,000 12,000 240 120 1,62,360 16,236 1,78,596 2,20,000 8,800 176 88 2,29,064 4,581 2,33,645 4,50,000 36,000 720 360 4,87,080 48,708 5,35,788 1,50,000 2,24,581 4,50,000 6,00,000 2,00,000 50,000 16,74,581 2,67,933 5,359 2,679 19,50,552 1,95,055 21,45,607 EC @ 2% SHEC @ 1% ` ` 240.00 120.00 176.00 88.00 720.00 360.00 400.00 200.00 1,536.00 768.00 Delhi VAT ` 16,236.00

CENVAT/VAT ACCOUNT
(Invoice 1) Raw material R1 (Invoice 2) Raw material R2 (Invoice 3) Raw material R3 Service tax Total Final product Excise Duty / Service Tax ` 12,000.00 8,800.00 36,000.00 20,000.00 76,800.00

48,708.00 64,944.00

Value Added Tax Output tax Less: VAT/CENVAT Credit Net tax payable Solution 8: Computation of VAT payable Raw material R1 Purchase price Add: Excise duty @ 10% Add: Education cess @ 2% Add: SHEC @ 1% Add: Delhi VAT @ 10% Raw material R2 Purchase price 4,00,000.00 Add: Excise duty @12% Add: Education cess @ 2% Add: SHEC @1% Add: CST @ 2% Capital goods Purchase price 12,00,000.00 Add: Excise duty @10% Add: Education cess @ 2% Add: SHEC @1% Add: Delhi VAT @ 4% Services Service Tax @ 10% Add: Education cess @ 2% Add: SHEC @1% Cost of final product Raw material R1 Raw material R2 Capital goods (12,00,000 @ 20%) Services Other processing charges Profit Assessable Value Add: Excise duty @12% Add: Education cess @ 2% Add: SHEC @ 1% Add: Delhi VAT @ 10% CENVAT/VAT ACCOUNT 2,67,933.00 (76,800.00) 1,91,133.00 5,359.00 (1,536.00) 3,823.00 2,679.00 (768.00) 1,911.00

99 1,95,055.00 (64,944.00) 1,30,111.00 ` 3,00,000.00 30,000.00 600.00 300.00 3,30,900.00 33,090.00 3,63,990.00

48,000.00 960.00 480.00 4,49,440.00 8,988.80 4,58,428.80

1,20,000.00 2,400.00 1,200.00 13,23,600.00 52,944.00 13,76,544.00 4,00,000.00 40,000.00 800.00 400.00 4,41,200.00 3,00,000.00 4,08,988.80 2,40,000.00 4,00,000.00 6,00,000.00 4,00,000.00 23,48,988.80 2,81,878.66 5,637.57 2,818.78 26,39,323.81 2,63,932.38

Value Added Tax Excise Duty / Service Tax Raw material R1 Raw material R2 Plant and machinery Service tax Total Final product Output tax Less: VAT/CENVAT Credit Net tax payable Rounded Off 30,000 48,000 1,20,000 40,000 2,38,000 2,81,878.66 2,38,000.00 43,878.66 43,879.00 EC @ 2% 600 960 2,400 800 4,760 5,637.57 4,760.00 877.57 878.00 SHEC @ 1% 300 480 1,200 400 2,380 2,818.78 2,380.00 438.78 439.00

100 Delhi VAT / CST 33,090 52,944 86,034 2,63,932.38 86,034.00 1,77,898.38 1,77,898.00

Solution 8(b): In this case the manufacturer shall charge central sales tax on the sale instead of Delhi Value Added Tax Act. Since CST shall also be paid to the Delhi Government, VAT credit shall be allowed in the normal manner and it can be adjusted against output CST and tax treatment shall be as given below: Computation of VAT payable Raw material R1 Purchase price Add: Excise duty @ 10% Add: Education cess @ 2% Add: SHEC @ 1% Add: Delhi VAT @ 10% Raw material R2 Purchase price 4,00,000.00 Add: Excise duty @12% Add: Education cess @ 2% Add: SHEC @1% Add: CST @ 2% Capital goods Purchase price 12,00,000.00 Add: Excise duty @10% Add: Education cess @ 2% Add: SHEC @1% Add: Delhi VAT @ 4% Services Service Tax @ 10% Add: Education cess @ 2% Add: SHEC @1% Cost of final product

3,00,000.00 30,000.00 600.00 300.00 3,30,900.00 33,090.00 3,63,990.00

48,000.00 960.00 480.00 4,49,440.00 8,988.80 4,58,428.80

1,20,000.00 2,400.00 1,200.00 13,23,600.00 52,944.00 13,76,544.00 4,00,000.00 40,000.00 800.00 400.00 4,41,200.00

Value Added Tax Raw material R1 Raw material R2 Capital goods (12,00,000 @ 20%) Services Other processing charges Profit Assessable Value Add: Excise duty @12% Add: Education cess @ 2% Add: SHEC @ 1% Add: CST @ 2%- output tax CENVAT/VAT ACCOUNT Excise Duty / Service Tax EC @ 2% Raw material R1 Raw material R2 Plant and machinery Service tax Total Final product Output tax Less: VAT/CENVAT Credit Net tax payable Net credit balance Rounded Off 30,000 48,000 1,20,000 40,000 2,38,000 2,81,878.66 2,38,000.00 43,878.66 43,879.00 600 960 2,400 800 4,760 5,637.57 4,760.00 877.57 878.00 SHEC @ 1% 300 480 1,200 400 2,380 2,818.78 2,380.00 438.78 439.00

101 3,00,000.00 4,08,988.80 2,40,000.00 4,00,000.00 6,00,000.00 4,00,000.00 23,48,988.80 2,81,878.66 5,637.57 2,818.78 26,39,323.81 52,786.47 Delhi VAT 33,090 52,944 86,034 52,786.47 86,034.00 33,247.53 33,248.00

Solution 8 (c): Since the goods have been exported, there will not be any output tax and cenvat credit/ VAT credit shall be refunded. CENVAT/VAT ACCOUNT Excise Duty / Service Tax EC @ 2% SHEC @ Delhi VAT 1% Raw material R1 30,000 600 300 33,090 Raw material R2 48,000 960 480 Plant and machinery 1,20,000 2,400 1,200 52,944 Service tax 40,000 800 400 Total 2,38,000 4,760 2,380 86,034 Output tax Nil Nil Nil Nil Solution 9: Raw material Excise duty @ 10% EC @ 2% SHEC@ 1% Total Delhi VAT @ 4% (i) ` 12,00,000 1,20,000 2,400 1,200 13,23,600 52,944 13,76,544

Value Added Tax

102

Since 1/6th of the stock has been transferred to some other state, VAT credit allowed for such stock transfer shall be 2% (4% - 2%). Tax paid (52,944 x 1/6) 8,824 VAT credit (4% - 2%) 4,412 (In case of stock transfer VAT credit is allowed after retaining 2%) (ii) Raw material 2,00,000 VAT (`52,944 x 1/6) 8,824 Processing charges 1,00,000 Assessable value 3,08,824 Excise duty @ 10% 30,882 EC @ 2% 618 SHEC@ 1% 309 Total 3,40,633 VAT Nil (Since output VAT is exempt, hence VAT credit for input VAT is not allowed and it will be added in the cost) (iii) Raw material Excise duty @ 10% EC @ 2% SHEC@ 1% Processing charges Total Delhi VAT @ 12.5% 2,00,000 20,000 400 200 1,00,000 3,20,600 40,075 3,60,675 (Since output excise duty is exempt, Cenvat credit for input excise duty is not allowed and it will be added in the cost) (iv) No tax is payable in case of export but Tax credit will be allowed. (v) Raw material Processing charges Assessable value Excise duty @ 10% EC @ 2% SHEC@ 1% Total Central sales tax @ 2% (vi) Raw material Processing charges Assessable value Excise duty @ 10% EC @ 2% SHEC @ 1% 2,00,000 1,00,000 3,00,000 30,000 600 300 3,30,900 6618 3,37,518 2,00,000 1,00,000 3,00,000 30,000 600 300

Value Added Tax Total Delhi VAT @ 12.5%

103 3,30,900 41,363 3,72,263

Excise Duty EC @ 2% SHEC @ 1% DVAT CST Particulars ` ` ` ` ` OUTPUT TAX 1/6th raw material (Stock transfer) th 1/6 Final Product (Exempt from 30,882 618 309 sales tax) 1/6th Final Product (Exempt from 40,075 excise duty) 1/6th Final product Exported 1/6th Final product sold in some other 30,000 600 300 6,618 State 1/6th Final product sold in same State 30,000 600 300 41,363 Total 90,882 1,818 909 81,438 6,618 Less: INPUT TAX CREDIT 1/6th raw material (Stock transfer) 20,000 400 200 4,412 th 1/6 Final Product (Exempt from 20,000 400 200 sales tax) 1/6th Final Product (Exempt from 8,824 excise duty) 1/6th Final product Exported 20,000 400 200 8,824 1/6th Final product sold in some other 20,000 400 200 8,824 State 1/6th Final product sold in same State 20,000 400 200 8,824 Total 1,00,000 2,000 1,000 39,708 Tax payable 41,730 6,618 VAT / CENVAT Credit Balance 9,118 182 91 Cenvat credit for excise duty and service tax shall be refundable only in case of exports, otherwise its carry forward is allowed. Solution 10: Computation of Turnover and VAT Cost of Raw Material Purchased VAT @ 12.5% 9,375.00 Other material Local Purchases VAT @ 12.5% Other Material - Interstate Purchases Manufacturing expenses Cost of Product Selling Price (3,60,600 x 100% / 80%) VAT @ 12.5% Invoice value of sale Computation of VAT Payable Taxable 75% 2,25,000.00 --37,500.00 --61,200.00 36,900.00 3,60,600.00 4,50,750.00 56,433.75 5,07,093.75 Exempt 25% 75,000.00 12,500.00 1,562.50 20,400.00 12,300.00 1,31,137.50 1,63,921.88 Nil 1,63,921.88

Value Added Tax Output VAT Less: Input VAT credit Raw Material (37,500 x 75%) Other raw Material (6,250 x 75%) Net VAT Payable Rounded off Solution 11:

104 56,433.75 28,125.00 4,687.50 23,621.25 23,621.00 `

Computation of VAT payable by Bhim for the month of February 2012 Raw material purchased from another State Purchase Price Add: CST @ 2% Total purchase price Raw material X purchased within the State Purchase Price Add: VAT @ 1% Raw material Y imported from Singapore Purchase Price Raw material Z purchased within the State Purchase Price Add: VAT @ 12.5% Sale of goods produced from raw material X. Sale Price Add: VAT @ 4% Sale of goods produced from inter-State purchase and imported raw materials. Sale Price Add: VAT @ 1% Sale of goods produced from raw material Z. Sale Price Add: VAT @ 12.5% Net Tax payable Output tax (1,20,000 + 34,000 + 1,50,000) Less: Tax credit (18,000 + 1,00,000) Net tax payable Solution 12:

12,00,000 24,000 12,24,000 18,00,000 18,000 13,00,000 8,00,000 1,00,000 30,00,000 1,20,000 34,00,000 34,000 12,00,000 1,50,000 3,04,000 1,18,000 1,86,000 `

(i) Gross Product Variant Raw material purchased Add: VAT @ 4% Sale price Add: VAT @ 4%

9,00,000 36,000 15,00,000 60,000

Value Added Tax Plant and machinery Purchase price Add: VAT @ 4% Net tax payable Output tax Less: Tax credit on raw material Net tax payable (ii) Consumption Variant Net tax payable Output tax Less: Tax credit on raw material Less: Tax credit on plant and machinery Net tax payable Consumption Variant is beneficial to the dealer Solution 13: Computation of VAT liability of Kapil & Co. for the year ended 31.03.2012:Particulars Input tax credit: Intra-State purchases of 1000 units of raw materials

105 4,00,000 16,000 60,000 36,000 24,000

60,000 36,000 16,000 8,000

Amount (`)
7,42,000 6 106

42,000 Inter-State purchases of raw materials Import of raw materials Purchase of Capital Goods

--5,50,000 10 110 2

25,000 Other manufacturing expenses Total input tax credit available : Output VAT payable: Sale of taxable goods within State [(10,40,000 x 4)/104] Sale of exempted goods within State [Refer Note 2] VAT credit to be carried forward (40,000 67,000)

-67,000 40,000 -(27000)

Notes:1. VAT paid on purchase of capital goods is eligible for input tax credit. However, the same has to be spread over a period of two years. 2. VAT system allows credit in respect of purchases made during a period to be set-off against the taxable sales during that period, irrespective of when the supplies/inputs purchased are utilized/sold. Therefore, input tax credit in respect of closing stock of raw materials need not be reduced from total input tax credit available.

Value Added Tax

106

Note: The statement in the question, Input credit is allowed only on raw materials used in manufacture of the taxable goods, implies that the same is not allowable in respect of sale of goods within the State which are exempt from levy of VAT. Solution 14: Computation of VAT liability of Rahul:Particulars Raw materials purchased from foreign market (including basic custom duty @ 20% plus EC) Raw material purchased from local market:Cost of raw material Add: Excise duty @ 16% Raw material purchased from neighbouring State (including CST @ 2%) Storage and transportation cost Manufacturing expenses Cost of production Add: Profit @ 12% of cost of production Sale Price VAT @ 4% on `7,26,432 Net VAT liability of Rahul:VAT on sale price 29,057 Less: Input tax credit Basic custom duty paid on imports CST paid on inter-state purchases VAT paid on local purchases Net VAT payable by Rahul 12,817 ` ` 1,36,500

3,50,000 56,000

4,06,000 56,100 14,000 36,000 6,48,600 77,832 7,26,432 29,057

Nil Nil 16,240

Solution 15: Computation of VAT Liability of Mr. Pankaj Ahuja for the month of October 2011 using 'invoice method' of computation of VAT: Particulars ` Purchase price (including VAT @ 4%) 81,120 Less: VAT paid on purchases (81,120 4 / 104) 3,120 Add: Storage cost 2,800 Add: Transportation cost 3,200 Cost Price 84,000 Add: Profit @ 5% of cost price 4,200 Sale price before VAT 88,200 VAT @ 12.5% (` 88,200 12.5%) 11,025 Less: VAT paid on purchases 3,120 VAT Liability of Mr. Pankaj Ahuja 7,905

Value Added Tax

107

EXAMINATION QUESTIONS
IPCC NOV 2011
Question 1 (5 Marks) Laxman, a registered dealer under DVAT /CST Act submits the following information for the month of February, 2012. Particulars Details of purchase Raw material purchased from another State (CST @ 2%). Raw material X purchased within the State Raw material Y imported from Singapore (includes custom duty paid @ 10%) Raw material Z purchased within the State. Details of sales Sale of goods produced from raw material X. Amount ` 10,00,000 15,00,000 11,00,000 6,00,000 27,00,000 Rate of VAT 1% 12.5% 4%

Value Added Tax Sale of goods produced from inter-State purchase and imported raw materials. Sale of goods produced from raw material Z. 32,00,000 8,00,000

108 1% 12.5%

Note: The purchase and sales figures given above do not include VAT/CST. Assume that there was no opening or closing inventory. Compute the amount of Value Added Tax (VAT) payable by Laxman for the month of February, 2012. (Modified) Answer: ` Computation of VAT payable by Laxman for the month of February 2012 Raw material purchased from another State Purchase Price 10,00,000 Add: CST @ 2% 20,000 Total purchase price 10,20,000 Raw material X purchased within the State Purchase Price Add: VAT @ 1% Raw material Y imported from Singapore Purchase Price Raw material Z purchased within the State Purchase Price Add: VAT @ 12.5% Sale of goods produced from raw material X. Sale Price Add: VAT @ 4% Sale of goods produced from inter-State purchase and imported raw materials. Sale Price Add: VAT @ 1% Sale of goods produced from raw material Z. Sale Price Add: VAT @ 12.5% Net Tax payable Output tax (1,08,000 + 32,000 + 1,00,000) Less: Tax credit (15,000 + 75,000) Net tax payable 15,00,000 15,000 11,00,000 6,00,000 75,000 27,00,000 1,08,000 32,00,000 32,000 8,00,000 1,00,000 2,40,000 90,000 1,50,000

Question 2 (4 Marks) Explain the role of chartered Accountants in proper compliance of VAT. (Any 4 points). Answer. Under the VAT system, trust has been reposed on tax payers, as there will be no regular assessment of all VAT returns, but only a few VAT returns will be taken up for scrutiny assessment. In other cases, the return filed by the trader will be accepted. It will not be also seen whether proper records have been maintained by the trader. As a consequence, a check on compliance becomes essential. Chartered Accountants can ensure tax compliance by:-

Value Added Tax (i) helping the client in systematic record keeping; (ii) helping the client in interpretation of the provisions of VAT law, and (iii) performing audit of VAT accounts. (iv) reporting the under-assessment, if any, made by the dealer requiring additional payment or (v) reporting any excess payment of tax warranting refund to the tax payers.

109

Question 3 (4 Marks) State any two benefits and two drawbacks for a dealer who opts for composition scheme under VAT as per White Paper. Answer: Benefits Drawback 1) Dealer opting for the scheme is not required to 1) Dealer will not be able to take the credit of the vat maintain lengthy records. paid by him on purchases. 2) It reduces the administrative cost to the dealer. 2) Dealer cannot pass on the credit of VAT, which will increase the cost of the product and adversly effect the business. Question 4 (4 Marks) State with reasons whether the following are true or false in the context of VAT as per White Paper: (i) No declaration form is prescribed under VAT system. (ii) Taxpayers Identification Number (TIN) is a 10 digit alpha numeral. (iii) Self assessment concept on deemed basis is one of the important features of VAT. (iv) Set off of input tax credit on capital goods is available only to manufacturers and not to traders. Answer: (i) The statement is true. In view of the fact that a lot of time and energy is wasted by the dealer in getting the declaration forms from the Department, most of earlier forms have been dispensed with. There is no declration form prescribed under VAT. (ii) The statement is false. Taxpayers identification Number (TIN) is a 11 digit numeral. The first two characters represent State Code as used by the Union Ministry of Home Affairs and the next nine characters will be different in different states. (iii) The statement is true. Under VAT, a dealer assesses the VAT liability on his own and submits the return. The dealer is deemed to be self-assesseed on the basis of the return filed by him. If he does not receive any notice proposing departmental audit of his books of account within the time-limit specified in the VAT Act of the respective State. (iv) The statement is false. As per the White Paper on VAT, set off of input tax credit on capital goods is available to both manufacturers and traders. Question 5 (4 Marks)

Value Added Tax

110

Ashok, purchased raw material A for `30,00,000 plus VAT @ 4%. Out of such raw material 60% was used for manufacture of taxable goods and the remaining for manufacture of goods which are exempt from VAT. Another raw material B was purchased for `15,00,000 on which VAT was paid @ 1%. Entire raw material B was used for manufacture of taxable goods only. The entire taxable goods were sold for `50,00,000 plus VAT @ 12.5%. Compute VAT liability of Ashok on the assumption that there was no opening or closing inventory. Note: Ashok is not a dealer who opted for Composition Scheme. Answer: Raw Material A Purchase 30,00,000 Add: VAT @ 4% Raw Material B Purchase 15,00,000 Add: VAT @ 1% Sale price Add: VAT @ 12.5% Net tax payable Output Tax Less: Input tax credit Raw Material A (1,20,000 x 60%) Less: Input tax credit Raw Material B Net Tax Payable (Modified) ` price 1,20,000 price 15,000 50,00,000 6,25,000 6,25,000 72,000 15,000 5,38,000

Question 6 (4 Marks) Briefly explain the benefits of the system of cross-checking under VAT as per White Paper. Answer: System of Cross Checking In the VAT system more emphasis has been laid on self-assessment. Hence, a system of cross-checking is essential. Dealers may be asked to submit the list of sales or purchases above a certain monetary value or to give the dealer-wise list from whom or to whom the goods have been purchased/sold for values exceeding a prescribed monetary ceiling. A cross-checking computerized system is being worked out on the basis of coordination between the tax authorities of the State Government and the authorities of Central Excise and Income-tax to compare constantly the tax returns and set-off documents of VAT system of the States and those of Central Excise and Income-tax. This comprehensive cross-checking system will help reduce tax evasion and also lead to significant growth of tax revenue. At the same time, by protecting the interests of tax-complying dealers against the unfair practices of tax-evaders, the system will also bring in more equal competition in the sphere of trade and industry. Question 7 X Co., furnishes you the following information: (4 Marks)

Value Added Tax Raw material purchased `5,00,000 plus VAT @ 4%. Manufacturing expenses (revenue nature) `2,00,000. Sale price `8,00,000 plus VAT @ 4% Plant & machinery acquired `2,50,000 plus VAT @ 4%. Compute VAT liability under (i) Gross Product Variant. (ii) Consumption Variant. State which variant is beneficial to the dealer? Answer: (i) Gross Product Variant Raw material purchased Add: VAT @ 4% Sale price Add: VAT @ 4% Plant and machinery Purchase price Add: VAT @ 4% Net tax payable Output tax Less: Tax credit on raw material Net tax payable (ii) Consumption Variant Net tax payable Output tax Less: Tax credit on raw material Less: Tax credit on plant and machinery Net tax payable Consumption Variant is beneficial to the dealer

111

` 5,00,000 20,000 8,00,000 32,000 2,50,000 10,000 32,000 20,000 12,000

32,000 20,000 10,000 2,000

PCC NOV 2011


Question 1 (5 Marks) The following particulars are provided by Mr. Prohit of Calcutta, who has purchased Raw materials for manufacturing PVC Cans and PVC Pipes from Mr. Arvind. The State VAT for Raw Materials and other materials was 12.5%. ` 1. Cost of Raw materials purchased 1,00,000 2. VAT paid to Mr. Arvind 3. Cost of other materials - Local Purchases - Interstate Purchases 12,500 20,000 40,000

Value Added Tax 4. VAT paid on Local Materials Purchased-12.5% 5. CST Paid @ 2% 6. Manufacturing Expenses 7. Profit Margin (on Sale Value)

112 2,500 800 39,200 20%

Mr. Prohit utilized and manufactured 75% of production as PVC Cans and 25% of production as PVC Pipes. While PVC Cans are subject to 12.5% VAT, PVC Pipes are exempt. All materials were used in production and there was no closing stock of Raw materials and other materials. What would be the invoice value of Sales charged by Mr. Prohit if all the manufactured goods were sold within the State? What would be his liability under VAT? Answer: Computation of Turnover and VAT Taxable Exempt 75% 25% Cost of Raw Material Purchased 75,000.00 25,000.00 VAT @ 12.5% --3,125.00 Other material Local Purchases 15,000.00 5,000.00 VAT @ 12.5% --625.00 Other Material - Interstate Purchases 30,600.00 10,200.00 Manufacturing expenses 29,400.00 9,800.00 Cost of Product 1,50,000.00 53,750.00 Selling Price (1,50,000 x 100% / 80%) 1,87,500.00 67,187.50 VAT @ 12.5% 23,437.50 Nil Invoice value of sale 2,10,937.50 67,187.50 Computation of VAT Payable Output VAT 23,437.50 Less: Input VAT credit Raw Material (12,500 x 75%) 9,375.00 Other raw Material (2,500 x 75%) 1,875.00 Net VAT Payable 12,187.50 Rounded off 12,188.00 Question 5 (4 Marks) Briefly explain the consumption variant of VAT and reasons for its preference over other variants. Answer. The three variants of VAT are: (a) Gross product variant (b) Income variant (c) Consumption variant. Among the three variants, the consumption variant is most widely used. The reasons are1. It does not affect decisions regarding investment because the tax on capital goods is also set off against the VAT liability. Hence, the system is tax neutral in respect of techniques of production.

Value Added Tax

113

2. The consumption variant is convenient from the point of administrative expediency as it simplifies tax administration by obviating the need to distinguish between capital goods on one hand and consumption goods on the other hand.

IPCC MAY 2011


Question 1 (5 Marks) The following are details of purchases, sales, etc. effected by Vasudha & Co., a registered dealer, for the year ended 31.03.2012: Particulars Amount (`) Purchase of raw materials within State, 1000 units, inclusive of VAT levy at 6% 5,30,000 Inter-State purchase of raw materials, inclusive of CST at 2% Import of raw materials, inclusive of basic customs duty plus education cess of `36,050 Capital goods purchased on 01.05.2011, inclusive of VAT levy at 10% (input credit to be spread over 2 financial years) Other manufacturing expenses Sale of taxable goods within State, inclusive of VAT levy at 4% Sale of goods within State, exempt from levy of VAT (Goods were manufactured from the Inter-State purchase of raw materials) Closing stock as on 31.03.2012 was 100 units of raw materials purchased within the State Input credit is allowed only on raw material used in manufacture of the taxable goods. Compute the VAT liability of the dealer for the year ended 31.03.2012. (Modified) Answer. Computation of VAT liability of Vasudha & Co. for the year ended 31.03.2012:Particulars Input tax credit: Intra-State purchases of 1000 units of raw materials
5,30,000 6 106

2,04,000 4,35,000 3,30,000 1,50,000 7,28,000 1,20,000

Amount (`)

30,000 Inter-State purchases of raw materials Import of raw materials Purchase of Capital Goods

--3,30,000 10 110 2

15,000 Other manufacturing expenses Total input tax credit available : Output VAT payable: Sale of taxable goods within State [(7,28,000 x 4)/104] Sale of exempted goods within State [Refer Note 2]

-45,000 28,000 --

Value Added Tax VAT credit to be carried forward (28,000 45,000)

114 (17000)

Notes:1. VAT paid on purchase of capital goods is eligible for input tax credit. However, the same has to be spread over a period of two years. 2. VAT system allows credit in respect of purchases made during a period to be set-off against the taxable sales during that period, irrespective of when the supplies/inputs purchased are utilized/sold. Therefore, input tax credit in respect of closing stock of raw materials need not be reduced from total input tax credit available.

Note: The statement in the question, Input credit is allowed only on raw materials used in manufacture of the taxable goods, implies that the same is not allowable in respect of sale of goods within the State which are exempt from levy of VAT. Question 2 (4 Marks) Which variant of VAT is most widely used in the world and why? Are some services also included in the VAT net by such countries? Answer. Among the three variants of VAT, the consumption variant is widely used. Several countries of Europe and other countries have adopted this variant. The reasons for preference of this variant are: Firstly, it does not affect decisions regarding investment because the tax on capital goods is also set-off against the VAT liability. Hence, the system is tax neutral in respect of techniques of production. Secondly, the consumption variant is convenient from the point of administrative expediency as it simplifies tax administration by obviating (to remove problem) the need to distinguish between capital goods on the one hand and consumption goods on the other hand. In practice, therefore, most countries use the consumption variant. Also, most VAT countries include many services in the tax base. Since the business gets set-off for the tax on services, it does not cause any cascading effect. Question 3 (4 Marks) M/s. Staruss & Co., a registered dealer under the local VAT law, having stock of goods purchased from outside the State, wishes to opt for the Composition Scheme. Advise him whether the same is possible. Will the VAT chain be broken if the dealer opts for the said scheme? Answer: As per the principles laid down in the White Paper, a dealer desirous of availing the benefits of VAT Composition Scheme should not have stock of the goods purchased from outside the State. Therefore, if the dealer wishes to avail the benefit of the scheme, he must ensure that he does not possess stock of such goods as on the date of exercise of option. Advice is to be tendered on above lines. The selling dealer will not be able to pass on the benefit of the input credit when he opts for the Composition Scheme. A purchasing dealer buying goods from a dealer operating under the Composition Scheme will not get any tax credit for the goods purchased. Hence, as soon as any dealer opts for the Composition Scheme, the VAT chain is broken. Question 4 (4 Marks) What is meant by input tax credit in the context of VAT provisions? How does input tax credit help in achieving the essence of VAT?

Value Added Tax

115

Answer. The tax paid by a registered dealer at the earlier point is called input tax. This amount is adjusted/rebated against the tax payable by the purchasing dealer on his sales. This credit availability is called input tax credit (ITC). It can also be referred to as tax credit on a sale within the State or in the course of inter-State trade or commerce. The essence of VAT is in providing set-off for the tax paid earlier, and this is given effect through the concept of input tax credit/rebate. Thus, input tax credit in relation to any period can be set off by the registered dealer against the amount of his output tax. Question 5 What are the major deficiencies of VAT system in India? Answer: The major deficiencies of VAT system in India are as under: (4 Marks)

(1) There is lack of uniformity in the rates of VAT in different States. Distortion occurs on account of different rates of VAT, Composition scheme, Exemptions, difference in classification of goods, etc. (2) Central Sales Tax is not integrated with the State VAT. Therefore, it is difficult to put the purchases from other States at par with the purchases within the State. Consequently, the advantage of neutrality is confined only for purchases within the State. (3) For complying with the VAT provisions, the accounting cost has increased which may not be commensurate with the benefits to traders and small firms. (4) VAT is paid at various stages and not at last stage. This has increased the requirement of working capital and the interest burden on the same. (5) VAT, being a consumption tax, tends to be regressive since the proportion of income spent on consumption is large for the poor than the rich. (6) As a result of introduction of VAT, the administrative cost to the States has increased. Question 6 (4 Marks) How can a Chartered Accountant help a client in the handling of VAT audit called for by the Department and in conducting external audit of VAT records?

Answer: Handling audit by Departmental Auditors There are audit wings in VAT Departments and certain percentage of dealers are taken up for audit every year on scientific basis. Chartered Accountants can ensure proper record keeping to satisfy the Departmental auditors. The professional expertise of a Chartered Accountant will help him in effectively replying audit queries and sorting out audit objections. External audit of VAT records Under VAT system, trust has been reposed on tax payers as there will be no regular assessment of all VAT returns, but only few returns will be scrutinized. In other cases, returns filed by dealers will be accepted. Thus, a check on compliance becomes necessary. Chartered Accountants can play a very vital role in ensuring tax compliance by audit of VAT accounts. Question 7 (4 Marks)

Value Added Tax

116

Briefly list out the contents of VAT invoice. Answer: VAT legislations of all the States provide for the tax invoice. Generally, the various legislations provide that the tax invoice should have the following contents: (i) the words tax invoice in a prominent place; (ii) name and address of the selling dealer; (iii) registration number of the selling dealer; (iv) name and address of the purchasing dealer; (v) registration number of the purchasing dealer (may not be required under all VAT legislations); (vi) pre-printed or self-generated serial numbers; (vii) date of issue; (viii) description, quantity and value of goods sold; (ix) rate and amount of tax charged in respect of taxable goods; (x) signature of the selling dealer or his regular employee duly authorized by him for such purposes.

PCC MAY 2011


Question 4 (4 Marks) A Manufacturer (Registered Dealer) sold goods to Distributor (Registered Dealer) for `20,000. The Distributor sold the goods to the Wholesaler (Registered Dealer) for `24,000. The Wholesaler sold the goods to the Retailer (Registered Dealer) for `30,000. The Retailer sold the goods to the final consumer for `40,000. The VAT rate is 12.5% which is charged separately. Compute VAT liability under Invoice method. State why this method is preferable?

Answer. Computation of VAT liability under invoice method Particulars Output Tax Tax Credit ` ` Manufacturer sells the goods to distributor. 2,500 -` 20,000 x 12.5% Distributor sells the goods to wholesaler. 3,000 2,500 ` 24,000 x 12.5% Wholesaler sells the goods to a retailer. 3,750 3,000 ` 30,000 x 12.5% Retailer sold the goods to consumer. 5,000 3,750 ` 40,000 x 12.5% Total 14,250 9,250 Net Tax ` 2,500 500 7,50 1,250 5,000

Value Added Tax

117

This method is preferable as the tax is charged at each stage of sales on the entire sales value and the tax paid at the earlier stage is allowed as set off. This method ensures payment of tax at the earlier stage. Question 7 (4 Marks) Briefly state the contents of VAT invoice (any 8 items). Answer: Refer to Answer given in IPCC MAY 2011Question No.7

IPCC NOV 2010


Question 1 (5 Marks) Mr. Rajesh is a registered dealer and gives the following information. You are required to compute the net tax liability and total sales value under Value Added Tax: Rajesh sells his products to dealers in his State and in other States. The profit margin is 15% of cost of production and VAT rate is 12.5% of sales. Intra State purchases of raw material `2,50,000/- (excluding VAT @ 4%) (i) (ii) (iii) (iv) (v) Purchases of raw material from an unregistered dealer `80,000 High seas (import) purchases of raw material are `1,85,000/- (excluding basic custom duty @ 10% and education cess and secondary and higher education cess @ 3%) Purchases of raw materials from other States (excluding CST @ 2%) `50,000 Transportation charges, wages and other manufacturing expenses excluding tax `1,45,000 Interest paid on bank loan `70,000/-. (Modified)

Answer: Computation of net VAT liability and total sales value Intra-State purchases of raw material (excluding VAT ` 10,000) Purchases of raw materials from unregistered dealer High seas purchases of raw materials [Refer Note (i)] Purchase of raw materials from other States [Refer Note (ii)] Transportation charges, wages and manufacturing expenses Cost of production Add : Profit margin 15% Add: VAT @ 12.5% ` 2,50,000 80,000 2,04,055 51,000 1,45,000 7,30,055 1,09,508 8,39,563 1,04,945 9,44,508

Total sales value Computation of VAT liability:` VAT on above sales price @ 12.5% 1,04,945 Less: Set off of VAT on purchases: From high seas Nil From intra-State [Refer Note (iii)] 10,000 From inter-State Nil From unregistered dealer Nil 10,000 Net VAT payable 94,945 Notes: (i) Duty paid on high seas purchases i.e., imports is not a State VAT, so the input tax credit is not available in respect of the same and it is a part of cost of production. (ii) Set-off of tax paid on inter-state purchases is not allowed.

Value Added Tax

118

(iii) Tax on intra-State purchases is `10,000. As credit of the same will be available, it is not included in the cost of production. (iv) Interest on loan has been excluded for calculating the cost of production on the presumption that the loan is availed for purposes other than working capital. (v) It has been assumed that the entire production is sold. Question 2 What record should be maintained under VAT system by a registered dealer? Answer: The following records should be maintained under VAT system: (i) (ii) (iii) (iv) Purchase records; Sales records; VAT account; Separate record of any exempt sale. (4 Marks)

Further, the following records should also be kept and produced to an Officer: (i) (ii) (iii) (iv) (v) (vi) (vii) Copies of invoices issued, in serial number; Copies of all credit & debit notes issued, in chronological order; All purchase invoices, copies of customs entries, receipt for payment of customs duty or tax and credit and debit notes received to be filed chronologically either by date of receipt or under each suppliers name; Details of amount of tax charged on each sale or purchase; Total of the output tax and the input tax in each period and a net total of the tax payable or the excess carried forward, as the case may be, at the end of each month; Details of goods manufactured and delivered from the factory of the taxable person; Details of each supply of goods from the business premises, unless such details are available at the time of supply in invoices issued at, or before, that time. (4 Marks)

Question 3 State the Variants of VAT. Present them in schematic diagram and explain each one briefly. Answer: VAT has following three variants: (a) Gross product variant (b) Income variant (c) Consumption variant These variants are presented in a schematic diagram given below: Different variants of VAT Gross product variant Tax is levied on all sales and deduction for tax paid on inputs excluding capital inputs is allowed. (a) Gross product variant Income variant Tax is levied on all sales with set-off for tax paid on inputs and only depreciation on capital goods.

Consumption variant Tax is levied on all sales with deduction for tax paid on all business inputs (including capital goods).

Value Added Tax

119

The gross product variant allows deductions for taxes on all purchases of raw materials and components, but no deduction is allowed for taxes on capital inputs. That is, taxes on capital goods such as plant and machinery are not deductible from the tax base in the year of purchase and tax on the depreciated part of the plant and machinery is not deductible in the subsequent years. (b) Income variant The income variant of VAT on the other hand allows for deductions on purchases of raw materials and components as well as depreciation on capital goods. This method provides incentives to classify purchases as current expenditure to claim set-off. In practice, however, there are many difficulties connected with the specification of any method of measuring depreciation, which basically depends on the life of an asset as well as on the rate of inflation. (c) Consumption variant Consumption variant of VAT allows for deduction on all business purchases including capital assets. Thus, gross investment is deductible in calculating value added. It neither distinguishes between capital and current expenditures nor specifies the life of assets or depreciation allowances for different assets. Question 4 (4 marks) State with reasons in brief whether the following statements are correct or incorrect with reference to the provision of Value Added Tax. (i) It is permitted to issue tax invoice inclusive of VAT i.e. aggregate sales price & VAT. (ii) A registered dealer is compulsorily required to get its books of accounts audited under VAT Laws of different states irrespective of limit of turnover. Answer: (i) Incorrect Reason: One of the requirements under the contents of the tax invoice is that rate and amount of tax charged in respect of taxable goods should be distinctly shown in the tax invoice, in order to claim input credit. (ii)Incorrect Reason: Different States have determined different turnover limits above which a registered dealer will have to get its books of accounts audited under VAT laws. Question 5 (4 Marks) What are the conditions to be fulfilled by the dealer accepting the composition scheme under the Value Added Tax? Answer: The dealer accepting the composition scheme should fulfill the following conditions: (i) (ii) He should intimate to the Commissioner of VAT in writing that he is opting to such scheme for a year or a part of the year in which he gets himself registered. If he avails this scheme, he is not required to maintain any statutory records as prescribed under the Act. Only the records for purchases, sales and inventory should be maintained.

(iii) The dealer should not have any stock of goods which were brought from outside the State on the date he opts to pay tax under composition scheme and should not use such goods after such date. (iv) The dealer should not claim input tax credit on the inventory available on the date on which he opts for composition scheme. Question 6 (4 Marks) Mention the purchases which are not eligible for input tax credit (any eight items) under Value Added Tax. Answer: The following purchases are not eligible for input tax credit:

Value Added Tax (i) (ii) (iii) (iv) (v) (vi) (vii) (viii) (ix) (x) purchases from unregistered dealer; purchases from a registered dealer who opts for composition scheme; purchases of goods as may be notified by the State Government; purchases of goods where the purchase invoice is not available with the claimant; purchases of goods where invoice does not show the amount of tax separately; purchases of goods which are being utilized in the manufacture of exempted goods; purchases of goods used for personal use or provided free of charge as gifts; imports from outside the territory of India; imports from other States;

120

goods in stock, which have suffered tax under an earlier Act, but under the VAT Act they are covered under exempted items. (Note: Any eight points may be given.)

Question 7 (4 Marks) Compute the VAT amount payable by Mr. Shyam, who purchased goods from a manufacturer on payment of `4,16,000 (including VAT) and earned 20% profit on purchase price. VAT rate on both purchases and sales is 4%. Answer: Computation of VAT payable by Mr. Shyam ` Payment made to manufacturer 4,16,000 Less: VAT paid [(4,16,000/104) x 4] 16,000 Purchase price 4,00,000 Add: Profit margin @ 20% on purchase price 80,000 Sale price before VAT 4,80,000 Add: VAT @ 4% on `4,80,000 19,200 Less: Input credit 16,000 VAT payable by Mr. Shyam 3,200

PCC NOV 2010


Question 1 Compute net VAT liability of Rishi from the following information: Particulars Raw materials from foreign market (Including Basic Custom duty @ 20% plus EC) Raw material purchased from local market Cost of raw material Add: Excise duty @ 16% Add: VAT @ 4% ` 2,50,000 40,000 2,90,000 11,600 (5 Marks) ` 1,23,600

3,01,600

Value Added Tax Raw material purchased from neighbouring State (Includes CST @ 2%) Storage and transportation cost Manufacturing expenses

121 51,000 9,000 30,000

Rishi sold goods to Madan and earned profit @ 12% on the cost of production. VAT rate on sale of such goods is 4%. (Modified) Answer. Computation of VAT liability of Rishi:Particulars ` ` Raw materials purchased from foreign market (including basic custom duty 1,23,600 @ 20% plus EC) Raw material purchased from local market:Cost of raw material 2,50,000 Add: Excise duty @ 16% 40,000 2,90,000 Raw material purchased from neighbouring State (including CST @ 2%) 51,000 Storage and transportation cost 9,000 Manufacturing expenses 30,000 Cost of production 5,03,600 Add: Profit @ 12% of cost of production 60,432 Sale Price 5,64,032 VAT @ 4% on `5,64,032 22,561 Net VAT liability of Rishi:VAT on sale price 22,561 Less: Input tax credit Basic custom duty paid on imports CST paid on inter-state purchases VAT paid on local purchases Net VAT payable by Rishi

Nil Nil 11,600 10,961

Question 7 (4 Marks) List out the merits of VAT. Answer. Merits of VAT 1. No tax evasion is possible as the credit of duty paid is allowed against the liability on the final product manufactured or sold. Under VAT, unless proper records are kept in respect of various inputs, it is not possible to claim credit. A perfect system of VAT is a perfect chain where tax evasion is difficult. 2. Neutrality is the greatest advantage of VAT. VAT does not interfere in the choice of decision for purchases because it has anti-cascading effect. The system is neutral with regard to choice of production technique, as well as business organisation. All other things remaining the same, the issue of tax liability does not vary the decision about the source of purchase. 3. It has a certainty as it is based simply on transactions. There is no need to go through complicated definitions like sales, sales price, turnover of purchases and turnover of sales. The tax is also broad-based and applicable to all sales in business leaving little room for different interpretations. 4. Transparency is ensured as the buyer knows, out of the total amount paid for purchases of material, what is the amount paid towards VAT. This transparency enables the State Governments to know as to what is the

Value Added Tax

122

exact amount of tax coming at each stage. Thus, it is a great aid to the Government while taking decisions with regard to rate of tax etc. 5. For Government, better revenue collection and stability is achieved as the tax credit will be given only if the proof of tax paid at an earlier stage is produced. This means that if the tax is evaded at one stage, full tax will be recoverable from the person at the subsequent stage or from a person unable to produce proof of such tax payment. 6. Since the tax paid on an earlier stage is to be received back, the system promotes better accounting systems. Note: Any four points may be given.

IPCC MAY 2010


Question 7 (4 x 2 = 8 Marks) (a) What are the items aggregated in the Addition method to calculate the VAT payable? When is this method mainly used? (b) Is any threshold exemption limit fixed for dealers to obtain VAT registration, as per the White Paper? If yes, why is the same provided? (c) Is the VAT chain continued when a purchasing dealer opts for VAT composition scheme? What is the loss to the seller and buyer opting for the composition scheme, and the subsequent buyers? (d) Can it be said that VAT brings about certainty to a great extent in the matter of interpretational issues? If so, how? Answer 7(a): (a) In the addition method, (i) All the factor payments (rent, interest, and wages), and (ii) Profit, are added to arrive at the value addition on which VAT rate is applied to compute the VAT payable. Answer 7(b): The threshold limit far small traders, as per the White Paper is `5 lakh. The same was subsequently increased to `10 lakh. The same is fixed to provide relief to small traders. Answer 7(c): As soon as the dealer opts for the composition scheme, the VAT chain is broken. When a composition scheme is availed by a seller or buyer, he cannot claim input credit of the tax paid on the purchases. This will add to the cost of the goods. The benefit of tax paid earlier will not be passed on to subsequent buyers. Answer 7(d): VAT is a system, based simply on transactions; hence there is no need to go through complicated definitions like sales, turnover, etc. The tax is also broad based and is applicable to all sales of the commodity in question, leaving little room for different interpretations. Hence it can be said that VAT brings certainty to a great extent. Question 8 (a) (8 Marks)

Value Added Tax

123

Mr. X, a dealer in Mumbai dealing in consumer goods, submits the following information pertaining (to apply in a particular situation) to the Month of March, 2012: (i) Exempt goods A purchased for `2,00,000 and sold for `2,50,000. (ii) Goods B purchased for `2,25,000 (including VAT) and sold at a margin of 10% profit on purchase (VAT rate 12.5%); (iii) Goods C purchased for `1,00,000 (excluding VAT) and sold for `1,50,000 (VAT rate 4%); (iv) His unutilized balance in VAT input credit on 01.03.2012 was `1,500. Compute the turnover, Input VAT, Output VAT and Net VAT payable by Mr. X. Answer 8(a): Goods ` ` A (Exempt) B (2,25,000 12.5 / 112.5) C Purchases 2,00,000 2,00,000 1,00,000 5,00,000 Input VAT credit ` 25,000 4,000 29,000 ` 2,50,000 2,20,000 1,50,000 6,20,000 27,500 6,000 33,500 ` 33,500 (1,500) (29,000) 3,000 2,25,000 25,000 2,00,000 20,000 2,20,000 27,500 Sales (Turnover) (Modified) Output VAT `

Computation of Net VAT Output VAT Less: Opening balance of Input VAT credit Less: Input VAT credit for March, 2012 Net VAT payable Computation of purchase price and sale price of goods B Goods B purchase value (including VAT) Less: VAT included in above 2,25,000 12.5 / 112.5 Purchase price excluding VAT Add: Profit on above @ 10% Selling price before VAT VAT @ 12.5% on selling price

Question 8(b) (3 x 3 = 9 Marks) (i) What are the merits of VAT in the context of tax evasion, neutrality and transparency? (ii) State the importance of VAT invoice/tax invoice in administering VAT. (iii) Discuss the tax consequences of Stock transfer under the VAT scheme. Answer 8(b)(i): Refer to Answer given in PCC NOV 2010 Question No.7 Answer 8(b)(ii): Invoices are crucial documents for administering VAT. In the absence of invoices VAT paid by the dealer earlier cannot be claimed as set off. A VAT invoice:

Value Added Tax (i) helps in determining the input tax credit and prevents cascading effect of taxes; (ii) facilitates multi-point taxation on the value addition; (iii) promotes assurance of invoices;

124

(iv) assists in performing audit and investigation activities effectively and checks evasion of tax. Answer 8(b)(iii): Inter-State stock transfers do not involve sale and, therefore they are not subjected to sales tax. The same position is continued under VAT. However, the tax paid on: (i) inputs used in the manufacture of finished goods which are stock transferred; or (ii) purchases of goods which are stock transferred is available as input tax credit after retention of 2% of such tax by the State Governments.

PCC MAY 2010


Question 6 (2 Marks) Do you agree with the statement that tax cannot be evaded under VAT system? Answer. The statement that tax cannot be evaded under VAT system is correct. It is said that VAT is a logical beauty. Under VAT, credit for duty paid is allowed against the liability on the final product manufactured or sold. Therefore, unless proper records are kept in respect of various inputs, it is not possible to claim credit. Hence, suppression of purchases or production will be difficult because it will lead to loss of revenue. A perfect system of VAT is a perfect chain where tax evasion is difficult. Question 8 (3 Marks Each) (a) Compute the VAT liability of Mr. P Kapoor for the month of October, 2011, using the Invoice method of computation of VAT. Purchases from the local market (Includes VAT 4%) Storage cost incurred Transportation cost Goods sold at a margin of 5% on the cost of such goods VAT rate on sales 12.5%. `65,000 ` 750 ` 1,750

(b) What are the three variants of VAT? Which of these methods is most widely used and why? Answer 8(a). Computation of VAT Liability of Mr. P. Kapoor for the month of October 2011 using method' of computation of VAT: Particulars Purchase price (including VAT @ 4%) Less: VAT paid on purchases (65,000 4 / 104) Add: Storage cost Add: Transportation cost Cost Price Add: Profit @ 5% of cost price Sale price before VAT VAT @ 12.5% (` 68,250 12.5%) Less: VAT paid on purchases VAT Liability of Mr. P. Kapoor

'invoice ` 65,000 2,500 750 1,750 65,000 3,250 68,250 8,531 2,500 6,031

Value Added Tax Answer 8(b). Refer to Answer given in PCC NOV 2011Question No.5

125

IPCC NOV 2009


Question 7 (2 x 4 = 8 Marks) Answer the following : (a) What are the different rates under VAT system? (b) Under what circumstances registration can be cancelled under VAT? (c) Briefly explain the income variant of VAT. (d) State with reasons in brief whether the following statement is true or false with reference to the provision of Value Added Tax. The VAT rate on sale of Lottery Ticket is 4%. Answer 7(a): To reduce the multiplicity of sales-tax rates between various States in India, it was recommended that VAT will have broadly the following tax rates: (a) Zero rate for tax free goods, (b) 1% on precious or semi-precious metals i.e., bullion etc. (c) 4% on items of basic necessities, agricultural and industrial inputs, capital goods and declared goods (d) 20% on non VAT goods (e) 12.5% on other goods. Answer 7(b): VAT registration can be cancelled on: (i) discontinuance of business; or (ii) disposal of business; or (iii) transfer of business to new location; or (iv) annual turnover falling below the specified limit. Answer 7(c): The income variant of VAT allows deduction of purchases of raw material and components as well as depreciation of capital goods. This method provides incentive to classify purchases as current expenditure to claim set off. In practice, however, there are many difficulties connected with the specification of any method of measuring depreciation, which basically depends on the life of an asset as well as on the rate of inflation. Answer 7(d): False. VAT rate for lottery tickets is different in different States e.g. in Delhi it is 20% Question 8 (8 Marks) Mr. X (Registered Dealer) is a manufacturer sells goods to Mr. B (Registered Dealer), a distributor for `2,000 (excluding of VAT). Mr. B sells goods to Mr. K (Registered Dealer), a wholesale dealer for `2,400. The wholesale dealer sells the goods to a retailer (Registered Dealer) for `3,000, who ultimately sells to the consumers for `4,000. Compute the Tax Liability, input credit availed and tax payable by the manufacturer, distributor, wholesale dealer and retailer under Invoice method assuming VAT rate @ 12.5%. Answer: Computation of tax liability, input tax credit availed and tax payable under invoice method

Value Added Tax Stage 1. 2. Particulars X, the manufacturer, sells to B, the distributor, for `2,000. Therefore his tax liability will be `250 (`2,000 @ 12.5%). He will not have any VAT credit. B, the distributor, sells goods to K, the wholesale dealer, for `2,400. Bs tax liability will be `300 (`2,400 @ 12.5%). He will get set off of tax paid at earlier stage of `250. Thus, tax payable by him will be `50. K, the wholesaler dealer, sells to retailer at `3,000. Ks tax liability will be `375 (`3,000 @ 12.5%). He will get set off of tax paid at earlier stage of `300. Thus, tax payable by him will be `75. Retailer sells goods to consumers at `4,000. His tax liability will be `500 (`4,000 @ 12.5%). He will get set off of tax paid at earlier stage of `375. Thus, tax payable by him will be `125/Output Tax 250 300 Less VAT Credit 250

126 Net Tax 250 50

3.

375

300

75

4.

500

375

125

Note: It has been assumed that sales made by the distributor, the wholesale dealer and the retailer are also exclusive of VAT. Question 8 (3 x 3 = 9 Marks) (i) What are the different stages of VAT? Can it be said that entire burden falls on the final consumer? (ii) Discuss filing of Return under VAT. (iii) List six purchases which are not eligible for input tax credit. Answer 8(i): The Value Added Tax (VAT) is a multistage tax levied (to use official authority to demand and collect a payment, tax) as a proportion of the value added (i.e. sale minus purchase) which is equivalent to wages plus interest, other costs and profits. In an economy, apart from the manufacturers and final consumers, there would be wholesalers and retailers also. VAT is collected at each stage of production and distribution process, and in principle, its entire burden falls on the final consumer, who does not get any tax credit. Thus, VAT is a broad based tax covering the value added to each commodity by parties during the various stages of production and distribution. Answer 8(ii): VAT returns are to be filed monthly/quarterly/half-yearly/annually along with tax paid challans according to the provisions of the State Acts. They should contain details of output tax liability, value of input tax credit and payment of VAT and should be filed within the prescribed time schedule. In case of any mistakes, revised returns may be filed. The returns will be checked and any deficiency in payment of tax may have to be made good. Filing of returns are designed with a view: (i) to reduce cost of compliance (ii) to encourage businesses to comply with their obligations; and (iii) to ensure efficient processing of data.

Value Added Tax Answer 8(iii): Refer to Answer given in IPCC NOV 2010 Question No.6

127

PCC NOV 2009


Question 7 (5 Marks) Mr. Goenka, a Registered Dealer, is selling raw materials to a manufacturer of finished products. He imports (purchases from other states is also called import) his stock in trade as well as purchases the same from the local markets. Following transactions took place during financial year 2011-12. Calculate the VAT and invoice value charged by him to a manufacturer. Assume the rate of VAT @ 12.5%: (1) Cost of imported materials (from other State) excluding tax (2) Cost of local materials including VAT (3) Other expenditure includes storage, transport, interest and loading and unloading and profit earned by him Answer. Sales Price of goods:Particulars Imported material cost Add: CST @ 2% Add: Cost of local materials 2,25,000 Less: VAT @12.5% (2,25,000 x 12.5 / 112.5) 25,000 [Since, credit of ` 25,000 would be available, it will not be included in cost of input] Add: Other expenses and profit Sales Price of goods Add: Output VAT @12.5% Invoice value charged by Mr. Goenka to a manufacturer VAT Payable by Mr. Goenka (48,688 25,000 ) ` 1,00,000 2,25,000 87,500 (Modified)

` 1,00,000.00 2,000.00 2,00,000.00 87,500.00 3,89,500.00 48,688.00 4,38,188.00 23,688.00

Question 8 (3 Marks) VAT would increase the working capital requirements and the interest burden. Discuss. Answer. One of the demerits of VAT is that it increases the working capital requirements and the interest burden. The tax is imposed or paid at various stages and not on last stage only. It increases the requirement of working capital and also the interest element as compared to single stage-last point taxation system.

PCC JUNE 2009


Question 6 (2 Marks) Discuss the word transparency in the context of VAT system. Answer. Out of total consideration paid for purchase of material, the buyer knows the tax component under a VAT system. Thus, the system ensures transparency. This transparency enables the State Government to know as

Value Added Tax

128

to what is the exact amount of tax coming at each stage. Thus, it is a great aid to the Government while taking decisions with regards to rate of tax etc. Question 7 (3 Marks) Compute the VAT amount payable by Mr. A (Registered Dealer) who purchases goods from a Manufacturer (Registered Dealer) on payment of `2,25,000 (including VAT) and earns 10% profit on purchase. The goods have been sold to retailers and VAT rate on purchase and sale is 12.5%. Answer. Computation of VAT payable by Mr. A:Payment made to manufacturer Less: VAT paid (2,25,000 x 12.5)/112.5 Purchase price Add: Profit margin (10% of Cost Price) Sale price before VAT Add: VAT @ 12.5% on ` 2,20,000 Output tax Less: Tax credit VAT payable by Mr. A (Modified) Amount (`) 2,25,000 25,000 2,00,000 20,000 2,20,000 27,500 27,500 25,000 2,500

Question 8 (3 Marks Each ) (a) How can an auditor play role to ensure that the tax payers discharge their tax liability properly under the VAT system. (b) Discuss the subtraction method for computation of VAT. Answer 8(a). Refer to Answer given in IPCC NOV 2011 Question No.2 Answer 8(b). Under the subtraction method, the tax is charged only on the value added at each stage of the sale of the goods. Since, the total value of goods sold is not taken into account, the question of grant of claim for set-off or tax credit does not arise. This method is normally applied where the tax is not charged separately. Under this method for imposing tax, value added is simply taken as the difference between sales and purchases.

PCC NOV 2008


Question 6 (2 Marks) Can we say that levy of VAT will have effect on retail price of goods? Answer. A persistent criticism of the VAT form has been that since the tax is payable on the final sale price, the VAT usually increases the prices of the goods. However, VAT does not have any inflationary impact as it merely replaces the existing equal sales tax. It may also be pointed out that with the introduction of VAT, the tax impact on raw material is to be totally eliminated. Therefore, there may not be any increase in the prices. Question 8 (a) Explain Input Tax Credit in context of VAT. (b) What are the exceptions to input tax credit? Answer 8(a). Refer to Answer given in IPCC MAY 2011 Question No.4 (3 Marks Each)

Value Added Tax Answer 8(b). Input tax credit may not be allowed in the following circumstances: (i) purchases from unregistered dealers; (ii) purchases from registered dealer who opt for composition scheme under the provisions of the Act; (iii) purchase of goods as may be notified by the State Government;

129

(iv) purchase of goods where the purchase invoice is not available with the claimant or there is evidence that the same has not been issued by the selling registered dealer from whom the goods are purported (claimed) to have been purchased; (v) purchase of goods where invoice does not show the amount of tax separately; (vi) purchase of goods, which are being utilized in the manufacture of, exempted goods; (vii) goods in stock, which have suffered tax under an earlier Act but under VAT Act they are covered under exempted items; (viii) purchase of goods used for personal use/consumption or provided free of charge as gifts; (ix) goods imported from outside the territory of India (commonly known as high seas purchases); (x) goods imported from other States viz. (namely) inter-State purchases . Note: Any three points can be given in the above answer.

PCC MAY 2008


Question 6 (2 x 2 = 4 Marks) (a) Briefly explain the income variant of VAT. (b) What is the demerit of VAT from the view point that it is a form of consumption tax? Answer 6(a): Refer to Answer given in IPCC NOV 2009 Question No.7 Answer 6(b). Demerit of VAT VAT is a form of consumption tax. Since the proportion of income spent on consumption is larger for the poor than for the rich, VAT tends to be regressive. However, this weakness is inherent in all the forms of consumption tax. While it may be possible to moderate the distribution impact of VAT by taxing necessities at a lower rate, it is always advisable to moderate the distribution considerations through other programmes rather than concessions or exemptions, which create complications for administration. Question 8 (3 Marks) (a) What are the different stages of VAT? Can it be said that the entire burden falls on the final consumer? (b) Briefly explain, how VAT helps in checking tax evasion and in achieving neutrality. Answer 8(a). Different Stages of VAT

Value Added Tax

130

The Value Added Tax (VAT) is a multistage tax levied as a proportion of the value added (i.e. Sale minus purchase) which is equivalent to wages plus interest, other costs and profits. In an economy, apart from the manufacturers and final consumers, there would be wholesalers and retailers also. The wholesaler might supply to retailer, and each one of them could supply to the manufacturer and the end consumer. VAT will be collected at each stage, and wherever applicable, the manufacturer or retailer will claim input credit. Thus, VAT is collected at each stage of production and distribution process, and in principle, its entire burden falls on the final consumer, who does not get any tax credit. Thus VAT is a broad-based (based on a wide variety of things) tax covering the value added to each commodity by parties during the various stages of production and distribution. Answer 8(b). No Tax Evasion It is said that VAT is a logical beauty. Under VAT, credit of duty paid is allowed against the liability on the final product manufactured or sold. Therefore, unless proper records are kept in respect of various inputs, it is not possible to claim credit. Hence, suppression of purchases or production will be difficult because it will lead to loss of revenue. A perfect system of VAT will be a perfect chain where tax evasion is difficult. Neutrality The greatest advantage of the system is that it does not interfere in the choice of decision for purchases. This is because the system has anti-cascading effect. How much value is added and at what stage it is added in the system of production/distribution is of no consequence. The system is neutral with regard to choice of production technique, as well as business organisation. All other things remaining the same, the issue of tax liability does not vary the decision about the source of purchase. VAT facilitates precise identification and rebate of the tax on purchases and thus ensures that there is no cascading effect of tax. In short, the allocation of resources is left to be decided by the free play of market forces and competition.

PCC NOV 2007


Question 6 (2 x 2 = 4 Marks) (a) Does the VAT system bring certainty to a great extent? (b) Can VAT be said to be non-beneficial as compared to single stage-last point system? Answer 6(a). The VAT is a system based simply on transactions. Thus there is no need to go through complicated definitions like sales, sales price, turnover of purchases and turnover of sales. The tax is also broad-based and applicable to all sales in business leaving little room for different interpretations. Thus, this system brings certainty to a great extent. Answer 6(b). VAT system has many advantages like no tax evasion, transparency, certainty, reduction in cascading effect of taxes etc. However, since the VAT is imposed or paid at various stages and not at last stage, it increases the working capital requirements and the interest burden on the same. In this way, it may be considered to be non-beneficial as compared to the single stage-last point taxation system though to a certain extent, this rigour (problem) can be brought down through input credits on purchases. Question 7 (3 Marks) Compute the invoice value to be charged and amount of tax payable under VAT by a Registered Dealer who had purchased goods for ` 1,20,000 and after adding for expenses of ` 10,000 and of profit ` 15,000 had sold out the same.

Value Added Tax The rate of VAT on purchases and sales is 12.5%. Answer. Computation of Invoice Value Particulars Cost of goods purchased Add: Expenses Add: Profit margin Product Sale Value Add: VAT @ 12.5% Invoice Value Computation of VAT payable VAT charged on sales Less: Input credit of VAT paid on purchases @ 12.5% on 1,20,000 VAT Payable Note: It has been assumed that the purchase price of ` 1,20,000 is exclusive of VAT. ` 10,000 15,000

131 (Modified)

` 1,20,000 25,000 1,45,000 18,125 1,63,125 18,125 15,000 3,125

Question 8 (3 Marks) What are the different variants of VAT and how is deduction available for tax paid on inputs including capital inputs? Answer. Refer to Answer given in IPCC NOV 2010 Question No.6

PCC MAY 2007


Question 6 (2 x 2 = 4 Marks) (a) Which is the most popular and common method for computing VAT liability and at what stage is the tax imposed? (b) Is it correct to state that VAT usually increases the retail price, as the tax is payable on the first sale price? Answer 6(a) . Invoice method is the most common and popular method for computing the tax liability under the VAT system. Under this method, tax is imposed at each and every stage of sales on the entire sale value, and the tax paid at the earlier stage is allowed as set-off. Answer 6(b). The statement is not correct as VAT is a multi-point tax where tax is imposed at each and every stage of sales and tax paid at the earlier stage is allowed as set-off. Question 8 (3 Marks) Briefly explain the invoice method of computing tax liability under the VAT system. What are its other names? Answer. Invoice method is the most common and popular method for computing the tax liability under VAT system. Under this method, tax is imposed at each stage of sales on the entire sale value and the tax paid at the earlier stage is allowed as set-off. In other words, out of tax so calculated, tax paid at the earlier stage i.e., at the stage of purchases is set-off, and at every stage the differential tax is being paid. The most important aspect of this method is that at each stage, tax is to be charged separately in the invoice. This method is very popular in western countries. In India also, under the VAT law as introduced in several Sates and Central Excise Law, this method is followed.

Value Added Tax This method is also called the Tax Credit Method or Voucher Method.

132

FORM DVAT 04 COVER PAGE (See Rule 12 of the Delhi Value Added Tax Rules, 2005)
Application for Registration under Delhi Value Added Tax Act, 2004

Value Added Tax Checklist of Supporting Documents Please tick as applicable Mandatory Supporting Documents

133

Annexures of the Form duly filled in (in case any of the annexures is not applicable, please mention the same ) Proof of incorporation of the applicant dealer i.e. Copy of deed of constitution (partnership deed (if any), certificate of registration under the Societies Act, Trust deed, Memorandum and Articles of Association etc) duly certified by the authorized signatory Proof of identity of authorized signatory signing the Registration Application Form Two self addressed envelopes (Without stamps) In case of a dealer applying for registration and simultaneously opting for payment of tax under composition scheme, please attach application in Form DVAT 01 along with this application Proof of Security along with duly filled form DVAT-12 Optional Supporting Documents (For reduction in Security Amount) Proof of ownership of principle place of business Proof of ownership of residential property by proprietor/ managing partner Copy of passport of proprietor/ managing partner Copy of Permanent Account Number in the name of the business allotted by the Income Tax Department Copy of last electricity bill (The bill should be in the name of the business and for the address specified as the main place of business in the registration form) Copy of last telephone bill (The bill should be in the name of the business and for the address specified as the main place of business in the registration form) Reasons for Rejection (For Office Use Only) Please tick as applicable Not attached Mandatory Supporting Document(s) _________________________ Other____________________________________________________________

FORM DVAT 04
(See Rule 12 the Delhi Value added Tax Rules, 2005)

Value Added Tax Application for Registration under Delhi Value Added Tax Act, 2004 1. Full Name of Applicant Dealer (For individual, provide in order of first name, middle name, surname) 2. Trade Name (if any) 3. Nature of Business (Tick [] all applicable) 4. Constitution of Business (Tick [] all applicable) Manufa cture Tr ader lea sing Work contractor

134

Other(spe cify)

Proprietorship Partnership HUF Others, please Specify Tike [ ] one

Private Ltd. Company

Public Sector Undertaking

Government Company Public Ltd. Company

Government Corporation Govt. Deptt/Society /Club/Trust

5. Type of Registration

Mediatory

Voluntary Yes No

5A. Opting for composition scheme under section 16 of the Act? Tick [] one 6. Annual Turnover Category Tick** one less than ` 10 lacs ` `

` 10 lacs or above

(a) Turnover in preceding financial year (b) Expected turnover in the current Financial year

7 Date from which liable for registration under Delhi Value Added Tax Act,2004 Day 8. Permanent Account Number of the applicant dealer (PAN) 9. Registration number under Central Excise Act (if applicable) 10. Principal Place of Business Building Name/ Number Area/ Road Locality/Market Pin Code Email Id Telephone Number Fax Number

/ Month

/ Year

Value Added Tax 11. Address for Service of Notice (if different from principal place of business) Building Name/ Number Area/ Road Locality/Market Pin Code Email Id Telephone Number Fax Number Godown /Warehouse Factory Shop Other place of business

135

12. Number of additional places of business within or outside the State (also please complete part C)

13. Details of main Bank Account

Account Number MICR Number Name of Bank Address of Bank

14. Details of investment in the business (details should be current as on date of application)

Own Capital (`) Loans from Banks (`) Others loans and borrowing (`) Plant & Machinery (`) Land & Building (`) Others assets & investments (`) Description of items 1 2 3 4 5

15. Description of top 5 items you deal or propose to deal in (1-higest volume to 5-lowest volume)

16. Accounting Basis

Tick [ ] One

Accrual Monthly `

Cash Quartely

17. Frequency of filing of returns (to be filled in by the dealer whose turnover is less than ` 5 crores in the preceeding year) Tick One [ ] if appicable 18. Security (a) Amount of Security

Value Added Tax (b) Type of Security (c) Date of expiry of Security Day 19. Number of person having interest in business (also please complete part B for each such person 20. Number of managers 21. Numbers of authorized Signatories 22. Name of manager First Name Middle Name Surname *If more than one manager, attach particulars for additional managers on a separate sheet. 23. Name of authorized signatory * *please complete Part D First Name Middle Name Surname / Month / Year

136

24. Verification I/We ________________________________ hereby solemnly affirm and declare that the information given hereinabove is true and correct to the best of my/our knowledge and belief and nothing has been concealed there from. Signature of Authorized Signatory Name. Designation/Status Place Date: Day

Month

Year

FORM DVAT 04 : Annexure I


Particulars of person [propertor/karta/partners/directors in the business/members of executives committee of socities, clubs ete.] having interest in the business

Value Added Tax Please affix a passport size photograph of the person whose particulars are being given in this form 1. Full Name of Applicant Dealer (For individual, provide in order of first name, middle name, surname) 2. Registration No.* *This field is applicable when applying for amendment of registration in Form DVAT 07 3. Full name of person (Provide in order of first name, middle name surname) 4.Date of birth 5. Gender (tick [] one) Male / Female First Name 7. PAN : 8. Passport No. 9. E-mail address 10. Residential Address (If different from principle place of Business) Building Name/ Number Area/ Road Locality/ Market Pin Code Telephone Number Fax Number Building Name/ Number Area/ Road Locality/ Market Pin Code Telephone Number Fax Number Middle Name Surname /

137

6. Fathers / Husbands name

11. Permanent Address (If different from residential address)

12. Verification I/We __________________________________________ hereby solemnly affirm and declare that the information given hereinabove is true and correct to the best of my/our knowledge and belief and nothing has been concealed there from.

Value Added Tax Signature of Authorized Signatory . Full Name (first name, middle, surname) Designation/Status Place: Date: Day Month Year

138

FORM DVAT 04 : Annexure II


Details of additional places of business 1. Full Name of Applicant Dealer

Value Added Tax (For individual, provide in order of first name, middle name, surname) 2. Registration No.* *This field is applicable when applying for amendment of registration in Form DVAT 07 3. Details of Additional Places of Business (attach additional sheets if required) Type Address Godown/Warehouse Factory Building Name/ Number Area/ Road Locality/ Market Pin Code Email Id Telephone Number Fax Number Date of establishment State local sales tax/VAT/CST registration number (if place of business is situated outside Delhi) Type Address Godown/Warehouse Factory Building Name/Number Area/ Road Locality/ Market Distt. State Pin Code Email Id Telephone Number Fax Number Date of establishment State local sales tax/VAT/CST registration number (if place of business is situated outside Delhi) Type Address Godown/Warehouse Factory Building Name/Number Area/ Road Locality/ Market Distt. State Pin Code Email Id Telephone Number Fax Number Date of establishment State local sales tax/VAT/CST registration Shop Other place of business

139

/ Day Month

/ Year

Shop

Other place of business

/ Date Month

/ Year

Shop

Other place of business

/ Date Month

/ Year

Value Added Tax number (if place of business is situated outside Delhi) Type Address Godown/Warehouse Factory Building Name/Number Area/ Road Locality/ Market Distt. State Pin Code Email Id Telephone Number Fax Number Date of establishment State local sales tax/VAT/CST registration number (if place of business is situated outside Delhi) Shop Other place of business

140

/ Date Month

/ Year

4. Verification I/We __________________________________________ hereby solemnly affirm and declare that the information given hereinabove is true and correct to the best of my/our knowledge and belief and nothing has been concealed there from. Signature of Authorized Signatory . Full Name (first name, middle, surname) . Designation/Status . Place: .. Date: Day Month Year

FORM DVAT 04 : Annexure III


Particulars of the authorised signatory

Value Added Tax 1. Full Name of Applicant Dealer (For individual, provide in order of first name, middle name, surname) 2. Registration No.* *This field is applicable when applying for amendment of registration in Form DVAT 07 3. Name of Authorised Signatory (Provide in order of first name, middle name, surname) 4. Date of birth 5. Gender (tick [] one) / Male / Female First Name 7. PAN: 8. Passport No. 9. E-mail address 10. Residential Address (If different from principle place of Business) Building Name/ Number Area/ Road Locality/ Market Distt. State Pin Code Telephone Number Fax Number Building Name/ Number Area/ Road Locality/ Market Distt. State Pin Code Telephone Number Fax Number Middle Name Surname

141

6. Fathers / Husbands name

11. Permanent Address (If different from residential address)

12. Declaration I/We ________________________________________________ hereby solemnly affirm and declare that the person named above is authorized to act as an authorized signatory for the above referred business for which application for registration is being filed/ is registered under the Delhi VAT Act, 2004. All his actions in relation to this business will be binding on us. S.No. __________________

Value Added Tax Full Name (First name, Middle Name, Surname) Designation ______________________ Signature _______________________ __________________________

142

13. Acceptance as an authorized signatory I __________________________________________ hereby solemnly accord my acceptance to act as authorized signatory for the above referred business and all my acts shall be binding on the business. Signature of Authorised Signatory Full Name (First name, middle, surname) .. Designation/Status Place: Date: Day Month Year

Instructions for filling Registration Form (DVAT-04) (For details refer to Section 19 and Rule 12) 1. Please fill in all the details in CAPITAL letters.

Value Added Tax 2. Please note that you are mandatorily required to register if you:

143

(i) had turnover of more than Rupees 10 lakhs in the preceding financial year; or (ii) exceed turnover of Rupees 10 lakhs in the current year; or (iii) are liable to pay tax, or are registered or required to be registered under Central Sales Tax Act, 1956 3. Please note that irrespective of the quantum of turnover of the business, a dealer may apply for voluntary registration under the Delhi Value Added Tax Act, 2004. 4. For field 3, an importer means (i) a person who brings his own goods into Delhi; or (ii) a person on whose behalf another person brings goods into Delhi; or (iii) in the case of a sale occurring in the circumstances referred to in sub-section 2 of section 6 of the Central Sales Tax Act, 1956, the person in Delhi to whom the goods are delivered 5. The application for registration under this Act should be filed within 30 days from the date of person becoming liable for payment of tax. 6. For field 8, if the business does not have a PAN, then please mark Applied for or N/A as applicable. 7. For field 15, please fill the description of top 5 items on the basis of value of goods sold. 8. In case any of these details change, the dealer is required to intimate the department of the amendments within one month of the change. 9. The form has to be filled and signed by the authorised signatory of the business. 10. Businesses with a turnover of more than ` 5 crores are mandatorily required to file returns every month. Businesses with a turnover of less than ` 5 crores are required to file returns every quarter. They may however, elect to file their returns every month. 11. Registration application should be verified and signed by the following: (i) in the case of an individual, by the individual himself, and where the individual is absent from India, either by the individual or by some person duly authorised by him in this behalf and where the individual is mentally incapacitated from attending to his affairs, by his guardian or by any other person competent to act on his behalf; (ii) in the case of a Hindu Undivided Family, by a Karta and where the Karta is absent from India or is mentally incapacitated from attending to his affairs, by any other adult member of such family; (iii) in the case of a company or local authority, by the principle officer thereof; (iv) in the case of a firm, by any partner thereof, not being a minor; (v) in the case of any other association, by any member of the association or persons; (vi) in the case of a trust, by the trustee or any trustee; and (vii) in the case of any other person, by some person competent to act on his behalf.

Instructions for filling Registration Form (Annexures I, II and III) 1. In case of partnerships, Annexure I to be filled and signed by the managing partner plus top 4 other partners.

Value Added Tax

144

2. In case of companies, Annexure I to be filled and signed by the company secretary, the managing director and 3 other directors. 3. If required, make additional copies of the Annexures and attach with application form for registration (DVAT-04). 4. An amendment would be required each time a person changes (and not when the details of an existing person change) 5. In case of minors, the specimen signature of guardian/ trustee should be furnished. 6. In case of Annexure III, it is to be filled and signed by the person whose details are given in the Annexure. 7. Every sheet filled in the part has to be signed by the same person (authorized signatory) who has signed the registration application. 8. In case any of the Parts are not applicable, please strike off the same and write Not Applicable on the said Part. Method of Calculating Security Amount Prescribed Security Amount Reduction sought (Maximum reduction available ` 50,000) 1. 2. 3. 4. 5. 6. Proof of ownership of principle place of business Proof of ownership of residential property by proprietor/ managing partner Copy of passport of proprietor/ managing partner Copy of Permanent Account Number in the name of the business allotted by the Income Tax Department Copy of last electricity bill (The bill should be in the name of the business and for the address specified as the main place of business in the registration form) Copy of last telephone bill (The bill should be in the name of the business and for the address specified as the main place of business in the registration form) (`) 1,00,000 Rebate (`) 30,000 20,000 10,000 10,000 10,000 5,000

Original/Revised If revisedRefund Claimed? Yes No

FORM DVAT 16
(Refer Rules 28 and 29 of Delhi Value) Added Tax Rules, 2005

1. Date of original return2. Acknowledgement / Receipt No.3. Date of discovery of mistake or error. Attach a note explaining the revisions

Value Added Tax DELHI VALUE ADDED TAX RETURN

145

R1 Tax period

Form mm

/ dd

/ yy

To mm

/ dd

/ yy

R2.1 Registration No/TIN R2.2 Full Name of Dealer R2.3 Address R3 Description of top 3 items you deal in (In order of volume of sale for the Tax period, 1-highlights volume to 3-lowest volume 1 2 3 Output Tax (`)

Total Turnover ( `) R4 Turnover R4.1 Goods Taxable At 1% R4.2 Goods Taxable At 4% R4.3 Goods Taxable At 12.5% R4.4 Goods Taxable At 20% R4.5 Works contract taxable @ 12.5% R4.6 Exempt sales R4.7 Output Tax before adjustments Sub Total ( A ) R4.8 Adjustments to output tax (Complete Schedule 1 and enter Total S1.2 here ( B ) R4.9 Total Output Tax ( A +B)

R5 Computation of output tax Turnover (`) Output tax(`) R5.1 goods taxable at 1% R5.2 good taxable at 4% R5.2(1) Goods taxable at 5% R5.3good taxable at 12.5% R5.4 Good taxable at 20% R5.5 work contract taxable at 4% R5.5(1) works contract taxable @ 5% R5.6 work contract taxable at 12.5% R5.7 Exempt sales R5.8 Output Taxable adjustments Sub total (A) R5.9 Adjustments to output tax (Complete Annexure and enter Total 2 here) R5.10 Total output Tax Total A 2 From ANNEXURE

(B) (A+B)

Value Added Tax

146

R6 Turnover of Purchases in Delhi (including Purchases (`) Tax) & Tax credit R6.1 Capital goods R6.2 Other goods R6.2(1) Goods taxable at 1% R6.2(2) Goods taxable at 4% R6.2(3) Goods taxable at 5% R6.2(4) Goods taxable at 12.5% R6.2(5) Goods taxable at 20% R6.2(6) Works Contract taxable at 4% R6.2(6)(1) works contract taxable @ 5% R6.2(7) Works Contract taxable at 12.5% R6.2(8) Exempted purchase R6.3 Tax Credit before adjustment sub total (A) R6.4 Adjustment to tax credits Total A4 from (B) (Complete Annexure and enter Total A4 here) Annexure R6.5 Total Tax credits (A + B) R7.1 Net Tax (R5.10)-(R6.5) R7.2 Add: Interest, if payable R7.3 Add: penalty, if payable R7.4 Less: Tax deducted at source (attached No. of TDS certificates in original) R7.5 Balance payable (R7.1+R7.2-7.4) R7.6 Less: Amount deposited by the dealer (attach proof of payment) S. No. Date of deposit Challan No. Name of Bank and Branch Amount

Tax Credit (`)

R8 Net Balance*

(R7.5-R7.6)

*The net balance should not be positive as the amount due has to be deposited before filing the return: IF THE BALANCE OF LINE R8 IS NEGATIVE, PROVIDE DETAILS IN THIS BOX Balance brought forward from line R8 R9.1 Adjusted against liability under Central Sales Tax R9.2 Refund Claimed R9.3 Balance carried forward to next tax period IF REFUND IS CLAMIED,PROVIDE DETAILS IN THIS BOOK R10 Details OF Bank Account R10.1 Account No. R10.2 Account Type (Saving/Current etc.) R10.3 MICR No. R.10.4 Name of Bank & Branch R11 Inter-state trade and exports/imports Inter-state Inter-state

Value Added Tax Sales/Exports R11.1 Against C Forms R11.2 Against C+E1/E2 Forms R 11.3 Inward/outward stock Transfers against F Forms R11.4 Against H Forms R 11.5 Against I Forms R 11.6 Against J Forms R 11.7 Export to/Imports from outside India R11.7(1)Exempted sale / purchase including High Sea Sale etc R 11.8 Other (not supported by any Form) R 11.9 Capital goods R 11.10 Total Purchases/Imports

147

R12 Verification I/We________________________________________ hereby solemnly affirm and declare that the information given hereinabove is true that the best of my/our knowledge and belief and nothing has been concealed there from. Signature of Authorized Signatory Full Name (first name, middle, surname) . Designation/State Place Date: Day Month

Year

Instructions for filling return form 1. Please complete all the fields in the form. 2. Insert N/A in any filed which is not applicable to you. 3. Return has to be field within the time limit prescribe in rule 28 of the DVAT Rules. 4. Each page of the return form shall be signed by the authorized signatory. 5. For reporting adjustments, please use the following conversation: (a) Any amount that decrease the output tax or tax credits should be entered as a negative amount with a negative sign (-) before it. (b) Any amount that increases the output tax or tax credit should be entered as a positive amount.

ANNEXURE (To be attached with the return where adjustment is Output Tax or Tax Credit are made) A.1 Adjustment to Output Tax

Value Added Tax Nature of adjustment Increase in Output Tax (A) Decrease in Output Tax (B)

148

A1.1 Sale cancelled [Section8(1)(a)] A1.2 Nature of sale changed [Section 8(1)(b)] A1.3 Change is agreed consideration [Section 8(1)(c)] A1.4 Goods sold returned [Section 8(1)(d)] A1.5 Bed debts written off [Section 8(1)(e) and Rule 7A] A 1.6 Bed debts recovered [Rule 7 A(3)] A 1.7 Tax payable on goods held on the date of cancellation of registration (Section 23) A 1.8 others adjustments, if any (specify) Total A.2 Total Net Increase/(decrease) in Output Tax A.3 Adjustments to Tax Credits Nature of Adjustment (A-B) Increase in Tax Credit (C) Decrease in Tax Credit (D)

A3.1 Tax credit carried forward from previous tax period A 3.2 Receipt of debit note from the seller [Section 10(1)] A 3.3 Receipt of credit notes from seller [Section 10(1)] A 3.4 Goods purchased returned or rejected [Section 10(1)] A 3.5 Change in use of goods, for purposes other than for which credit is allowed [ Section 10(2)(a) A 3.6 Change is use of goods, for purposes for which credit is allowed [Section 10(2)(b)] A 3.7 Tax credit disallowed in respect of stock transfer out of Delhi [Section 14) A 3.8 Tax credit for transitional stock held on 1st April 2005 (Section 14) A 3.9 Tax credit for purchases of second hand goods (Section 15) A 3.10 Tax credits foe goods held on the date of withdrawal from composition Scheme (Section 16) A 3.11 The credit for trading stock and raw materials held at the time of registration (Section 20) A 3.12 Tax credit allowed for goods lost or destroyed (Rule 7) A 3.13 Tax credit adjustment on sale or stock transfer of capital goods [Section 9(9)(a)] A 3.14 Other adjustments if any (specify) Total A4. Total net Increase/(decrease) in Tax Credit (C-D)

Annexure 2A (See instruction 6) SUMMARY OF PURCHASE / INWARD BRANCH TRANSFER REGISTER (Month-wise) (To be filled along with return)

Value Added Tax

149 Name of the Dealer: Purchase for the tax period: From_____to_____

TIN : Address (Summary of Purchase (As per DVAT-30)


Sr. No. Month Sellers TIN Sellers Name Purchases not eligible for credit of input tax Import from outside India Interstate purchases or stock transfers Purchases from exempted units Total Purchases including tax, if any 8. Purchases eligible for credit of input tax Local purchases Rate of tax Others 10. 11. 12. 13. Input Tax paid Tota purchase including tax

1.

2.

3.

4.

5.

6.

7.

Capital Goods 9.

TOTAL Note : Purchases made from un -registered dealers may be reported in one row for a month.

Signature of Dealer / Authorised Signatory

Annexure-2B (see instruction 6) SUMMARY OF SALE/OUTWARD BRANCH TRANSFER REGISTER (Month-wise) (To be filled along with return)

Value Added Tax

150 Name of the Dealer: Sale for the tax period : From____to_____

TIN Address: Summary of Sales (As per DVAT 31)


Sr. No. Month Buyers TIN Buyers Name Inter-state Branch/ Consignment transfer Exports Out of India Inter State Sales

Local Sales

Sale Price (excluding CST) 1. 2. 3. 4. 5. 6. 7.

Central Sales Tax 8.

Total (7 + 8) 9.

Sales Price (excluding Tax) 10.

Output Tax 11.

Total (10 +11) 12.

Total

Note : Sales Made to un-registered dealers may be reported in one row for a month.

Signature of Dealer / Authorised Signatory

A WHITE PAPER ON STATE-LEVEL VALUE ADDED TAX


By

Value Added Tax

151

The Empowered Committee of State Finance Ministers


(Constituted By the Ministry of Finance, Government of India on the Basis of Resolution Adopted in the Conference of the Chief Ministers on November 16, 1999)
New Delhi January 17, 2005
PREFACE This White Paper is a result of collective efforts of all the States in formulating the basic design of the Statelevel Value Added Tax (VAT) through repeated and candid discussions in the Empowered Committee of State Finance Ministers. The State-level VAT, as elaborated in this White Paper, has certain distinct advantages over the existing sales tax structure. The VAT will not only provide full set-off for input tax as well as tax on previous purchases, but it will also abolish the burden of several of the existing taxes, such as turnover tax, surcharge on sales tax, additional surcharge, special additional tax, etc. In addition, Central Sales Tax is also going to be phased out. As a result, the overall tax burden will be rationalised, and prices, in general, will fall. Moreover, VAT will replace the existing system of inspection by a system of built-in self-assessment by traders and manufacturers. The tax structure will become simple and more transparent. This will significantly improve tax compliance and will also help increase revenue growth. While this State-level VAT has all these advantages, it is a State subject derived from Entry 54 of the State List, for which the States are sovereign in taking decisions. On these decisions on VAT, the States, through discussion in the Empowered Committee, have found it in their interests, to avoid unhealthy competition and have certain features of VAT to be common for all the States. These features will constitute the basic design of VAT. At the same time, the States will have freedom for appropriate variations consistent with this basic design. This White Paper is a collective attempt of the States to strike a balance between this needed commonality and the desired federal flexibility in the VAT structure. The White Paper also strikes a balance between what is possible in the VAT design to begine with and what can be improved upon in subsequent years as we gather more experience. The White Paper further mentions how after working out a consensus on this VAT design, nearly all the States either have finalised their VAT Bills by now and are in the process of obtaining Presidential Assent, or will reach that stage very soon. Even for one major State where there are some ground-level problems, a positive interaction with the Empowered Committee has recently opened up the possibility of resolving most of these problems. These efforts of the States towards formulation of VAT design and its implementation have received full cooperation of the Finance Ministry, Government of India. At the same time, the Finance Ministry has never imposed their views on us. We, therefore, remain thankful to the former Union Finance MinistersDr. Manmohan Singh, Shri Yashwant Sinha and Shri Jaswant Singh. We are specially grateful to Shri P. Chidambaram, the present Union Finance Minister, for his active support over the last eight months, when he not only helped formulate the modality of Central financial support to the States for possible loss of revenue in the transitional years of implementation of VAT, but also took time off his busy schedule to participate with us in the campaign for VAT in the States. It has always been fruitful to have interaction with Dr. Parthasarathi Shome, Adviser to the Union Finance Minister, for his insightful observations on the analytical structure of VAT as well as his reference to vast

Value Added Tax

152

experience in the implementation of VAT. The Secretary, Revenue, Additional Secretary, Revenue and all the concerned officials of the Revenue Department of the Finance Ministry have helped us by participating in the discussions whenever we requested them, and also by assisting in various procedural matters. Interaction with Dr. Govinda Rao, the Chairman of Technical Experts Committee on VAT and other members of the Committee has also been useful. We take this opportunity to thank all of them. Discussions with the representatives of trade organizations and chambers of commerce and industry at the national level as well as in the States have been relevant in assessing the ground level difficulties. Together with them, we are determined to overcome these difficulties in implementing VAT in the States. We remain thankful to them, and our mutual interaction will take place regularly. Finally, this White Paper could be written only on the basis of lively support of the Finance Ministers of the States, and with constant help from the Finance Secretaries and the Commissioners of Commercial Taxes of the States. The Commissioners of Commercial Taxes have often burnt their midnight oil, and their contribution should be particularly recorded. Shri Ramesh Chandra, Member-Secretary of the Empowered Committee had to carry on the difficult administrative task in the functioning of the Empowered Committee. We appreciate the efforts of Shri Chandra and the staff of the Empowered Committee. Even after all these efforts, there may be some unavoidable shortcomings in this White Paper, which we will try to overcome as we learn more from the actual experience of implementation of VAT. With this background and the attitude, this White Paper is an expression of the genuine commitment of the States to the implementation of VAT from April 1, 2005, which we are all looking forward to.

New Delhi, January 17, 2005.

Asim Kumar Dasgupta Convenor, Empowered Committee of State Finance Ministers, and Finance Minister, Government of West Bengal.

A White Paper On State-Level Value Added Tax This White Paper on State-level Value Added Tax (VAT) is presented in three parts. To begin with, the justification of VAT and its background have been mentioned (Part 1). In Part 2, the main design of VAT, as evolved on the basis of a consensus among the States through repeated discussions in the Empowered Committee, has been elaborated. While doing so, it is recognized that this VAT is a State subject and

Value Added Tax

153

therefore the States will have freedom for appropriate variations consistent with the basic design as agreed upon at the Empowered Committee. Finally, in Part 3, the other related issues have been discussed for effective implementation of VAT. 1. Justification of VAT and Background 1.1 In the existing sales tax structure, there are problems of double taxation of commodities and multiplicity of taxes, resulting in a cascading tax burden. For instance, in the existing structure, before a commodity is produced, inputs are first taxed, and then after the commodity is produced with input tax load, output is taxed again. This causes an unfair double taxation with cascading effects. In the VAT, a set-off is given for input tax as well as tax paid on previous purchases. In the prevailing sales tax structure, there is in several States also a multiplicity of taxes, such as turnover tax, surcharge on sales tax, additional surcharge, etc. With introduction of VAT, these other taxes will be abolished. In addition, Central sales tax is also going to be phased out. As a result, overall tax burden will be rationalised, and prices in general will also fall. Moreover, VAT will replace the existing system of inspection by a system of built-in self-assessment by the dealers and auditing. The tax structure will become simple and more transparent. That will improve tax compliance and also augment revenue growth. Thus, to repeat, with the introduction of VAT, benefits will be as follows: a set-off will be given for input tax as well as tax paid on previous purchases other taxes, such as turnover tax, surcharge, additional surcharge, etc. will be abolished overall tax burden will be rationalized prices will in general fall there will be self-assessment by dealers transparency will increase there will be higher revenue growth The VAT will therefore help common people, traders, industrialists and also the Government. It is indeed a move towards more efficiency, equal competition and fairness in the taxation system. 1.2 For these beneficial effects, a full-fledged VAT was initiated first in Brazil in mid 1960s, then in European countries in 1970s and subsequently introduced in about 130 countries, including several federal countries. In Asia, it has been introduced by a large number of countries from China to Sri Lanka. Even in India, there has been a VAT system introduced by the Government of India for about last ten years in respect of Central excise duties. At the State-level, the VAT system as decided by the State Governments, would now be introduced in terms of Entry 54 of the State List of the Constitution. 1.3 The first preliminary discussion on State-level VAT took place in a meeting of Chief Ministers convened by Dr. Manmohan Singh, the then Union Finance Minister in 1995. In this meeting, the basic issues on VAT were discussed in general terms and this was followed up by periodic interactions of State Finance Ministers. Thereafter, in a significant meeting of all Chief Ministers, convened on November 16, 1999 by Shri Yashwant Sinha, the then Union Finance Minister, three important decisions were taken. First, before the introduction of State-level VAT, the unhealthy sales tax rate war among the States would have to end and sales tax rates would need to be harmonised by implementing uniform floor rates of sales tax for different categories of commodities with effect from January 1, 2000. Second, in the interest again of harmonisation of incidence of sales tax, the sales-tax-related industrial incentive schemes would also have to be discontinued with effect from January 1, 2000. Third, on the basis of achievement of the first two objectives, steps would be taken by the States for introduction of State-level VAT after adequate preparation. For implementing these decisions, an Empowered Committee of State Finance Ministers was set-up. 1.4 Thereafter, this Empowered Committee has met regularly, attended by the State Finance Ministers, and also by the Finance Secretaries and the Commissioners of Commercial Taxes of the State Governments as well as senior officials of the Revenue Department of the Ministry of Finance, Government of India. Through repeated discussions and collective efforts in the Empowered Committee, it was possible within a period of about a year and a half to achieve nearly 98 per cent success in the first two objectives on

Value Added Tax

154

harmonisation of sales tax structure through implementation of uniform floor rates of sales tax and discontinuation of sales-tax- related incentive schemes. As a part of regular monitoring, whenever any deviation is reported from the uniform floor rates of sales tax, or from decision on incentives, the Empowered Committee takes up the matter with the concerned State and also the Government of India for necessary rectification. 1.5 After reaching this stage, steps were initiated for systematic preparation for the introduction of Statelevel VAT. In order again to avoid any unhealthy competition among the States which may lead to distortions in manufacturing and trade, attempts have been made from the very beginning to harmonise the VAT design in the States, keeping also in view the distinctive features of each State and the need for federal flexibility. This has been done by the States collectively agreeing, through repeated discussions in the Empowered Committee, to certain common points of convergence regarding VAT, and allowing at the same time certain flexibility for the local characteristics of the States. 1.6 Along with these measures at ensuring convergence on the basic issues on VAT, steps have also been taken for necessary training, computerisation and interaction with trade and industry, particularly at the State levels. This interaction with trade and industry is being specially emphasised. 1.7 It may be noted that while such preparation was going on, the Chief Ministers of all the States in an important meeting on State-level VAT convened by the Prime Minister on October 18, 2002, when Shri Jaswant Singh, the then Union Finance Minister was present, clearly stated their intention of introducing VAT from April 1, 2003. About 29 States and Union Territories had expeditiously sent their Bills to the Ministry of Finance, Government of India for prior vetting. The Union Ministry of Finance had considered these Bills of States and Union Territories, and sent their comments/ suggestions to the States and Union Territories in line with the decisions of the Empowered Committee of the State Finance Ministers for incorporating the same in VAT Bills to be placed in the State legislatures and subsequent transmission to the Government of India for Presidential Assent. At this stage, there were certain developments which delayed the introduction of VAT. Despite these developments, most of the States remained positively interested in implementation of VAT. Madhya Pradesh VAT Bill had already been accorded Presidential Assent in November 2002. One State, namely, Haryana, has already introduced VAT on its own with good results on revenue growth. It is important to note that in the meeting of Empowered Committee on June 18, 2004 when Shri P. Chidambaram, the Union Finance Minister, was invited and was kindly present, all the States, excepting one, once again categorically renewed their commitment to the introduction of VAT from April 1, 2005. Even for this particular State with certain problems, a positive interaction has recently been organised with that State to resolve certain genuine ground-level problems. Now nearly all the States have either finalised their VAT Bills and are in the process of obtaining Presidential Assent, or will reach that stage very soon. 2. Design of State-Level VAT 2.1 As already mentioned, the design of State-level VAT has been worked out by the Empowered Committee through several rounds of discussion and striking a federal balance between the common points of convergence regarding VAT and flexibility for the local characteristics of the States. Since the State-level VAT is centred around the basic concept of set-off for the tax paid earlier, the needed common points of convergence also relate to this concept of set-off/input tax credit, its coverage and related issues as elaborated below.

Concept of VAT and Set-off / Input Tax Credit 2.2 The essence of VAT is in providing set-off for the tax paid earlier, and this is given effect through the concept of input tax credit/rebate. This input tax credit in relation to any period means setting off the amount of input tax by a registered dealer against the amount of his output tax. The Value Added Tax (VAT) is

Value Added Tax

155

based on the value addition to the goods, and the related VAT liability of the dealer is calculated by deducting input tax credit from tax collected on sales during the payment period (say, a month). If, for example, input worth Rs. 1,00,000/- is purchased and sales are worth Rs. 2,00,000/- in a month, and input tax rate and output tax rate are 4% and 10% respectively, then input tax credit/set-off and calculation of VAT will be as shown below: (a) Input purchased within the month : ` 1,00,000/(b) Output sold in the month : ` 2,00,000/(c) Input tax paid : ` 4,000/(d) Output tax payable : ` 20,000/(e) VAT payable during the month : ` 16,000/after set-off/input tax credit [(d) (c)] Coverage of Set-Off / Input Tax Credit 2.3 This input tax credit will be given for both manufacturers and traders for purchase of inputs/supplies meant for both sale within the State as well as to other States, irrespective of when these will be utilised/sold. This also reduces immediate tax liability. Even for stock transfer/consignment sale of goods out of the State, input tax paid in excess of 4% will be eligible for tax credit. Carrying Over of Tax Credit 2.4 If the tax credit exceeds the tax payable on sales in a month, the excess credit will be carried over to the end of next financial year. If there is any excess unadjusted input tax credit at the end of second year, then the same will be eligible for refund. Input tax credit on capital goods will also be available for traders and manufacturers. Tax credit on capital goods may beadjusted over a maximum of 36 equal monthly instalments. The States may at their option reduce this number of instalments. There will be a negative list for capital goods (on the basis of principles already decided by the Empowered Committee) not eligible for input tax credit. Treatment of Exports, etc. 2.5 For all exports made out of the country, tax paid within the State will be refunded in full, and this refund will be made within three months. Units located in SEZ and EOU will be granted either exemption from payment of input tax or refund of the input tax paid within three months. Inputs Procured from Other States 2.6 Tax paid on inputs procured from other States through inter-State sale and stock transfer will not be eligible for credit. However, a decision has been taken for duly phasing out of inter-State sales tax or Central sales tax. As a preparation for that, a comprehensive inter-State tax information exchange system is also being set up. Treatment of Opening Stock 2.7 All tax-paid goods purchased on or after April 1, 2004 and still in stock as on April 1, 2005 will be eligible to receive input tax credit, subject to submission of requisite documents. Resellers holding tax-paid goods on April 1, 2005 will also be eligible. VAT will be levied on the goods when sold on and after April

Value Added Tax

156

1, 2005 and input tax credit will be given for the sales tax already paid in the previous year. This tax credit will be available over a period of 6 months after an interval of 3 months needed for verification. Compulsory Issue of Tax Invoice, Cash Memo or Bill 2.8 This entire design of VAT with input tax credit is crucially based on documentation of tax invoice, cash memo or bill. Every registered dealer, having turnover of sales above an amount specified, shall issue to the purchaser serially numbered tax invoice with the prescribed particulars. This tax invoice will be signed and dated by the dealer or his regular employee, showing the required particulars. The dealer shall keep a counterfoil or duplicate of such tax invoice duly signed and dated. Failure to comply with the above will attract penalty. Registration, Small Dealers and Composition Scheme 2.9 Registration of dealers with gross annual turnover above `5 lakh will be compulsory. There will be provision for voluntary registration. All existing dealers will be automatically registered under the VAT Act. A new dealer will be allowed 30 days time from the date of liability to get registered. Small dealers with gross annual turnover not exceeding `5 lakh will not be liable to pay VAT. States will have flexibility to fix threshold limit within `5 lakh. Small dealers with annual gross turnover not exceeding ` 50 lakh who are otherwise liable to pay VAT, shall however have the option for a composition scheme with payment of tax at a small percentage of gross turnover. The dealers opting for this composition scheme will not be entitled to input tax credit. Tax Payers Identification Number (TIN) 2.10 The Tax Payers Identification Number will consist of 11 digit numerals throughout the country. First two characters will represent the State Code as used by the Union Ministry of Home Affairs. The set-up of the next nine characters may, however, be different in different States. Return 2.11 Under VAT, simplified form of returns will be notified. Returns are to be filed monthly/quarterly as specified in the State Acts/Rules, and will be accompanied with payment challans. Every return furnished by dealers will be scrutinized expeditiously within prescribed time limit from the date of filing the return. If any technical mistake is detected on scrutiny, the dealer will be required to pay the deficit appropriately. Procedure of Self-Assessment of VAT Liability 2.12 The basic simplification in VAT is that VAT liability will be self-assessed by the dealers themselves in terms of submission of returns upon setting off the tax credit. Return forms as well as other procedures will be simple in all States. There will no longer be compulsory assessment at the end of each year as is existing now. If no specific notice is issued proposing departmental audit of the books of accounts of the dealer within the time limit specified in the Act, the dealer will be deemed to have been self-assessed on the basis of returns submitted by him. Because of the importance of the concept of self-assessment in VAT, provision for self-assessment will be stated in the VAT Bills of the States. Audit 2.13 Correctness of self-assessment will be checked through a system of Departmental Audit. A certain percentage of the dealers will be taken up for audit every year on a scientific basis. If, however, evasion is detected on audit, the concerned dealer may be taken up for audit for previous periods. This Audit Wing will remain delinked from tax collection wing to remove any bias. The audit team will conduct its work in a time bound manner and audit will be completed within six months. The audit report will be transparently sent to the dealer also.

Value Added Tax

157

Simultaneously, a cross-checking, computerised system is being worked out on the basis of coordination between the tax authorities of the State Governments and the authorities of Central Excise and Income Tax to compare constantly the tax returns and set-off documents of VAT system of the States and those of Central Excise and Income Tax. This comprehensive cross-checking system will help reduce tax evasion and also lead to significant growth of tax revenue. At the same time, by protecting transparently the interests of tax-complying dealers against the unfair practices of tax-evaders, the system will also bring in more equal competition in the sphere of trade and industry. Declaration Form 2.14 There will be no need for any provision for concessional sale under the VAT Act since the provision for setoff makes the input zero-rated. Hence, there will be no need for declaration form, which will be a further relief for dealers. Incentives 2.15 Under the VAT system, the existing incentive schemes may be continued in the manner deemed appropriate by the States after ensuring that VAT chain is not affected. Other Taxes 2.16 As mentioned earlier, all other existing taxes such as turnover tax, surcharge, additional surcharge and Special Additional Tax (SAT) would be abolished. There will not be any reference to these taxes in the VAT Bills. The States that have already introduced entry tax and intend to continue with this tax should make it vatable. If not made vatable, entry tax will need to be abolished. However, this will not apply to entry tax that may be levied in lieu of octroi. Penal Provisions 2.17 Penal provisions in the VAT Bills should not be more stringent than in the existing Sales Tax Act. Coverage of Goods under VAT 2.18 In general, all the goods, including declared goods will be covered under VAT and will get the benefit of input tax credit. The only few goods which will be outside VAT will be liquor, lottery tickets, petrol, diesel, aviation turbine fuel and other motor spirit since their prices are not fully market determined. These will continue to be taxed under the Sales Tax Act or any other State Act or even by making special provisions in the VAT Act itself, and with uniform floor rates decided by the Empowered Committee. VAT Rates and Classification of Commodities 2.19 Under the VAT system covering about 550 goods, there will be only two basic VAT rates of 4% and 12.5%, plus a specific category of tax-exempted goods and a special VAT rate of 1% only for gold and silver ornaments, etc. Thus the multiplicity of rates in the existing structure will be done away with under the VAT system. Under exempted category, there will be about 46 commodities comprising of natural and unprocessed products in unorganized sector, items which are legally barred from taxation and items which have social implications. Included in this exempted category is a set of maximum of 10 commodities flexibly chosen by individual States from a list of goods (finalised by the Empowered Committee) which are of local social importance for the individual States without having any inter-state implication. The rest of the commodities in the list will be common for all the States. Under 4% VAT rate category, there will be the largest number of goods (about 270), common for all the States, comprising of items of basic necessities such as medicines and drugs, all agricultural and industrial inputs, capital goods and declared goods. The schedule of

Value Added Tax

158

commodities will be attached to the VAT Bill of every State. The remaining commodities, common for all the States, will fall under the general VAT rate of 12.5%. In terms of decision of the Empowered Committee, VAT on AED items relating to sugar, textile and tobacco, because of initial organisational difficulties, will not be imposed for one year after the introduction of VAT, and till then the existing arrangement will continue. The position will be reviewed after one year. Effects of the VAT System 2.20 This design of the State-level VAT has been carefully worked out by the Empowered Committee after repeated interactions with the States and others concerned and striking a balance between the needed convergence and federal flexibility as well as ground-level reality. If now all the components of the VAT design are taken together, then it will be seen that the total effect of this VAT system will be to rationalise the tax burden and bring down, in general, the price level. This will also stop unhealthy tax-rate war and trade diversion among the States, which had adversely affected interests of all the States in the past. Moreover, this VAT design will also significantly bring in simplicity and transparency in the tax structure, thereby improving tax-compliance and eventually also the revenue growth, as mentioned in the beginning. 3. Steps Taken by the States 3.1 It is now of significance to note that most of the States, after collective interaction in the Empowered Committee, have either already modified or agreed to modify their VAT Bills by incorporating these common points of convergence including flexibility as mentioned in the VAT design above, and are also taking other preparatory steps towards introduction of VAT from April 1, 2005. 3.2 As a part of the preparatory steps, the States have started the process of preparing the draft of VAT Rules, including Books of Accounts to be maintained. The objective will be to keep these as simple as possible so that it becomes easy for a small trader to comply with the requirements. 3.3 Moreover, the States have initiated, and in many cases also completed, steps for computerisation upto the levels of assessing officers and also at the check posts. This process will continue since this is extremely important for document-based verification and integration with Taxation Information Exchange System as well as with information of the Central excise and income tax systems as indicated earlier. 3.4 It may be mentioned here that appropriate Central funds for VAT-related computerisation in the NorthEastern States are also being released by the Government of India. 4. Related Issues 4.1 While the States have thus taken several steps towards introduction of VAT, certain supporting decisions were critically needed at the national level for more effective implementation of VAT from April 1, 2005. 4.2 It needs to be carefully noted that although introduction of VAT may, after a few years, lead to revenue growth, there may be a loss of revenue in some States in the initial years of transition. It is with this in view that the Government of India had agreed to compensate for 100 per cent of the loss in the first year, 75 per cent of the loss in the second year and 50 per cent of the loss in the third year of introduction of VAT, and the loss would be computed on the basis of an agreed formula. This position has not only been reaffirmed by the Union Finance Minister in his Budget Speech of 2004-05, but a concrete formula for this compensation has also now been worked out after interaction between the Union Finance Minister and the Empowered Committee. 4.3 As mentioned earlier, there is also a need, after introduction of VAT, for phasing out of Central Sales Tax (CST). However, the States are now collecting nearly ` 15 thousand crore every year from CST. There is accordingly a need of compensation from the Government of India for this loss of revenue as CST is phased out. Moreover, while CST is phased out, there is also a critical need for putting in place a regulatory

Value Added Tax

159

frame-work in terms of Taxation Information Exchange System to give a comprehensive picture of interState trade of all commodities. As already mentioned, this process of setting up of Taxation Information Exchange System has already been started by the Empowered Committee, and is expected to be completed within one year. The position regarding CST will be reviewed by the Empowered Committee during 200506, and suitable decision on the phasing out of CST will be taken. 4.4 It is also essential to bring imports into the VAT chain. Because of the set-off, this will not result in any tax cascading effect, but will only improve tax compliance. A proposal for VAT on imports, including the collection mechanism with adequate safeguards for the protection of interest of land-locked States, is being discussed with the Government of India. 4.5 Similarly, discussion between the Empowered Committee and the Government of India is going on for an early decision on the question of collection and appropriation of service tax by the Centre and the States. If decisions on VAT on imports and service tax are taken expeditiously at the national level, then these two important spheres of taxation can be integrated, along with the AED items as mentioned earlier, into the VAT system of the States from the second year of introduction of VAT. 4.6 It may be noted that this VAT design has been worked out carefully by the Empowered Committee to strike a balance not only between the common points of convergence and federal flexibility, but also a balance between what can be done to begin with and what should be incorporated subsequently for further perfection of the VAT system. 4.7 For successful implementation of State-level VAT, close interaction with trade and industry is specially important. The Empowered Committee has therefore also set up a Consultative Committee with one representative from each of the national level trade organisations and national level chambers of commerce and industry. This Committee has already started interacting with the Empowered Committee. This process of interaction will continue regularly to discuss issues and sort out problems of implementation of VAT. Such Consultative Committees will also be set up at the level of each State, and interaction with the State Government will take place in a similarly regular manner. 4.8 In course of discussion with representatives of trade and industry, reference has often been made to the earlier VAT Bills of some of the States. It should be clearly noted, as already mentioned before, that all the States have agreed to amend their earlier VAT Bills so as to conform broadly to the common design as elaborated in this White Paper. This process of amendment has also already started. The point of reference on VAT should therefore be this design of VAT as explained in this White Paper. It should also be mentioned that there are some important points on the ground-level implementation of VAT which have been raised by the representatives of trade and industry. Many of the points will be taken care of in the VAT rules of the States, with changes where necessary. 4.9 Finally, a comprehensive campaign on State-level will be launched to communicate in simple and transparent manner the benefit of VAT for common people, traders, industrialists and also the State Governments. This campaign will then be launched first at the national level on the basis of necessary coordination between the States and the Centre. This will then be simultaneously followed up at the level of every State and also in districts of the States. This campaign will be based on written materials as well as publicity through all media. The purpose of this campaign will be a two-way interaction between the Government and the trade and industry as well as the common people. There is now only looking forward to the introduction of State-level VAT by all the States and Union Territories from April 1, 2005. We seek cooperation of all sections of people in the country.

Value Added Tax

160

Service Tax

161

SERVICE TAX
INDEX

FINANCE ACT 1994


Extent, Commencement and Application Definition Classification of taxable services Charge of Service Tax Charge of service tax on services received from outside India Valuation of taxable services for charging service tax Payment of service tax Registration Furnishing of returns Scheme for submission of returns through service tax returns preparers Recovery of service tax not levied or paid or short-levied or short-paid or erroneously refunded Service tax collected from any person to be deposited with Central Government Interest on amount collected in excess Interest on delayed payment of service tax Penalty for failure to pay service tax Penalty for contravention of rules and provisions of Act for which no penalty is specified elsewhere. Penalty for suppressing, etc., of value of taxable services Penalty not to be imposed in certain cases Application of certain provisions of Central Excise Act, 1944 Appeals to the Commissioner of Central Excise (Appeals) Appeals to Appellate Tribunal Recovery of any amount due to Central Government Power to grant exemption from service tax Power to grant rebate (Duty drawback) in case of exports Power to make rules

SECTIONS 64 65 65A 66 66A 67 68 69 70 71 73 73A 73B 75 76 77 78 80 83 85 86 87 93 93A 94 Rule 2 4 5 6 7

SERVICE TAX RULES, 1994


Reverse Charge Registration Records Payments Return

POINT OF TAXATION RULES, 2011


Point of Taxation for Payment of Service Tax Point of Taxation for change in rate of service tax

Rule 3 4

Service Tax Point of Taxation for Service Tax on New Services Point of Taxation for continuous supply of Services Payment of Service Tax on Receipt Basis in certain cases 5 6 7

162

Q1: Explain Selective Approach/ Comprehensive Approach to Service Tax. Q2: Explain meaning of Taxable Service. Q3 (Imp.): Write a note on Constitutional validity of Service Tax. Q4 (V. Imp.): Write a note on sources of Service Tax Law. Q5: Write a note on Administration of Service Tax. Q6: Write a note on Service Tax Procedures. Q7 (Imp.): Write a note on role of a Chartered Accountant in service tax. Q8 (V. Imp.): Write a note on Extent, Commencement and Application of Service Tax. Q9: Is service tax destination based consumption tax? Q10: Write a note on charge of Service Tax. Q11 (V. Imp.): What is the exemption limit for the Small Service Provider. Q12 (V. Imp.): Write a note on General Exemptions under Service Tax. Q13 (V. Imp.): Write a note on Registration under Service Tax. Q14 (V. Imp.): Write a note on Centralized Registration under Service Tax. Q15: Write a note on Amendment of Registration Certificate. Q16: Write a note on Cancellation of Registration Certificate. Q17: Write a note on issue of Invoice/Bill by Service Provider. Q18: Explain Determination of Rate for Service Tax. Q19 (V. Imp): Write a note on payment of Service Tax. Q20 (V. Imp.): Write a note on Point of Taxation for Payment of Service Tax. Q21: Write a note on interest on delayed payment of Service Tax. Q22: Write a note on penalty for failure to pay Service Tax. Q23: Write a note on penalty for contravention of rules and provisions of Act for which no penalty is specified elsewhere. Q24: Write a note on penalty for suppressing (to conceal) value of taxable services. Q25 (Imp.): Write a note on Advance Payment of Service Tax. Q26 (Imp.): Write a note on Refund of Service Tax/ Issue of Credit Note. Q27 (Imp.): Write a note on Provisional Payment of Service Tax. Q28 (V. Imp.): Write a note on Self Adjustment of Service Tax. Q29 (Imp.): Explain payment of service tax in case of Associated Enterprises. Q30: Write a note on adjustment for Municipal Tax paid for House Property. Q31 (Imp.): Write a note on service tax collected wrongly or in excess from any person. Q32 (Imp.): Write a note on point of taxation for determining the rate of Service Tax. Q33: Write a note on liability to service tax in case of New Services. Q34: What are the provisions of service tax for Continuous Supply of Service. Q35: Write a note on determination of Point of Taxation in case of Copyrights, etc. (For Self Reading) Q36 (V. Imp.): Write a note on Filing of Service Tax Return. Q37: Write a note on Revision of Service Tax Return. Q38 (V. Imp.): Write a note on Delay Furnishing of Service Tax Return. Q39: Write a note on contents of the Service Tax Return. Q40: Write a note on documents to be submitted along with Service Tax Return. Q41 (V. Imp.): Write a note on E-Filing of Service Tax Return. Q42: Write a note on Service Tax Return Preparer Scheme. Q43: Write a note on Records under Service Tax. Q44 (V. Imp.): Write a note on option to pay service tax in case of Air Travel Agent.

Service Tax

163

Q45 (V. Imp.): Write a note on option to pay service tax by any insurer carrying on Life Insurance Business. Q46 (Imp.): Write a note on option to pay service tax in case of Money Changer. Q47: Write a note on option to pay service tax in case of Distribution of Lottery ticket. Q48 (V. Imp.): Write a note on Reverse Charge Mechanism. Q49 (V. Imp.): Write a note on Insurance Auxiliary (help) Service. Q50 (V. Imp.): Write a note on Import of Services. Q51 (V. Imp.): Write a note on Goods Transport Agency. Q52: Write a note on Business Auxiliary Services of Distribution of Mutual Fund. Q53 (V. Imp.): Write a note on Sponsorship Services. Q54 (V. Imp): Write a note on tax credit in case of Service Provider. Q55: Explain provisions of Rule 6 of Cenvat Credit Rules, 2004. Q56: Whether any section of Central Excise Act, 1944 is applicable to Service tax. (For Self Reading Only) Q57: Write a note on Appeals to the Commissioner of Central Excise (Appeals). (For Self Reading Only) Q58: Write a note on Appeals to Appellate Tribunal (CESTAT). (For Self Reading Only) Q59 (V. Imp.): Write a note on valuation of taxable services for charging Service Tax. Q60: Explain the concept of Pure Agent. Q61 (V. Imp.): Write a note on Export of Services. Q62 (V. Imp.): Explain meaning of Input Service / Output Service. Q63 (Imp.): Write a note on Input Service Distributor. Q64 (Imp.): Write a note on liability of service tax in case of supply of goods as well as services. Q65 (Imp.): Explain concept of Partial Abatement. Q66: Write a note on services rendered by Sub-Contractor. Q67: Explain Recovery of Service Tax. (Self reading) Q68: Explain provisions of Tax Deduction at Source (TDS) with regard to Service Tax. Q69 (V. Imp.): Explain provisions relating to Consulting Engineers Services. Q70 (V. Imp.): Explain provisions relating to Mandap Keepers Service. Q71 (V. Imp.): Explain provisions relating to Practising Chartered Accountant. Q72 (V. Imp.): Explain provisions relating to Scientific and Technical Consultancy Service. Q73 (V. Imp.): Explain provisions relating to Commercial Training or Coaching. Q74 (V. Imp.): Explain provisions relating to Technical Testing and Analysis Service. Q75 (V. Imp.): Explain provisions relating to Business Exhibition Service. Q76 (V. Imp.): Explain provisions relating to Information Technology Software Service.

Service Tax

164

Service Tax in India The genesis of service tax is the result of recommendations made in 1990s by the Tax Reforms Committee headed by Professor Dr. Raja J. Chelliah. Based on the above recommendations, Dr. Manmohan Singh, the Union Finance Minister, introduced the concept of service tax in 1994-95 w.e.f. 01.07.1994. Initially service tax was imposed only on three services i.e. telephones, non-life insurance and stock brokers but at present there are 119 services which are chargeable to service tax and are called Taxable Services and details are given in section 65(105). Question 1: Explain Selective Approach/ Comprehensive Approach to Service Tax. Answer: Each country has a taxation system on services adopting either a Comprehensive Approach or a Selective Approach. In Comprehensive Approach all services are made taxable and a list is given in case some services are to be exempted. In Selective Approach, selective services are subjected to service tax. While most of the developed countries tax all the services with very few and limited exemptions, most of the developing countries have opted for taxation of select services only. India has adopted a selective approach to taxation of services and the services which are chargeable to service tax as given in section 65(105) of Finance Act, 1994. Question 2: Explain meaning of Taxable Service. Answer: The services which are chargeable to service tax are called Taxable Service and the services which are not chargeable to service tax are called Exempted Services and at present there are 119 services which are Taxable Services and are given under section 65(105) of Finance Act, 1994 and are as given below: (Students need not learn the list given below rather it is only for reading) S. Taxable Service Section Effective No. from 1 Stock Broking 65(105)(a) 01.07.1994 2. General Insurance 65(105)(d) 01.07.1994 3. Advertising Agency Services 65(105)(e) 01.11.1996 4. Courier 65(105)(f) 01.11.1996 5. Consulting Engineer 65(105)(g) 07.07.1997 6. Custom House Agent 65(105)(h) 15.06.1997 7. Steamer Agent 65(105)(i) 15.06.1997 8. Clearing & Forwarding Agent 65(105)(j) 16.07.1997 9. Manpower Recruitment or Supply Agency 65(105)(k) 07.07.1997 10. Air Travel Agent 65(105)(l) 01.07.1997 11. Mandap Keeper 65(105)(m) 01.07.1997 12. Tour Operator 65(105)(n) 01.09.1997 13. Rent-a-Cab Operator 65(105)(o) 16.07.1997 14. Architect 65(105)(p) 16.10.1998 15. Interior Decorator 65(105)(q) 16.10.1998 16. Management or Business Consultant 65(105)(r) 16.10.1998 17. Practicing Chartered Accountants Services 65(105)(s) 16.10.1998 18. Practicing Cost Accountants Services 65(105)(t) 16.10.1998 19. Practicing Company Secretary Services 65(105)(u) 16.10.1998 20. Real Estate Agent 65(105)(v) 16.10.1998 21. Security Agency 65(105)(w) 16.10.1998 22. Credit Rating Agency 65(105)(x) 16.10.1998

Service Tax 23. 24. 25. 26. 27. 28. 29. 30. 31. 32. 33. 34. 35. 36. 37. 38. 39. 40. 41. 42. 43. 44. 45. 46. 47. 48. 49. 50. 51. 52. 53. 54. 55. 56. 57. 58. 59. 60. 61. 62. 63. 64. 65. 66. 67. 68. 69. 70. 71. 72. Market research agency Underwriter Scientific and Technical Consultancy Photography Convention On-line information and data base access or retrieval(computer network) Video Production Agency Sound Recording Broadcasting Insurance Auxiliary (General Insurance) Banking and Financial services Port Authorised service station (Motor Car / Two- wheelers /LMV) Beauty Treatment Cargo Handling Cable Dry Cleaning Event Management Fashion Designing Health and Fitness Life Insurance Insurance Auxiliary (life insurance) Rail Travel Agent Storage and Warehousing Business Auxiliary Services Commercial Training or Coaching Erection, Commissioning or Installation Franchise Internet Caf Management, Maintenance or Repair Technical Testing and Analysis Technical Inspection and Certification Foreign Exchange Broking Other Ports Airport Services Transport of Goods by Air Business Exhibition Transport of Goods by Road Commercial or Industrial Construction Intellectual Property Opinion Poll Outdoor Caterer Programme Production (of TV or Radio Programmes) Survey and Exploration of Mineral Pandal or Shamiana Travel Agent (other than Air Travel Agent and Rail Travel Agent) Forward Contract Transport of Goods (other than water) through Pipeline or Conduit Site Formation and Clearance etc. Dredging (to remove mud, stones, etc.) 65(105)(y) 65(105)(z) 65(105)(za) 65(105)(zb) 65(105)(zc) 65(105)(zh) 65(105)(zi) 65(105)(zj) 65(105)(zk) 65(105)(zl) 65(105)(zm) 65(105)(zn) 65(105)(zo) 65(105)(zq) 65(105)(zr) 65(105)(zs) 65(105)(zt) 65(105)(zu) 65(105)(zv) 65(105)(zw) 65(105)(zx) 65(105)(zy) 65(105)(zz) 65(105)(zza) 65(105)(zzb) 65(105)(zzc) 65(105)(zzd) 65(105)(zze) 65(105)(zzf) 65(105)(zzg) 65(105)(zzh) 65(105)(zzi) 65(105)(zzk) 65(105)(zzl) 65(105)(zzm) 65(105)(zzn) 65(105)(zzo) 65(105)(zzp) 65(105)(zzq) 65(105)(zzr) 65(105)(zzs) 65(105)(zzt) 65(105)(zzu) 65(105)(zzv) 65(105)(zzw) 65(105)(zzx) 65(105)(zzy) 65(105)(zzz) 65(105)(zzza) 65(105)(zzzb)

165 16.10.1998 16.10.1998 16.07.2001 16.07.2001 16.07.2001 16.07.2001 16.07.2001 16.07.2001 16.07.2001 16.07.2001 16.07.2001 16.07.2001 16.07.2001 16.08.2002 16.08.2002 16.08.2002 16.08.2002 16.08.2002 16.08.2002 16.08.2002 16.08.2002 16.08.2002 16.08.2002 16.08.2002 01.07.2003 01.07.2003 01.07.2003 01.07.2003 01.07.2003 01.07.2003 01.07.2003 01.07.2003 01.07.2003 16.07.2001 10.09.2004 10.09.2004 10.09.2004 01.01.2005 10.09.2004 10.09.2004 10.09.2004 10.09.2004 10.09.2004 10.09.2004 10.09.2004 10.09.2004 10.09.2004 16.06.2005 16.06.2005 16.06.2005

Service Tax 73. 74. 75. 76. 77. 78. 79. 80. 81. 82. 83. 84. 85. 86. 87. 88. 89. 90. 91. 92. 93. 94. 95. 96. 97. 98. 99. 100. 101. 102. 103. 104. 105. 106. 107. 108. 109. 110. 111. 112. 113. 114. 115. 116. 117. 118. 119. Survey and Map-Making Cleaning Activity Club or Associations Services Packaging Activity Mailing list Compilation and Mailing Construction of (Residential) Complex Registrar to An Issue Share Transfer Agent ATM Operations, Maintenance or Management Recovery Agent Sale of Advertising Space or Time Sponsorship Service Air Transport of Passengers Embarking (to get onto a ship) for International Travel Transport of Goods by Rail Business Support Services Auctioneers Service Public relations Management Service Ship Management Services Internet Telecommunication Transport of Person by Cruise Ships (to travel in a ship or boat visiting different places, especially as a holiday/vacation) Credit card, debit card, charge or other payment cards related services Telecommunication Services Mining of Mineral, Oil or Gas Services Renting of Immovable Property Works Contract Services Development and Supply of Content Services Asset Management Including Portfolio Management Design Services Information Technology Software Unit Linked Insurance Plan (ULIP) Management Stock Exchange Commodity Exchange Processing and Clearing House Supply of Tangible Goods for Use Cosmetic and Plastic Surgery Transport of Goods Through Waterways Legal Consultancy Promotion, Marketing, Organizing Games Clinical Establishment Service Medical Record Agency Promotion or Marketing of Brand Granting Right or Commercial Use of an Event Electricity Exchange Services Copyright Construction Services Preferential Location and Development Restaurant Service Short-Term Accommodation /Hotel Accommodation 65(105)(zzzc) 65(105)(zzzd) 65(105)(zzze) 65(105)(zzzf) 65(105)(zzzg) 65(105)(zzzh) 65(105)(zzzi) 65(105)(zzzj) 65(105)(zzzk) 65(105)(zzzl) 65(105)(zzzm) 65(105)(zzzn) 65(105)(zzzo) 65(105)(zzzp) 65(105)(zzzq) 65(105)(zzzr) 65(105)(zzzs) 65(105)(zzzt) 65(105)(zzzu) 65(105)(zzzv) 65(105)(zzzw) 65(105)(zzzx) 65(105)(zzzy) 65(105)(zzzz) 65(105)(zzzza) 65(105)(zzzzb) 65(105)(zzzzc) 65(105)(zzzzd) 65(105)(zzzze) 65(105)(zzzzf) 65(105)(zzzzg) 65(105)(zzzzh) 65(105)(zzzzi) 65(105)(zzzzj) 65(105)(zzzzk) 65(105)(zzzzl) 65(105)(zzzzm) 65(105)(zzzzn) 65(105)(zzzzo) 65(105)(zzzzp) 65(105)(zzzzq) 65(105)(zzzzr) 65(105)(zzzzs) 65(105)(zzzzt) 65(105)(zzzzu) 65(105) (zzzzv) 65(105)(zzzzw)

166 16.06.2005 16.06.2005 16.06.2005 16.06.2005 16.06.2005 16.06.2005 01.05.2006 01.05.2006 01.05.2006 01.05.2006 01.05.2006 01.05.2006 01.05.2006 01.05.2006 01.05.2006 01.05.2006 01.05.2006 01.05.2006 01.05.2006 01.05.2006 01.05.2006 01.06.2007 01.06.2007 01.06.2007 01.06.2007 01.06.2007 01.06.2007 01.06.2007 16.05.2008 16.05.2008 16.05.2008 16.05.2008 16.05.2008 16.05.2008 01.09.2009 01.09.2009 01.09.2009 01.07.2010 01.07.2010 01.07.2010 01.07.2010 01.07.2010 01.07.2010 01.07.2010 01.07.2010 01.05.2011 01.05.2011

Service Tax

167

Question 3 (Imp.): Write a note on Constitutional validity of Service Tax. Answer: The Government derives its power to levy taxes from Constitution of India. As per article 265, no tax shall be levied or collected except by authority of law. As per Article 246, the Constitution in its Schedule VII has mentioned the matters on which Central Government and State Government can make laws. Such matters are divided into three categories. List I: Union List (It contains the matters in respect of which only the Central Government can make law). (Total Entries 97) List II: State List (It contains the matters in respect of which only the State Government can make law). (Total Entries 66) List III: Concurrent List (It contains the matters in respect of which both the Central Government and the State Government have power of legistration). (Total Entries 46) Initially there was no specific entry in the Union List for levying service tax. Service tax was levied by the Central Government by drawing power from Entry 97 of the Union List. Entry 97 is a residuary entry in List-I, which has been reproduced below: 97 Any other matter not enumerated in List II or List III including any tax not mentioned in either of those Lists. The residuary entry provides wide power to the Central Government in respect of taxation of the subjects not mentioned in the Lists given by the constitution. Entry 92C was introduced in the VII schedule in the Union List with effect from 07.01.2004 vide article 268A. Entry 92C reads as under: 92C Taxes on services But till date this entry (92C) is kept dormant (ineffective). Question 4 (V. Imp.): Write a note on sources of Service Tax Law. Answer: Sources of Service Tax Law are as given below: 1. Finance Act, 1994 2. Rules 3. Notifications 4. Circulars 5. Trade Notices 1. Finance Act, 1994 The provisions relating to service tax are given in chapter V of Finance Act, 1994. Later, Finance Act, 2003 inserted chapter VA in Finance Act, 1994. It contains section 64 to 96-I. In the subsequent years additional provisions were introduced. So far there is no Service Tax Act. In the year 2004, the provisions relating to charging of education cess on the service tax were made applicable through Chapter VI of the Finance (No.2) Act, 2004. Thereafter secondary and higher education cess was levied through Chapter VI of Finance Act, 2007. 2. Rules on Service Tax Section 94/96-I of Finance Act, 1994 grants power to the Central Government to make rules for service tax. Central Government has made rules as given below: (i) (ii) (iii) (iv) (v) (vi) Service Tax Rules, 1994 Point of Taxation Rules, 2011 Service Tax (Determination of Value) Rules, 2006 Taxation of Services (Provided from Outside India and Received in India) Rules, 2006. Export of Service Rules, 2005 Service Tax Return Preparer Scheme, 2009

Service Tax (vii) Works contract (Composition Scheme for Payment of Service Tax) Rules, 2007 (viii) Service Tax (Registration of Special Category of Persons) Rules, 2005 (ix) CENVAT credit Rules, 2004

168

3. Notifications on Service Tax Section 93, 94, 96-I of Finance Act, 1994 empowers Central Government to issue notifications to exempt any service from service tax. 4. Circulars and office letters on Service Tax The Central Board of Excise and Custom issues circular and instruction to explain scope of taxable services and the scheme of service tax administration. The circulars bring out the real intention of the legislature. 5. Trade Notices on Service Tax Trade notice are issued by the commissioners for effective administration of service tax law. The trade notice are circulated among the field officers and also copies are given to Trade Associations. Question 5: Write a note on Administration of Service Tax. Answer: Ministry of Finance has two main Departments 1. Department of Revenue 2. Department of Expenditure All the taxes are covered under the Department of Revenue and the Department of Revenue has two Board of Taxes: 1. Central Board of Direct Taxes 2. Central Board of Excise and Customs Service Tax is administered by CBEC and the administration of service tax is as given below in the descending (decreasing) order: 1. Central Board of Excise and Customs 2. Chief Commissioners of Central Excise 3. Commissioners of Central Excise [Director General of Service Tax (Co-ordinator between 1 & 3)] The work relating to service tax is also under the control of CBEC, however in 1997 a new post of Director General (Service Tax) was formed. The functions and power of Direct General (Service Tax) are as follows: (1) To monitor the collection and assessment of service tax. (2) To study the staff requirement at field level. (3) To increase revenue collection or to streamline procedures. (4) To undertake study of law and procedures in relation to service tax with a view to simplify the service tax collection and assessment and make suggestions thereon. (5) To form a data base regarding the collection of service tax. (6) To inspect the service tax cells in the Commissionerate. (7) To undertake any other functions as assigned by the Board from time to time.

Service Tax Question 6: Write a note on Service Tax Procedures. Answer: Service tax procedures includes (i) Registration (ii) Maintenance of books and records (iii) Payment of service tax (iv) Availment and utilization of CENVAT credit (v) Filing of service tax returns (vi) Assessment and recovery of service tax (vii) Interest and penalties (viii) Rectification of mistakes (ix) Revision of assessment order (x) Appeals (xi) Search and seizure (xii) Advance rulings etc.

169

Question 7 (Imp.): Write a note on role of a Chartered Accountant in service tax. Answer: A Chartered Accountant is well equipped to position himself in the new role as an advisor and facilitator for due compliance of service tax law. The nature of services are: 1. Advisory services: A Chartered Accountant is considered to be qualified and competent to comprehend (understand) and interpret the service law to give required advice and guidance. 2. Procedural compliance: The service tax law requires Registration, Payment of tax, Filing of returns etc. A Chartered Accountant is well suited to assist the assessee in all the above functions and ensure compliance. 3. Personal representation: A Chartered Accountant is allowed to appear before the assessment authority, Commissioner (Appeals) and Custom, Excise and Service Tax Appellate Tribunal. With his expertise a Chartered Accountant can represent his clients. 4. Certification and audit: Service tax Returns and Financial Statements can be certified by the Chartered Accountant. Question 8 (V. Imp.): Write a note on Extent, Commencement and Application of Service Tax. Answer: Extent, Commencement and Application Section 64 The Finance Act, 1994 came into force from 01.07.1994. The Act extends to the whole of India except the state of Jammu and Kashmir. Thus, services provided in the state of Jammu and Kashmir are not liable to service tax. As per Article 370 of the Constitution, any Act of Parliament applies to Jammu & Kashmir only with concurrence (approval) of State Government. Since, no such concurrence has been obtained in respect of Finance Act, 1994, service tax provisions are not applicable in Jammu and Kashmir. Service tax will not be payable if service is provided in Jammu & Kashmir, however, if a person from Jammu & Kashmir provides the service outside Jammu & Kashmir, the service will be liable to service tax. Merely because the office of the service provider is situated in Jammu & Kashmir, it does not mean that service is provided in Jammu & Kashmir. Service provided within the Territorial Waters will be liable to service tax, as levy of service tax extends to

Service Tax

170

whole of India except Jammu and Kashmir and India includes territorial waters. Indian territorial waters extend upto 12 nautical miles from the Indian land mass. (1 nautical mile = 1.852 km) The provisions are applicable even in the Exclusive Economic Zone of India i.e. the provisions are applicable upto 200 nautical miles from the baseline (Notification no. 1/2002 dated 01.03.2002) As per Notification No. 21/2009 ST, dated 07.07.2009, Service provided to or from Installations, Structures and Vessels in entire Continental Self and Exclusive Economic Zone of India have been brought within the ambit of provisions relating to Service Tax. Question 9: Is service tax destination based consumption tax? Answer: True, service tax is a destination based consumption tax. It means that tax shall be payable only if service tax is imposed at the place where the service is used. Service tax is applicable all over India except the state of J & K. Therefore if services are used in J & K or outside India then it shall not be taxable. Example Mr. A of Delhi provided service to Mr. B of J & K. the service was provided and used in J & K. No service tax shall be payable in such cases. Example Salman and Associates is a firm of Chartered Accountant situated in the state of Jammu and Kashmir and provides accounting services to A Ltd. of Delhi. In this case, Salman and Associates is liable to pay service tax. As per section 64 The provisions of service tax apply to whole of India, except Jammu and Kashmir As service tax is a destination based consumption tax. Therefore services consumed in J & K are not liable to service tax. But in the given case the services are provided and consumed in Delhi (i.e. outside J & K) Hence Salman and Associates will be liable to charge tax from A Ltd. Example Bharti Ltd. is a company which provides telecommunication services under the brand name of Airtel. The Company has its registered office situated in Delhi. Mr. A is a resident of Jammu and is a subscriber of mobile connection of Airtel. In this case, Bharti Ltd. cannot charge service tax from Mr. A. Service tax is a destination based consumption tax. It is immaterial that where the office of the service provider is located. If service is received and consumed in J & K then it shall not be taxable. Therefore, Mr. A subscribing to the services of Airtel in Jammu and Kashmir will not be liable to pay tax. Question 10: Write a note on charge of Service Tax. Answer: Charge of Service Tax Section 66 Section 66 is the charging section which deals with levy and collection of service tax. It prescribes the rate applicable for levying of service tax. The prescribed manner for collection and payment of service tax is provided in the Service Tax Rules, 1994. The rate of service tax at present is 10% and education cess and secondary and higher education cess is payable @ 3% on service tax.

Service Tax

171

Service tax is payable on the value of taxable services and valuation is given in section 67 of the Finance Act, 1994.E.g. If a service provider has issued a bill of `10,00,000, service tax shall be `10,00,000 x 10.3% = `1,03,000 Alternative rate of service tax in case of certain services Although service tax is levied at the basic rate of 10% but in case of certain services, an alternative rate is also provided and is asunder: (1) In case of Life Insurance Services alternate mode of discharge of service tax liability has been provided and the rate of service tax in this case is 1.5% plus EC of total premium charged from the policy holders. (2) In case of Air Travel Agents Services, the Air Travel Agent, shall have the option of pay service tax of an amount calculated as under: (a) In case of domestic bookings 0.6% plus EC of the basic fare. (b) In case of international bookings 1.2% plus EC of the basic fare (3) In case of purchase and sale of foreign currency provided by foreign exchange broker, etc. provided consideration for the service provided is not shown separately in the invoice (a) 0.1% of the gross amount of currency exchanged upto ` 100,000, but minimum ` 25; (b) on next 9,00,000 @ 0.05% (c) on balance @ 0.01% but maximum `5,000. Question 11 (V. Imp.): What is the exemption limit for the Small Service Provider. Answer: As per Notification No.6/2005, dated 01.03.2005, exemption is allowed upto `10,00,000 provided in the preceding year, value of taxable services rendered is not exceeding `10,00,000. While computing amount of `10,00,000, value of services which are exempt from service tax shall not be considered. If any person is rendering services from more than one premises, value of services rendered from all the premises shall be taken into consideration. If any person is rendering more than one services, value of all the services shall be taken into consideration. If any abatement is available, value before permitting abatement shall be taken into consideration. Example If a Mandap Keeper has started rendering services in the Financial Year 2011-12 and he is providing catering services also and the amount to be received for the services rendered by him is `12,00,000, in this case, abatement shall be allowed for 40% and remaining 60% is taxable and in order to check the limit of 10,00,000, gross amount before abatement shall be taken into consideration and in this case service tax shall be payable with regard to `2,00,000 but after permitting abatement @ 40% and Mandap Keeper should charge service tax in the manner given below: Gross amount 12,00,000 Less: Exemption 10,00,000 Balance amount 2,00,000 60% of balance amount 1,20,000 Amount of service tax @ 10.3% 12,360 No CENVAT Credit is allowed till the service provider is availing exemption under Notification No.

Service Tax 6/2005.

172

Example A Chartered Accountant has started his practice wef 01.04.2011 and amount received upto 31.12.2011 is `10,00,000 and he has taken input services of `3,00,000 and paid service tax of `30,900, in this case, cenvat credit for input services is not allowed and also he cannot charge service tax on `10,00,000. If he has rejected general exemption of `10,00,000, in that case he should charge service tax of 10,00,000 x 10.3% = `1,03,000 and also cenvat credit is allowed for input services amounting to `30,900 and net tax payable shall be `72,100 The service provider has the option not to avail exemption and such option can not be withdrawn during the relevant year, however, it can be withdrawn from the beginning of next year. If any person is registered as per section 68(2), rule 2(1)(d), no such exemption shall be allowed and tax shall be payable right from the beginning i.e. in case of Reverse Charge mechanism, general exemption of `10,00,000 is not allowed. Question 12 (V. Imp.): Write a note on General Exemptions under Service Tax. Answer: 1. Exemption to all services provided to United Nations or International Organisation as per Notification No 16/2002 dated 02.08.2002 Central Government has exempted all the taxable services specified in section 65(105) of the Act provided by any person to the United Nations or an International Organisation, from whole of the service tax leviable thereon under section 66 of the Act. International Organisation means an International Organisation declared by the Central Government. 2. Exemption to services provided to a Developer or Units of Special Economic Zone as per Notification No. 17/2011 dated 01.03.2011 Taxable service provided to a developer of special economic zone or a unit (including a unit under construction) of special economic zone by any service provider, for consumption of the services within such special economic zone, are exempt from the whole of service tax. If the services are consumed outside SEZ, the service provider should charge service tax however, the recipient can claim refund. 3. Exemption to service provided by or to the Reserve Bank of India as per Notification No. 22/2006 ST dated 31.05.2006 Following taxable services are exempted from the whole of the service tax leviable thereon: (i) taxable services provided to any person, by the Reserve Bank of India; (ii) taxable services provided by any person, to the Reserve Bank of India when the service tax for such is liable to be paid by the Reserve Bank of India i.e. if reverse charge is not applicable, service tax shall be charged by the service provider from Reserve Bank and shall be paid to the Government. (iii) taxable services received in India from outside India by the Reserve Bank of India. 4. Service tax not leviable on fee collected by Public Authorities while performing Statutory Functions/ Duties under the provisions of law as per Circular No. 89/7/2006 ST dated 18.12.2006 / 96/7/2007 ST dated 23.08.2007

Service Tax

173

Service tax shall not be leviable on fee collected by public authorities while performing statutory functions/duties. A number of sovereign/public authorities (i.e. an agency constituted/set up by government) perform certain functions/duties, which are statutory in nature. For example the Regional Transport Officer (RTO) issues fitness certificate to the vehicles; or Explosive Department inspects and issues certificate for petroleum storage tank, LPG/CNG tank in terms of provisions of the relevant laws. Such activity is purely in public interest and it is undertaken as mandatory and statutory function. Therefore, no service tax is leviable on such activities. 5. Exemption to services provided to Foreign Diplomatic Mission or Consular Post in India as per Notification No. 33/2007 dated 23.05.2007 6. Exemption to services provided for personal use or for use of Family Members of Diplomatic Agents or Career Consular Officers posted in Foreign Diplomatic Mission / Consular Post in India as per Notification No. 34/2007 dated 23.05.2007 7. Exempts all taxable services, provided or to be provided, by a Technology Business Incubator (TBI) or a Science and Technology Entrepreneurship Park (STEP) Recognized by the National Science and Technology Entrepreneurship Development Board (NSTEDB) of the Department of Science and Technology, Government of India, from the service tax. As per Notification No. 9/2007, the Central Government, exempts all taxable services, provided or to be provided, by a Technology Business Incubator (TBI) or a Science and Technology Entrepreneurship Park (STEP) recognized by the National Science and Technology Entrepreneurship Development Board (NSTEDB) of the Department of Science and Technology, Government of India, from the whole of the service tax leviable thereon under section 66 of the said Finance Act. The STEP or the TBI, who intends to avail the exemption, shall furnish the requisite information in prescribed format containing the details of the incubator along with the information in prescribed format received from each incubatee to the concerned Assistant Commissioner or the Deputy Commissioner of Central Excise, as the case may be, before availing the exemption; and 8. Exempts taxable services provided by an entrepreneur located within the premises of a Technology Business Incubator (TBI) or Science and Technology Entrepreneurship Park (Step) Recognized by the National Science and Technology Entrepreneurship Development Board (NSTEDB) of the Department of Science and Technology Government of India from the service tax leviable. As per Notification No. 10/2007, Central Government, exempts taxable services provided by an entrepreneur located within the premises of a Technology Business Incubator (TBI) or Science and Technology Entrepreneurship Park (STEP) recognized by the National Science and Technology Entrepreneurship Development Board (NSTEDB) of the Department of Science and Technology Government of India from the whole of the service tax leviable thereon under section 66 of the said Finance Act subject to the following conditions, namely:(i) the entrepreneur enters into an agreement with the TBI or the STEP as an incubatee, to enable himself to develop and produce hi-tech and innovative products; and (ii) Exemption shall apply for a period of three years from the date on which such entrepreneur enters into an agreement with the TBI or the STEP.

Service Tax

174

(iii) The exemption shall not be available immediately after the total business turnover of the entrepreneur exceeds fifty lakh rupees during a given financial year. Question 13 (V. Imp.): Write a note on Registration under Service Tax. Answer: Registration Section 69/Rule 4(1) of STR, 1994 A service provider can charge service tax only if he is registered with the Service Tax Department. As per Notification No. 26/2005 dated 07.06.2005, every service provider in whose case, gross receipt has exceeded `9,00,000 shall apply for registration to the Service Tax Department in Form No ST-1 within 30 days from the date of crossing the limit of `9,00,000, however, he will charge service tax after crossing the limit of `10,00,000. He will also submit a copy of Permanent Account Number and proof of residence. Department shall grant him a registration certificate in Form No ST-2 within 7 days from the date of receipt of application otherwise the service provider shall be deemed to be registered. The service provider shall be given a Registration Number by the Department which will be called STP code i.e. Service Tax Payer Code and it will be 15 digit PAN based number and first 10 digit shall be that of PAN and remaining 5 digit shall be allotted by Service Tax Department e.g. AAEPC1298D ST-001 The last three digit shall indicate total number of registration for the same permanent Account Number. If a person is providing more than one taxable service, he may give only one application. He should mention in the application all the taxable services provided by him. A person may apply for voluntary registration at any time and also a person may forgo (reject) the general exemption of `10,00,000. As per section 77, any person who has failed to take registration shall pay penalty of `200 per day or `10,000 whichever is higher. Question 14 (V. Imp.): Write a note on Centralized Registration under Service Tax. Answer: Centralised Registration Rule 4(2) of STR, 1994 If a service provider is providing services from more than one premises, in such cases he can apply for separate registration for each of such premises. He may apply for a single registration called Centralized Registration provided he has either centralized accounting or centralized billing system. In such cases one of the places shall be considered to be Head Office and all other premises shall be considered to be branches. A single registration certificate shall be issued. If the service provider do not have centralized accounting and also there is no centralized billing system, he will not be allowed to apply for centralized registration. Question 15: Write a note on Amendment of Registration Certificate. Answer: Amendment of Registration Certificate Rule 4(5A) of STR, 1994 A service provider may apply for amendment in registration in the following cases: 1. Change in place of business 2. Change in the name of business 3. Change in services rendered i.e. there may be addition / deletion 4. Any other similar change If there is any such change, service provider should apply to the department within 30 days for effecting the change.

Service Tax

175

Question 16: Write a note on Cancellation of Registration Certificate. Answer: Cancellation of Registration Certificate Rule 4(7), 4(8) of STR, 1994 Every registered assessee, who ceases to provide the taxable service for which he is registered or if he dies, registration certificate shall be cancelled. The Superintendent of Central Excise shall ensure that the assessee has paid all monies due to the Central Government under the provisions of the Act, and the rules and the notifications issued thereunder, and thereupon cancel the registration certificate. Where a registered assessee transfers his business to another person, the transferee shall obtain a fresh certificate of registration. Question 17: Write a note on issue of Invoice/Bill by Service Provider. Answer: As per Rule 4A of STR, 1994, every person providing taxable service, not later than fourteen days from the date of completion of such taxable service or receipt of any payment towards the value of such taxable service, whichever is earlier, shall issue an invoice, a bill or, signed by such person or a person authorized by him and such challan etc. shall be serially numbered and shall contain the following, namely: (i) the name, address and the registration number of such person; (ii) the name and address of the person receiving taxable service; (iii)description, classification and value of taxable service provided or to be provided; and (iv) the service tax payable thereon : Provided also that in case of continuous supply of service, every person providing such taxable service shall issue an invoice, bill or challan, as the case may be, within fourteen days of the date when each event specified in the contract, which requires the service receiver to make any payment to service provider, is completed: Every input service distributor distributing credit of taxable services shall, in respect of credit distributed, issue an invoice, a bill or, as the case may be, a challan signed by such person or a person authorized by him, for each of the recipient of the credit distri-buted, and such invoice, bill or, as the case may be, challan shall be serially numbered and shall contain the following, namely: (i) the name, address and registration number of the person providing input services and the serial number and date of invoice, bill, or as the case may be; (ii) the name and address of the said input service distributor; (iii) the name and address of the recipient of the credit distributed; (iv) the amount of the credit distributed : Question 18: Explain Determination of Rate for Service Tax. Answer: As per rule 5B of STR, 1994, the rate of tax in case of services provided, or to be provided, shall be the rate prevailing at the time when the services are deemed to have been provided under the rules made in this regard. Question 19 (V. Imp): Write a note on payment of Service Tax. Answer: Payment of Service Tax Section 68/ Rule 6 of STR, 1994

Service Tax

176

An individual or proprietary firm or partnership firm shall be required to pay service tax on quarterly basis and should be paid upto 5th of the month subsequent to the quarter in which the service is deemed to be provided (i.e. the quarter in which point of taxation falls) but if the payment is being made electronically through internet banking, payment can be made upto 6 th of such month. However, payment for the last quarter should be made upto 31st March of that year. Any other service provider like Company or HUF etc. shall be required to pay service tax on monthly basis and it should be paid upto 5th of the month succeeding the month in which service is deemed to be provided but if payment is being made electronically through internet banking, payment can be made upto 6 th of the month however, payment for last month should be made upto 31st March of that year If an assessee has paid a total service tax of rupees ten lakh or more including the amount paid by utilization of Cenvat Credit, in the preceding financial year, he shall deposit the service tax liable to be paid by him electronically, through internet banking. As per section 77, if any person has failed to pay service tax electronically, he shall pay penalty which may extend upto `10,000. The assessee shall deposit the service tax liable to be paid by him in Form G.A.R-7 with the bank designated by the Central Board of Excise and Customs for this purpose. If the assessee deposits the service tax by cheque, the date of presentation of cheque to the bank designated by the Central Board of Excise and Customs for this purpose shall be deemed to be the date on which service tax has been paid subject to realization of that cheque. Rounding off of Duty, etc Section 37D (Central Excise Act, 1944) The amount of duty, interest, penalty, fine or any other sum payable, and the amount of refund or any other sum due, under the provisions of this Act shall be rounded off to the nearest rupee and, for this purpose, where such amount contains a part of a rupee consisting of paise then, if such part is fifty paise or more, it shall be increased to one rupee and if such part is less than fifty paise it shall be ignored. Question 20 (V. Imp.): Write a note on Point of Taxation for Payment of Service Tax. Answer: Point of Taxation for Payment of Service Tax Rule 3 Prior to 01.04.2011, service tax was to be paid on actual receipt basis but w.e.f. 01.04.2011, service tax shall be paid on the basis of point of taxation i.e. the point in time when the service shall be deemed to have been provided shall be taken into consideration. As per Rule 3, services shall be deemed to have been provided on the date of invoice issued by the service provider but if the invoice has been issued after expiry of 14 days from the date of completion of service, services shall be deemed to have been provided on the date of completion of service. If payment has been received before the date determined above, the date of providing the services shall be the date of receiving the payment and service tax should be paid accordingly. It can be further shown as given below: S. No. 1. Date of completion of service April 10, 2011 April 20, 2011 Date of invoice Date on which payment recd. April 30, 2011 Point of Taxation April 20, 2011 Invoice issued in 14 days Remarks

Service Tax

177 and before receipt of payment Invoice not issued within 14 days and payment received after completion of service Invoice issued in 14 days but payment received before invoice Invoice not issued in 14 days, point of taxation 10th April but for part payment received on 05th April 2011, POT shall be 05th April 2011. Invoice issued before completion of service

2. 3. 4.

April 10, 2011 April 10, 2011 April 10, 2011

April 26, 2011 April 20, 2011 April 26, 2011

April 30, 2011 April 15, 2011 April 5, 2011 (part payment) and April 25, 2011 (balance payment) 15th April 2011

April 10, 2011 April 15, 2011 April 5, 2011 (part payment) and April 10, 2011 (balance payment) 01st April 2011

5.

April 10, 2011

1st April 2011

Service tax to be paid on actual receipt basis in certain cases Rule 7 As per rule 7 of point of taxation in the following cases, service tax shall be paid on actual receipt basis. 1. Individuals or proprietary firms or partnership firms providing taxable services referred below: (i) Consulting Engineer (ii) Practising Chartered Accountant (iii) Scientist or a Technocrat (iv) Legal Consultancy Services (v) Practising Company Secretary (vi) Practising Cost Accountant (vii) Interior Decorator (viii) Architect 2. In case of Reverse Charge, but payment should be made within a period of 6 months of the date of invoice otherwise point of taxation shall be determined in the normal manner. Question 21: Write a note on interest on delayed payment of Service Tax. Answer: Interest on delayed payment of service tax Section 75 If any person has failed to credit the tax to the Central Government within the prescribed time period, such person shall pay interest for the period of delay at the prescribed rate. As per notification no. 14/2011 dated 01.03.2011, the rate of interest shall be 18% p.a. Provided further that in the case of a service provider, whose value of taxable services provided in a financial year does not exceed sixty lakh rupees during any of the financial years covered by the notice or during the last preceding financial year, as the case may be, such rate of interest shall be reduced by three per cent per annum i.e. rate shall be 15% p.a. Question 22: Write a note on penalty for failure to pay Service Tax. Answer: Penalty for failure to pay service tax Section 76 If any person liable to pay service tax has failed to pay service tax, he shall be liable to pay service tax alongwith interest @ 18% p.a. and also penalty may be imposed which shall be `100 per day or 1% of such tax, per month whichever is higher starting with first day after the due date till the date of actual payment. However total amount of penalty shall not exceed 50% of service tax payable.

Service Tax

178

Example ABC Ltd, an assessee, fails to pay service tax of ten lakh rupees payable by 5 th March. ABC Ltd. pays amount on the 15th March. The default has continued for ten days The penalty payable by ABC Ltd. is computed as follows: 100 x 10 = `1,000 10,00,000 x 1% x 10/31 = `3,225.80 whichever is higher Penalty liable to be paid is ` 3226.00. Example Mr. X, an assessee, fails to pay service tax of `3,000 payable by 31st March. Mr. X pays the amount on 30 th April. The default has continued for 30 days. The penalty payable by Mr. X is computed as follows: 100 x 30 = `3,000 3,000 x 1% x 30/30 = `30 whichever is higher. Penalty liable to be paid is `3,000 but penalty cannot exceed 50% of the amount of service tax, hence amount of penalty shall be `1,500 Question 23: Write a note on penalty for contravention of rules and provisions of Act for which no penalty is specified elsewhere. Answer: Penalty for contravention of rules and provisions of Act for which no penalty is specified elsewhere Section 77 (1) Any person who has failed to take registration shall pay penalty of `200 per day or `10,000 whichever is higher (2) Any person who has failed to maintain the book of accounts shall pay penalty which may extend upto `10,000. (3) Any person who has failed to furnish information or failed to furnish any document to the service tax officer shall pay penalty of `200 per day or `10,000 whichever is higher. (4) If any person has failed to pay service tax electronically shall pay penalty which may extend upto `10,000 (5) Any person, who contravenes any of the provisions of this Chapter or any rules made thereunder for which no penalty is separately provided in this Chapter, shall be liable to a penalty which may extend to ten thousand rupees. Question 24: Write a note on penalty for suppressing (to conceal) value of taxable services. Answer: Penalty for suppressing value of taxable service Section 78 If any person has not paid service tax or has short paid service tax or there was a refund of service tax because of fraud, wilful mis-statement etc., in such cases he shall be liable to pay service tax alongwith interest and also penalty shall be payable which shall be equal to the amount of service tax. Provided that where true and complete details of the transactions are available in the specified records, penalty shall be reduced to 50% of the service tax so not levied or paid or short-levied or short-paid or

Service Tax

179

erroneously (wrongly) refunded. If the tax and interest has been paid within 30 days of receiving the orders from the Central Excise Officer, penalty payable shall be 25% of such service tax. Further such penalty should also be paid within the period of 30 days as mentioned above. If the service provider whose value of taxable services do not exceeds `60 lakhs during any of the year covered by the notice or during the last preceding financial year, the period of 30 days shall be taken as 90 days. Provided also that if the penalty is payable under this section, the provisions of section 76 shall not apply. Question 25 (Imp.): Write a note on Advance Payment of Service Tax. Answer: Advance payment of Service Tax Rule 6 of STR, 1994 Every person liable to pay service tax, may, on his own volition (voluntary), pay an amount as service tax in advance, to the credit of the Central Government and adjust the amount so paid against the service tax which he is liable to pay for the subsequent period : Provided that the assessee shall, (i)intimate the details of the amount of service tax paid in advance, to the jurisdictional Superintendent of Central Excise within a period of fifteen days from the date of such payment; and (ii) indicate the details of the advance payment made, and its adjustment, if any in the subsequent return to be filed under section 70 of the Act. Question 26 (Imp.): Write a note on Refund of Service Tax/ Issue of Credit Note. Answer: Refund of Service Tax / Issue of Credit Note Rule 6 of STR, 1994 Where an assessee has issued an invoice, or received any payment, against a service to be provided which is not so provided by him either wholly or partially for any reason or where the amount of invoice is renegotiated due to deficient provision of service, or any terms contained in a contract, the assessee may take the credit of such excess service tax paid by him, if the assessee, (a) has refunded the payment or part thereof, so received for the service provided to the person from whom it was received; or (b) has issued a credit note for the value of the service not so provided to the person to whom such an invoice had been issued. Question 27 (Imp.): Write a note on Provisional Payment of Service Tax. Answer: Provisional payment of Service Tax Rule 6 of STR, 1994 In case the assessee is unable to correctly estimate, at the time of the deposit, the actual amount of service tax for any month or quarter, he may make a written request to Assistant/ Deputy Commissioner of Central Excise for making payment of service tax on provisional basis. The concerned officer may allow payment of service tax on provisional basis on such value of taxable service as may be specified by him. For the purpose of provisional assessment at the time of filing the return, the assessee is required to file a statement in form ST - 3A giving detail of difference between service tax deposited and the service tax liable to be paid for each month. The Assistant/Deputy Commissioner of Central Excise, on the basis of memorandum in form ST - 3A may complete the assessment after calling for necessary documents or records, if need be.

Service Tax

180

Question 28 (V. Imp.): Write a note on Self Adjustment of Service Tax. Answer: Self adjustment of Service Tax Rule 6 of STR, 1994 Where an assessee has paid to the credit of Central Government any amount in excess of the amount required to be paid towards service tax liability for a month or quarter, as the case may be, the assessee may adjust such excess amount paid by him against his service tax liability for the succeeding month or quarter, as the case may be. The adjustment of excess amount paid, shall be subject to the following conditions, namely: (i) excess amount paid is on account of reasons not involving interpretation of law, taxability, classification, valuation or applicability of any exemption notification, (ii) excess amount paid by an assessee having centralised registration, on account of delayed receipt of details of payments towards taxable services may be adjusted without monetary limit, (iii) in cases other than specified in clause (ii) above, the excess amount paid may be adjusted with a monetary limit of two lakh rupees for a relevant month or quarter, as the case may be, (iv) the details and reasons for such adjustment shall be intimated to the jurisdictional Superintendent of Central Excise within a period of fifteen days from the date of such adjustment. As per section 11B of Central Excise Act, 1944 if excess amount paid is on account of reasons involving interpretation of law, taxability, classification, valuation or applicability of any exemption notification, in such cases, the service provider should claim refund and for this purpose he should make an application before expiry of one year from the date of making the payment to Central Excise Officer and he shall grant him refund provided incidence of service tax has not been passed on to some other person otherwise amount shall be credited to Consumer Welfare Fund. As per section 11BB if refund has not been granted within 3 months from the date of receipt of the application, interest shall be allowed @ 6% p.a. from the date immediately after expiry of the period of 3 months till the date of refund (Notification No. 67/2003 dated: 12.09.2003) Question 29 (Imp.): Explain payment of service tax in case of Associated Enterprises. Answer: The term associated enterprises is not defined under service tax provision rather it is defined under section 92A of Income Tax Act and it means an enterprise (a) which participates, directly or indirectly, or through one or more intermediaries, in the management or control or capital of the other enterprise; or (b) in respect of which one or more persons who participate, directly or indirectly, or through one or more intermediaries, in its management or control or capital, are the same persons who participate, directly or indirectly, or through one or more intermediaries, in the management or control or capital of the other enterprise. Example Telco manufactures Mercedes cars in India using the know how of Mercedes Germany. Mercedes cars cannot be manufactured in India without such know how and therefore Telco & Mercedes Germany are associated Enterprises.

Service Tax

181

Example Unilever U.K. holds 26% equity shares of Hindustan Lever. Therefore Unilever & Hindustan Lever are associated Enterprises. Point of Taxation As per Rule 7 of Point of Taxation Rule, in case of associated enterprises, where the person providing the service is located outside India, the point of taxation shall be the date of credit in the books of account of the person receiving the service or date of making the payment whichever is earlier. Question 30: Write a note on adjustment for Municipal Tax paid for House Property. Answer: Adjustment for Municipal Tax paid for house property Rule 6 of STR, 1994 Where the person liable to pay service tax in respect of renting of immovable property, has paid any amount in excess of the amount required to be paid towards service tax liability on account of non-availment of deduction of property tax paid, from the gross amount charged for renting of the immovable property, the assessee may adjust such excess amount paid by him against his service tax liability within one year from the date of payment of such property tax. The details of such adjustment shall be intimated to the Superintendent of Central Excise within a period of fifteen days from the date of such adjustment. Example X Ltd. Gives a commercial property on rent to Y Ltd. (monthly rent being `2,00,000) Y Ltd. Pays rent on the last day of each month. Municipal tax for the financial year 2011-12 is Rs.4,44,000 (due date of payment is June 30,2011, whereas it is actually paid on September 30, 2011 along with late payment interest of Rs.16,200) Value of taxable service for the financial Year 2011-12 will be as follows: Financial Year 2011-12 April May June July August September October November December January February March Gross rent ` 2,00,000 2,00,000 2,00,000 2,00,000 2,00,000 2,00,000 2,00,000 2,00,000 2,00,000 2,00,000 2,00,000 2,00,000 Municipal tax (`4,44,000 12 = `37,000) ` 1,85,000 + 37,000 = 2,22,000 37,000 + 22,000 = 59,000 37,000 37,000 37,000 37,000 37,000 Taxable Value of service ` 2,00,000 2,00,000 2,00,000 2,00,000 2,00,000 (-) 22,000 1,41,000 1,63,000 1,63,000 1,63,000 1,63,000 1,63,000

Municipal tax is paid on September 30, 2011. Municipal tax liability from April 2011 till August 2011 comes to Rs.1,85,000 (i.e., `37,000 5). This unadjusted amount can be adjusted at any time within 1Year from the date of payment of municipal tax. The details of such adjustment should be informed to jurisdictional Superintendent of Central Excise within period of 15 days from date of adjustment [rule 6(4c) of Service Tax Rules]

Service Tax

182

Question 31 (Imp.): Write a note on service tax collected wrongly or in excess from any person. Answer: As per section 73A, if any person has collected any service tax in excess or wrongly from the recipient of taxable service in any manner, in that case such person should deposit such amount with the Central Government. Central Government shall refund the amount to the person from whom tax has been collected and for this purpose Central Government may issue a public notice and such person can make an application within 6 months from the date of public notice. If the amount has not been claimed by any person, it will be deposited in the Consumer Welfare Fund which is regulated by a committee and its members are nominated by the Government. The amount is utilized for the welfare of consumer in India. As per section 73B, such person shall also pay interest @ 18% p.a. from the first day of the month succeeding the month in which the amount ought to have been paid, till the date of payment of such amount. However if the total value of taxable services in the relevant financial year or in the preceding year is not exceeding `60,00,000, rate of interest shall be 15% p.a. Question 32 (Imp.): Write a note on Point of Taxation for determining the rate of Service Tax. Answer: Point of taxation for determining the rate of Service Tax Rule 4 of POT Rules, 2011 Rate of service tax shall be determined as per Rule 4 in the manner given below: Case 1: Taxable services have been completed before the change in effective rate of tax Situation where the invoice issued as well as the payment received after the change in effective rate of tax where the invoice is issued prior to change in effective rate of tax but the payment is received after the change in effective rate of tax where the payment is received before the change in effective rate of tax, but the invoice for the same has been issued after the change in effective rate of tax It will be further clear with help of the following illustrations. S. No. 1. 2. 3. Date of completion of service April 10, 2011 April 10, 2011 April 10, 2011 Change in effective rate of tax April 25, 2011 April 25, 2011 April 25, 2011 April 26, 2011 April 15, 2011 April 30, 2011 April 30, 2011 April 30, 2011 April 15, 2011 April 26, 2011 April 15, 2011 April 15, 2011 New rate Old rate Old rate Date of invoice Date on which payment recd. Point of Taxation Rate applicable Date of Receipt Point of Taxation Date of Receipt or issuing of invoice, whichever is earlier Date Of Issue Of Invoice

Case 2: Services have been completed after the change in effective rate of tax Situation Point of Taxation

Service Tax where the payment for the invoice is made after the date of Receipt of payment change in effective rate of tax but the invoice has been

183

issued prior to the change in effective rate of tax where the invoice has been issued and the payment date of receipt of payment or date of issuance of for the invoice received before the change in effective invoice, whichever is earlier rate of tax where the invoice has also been raised after the date of issuing of invoice change in effective rate of tax but the payment has been received before the change in effective rate of tax The applicability of the rule will be clear from the illustrations in the following table: S. No. 1. 2. 3. Date of completion of service April 30, 2011 April 30, 2011 April 30, 2011 Change in effective rate of tax April 25, 2011 April 25, 2011 April 25, 2011 Date of invoice April 20, 2011 April 20, 2011 April 30, 2011 Date on which payment recd. April 30, 2011 April 15, 2011 April 20, 2011 Point of Taxation April 30, 2011 April 15, 2011 April 30, 2011 Rate of Tax New rate Old rate New rate

Question 33: Write a note on liability to service tax in case of New Services. Answer: New Services Rule 5 of POT Rules, 2011 As per Rule 5, in case of new services, no service tax shall be charged if invoice has been issued and also payment has been received before such service became taxable. If payment has been received before the service became taxable and also invoice has been issued within 14 days, no service tax shall be charged. Example Hotel accommodation services have become taxable w.e.f. 01.05.2011 and Ashoka Hotel has received ` 12,00,000 from ABC Ltd. in connection with stay of their employees for one month from 01.05.2011 to 30.06.2011. Full payment was received on 18.04.2011 and invoice was issued on the same date, in this case, no service tax is payable. If in this case, payment was received on 27.04.2011 and invoice was issued on 07.05.2011, even then services are exempt. If payment was received on 01.05.2011 and invoice was issued on 03.05.2011, in this case, service tax shall be paid however, exemption shall be allowed for `10,00,000 and service tax shall be charged on the balance amount.

Question 34: What are the provisions of service tax for Continuous Supply of Service. Answer: Continuous Supply of Service Rule 6 of POT Rules, 2011 As per Rule 2 Continuous Supply of service means any service which is provided, or to be provided continuously, under a contract, for a period exceeding three months, or where the Central Government, by a

Service Tax

184

notification specify such service to be a continuous supply of service, whether or not subject to any condition. As per Rule 6 of point of taxation for continuous supply of service shall be determined as below Situation Point of Taxation For the service provided or to be provided The time when the invoice is issued Where the invoice is not issued within fourteen days of the Date of completion of Service completion of the provision of the service If Advance is Received Date of receipt of each such advance

Where the provision of the whole or part of the service is determined periodically on the completion of an event in terms of a contract, which requires the service receiver to make any payment to service provider, the date of completion of each such event as specified in the contract shall be deemed to be the date of completion of provision of service. For example, in the case of construction services if the payments are linked to stage-by-stage completion of construction, the provision of service shall be deemed to be completed in part when each such stage of construction is completed. The following services have been notified as continuous supply of services. (a) Telecommunication service [65(105)(zzzx)] (b) Commercial or industrial construction [65(105)(zzq)] (c) Construction of residential complex [65(105)(zzzh)] (d) Internet Telecommunication Service [65(105)(zzzu)] (e) Works contract service [65(105)(zzzza)] Thus these services will constitute continuous supply of services irrespective of the period for which they are provided or agreed to be provided. Other services will be considered continuous supply only if they are provided or agreed to be provided continuously for a period exceeding three months. Illustration 1: ABC Ltd. is a construction company and it has entered into an agreement with XYZ Ltd. to construct one building which will have 4 floor and as per the contract. Each floor shall be considered to be one stage of completion. ABC Ltd. submitted particulars as given below: Completed Ground Floor Issued Bill Half payment of ` 10,00,000 plus service tax received And balance payment of ` 10,00,000 plus Service Tax received Completed 1st floor Issued Bill Half payment of ` 9,00,000 plus service tax received And balance payment of ` 9,00,000 plus Service Tax received Completed 2nd Floor Issued Bill Half payment of ` 8,00,000 plus service tax received And balance payment of ` 8,00,000 plus Service Tax received on 16/06/2011 on 01/07/2011 on 27/06/2011 on 10/07/2011 on 27/08/2011 on 07/09/2011 on 31/08/2011 on 10/09/2011 on 30/11/2011 on 10/11/2011 on 30/10/2011 on 01/12/2011

Service Tax Completed 3rd Floor Issued Bill Full amount of ` 10,00,000 plus Service Tax received on 10/03/2012 on 01/04/2012 on 01/08/2012

185

In FY 10-11, Bill issued by company were ` 200 lakh and service tax was paid @ 10.3 % Payment of service tax for March 2012 was made on 21/04/2012 Show tax treatment of service tax for each month and also compute interest and penalty payable u/s 75 and 76 respectively. Solution: April 2011 May 2011 June 2011 Taxable value Service tax @10.3% July 2011 Aug 2011 Taxable value Service tax @ 10.3% Sept 2011 Taxable value Service tax @ 10.3 % Oct 2011 Taxable value Service tax @ 10.3 % Nov 2011 Taxable value Service tax @ 10.3 % Dec 2011 Jan, Feb 2012 March 2012 Taxable value Service tax @10.3% Delay in payment 21 days Interest u/s 75 ` 1,03,000 x 18% x 21/365 = 1066.68 Penalty u/s 76 ` NIL NIL 20,00,000 20,6000 NIL 9,00,000 92,700 9,00,000 92,700 8,00,000 82,400 8,00,000 82,400 NIL NIL 10,00,000 1,03,000 1,067

Service Tax ` 1,03,000 x 1% x 21/30 = ` 721 OR 21 x 100 = ` 2100 Whichever is higher Thus penalty will be

186

2,100

Question 35: Write a note on determination of Point of Taxation in case of Copyrights, etc. (For Self Reading) Answer: Determination of Point of Taxation in case of Copyrights, etc. Rule 8 In respect of royalties and payments pertaining to copyrights, trademarks, designs or patents, where the whole amount of the consideration for the provision of service is not ascertainable at the time when service was performed, and subsequently the use or the benefit of these services by a person other than the provider gives rise to any payment of consideration, the service shall be treated as having been provided each time when a payment in respect of such use or the benefit is received by the provider in respect thereof, or an invoice is issued by the provider, whichever is earlier. Question 36(V. Imp.): Write a note on Filing of Service Tax Return. Answer: Furnishing of Returns Section 70 / Rule 7 of STR, 1994 Every assessee liable to pay service tax shall submit a half yearly return in From ST-3 or ST-3A, as the case may be, along with a copy of the Form GAR-7 for the months / quarters covered in the half-yearly return. Every assessee shall submit the half yearly return by the 25th of the month following the particular half-year. If the last date is a holiday, return can be filed on the next day without any penalty. Every assessee shall submit the half-yearly return electronically. Every service provider who is registered shall also be required to file the return even if service tax liability is nil i.e. if any service provider has gross receipt exceeding `9,00,000 shall be required to get registered and in that case return has to be filed even if service tax liability is nil. Every assessee shall furnish to the Superintendent of Central Excise at the time of filing his return for the first time a list of all accounts maintained by the assessee in relation to service tax including memoranda received from his branch offices. Question 37: Write a note on Revision of Service Tax Return. Answer: Revision of Return Rule 7B of STR, 1994 An assessee may submit a revised return, in Form ST-3, in triplicate, to correct a mistake or omission, within a period of 90 days from the date of submission of the return under rule 7. Question 38 (V. Imp.): Write a note on Delay Furnishing of Service Tax Return. Answer: Amount to be paid for Delay in Furnishing Return Rule 7C of STR, 1994 Where the return prescribed under rule 7 is furnished after the date prescribed for submission of such return, the person liable to furnish the said return shall pay to the credit of the Central Government, for the period of delay of(i) 15 days from the date prescribed for submission of such return, an amount of `500. (ii) beyond 15 days but not later than 30 days from the date prescribed for submission of such return, an amount of `1,000; and

Service Tax

187

(iii) beyond 30 days from the date prescribed for submission of such return an amount of `1,000 plus `100 for every day from the thirty first day till the date of furnishing the said return. Provided that the total amount payable in terms of this rule, for delayed submission of return, shall not exceed `20,000. Provided also that where the gross amount of service tax payable is nil, the Central Excise Officer may, on being satisfied that there is sufficient reason for not filing the return, reduce or waive the penalty. Question 39: Write a note on contents of the Service Tax Return. Answer: Contents of the Return 1. Half year for which return is being filed 2. Name of the assessee 3. STC Number 4. Address 5. Constitution of the assessee like individual or partnership firm etc. 6. Assessee liable to pay service tax as service provider or service receiver under reverse charge 7. Gross amount received 8. Amount of service tax and education cess 9. Tax credit allowed 10. Net tax payable 11. Details of services rendered 12. Verification of the return 13. Signature of service provider or service recipient under reverse charge Question 40: Write a note on documents to be submitted along with Service Tax Return. Answer: Documents to be submitted along with the return Alongwith ST-3 return following documents should be attached: (i) copies of GAR-7 challans which indicate the payment of service tax. (ii) a memorandum in form ST-3A giving details of the difference between the amount of provisional tax deposited and the actual amount payable for each month. Every service tax assessee should furnish to the Superintendent of Central Excise, at the time of filing first half yearly return, a list of all accounts maintained by him in relation to service tax. The service tax return should be filed in triplicate to the Superintendent of Central Excise . The assessee may also file the return with the concerned Divisional Office by registered post However, the assesses should ensure that the return reaches the Divisional Office on or before the due date. For an assessee who provides more than one taxable service, filing of a single return is sufficient . However, the details in each of the columns of the Form ST-3 have to be furnished separately for each of the taxable service rendered by him. Even if no service has been provided during a half year and no service tax is payable; the assessee has to file a Nil return within the prescribed time limit. Question 41 (V. Imp.): Write a note on E-Filing of Service Tax Return. Answer: E-Filing of Return With effect from 01.10.2011, e-filing of service tax returns has been made mandatory for all the assesses (Notification No. 43/2011 dated: 25.08.2011).

Service Tax

188

The assessee can e-file the return through software ACES i.e. AUTOMATION OF CENTRAL EXCISE AND SERVICE TAX. The assessee should login the relevant site www.aces.gov.in. To transact business on ACES a user has to first register himself with ACES through a process called Registration with ACES. The assessee should have STP code i.e. PAN based 15 digit registration number allotted by Service Tax Department. The assessee should fill in self-chosen user-ID and his e-mail ID and system will check the availability of the chosen user ID and then generate a password and will send it to the assessee on the e-mail ID mentioned by him and the assessee can submit the return in the prescribed form. Benefits of E-Filing for assesee: 1) Reduce Physical Interface with the Department; 2) Save Time; 3) Reduce Paper Work; 4) Online Registration and Amendment of Registration; 5) Electronic filing of all documents such as applications for registration, returns, claims, permissions and intimations; export-related documents, refund request; 6) System-generated E-Acknowledgement; 7) Online tracking of the status of selected documents. Question 42: Write a note on Service Tax Return Preparer Scheme. Answer: Service Tax Return Preparer Scheme Section 71 Department shall appoint Service Tax Return Preparer to assist the persons to file service tax return. The STRP shall have required qualification i.e. he should have passed senior secondary level examination. The Service Tax Return Preparer shall (a) (b) (c) (d) (e) prepare the return with due diligence; affix his signature on the return prepared by him; furnish the return as specified; hand over a copy of the return to the person whose return is prepared and furnished by him; retain a copy of the acknowledgment of having furnished the return

In respect of returns prepared and furnished by him maintain record of the following, namely:(i) the name of assessees whose returns have been prepared and furnished by him during that month; (ii) the Service Tax Code(STC) number and premises code of such assessees; (iii) period for which return is filed; (iv) date of furnishing the return; (v) authority with whom return is filed; (vi) amount of tax payable; (vii) amount of tax paid; (viii) the fee charged and received by him Incentive to Service Tax Return Preparers An assessee shall pay a fee as may be mutually agreed upon between an assessee and the Service Tax Return Preparer. The Board recommends, as a yardstick, a fee of rupees one thousand rupees per return prepared by the Service Tax Return Preparer.

Question 43: Write a note on Records under Service Tax. Answer: Records Rule 5 of STR, 1994 The records including computerised data, as maintained by an assessee in accordance with the various laws in force from time to time shall be acceptable. All such records shall be preserved at least for a period of 5 years immediately after the financial year to which such records pertain. Every assessee shall make

Service Tax

189

available, at the registered premises, at all reasonable time, all such records, for inspection and examination by the Central Excise Officer authorised in writing by the jurisdictional Assistant Commissioner or Deputy Commissioner of Central Excise, as the case may be. Explanation. - For the purposes of this rule, registered premises includes all premises or offices from where an assessee is providing taxable services. As per section 77, any person who has failed to maintain the book of accounts shall pay penalty which may extend upto `10,000. Question 44 (V. Imp.): Write a note on option to pay service tax in case of Air Travel Agent. Answer: Option to pay service tax in case of Air Travel Agent Rule 6(7) As per section 65(105)(l) - Taxable service means any service provided or to be provided, to any person, by an Air Travel Agent in relation to the booking of passage for travel by air. As per section 65(4) -Air Travel Agent means any person engaged in providing any service connected with the booking of passage for travel by air. The person liable for paying the service tax in relation to the services provided by an air travel agent, shall have the option, to pay an amount at the rate of 0.6% plus EC of the basic fare in the case of domestic bookings, and at the rate of 1.2% plus EC of the basic fare in the case of international bookings, of passage for travel by air, during any calendar month or quarter, as the case may be, towards the discharge of his service tax liability instead of paying service tax at the normal rate and the option, once exercised, shall apply uniformly in respect of all the bookings of passage for travel by air made by him and shall not be changed during a financial year under any circumstances. Basic Fare means that part of the air fare on which commission is normally paid to the air travel agent by the airline. Illustration 2: Mr. Anurag is an Air Travel Agent and provides the following information for the month of October, 2011. Date Particulars Basic Fare Taxes Other Commission Total Value than Service tax 2/10/2011 Booked by Mr. A for 3,000 2,500 500 6,000 Delhi to Chennai 10/10/2011 Booked by Mr. Somnath 15,000 2,500 1,500 19,000 for Delhi to Bangkok 24/10/2011 Booked by Mr. Raj for 2,600 2,400 500 5,500 Delhi to Bhubaneswar Compute service tax payable by Mr. Anurag under various methods applicable for air travel agent. Solution: Method 1 Total Commission Service tax @ 10.3% Rounded off u/s 37D

2,500.00 257.50 258.00

Service Tax Method 2 As per Rule 6(7) of Service Tax Rules 1994 02.10.2011 3,000 x 0.618% 10.10.2011 15,000 x 1.236% 24.10.2011 2,600 x 0.618% Total Service Tax Payable Rounded off u/s 37D `

190

18.54 185.40 16.07 220.01 220.00

Question 45 (V. Imp.): Write a note on option to pay service tax by any insurer carrying on Life Insurance Business. Answer: Option to pay service tax by any insurer carrying on life insurance business Rule 6(7A) As per section 65(105)(zx) - Taxable service means any service provided or to be provided, to a policy holder or any person, by an insurer, including re-insurer carrying on life insurance business. An insurer carrying on life insurance business shall have the option to pay tax: (i) on the gross premium charged from a policy holder reduced by the amount allocated for investment, or savings on behalf of policy holder, if such amount is intimated to the policy holder at the time of providing of service; (ii) 1.5 per cent plus EC of the gross amount of premium charged from a policy holder in all other cases; towards the discharge of his service tax liability instead of paying service tax at the normal rate. Provided that such option shall not be available in cases where the entire premium paid by the policy holder is only towards risk cover in life insurance. Illustration 3: Mr. X is taken a policy from LIC and premium payable is `50,000 per annum for 20 years (including Risk Premium of `10,000). Calculate the service tax to be charged by LIC under various methods. Solution: Method 1 Risk Premium x service tax rate 10,000 x 10.3% = `1,030 Method 2 Gross Amount charged x 1.545 % 50,000 x 1.545% = `772.5 Rounded off U/s 37D = ` 773 Question 46 (Imp.): Write a note on option to pay service tax in case of Money Changer. Answer: Option to pay service tax in case of a Money Changer Rule 6(7B) As per section 65(105)(zzk) - Money Changer service means any service provided or to be provided, to any person, by a Foreign Exchange Broker, including an authorised dealer in foreign exchange or an authorised money changer. The person liable to pay service tax in relation to purchase or sale of foreign currency, including money changing, provided by a foreign exchange broker, shall have the option to pay an amount calculated at the following rate towards discharge of his service tax liability instead of paying service tax at the normal rate:

Service Tax (a) 0.1% of the gross amount of currency exchanged upto ` 100,000, but minimum ` 25; (b) on next 9,00,000 @ 0.05% (c) on balance @ 0.01% but maximum `5,000.

191

Provided that the person providing the service shall exercise such option for a financial year and such option shall not be withdrawn during the remaining part of that financial year. Alternatively, the assessee can find value of service as provided in rule 2B of Service Tax (Determination of Value) Rules, 2006. As per these rules, the value of service will be equal to difference in buying or selling rate (as the case may be) and the RBI reference rate for the currency at that time. [RBI Reference Rate is mean of selling and buying rate]. Example I: US$1000 are sold by a customer at the rate of Rupees 45 per US$. RBI reference rate for US$ is Rupees 45.50 for that day. The taxable value shall be Rupees 500. Example II: INR70000 is changed into Great Britain Pound (GBP) and the exchange rate offered is Rupees 70, thereby giving GBP 1000. RBI reference rate for that day for GBP is Rupees 69. The taxable value shall be Rupees 1000. Illustration 4: Mr. X is a dealer and engaged in sale & purchase of Foreign currency. ABC Ltd requires 10000 US Dollar to make foreign payment. Mr. X quotes ` 51 per US Dollar and RBI reference rate is `50 per dollar. Calculate the amount of service tax under different option: Solution: Option 1 10,000 (51-50) x 10.3% Option 2 Gross Amount Charged 51 x 10,000 Service Tax Up to 1,00,000 Next 4,10,000 Education cess @ 3% Service tax payable Rounded Off u/s 37D Question 47: Write a note on option to pay service tax in case of Distribution of Lottery ticket. Answer: Option to pay service tax in case of distributor of lottery tickets Rule 6(7C) ` 1,030 5,10,000

1,00,000 x 0.1% 4,10,000 x 0.05%

100.00 205.00 305.00 9.15 314.15 314.00

Service Tax

192

As per section 65(105)(zzzzn) -Distribution of lottery ticket service means any service provided or to be provided to any person, by any other person, for promotion, marketing, organising or in any other manner assisting in organising games of chance, including lottery, Bingo or Lotto in whatever form or by whatever name called, whether or not conducted through internet or other electronic networks. The distributor or selling agent, liable to pay service tax for the taxable service of promotion, marketing, organising or in any other manner assisting in organising lottery, referred to in sub-clause ( zzzzn) of clause (105) of section 65 of the said Act, shall have the option to pay an amount at the rate mentioned below: Sl. No. (1) 1. Rate (2) ` 6,000 on every ` 10 lakh (or part of ` 10 lakh) of aggregate face value of lottery tickets printed by the organising State for a draw ` 9,000 on every ` 10 lakh (or part of ` 10 lakh) of aggregate face value of lottery tickets printed by the organising State for a draw Condition (3) If the lottery or lottery scheme is one where the guaranteed prize payout is more than 80% If the lottery or lottery scheme is one where the guaranteed prize payout is less than 80% :

2.

In case of online lottery, the aggregate face value of lottery tickets shall be taken as the aggregate value of tickets sold. The distributor or selling agent shall exercise such option within a period of one month of the beginning of each financial year and such option shall not be withdrawn during the remaining part of the financial year. Illustration 5: M/s Future Gaming Solutions India Private Limited is a distributor of lottery organized by State of Sikkim provides following information: Particulars Diwali Bumper Diwali Dhamaka Total No of Tickets 2,50,000 2,50,000 Face Value of Tickets 100 100 Value of Guaranteed Prize Payout 51.11% 51.11% Actual Number of ticket sold 2,00,000 2,00,000 Mode of conducting the scheme Printed On line Compute Service tax payable under Rule 6(7C) of service tax Rules, 1994. Solution: ` Printed Mode 2,50,000 x 100 x 9,000 = 10,00,000 Education Cess @ 3% On Line Mode 2,00,000 x 100 x 9,000 10,00,000 2,25,000 6,750 2,31,750 1,80,000

Service Tax Education Cess @ 3%

193 5,400 1,85,400

Illustration 6: Mr. Amit Rastogi is a distributor of lotteries organized by the State of Haryana. He is running two schemes of lotteries as followedTotal No. of ticket proposed under the scheme Face value per ticket Value of guaranteed prize payouts Actual no. of tickets sold Mode of conducting the Scheme Scheme A 22,75,000 10 75% 17,50,000 Printed 22,75,000 Scheme B 50,000 250 85% 46,250 Online

Compute service tax payable under Rule 6(7C) of Service Tax Rule, 1994. Solution: Computation of Service Tax Liability of Mr. Amit Rastogi Statement showing computation of Service Tax liability under Composition Scheme (Rule 6(7C) of the STR, 1994) of Mr. Amit Rastogi. Particulars Scheme A Scheme B Total no. of tickets 22,75,000 46,250 Face value per ticket 10 250 Aggregate value of lottery tickets 2,27,50,000 1,15,62,500 Value of guaranteed prize payouts 75% 85% No. of units of 10 lakhs or part thereof 23 12 Service tax payable(for every 10 lakhs or part thereof) 9,000 6,000 Service tax payable 2,07,000 72,000 Total service tax 2,79,000 Add: EC @ 2% 5,580 Add: SHEC @ 1% 2,790 Total service tax liability 2,87,370 Question 48 (V. Imp.): Write a note on Reverse Charge Mechanism. Answer: Reverse Charge Mechanism Section 68(2)/ Rule 2(1)(d) In general service provider has to charge service tax from the service recipient and has to pay to the government but in some of the services, service recipient shall be liable to pay service tax directly to the government and it is called reverse charge mechanism and it is applicable in the following cases: 1. Insurance Auxiliary Service In relation to insurance auxiliary service by an insurance agent, any person carrying on the general insurance business or the life insurance business, as the case may be, in India. 2. Import of Services In relation to any taxable service provided or to be provided by any person from a country other than India and received by any person in India under section 66A of the Act, the recipient of such service (import of services). 3. Goods Transport Agency In case of transportation of goods by goods transport agency, service tax shall be paid by the consignor or the consignee whosoever is liable to pay freight charges under Reverse Charge but Reverse Charge is

Service Tax

194

applicable only when consignor or consignee is one of the persons mentioned below otherwise Reverse Charge is not applicable. (a)any factory registered under or governed by the Factories Act, 1948; (b)any company formed or registered under the Companies Act, 1956; (c)any corporation established by or under any law; (d)any society registered under the Societies Registration Act, 1860 (e)any co-operative society established by or under any law; (f) any dealer of excisable goods, who is registered under the Central Excise Act, 1944 or the rules made thereunder; or (g) any body corporate established, or a partnership firm registered, by or under any law, 4. Business Auxiliary Service of Distribution of Mutual Fund In relation to business auxiliary service of distribution of mutual fund by a mutual fund distributor or an agent, as the case may be, the mutual fund or asset management company, as the case may be, receiving such service. 5. Sponsorship Service In relation to sponsorship service provided to any body corporate or firm located in India, the body corporate or, as the case may be the firm who receives such sponsorship service shall be liable to pay service tax but if the person sponsoring the programme is located outside India, Reverse Charge shall not be applicable. The above persons shall apply for registration as if they are service provider and all the provisions of section 69 and rule 4 shall apply but general exemption as per notification no 6/2005 shall not apply. The service recipient shall be allowed Tax credit for the service tax paid by him under reverse charge mechanism, however it cannot be paid through Tax credit rather payment should be made and only after that its CENVAT credit shall be allowed. As per Rule 7 of point of Taxation Rule, 2011, in case of reverse charge point of taxation shall be the date on which actual payment has been made but if such payment is not made within 6 months of the date of invoice, in that case point of taxation shall be determined in the normal manner. Question 49 (V. Imp.): Write a note on Insurance Auxiliary (help) Service. Answer: In relation to insurance auxiliary service provided by an insurance agent, the person liable to pay service tax is the person carrying on general insurance business or the life insurance business as the case may be, in India. Example If LIC has to pay commission of `2,00,000 to their agent Mr. X, in this case LIC is the service recipient but still LIC has to pay service tax under reverse charge and service tax payable shall be `2,00,000 x 10.3% = 20,600 and amount of TDS under section 194D shall be 2,00,000 x 10% = `20,000 Question 50 (V. Imp.): Write a note on Import of Services. Answer: Section 66A/ Taxation of services (provided from outside India and received in India) Rules, 2006

Service Tax

195

Where any taxable service have been provided by any person from outside India to any person in India in such cases the person who has taken services from outside India shall be liable to pay service tax as if he is the service provider and he should apply for registration even if the gross amount is less than `9 lakh and also he should pay service tax even if the gross amount is not exceeding `10 lakhs and it is called Reverse Charge Mechanism. If the service recipient is an individual and services are not taken for business or commerce, in that case, he is not liable to pay service tax. Question 51 (V. Imp.): Write a note on Goods Transport Agency. Answer: As per section 65(105)(zzp) - Goods Transport Agency Service means any service provided or to be provided, to any person, by a goods transport agency, in relation to transport of goods by road in a goods carriage. As per section 65(50b) - Goods Transport Agency means any person who provides service in relation to transport of goods by road. In relation to taxable service provided by a goods transport agency, Reverse Charge shall be applicable and service tax shall be paid by the consignor or consignee whosoever is liable to pay freight (transportation charges) and Reverse Charge shall be applicable where the consignor or consignee of goods is atleast one of the persons mentioned below: (a) Any factory registered under by the Factories Act, 1948; (b) Any company registered under the Companies Act, 1956; (c) Any corporation established under any law; (d) Any society registered under the Societies Registration Act, 1860; (e) Any co-operative society; (f) Any dealer of excisable goods, who is registered under the Central Excise Act, 1944; (g) Any body corporate established, under any law (h) Any partnership firm registered. If neither the consignor nor the consignee is one of the persons mentioned above, in that case service tax shall be paid by the Goods Transport Agency and Reverse Charge shall not be applicable. Example If ABC Ltd. of Delhi has taken services of GTA and goods have been sent to Bombay and freight is payable by the consignee at Bombay, in this case service tax shall be paid by the consignee and not by the GTA. Question 52: Write a note on Business Auxiliary Services of Distribution of Mutual Fund. Answer: In relation to Business Auxiliary Service of Distribution of Mutual Fund by a mutual fund distributor or an agent, as the case be, the person liable for paying service tax is the mutual fund or asset management company, as the case may be, receiving such service.

Question 53 (V. Imp.): Write a note on Sponsorship Services. Answer:

Service Tax

196

As per section 65(99a) - Sponsorship includes naming an event after the sponsor, displaying the sponsors company logo or trading name, giving the sponsor exclusive or priority booking rights, sponsoring prizes or trophies for competition; but does not include any financial or other support in the form of donations or gifts, given by the donors subject to the condition that the service provider is under no obligation to provide anything in return to such donors. In relation to sponsorship service provided to any body corporate or firm located in India, the person liable to pay service tax is the body corporate or firm, as the case may be who receives such sponsorship service. If the person sponsoring the event is out of India, in that case, reverse charge shall not be applicable rather the person organizing the event i.e. the service provider shall be liable to pay service tax. Question 54 (V. Imp): Write a note on tax credit in case of Service Provider. Answer: A Service provider shall be allowed tax credit for the following tax paid by him. 1. Excise duty on inputs (raw material etc) or capital goods (plant and machinery, furniture and fixtures etc) used in or in connection with providing of output services. 2. Service Tax paid on the input services taken by him in connection with providing of output services and such input services may be insurance services, banking services, renting of immovable property or other similar services. 3. Education cess paid on Excise Duty or Countervailing Duty or Service Tax shall also be eligible for tax credit. The service provider can utilize the tax credit in the manner given below: 1. Tax credit for Excise Duty or Service Tax can be utilized against output service tax. 2. Tax credit for Education cess of 2% can be utilized for payment of education cess of 2% on output service tax. 3. Tax credit for SHEC of 1% can be utilized for payment of SHEC of 1% on output service tax. Inter-adjustment of Tax credit of Excise Duty, Service Tax is allowed to the service provider because all these taxes are collected by Central Government. Tax credit for Excise Duty, Service Tax is called Cenvat Credit and is regulated through Cenvat Credit Rules, 2004. Question 55: Explain provisions of Rule 6 of Cenvat Credit Rules, 2004. Answer: As per basic principle of cenvat, credit of duty or tax can be availed only for payment of duty on final product or output services. In other words, if no duty is payable on final product or output services, credit of duty/tax paid on inputs or input services cannot be availed. As per Rule 6 of Cenvat Credit Rules 2004, Cenvat credit shall not be allowed on such quantity of input used in or in relation to the manufacture of exempted goods or for provision of exempted services, or input service used in or in relation to manufacture of exempted goods or for provision of exempted services.

Service Tax

197

If a manufacturer is manufacturing both exempt and dutiable goods (or service provider providing taxable as well as exempt services), it may happen that same inputs/input services are used partly for manufacture of dutiable goods/ taxable services and partly for exempted goods/services. In such cases, the manufacturer/service provider has following 2 options for availing Cenvat credit : (i) Maintain separate Accounts: The manufacturer or service provider of output service shall maintain separate accounts for:(a) receipt, consumption and inventory of input and input service meant for use in the manufacture of dutiable final products or in providing output service and (b) the quantity of input meant for use in the manufacture of exempted goods or service, and take Cenvat credit only on that quantity of input or input service which is intended for use in the manufacture of dutiable goods or in providing taxable output service. (ii) Option not to maintain separate accounts: The manufacturer of goods or the provider of output service, opting not to maintain separate accounts, shall have to pay amount equal to 5% of value of exempted goods (if manufacturer) and of value of exempted services (if service provider). Example 1: Mr. Sachin, a service provider, has provided services of `50,00,000. Out of this, ` 35,00,000 are taxable output services and `15,00,000 are exempt output services. Mr. Sachin has opted not to maintain separate inventory and accounts and pay prescribed amount on value of exempt output services. Service tax paid on his input services, is `3,00,000 plus EC plus SHEC. Rate of service tax, excluding EC and SHEC, is 10%. Calculate the total amount payable including service tax, EC and SHEC by Mr. Sachin by GAR-7 challan. Solution: Calculation of service tax and total amount payable under rule 6 of the CENVAT Credit Rules, 2004 Particulars Amount payable on taxable services Amount payable on exempt services under rule 6 (15,00,000 x 5%) Service tax 3,50,000 75,000 Education cess 7,000 Secondary and higher education cess 3,500 3,500 3,000 500

4,25,000 7,000 Less: CENVAT Credit 3,00,000 6,000 Net tax 1,25,000 1,000 Amount payable by GAR-7 challan = ` 1,25,000 + ` 1,000 + ` 500 = `1,26,500

Notes: (1) Education cess & secondary and higher education cess are not payable on amount payable @ 5% of the exempt services under rule 6 of the CENVAT Credit Rules, 2004. (2) Credit of education cess and secondary and higher education cess on input services can be utilized only for the payment of education cess and secondary and higher education cess payable on output services (or on excisable goods) respectively. Example 2: Mr. X is engaged in manufacturing of Excisable as well as Exempted goods. During the year following sales were made

Service Tax i) ii) ` 50,00,000 ( Dutiable @ 10.3%) ` 50,00,000 ( Exempted goods)

198

He purchased raw material on which he paid Excise duty of ` 2,50,000 plus education cess and SHEC and receive Services in connection to his business and paid service tax of ` 1,00,000 plus education cess and SHEC. Note: i) Raw material purchased were used to manufacture excisable as well as exempted goods. Compute Cenvat credit available and net duty payable. Solution: Since, manufacturer is engaged in manufacturing excisable as well as exempted goods by using common raw material, in such case, he has two options for availing Cenvat credit as per Rule 6 of Cenvat credit Rule, 2004. i) Maintain Separate Accounts ii) Pay an amount equal to 5% of value of exempted goods i.e. Particulars Amount payable on taxable goods Amount payable on exempt goods i.e. 5% of `50,00,000 Less: CENVAT Credit (2,50,000 + 1,00,000) Net tax Excise Duty 5,00,000 2,50,000 7,50,000 3,50,000 4,00,000 Education cess 10,000 10,000 7,000 3,000 Secondary and higher education cess 5,000 5,000 3,500 1,500

Question 56: Whether any section of Central Excise Act, 1944 is applicable to Service tax. (For Self Reading Only) Answer: Application of certain provisions of Central Excise Act 1944 Section 83 The provisions of the following section of the Central Excise Act, 1944, as in force from time to time, shall apply, so far as may be, in relation to service tax as they apply in relation to a duty of excise: 9C, 9D, 11B, 11BB, 11C,12 12A, 12B. 12C, 12D, 12E, 14, 14AA, 15, 33A, 35F, 35FF to 35-O (both inclusive), 35Q, 36, 36A, 36B, 37A, 37B, 37C, 37D, 38A and 40. Question 57: Write a note on Appeals to the Commissioner of Central Excise (Appeals). (For Self Reading Only) Answer: Appeals to the Commissioner of Central Excise (Appeals) Section 85 An appeal can be filed to the Commissioner of Central Excise (Appeals) against the orders of Central Excise Officer within 3 months of receiving the orders and appeal should be filed in Form No. ST-4. Commissioner of Central Excise (Appeals) may admit the appeal even after expiry of the period of 3 months provided there were sufficient reason for the delay. Question 58: Write a note on Appeals to Appellate Tribunal (CESTAT). (For Self Reading Only) Answer: Appeals to Appellate Tribunal (CESTAT) Section 86 Appeal can be filed to the Custom Excise Service Tax Appellate Tribunal against the orders of Commissioner of Central Excsie or the order of Commissioner of Central Excise (Appeals). Appeal can be filed maximum within 3 months of the date of receiving the orders however Appellate

Service Tax

199

Tribunal may admit the appeal even after the expiry of the prescribed period provided there were sufficient reasons. Appeal should be filed in Form No. ST-5. Question 59 (V. Imp.): Write a note on valuation of taxable services for charging Service Tax. Answer: Valuation of taxable services for charging service tax Section 67 Value shall be determined in the manner given below: (a) If consideration is in terms of money, the money value of the services e.g. If a TV Programme Production services charges `5,00,000 (exclusive of service tax) as fee from its client, the value of the taxable service rendered will be `5,00,000 and service tax payable shall be `51,500 (5,00,000 x 10.3%). Where the gross amount charged by a service provider, for the service provided or to be provided is inclusive of service tax payable, the value of such taxable service shall be such amount as, with the addition of tax payable, is equal to the gross amount charged. E.g. If total amount charged is `5,00,000 and service tax has not been charged separately, in that case amount of service tax shall be 5,00,000 / 110.3 x 10.3 = `46,690.84 Rounded Off `46,691 (b) If consideration is in kind, market value of such consideration and it will be considered to be inclusive of service tax. E.g. One service provider has rendered services and service recipient has given one gold watch with market value `50,000, in this case amount of service tax shall be 50,000 / 110.3 x 10.3 = `4,669.08 rounded off `4,669 (c) If consideration is partly in cash and partly in kind, it will be total of cash plus market value of consideration in kind and it will be considered to be inclusive of service tax. e.g. A Survey and Map Making Agency provides taxable professional services to one of its clients and it charges `1,00,000 in cash and `75,000 in kind, in this case total consideration shall be `1,75,000 but it will be inclusive of service tax and accordingly amount of service tax shall be 10.3/ 110.3 x 1,75,000 = `16,341.79 Rounded off u/s 37D = `16,342 (d) If any person has rendered free services, no service tax is payable. Question 60: Explain the concept of Pure Agent. Answer: As per Service Tax (Determination of Value) Rules, 2006, Pure agent means a person who (a) enters into a contractual agreement with the recipient of service to act as his pure agent to incur expenditure or costs in the course of providing taxable service; (b) do not hold any title to the goods or services so procured as pure agent of the recipient of service; (c) does not use such goods or services so procured; and (d) receives only the actual amount incurred to procure such goods or services.

Service Tax

200

The expenditure or costs incurred by the service provider as a pure agent of the recipient of service, shall be excluded from the value of the taxable service. If the service provider is not a pure agent, the value of the taxable service is the total amount of consideration consisting of all components of the taxable service and it is immaterial that the details of individual components of the total consideration is indicated separately in the invoice. Illustration. X contracts with Y, a real estate agent to sell his house and thereupon Y gives an advertisement in television. Y billed X including charges for television advertisement and service tax paid on advertisement in the total bill. In such a case, consideration for the service provided is what X pays to Y. Y does not act as an agent on behalf of X when obtaining the television advertisement even if the cost of television advertisement is mentioned separately in the invoice issued by Y. Advertising service is an input service for the estate agent in order to enable or facilitate him to perform his services as an estate agent Illustration. In the course of providing a taxable service, a service provider incurs costs such as travelling expenses, postage, telephone, etc., and may indicate these items separately on the invoice issued to the recipient of service. In such a case, the service provider is not acting as an agent of the recipient of service but procures such inputs or input service on his own account for providing the taxable service. Such expenses do not become reimbursable expenditure merely because they are indicated separately in the invoice issued by the service provider to the recipient of service and thus shall form part of value of taxable service. Illustration. A contracts with B, an architect for building a house. During the course of providing the taxable service, B incurs expenses such as telephone charges, air travel tickets, hotel accommodation, etc., to enable him to effectively perform the provision of services to A. In such a case, in whatever form B recovers such expenditure from A, whether as a separately itemised expense or as part of an inclusive overall fee, service tax is payable on the total amount charged by B. Value of the taxable service for charging service tax is what A pays to B. Illustration. Company X provides a taxable service of rent-a-cab by providing chauffeur-driven cars for overseas visitors. The chauffeur is given a lump sum amount to cover his food and overnight accommodation and any other incidental expenses such as parking fees by the Company X during the tour. At the end of the tour, the chauffeur returns the balance of the amount with a statement of his expenses and the relevant bills. Company X charges these amounts from the recipients of service. The cost incurred by the chauffeur and billed to the recipient of service constitutes part of gross amount charged for the provision of services by the company X. Question 61 (V. Imp.): Write a note on Export of Services. Answer: Section 93A/ Export of services, Rules 2005 If any person has exported services from India, no service tax is payable however the payment should be received in India in foreign currency within the time allowed by Reserve Bank of India otherwise exemption is not allowed. If any person has taken input services for the purpose of export of services, service tax paid on such input services shall also be refunded. It will be considered to be export of services if services are provided from India and are used outside India. Similarly if any person has exported goods from India and has paid service tax on the services taken for the purpose of export of goods, service tax paid on such services shall also be refunded.

Service Tax Question 62 (V. Imp.): Explain meaning of Input Service / Output Service. Answer: Input service Rule 2(l) of Cenvat Credit Rules, 2004 means any service, (i)used by a provider of taxable service for providing an output service; or

201

(ii)used by a manufacturer, whether directly or indirectly, in or in relation to the manufacture of final products and clearance of final products upto the place of removal, and includes services used in relation to modernisation, renovation or repairs of a factory, premises of provider of output service or an office relating to such factory or premises, advertisement or sales promotion, market research, storage upto the place of removal, procurement of inputs, accounting, auditing, financing, recruitment and quality control, coaching and training, computer networking, credit rating, share registry, security, business exhibition, legal services, inward transportation of inputs or capital goods and outward transportation upto the place of removal. Output service Rule 2(p) of Cenvat credit Rules, 2004 means any taxable service, provided by the provider of taxable service, to a customer, client, subscriber, policy holder or any other person, as the case may be, and the expressions provider and provided shall be construed accordingly. Question 63 (Imp.): Write a note on Input Service Distributor. Answer: Input service distributor Rule 2(m) of Cenvat Credit Rules, 2004 means an office of the manufacturer or provider of output service, which receives invoices towards purchases of input services and issues invoice, for the purposes of distributing the credit of service tax paid on the said services to its manufacturing units or units providing output service. The ISD must be registered with Service Tax Department. Credit of service tax attributable to service used in a unit exclusively engaged in manufacture of exempted goods or providing of exempted services shall not be distributed. The credit so distributed should not exceed the amount of service tax paid on input services. Every input service distributor distributing credit shall, issue an invoice, signed by such person for each of the recipient of the credit distributed, and such invoice, shall be serially numbered and shall contain the following namely:(i) the name, address and registration number of the person providing input services and the serial number and date of invoice; (ii) the name and address of the input service distributor; (iii)the name and address of the recipient of the credit distributed; (iv) the amount of the credit distributed.

Service Tax

202

Question 64 (Imp.): Write a note on liability of service tax in case of supply of goods as well as services. Answer: As per notification No.12/2003 dated 20.06.2003, if any service provider is providing services as well as supplying the goods, in that case, so much of the value of all the taxable services, as is equal to the value of goods and materials sold by the service provider to the recipient of service shall not be chargeable to service tax provided there is documentary proof specifically indicating the value of the said goods and materials. If the goods have been consumed in the process of rendering services, service tax shall be payable on the value of such goods. However, Cenvat credit of duty paid on such goods and materials sold shall not be allowed under the provisions of the CENVAT Credit Rules, 2004 but cenvat credit for the goods consumed shall be allowed. Cenvat credit for capital goods used in providing services and cenvat credit of input services shall be allowed. Example If authorised service station of Maruti Udyog Limited has raised an invoice of `2,00,000 for repair of an accidental car with the bifurcation that `70,000 on account of service charge (labour charge etc) and `1,30,000 is the cost of spare parts and components etc., in this case service tax is payable only on `70,000 but if there is no such bifurcation, service tax is payable on the entire amount of `2,00,000. If there is a bifurcation, cenvat credit of excise duty shall not be allowed in connection with the spare parts etc. of `1,30,000. However, if the assessee has taken any input services, cenvat credit shall be allowed. Similarly, if a coaching institute is charging ` 12,000 as fees including the cost of assignments, in this case service tax is payable on the entire amount of `12,000 but if the coaching institute is providing study material in the form of standard books and price is also printed on such books and a separate invoice has been issued or amount has been shown separately in the same invoice, service tax shall not be charged on the value of study material and also cenvat credit shall not be allowed with regard to such study material i.e. excise duty paid on the value of the paper and other material. If any material has been consumed in the process of rendering services, in that case service tax has to be charged on the value of such material e.g. One beauty parlour has charged `1,00,000 for bridal makeup and material consumed in the process of rendering service is valued `60,000, in this case service tax should be charged on the entire amount and cenvat credit shall be allowed even for the material consumed in the process. Similarly, if a photography studio has rendered services and has charged `50,000 and has supplied the photographs and cost of photography paper and chemicals etc. is `20,000, in this case service tax shall be charged on the entire amount. Similarly, if a service provider is rendering Photostat services and has charged `1,00,000 which includes cost of paper `70,000, service tax shall be charged on the entire amount. Question 65 (Imp.): Explain concept of Partial Abatement. Answer: Partial Abatement (concession/reduction) Notification No. 1/2006 dated: 01.03.2006 If any service provider is rendering services as well as supplying material, in such cases, service tax shall not be charged on the value of goods or materials supplied as per Notification No.12/2003 but sometimes it is

Service Tax

203

not possible to bifurcate (divide) the value of services and the value of goods, in such cases, service tax shall not be charged on the full amount rather service tax shall be charged only on some part of the total amount i.e. some abatement/concession shall be allowed and only on the balance amount, service tax shall be charged and is as given below. In such cases, Cenvat credit on inputs, input services and capital goods used for providing such taxable services is not allowed. For the purpose of general exemption of `10,00,000, amount before permitting abatement shall be taken into consideration. Abatement has been allowed in case of the following services and balance percentage of the total value taxable is as given below: S. No. Particulars Abatement Taxable % % 1. Mandap Keepers Services (including catering services) 40% 60% 2. Tour Operators Service (a) In relation to booking of accommodation only 90% 10% (b) In respect of tour other than Package Tour (i.e. it 60% 40% will include expenses for stay) (c) In respect of package Tour (i.e. including 75% 25% accommodation, food etc) 3. Convention Services (i.e. services rendered by 40% 60% auditorium or hotel etc. in connection with official meeting) 4. Outdoor catering Services 50% 50% 5. Pandal or Shamiana Contractors Services 30% 70% 6. Services provided by a Restaurant 70% 30% 7. Short-Term Accommodation Services (Services 50% 50% rendered by hotel, inn, guest house etc) 8. Erection Commissioning and installation of Plant and 67% 33% Machinery 9. Rent-a-cab 60% 40% Question 66: Write a note on services rendered by Sub-Contractor. Answer: As per Circular No. 96/7/2007 dated 23.08.2007, A sub-contractor is essentially a taxable service provider. The fact that services provided by such sub-contractors are used by the main service provider for completion of his work does not in any way alter the fact of provision of taxable service by the subcontractor. Services provided by sub-contractors are in the nature of input services. Service tax is, therefore, leviable on any taxable services provided, whether or not the services are provided by a person in his capacity as a subcontractor and whether or not such services are used as input services. The fact that a given taxable service is intended for use as an input service by another service provider does not alter the taxability of the service provided. Question 67: Explain Recovery of Service Tax. (Self reading) Answer: Recovery of service tax not levied or not paid or erroneously (wrongly) refunded Section 73 If any service tax has not been levied or paid or erroneously refunded, the Central Excise Officer may, give a notice within one year from the relevant date, directing the service provider to pay such service tax

Service Tax

204

however, notice can be given upto 5 years instead of one year if service tax has not been paid because of fraud, collusion, wilful mis-statement, suppression of facts, contravention of any of the provisions with an intention to evade payment of service tax. No notice can be given after expiry of specified time period. Relevant date means, (a) If service tax return has been filed, the date of filing the return and if no return has been filed, the last date for filing service tax return. (b) in a case where any sum, relating to service tax, has erroneously been refunded, the date of such refund. Example If ABC Ltd. has not paid service tax for the month of May 2011, the relevant date shall be 25 th October 2011 and a notice can be given to the company by the service tax department upto one year from the relevant date. i.e. upto 25th October 2012 If the service tax was not paid as a result of fraud or suppression of facts etc., in that case notice can be given upto five years i.e. upto 25th October 2016. If service tax has been erroneously refunded, the relevant date shall be the date of such refund. Question 68: Explain provisions of Tax Deduction at Source (TDS) with regard to Service Tax. Answer: If any service recipient has to make the payment to the service provider and tax has to be deducted at source, in that case, while deducting tax at source, service tax shall also be included in some of the cases and it will not be included in some other cases and is as given below: TDS excluding Service Tax 1. If the service recipient is making payment of Rent, tax shall be deducted at source under section 194-I @ 10% and tax shall be deducted excluding service tax. Example ABC Ltd. has let out one commercial building to XYZ Ltd. and rent charged is `20,00,000 plus service tax, in this case, tax to be deducted at source by XYZ Ltd. shall be 20,00,000 x 10% = `2,00,000 and amount payable to ABC Ltd. shall be Rent 20,00,000 Add: Service Tax @ 10.3% 2,06,000 Total 22,06,000 Less: TDS (20,00,000 x 10%) 2,00,000 Amount Payable 20,06,000 2. If the service recipient is making payment of Commission / Brokerage, tax shall be deducted at source under section 194-H @ 10% and tax shall be deducted excluding service tax. Example Mr. X a property dealer has rendered services for getting one commercial building on rent to XYZ Ltd. and Commission charged is `5,00,000 plus service tax, in this case, tax to be deducted at source by XYZ Ltd. shall be 5,00,000 x 10% = `50,000 and amount payable to ABC Ltd. shall be Commission 5,00,000 Add: Service Tax @ 10.3% 51,500 Total 5,51,500 Less: TDS (5,00,000 x 10%) 50,000

Service Tax

205

Amount Payable 5,01,500 3. If the service recipient is making payment of Commission for Sale of Lottery Tickets , tax shall be deducted at source under section 194-G @ 10% and tax shall be deducted excluding service tax. Haryana Government has given contract for sale of lottery Tickets to XYZ Ltd. and XYZ Ltd. has charged commission of `20,00,000 plus service tax from Haryana Government, in this case, tax to be deducted at source by Haryana Government shall be 20,00,000 x 10% = `2,00,000 and amount payable to ABC Ltd. shall be Commission 20,00,000 Add: Service Tax @ 10.3% 2,06,000 Total 22,06,000 Less: TDS (20,00,000 x 10%) 2,00,000 Amount Payable 20,06,000 4. If the service recipient is making payment of Commission for Insurance Business, tax shall be deducted at source under section 194-D @ 10% and tax shall be deducted excluding service tax. There is a Reverse Charge and service tax has to be paid by the Insurance Company (Such services are called Insurance Auxiliary Services) Example If LIC has to commission of `2,00,000 to their agent Mr. X in connection with insurance business, amount of TDS shall be `2,00,000 x 10% = `20,000 and service tax of `20,600 shall be paid directly by LIC to the Government under Reverse Charge under Rule 2(1)(d). TDS including Service Tax 1. If the service recipient is making payment for Professional / Technical services, tax shall be deducted at source under section 194-J @ 10% and tax shall be deducted including service tax. Example Mr. X, a Chartered Accountant has given professional services to XYZ Ltd. and charged `20,00,000 plus service tax, in this case, tax to be deducted at source by XYZ Ltd. shall be 22,06,000 x 10% = `2,20,600 and amount payable to Mr. X shall be Professional Charges 20,00,000 Add: Service Tax @ 10.3% 2,06,000 Total 22,06,000 Less: TDS (22,06,000 x 10%) 2,20,600 Amount Payable 19,85,400 2. If the service recipient is making payment for any Contract e.g. Advertising Contract or Catering Contract etc tax shall be deducted at source under section 194-C @ 2% but if payment is being given to any individual or HUF, tax shall be deducted at source @ 1% and tax shall be deducted including service tax. Example ABC Ltd. has rendered advertising services to XYZ Ltd. and charged `20,00,000 plus service tax, in this case, tax to be deducted at source by XYZ Ltd. shall be 22,06,000 x 2% = `44,120 and amount payable to ABC Ltd. shall be Advertising Charges 20,00,000 Add: Service Tax @ 10.3% 2,06,000 Total 22,06,000 Less: TDS (22,06,000 x 2%) 44,120

Service Tax Amount Payable

206 21,61,880

INDIVIDUAL SERVICES
Question 69 (V. Imp.): Explain provisions relating to Consulting Engineers Services. Answer: Consulting Engineers Services Section 65(105)(g) th (w.e.f. 7 July 1997, Notification No. 23/1997-ST dated 2nd July 1997) Section 65(105)(g) Consulting Engineers Service means any service provided, to any person, by a consulting engineer in relation to advice, consultancy or technical assistance in any disciplines of engineering including the discipline of computer hardware engineering. Section 65(31) - Consulting engineer means any professionally qualified engineer or any body corporate or any other firm who, renders any advice, consultancy or technical assistance in any disciplines of engineering. Services rendered by a consulting engineer in connection with computer hardware engineering as well as software engineering or only in connection with hardware engineering, shall be taxable as consulting engineers services. If services are rendered only for computer software engineering, it will be taxable as information technology service. The scope of the services of a consultant may include: (i) Feasibility report; (ii) Basic design engineering; (iii) Detailed design engineering; (iv) Construction supervision and project management; (v) Supervision of commissioning and initial operation; (vi) Post-operation and management; (vii) Trouble shooting and technical services, including establishing systems and procedures for an existing plant.

Consulting Engineer will not include those qualified engineers who act as insurance surveyors and loss assessor for insurance companies and therefore service tax levy on the consulting engineer in any discipline of engineering will not cover the insurance surveying and loss assessment services rendered by a qualified engineer. As per rule 7 of Point of Taxation Rule, 2011, service tax shall be payable on actual receipt basis i.e. rule 3 of POT Rules, 2011 shall not be applicable. As per N. N.. 46/2011, dated 19.09.2011, Central Government, hereby exempts the taxable services provided by a consulting engineer to any person on transfer of technology from so much of the service tax leviable thereon under section 66 of the said Act, as is equivalent to the amount of cess paid on the said transfer of technology under the provisions of section 3 of the Research and Development Cess Act, 1986. (Section 3 of The Research and Development Cess Act, 1986

Service Tax

207

(1) There shall be levied and collected, for the purposes of this Act, a cess at such rate not exceeding five per cent. on all payments made towards the import of technology, as the Central Government may, from time to time, specify, by notification, in the Official Gazette. (2) The cess shall payable to the Central Government by an industrial concern which imports technology on or before making any payments towards such import and shall be paid by the industrial concern to any specified agency.) Illustration 7: Mr. Ramesh, a Consulting Engineer provides the following particulars in respect of various services rendered by him during the quarter ending December 2011: S. No. (i) (ii) (iii) (iv) (v) Particulars Professional advice to one of his friend Consultancy services in computer hardware engineering Technical assistance in computer software engineering and hardware engineering Advice in relation to metallurgical engineering Professional advice to his friend free of charge ` 5,000 15,000 25,000 10,000

Compute the service tax payable by Mr. Ramesh for the quarter ending December, 2011. Service tax has been charged separately by Mr. Ramesh and is not included in any of the receipts mentioned above. Mr. Ramesh is not entitled to the benefit of small service provider available under Notification No. 6/2005 ST dated 01.03.2005. Presume all the payments have been received upto 31st Dec, 2011. (b) Will your answer be different if the above services are rendered by AB Ltd., a consulting engineering company? Solution: Computation of service tax payable Particulars Professional advice to one of his friend Consultancy services in computer hardware engineering Technical assistance in computer software engineering and hardware engineering Advice in relation to metallurgical engineering Total Value of taxable services Service tax @10% Add: Education cess @ 2% Add: Secondary and higher education cess @ 1% Service tax payable

` 5,000 15,000 25,000 10,000 55,000 55,000 5,500 110 55 5,665

Notes: 1 Advice to friend is taxable as service rendered to any person is taxable. 2.Consultancy and technical assistance in relation to both computer hardware and software engineering are taxable. 3. Advice in relation to any branch of engineering is taxable. 4. Services rendered free of charge are not liable to service tax.

Service Tax (b) Answer will be the same as Consulting Engineer inter alia means any Body Corporate as well.

208

Illustration 8: Mr. X is a Consulting Engineer who is liable to pay service tax and has submitted information as given below: (i) Received advance `2,00,000 inclusive of service tax from ABC Ltd. on 10th April 2011 and services were rendered in September 2011 and bill was issued in October 2011.

(ii) Rendered services to Mr. A in October 2011 and a bill of `5,00,000 was issued inclusive of service tax and payment of `3,00,000 was received in February 2012 and balance is yet to be received. (iii) Rendered services to United Nations in December 2011 and payment of `7,00,000 was received in December 2011. No service tax has been collected. (iv) Rendered services to a unit in SEZ and `3,00,000 was received in March 2012 without service tax. (v) Rendered services to foreign diplomatic mission and `5,50,000 was received in January 2012 without service tax. (vi) Rendered services to family members of diplomatic agents and `1,50,000 was received in October 2011 without service tax. Show the tax treatment for Service Tax. Solution: (i) April to June 2011 Taxable value of services = 2,00,000 Service tax payable = 2,00,000 / 110.3 x 10.3 = 18,676.34 Rounded off under section 37D = 18,676 Last date for making payment shall be 5th July 2011 and if payment is through internet banking, last date shall be 6th July 2011. (ii) January to March 2012 Total bills raised = 5,00,000 Less: Amount received = 3,00,000 Amount not received = 2,00,000 Since amount charged is inclusive of service tax hence amount of service tax shall be = 3,00,000 / 110.3 x 10.3 = 28,014.51 Rounded off under section 37D = 28,015 Last date for making payment shall be 31st March 2012. (iii) Services provided to United Nations or an international organization is exempt from service tax. (iv) Services provided to a developer of SEZ or a unit of SEZ is exempt from service tax. (v) Services provided to foreign diplomatic mission are exempt from service tax. (vi) Services provided to family members of diplomatic agents are exempt from service tax.

Service Tax

209

Question 70 (V. Imp.): Explain provisions relating to Mandap Keepers Service. Answer: Mandap Keepers Service Section 65(105)(m) (w.e.f. 1st July 1997, Notification No. 19/1997-ST dated 26th June 1997) Section 65(105)(m) Mandap Keeper service means any service provided, to any person, by a mandap keeper in relation to the use of mandap in any manner including the facilities provided in relation to such use and also the services, if any, provided as a caterer. As per section 65(66) Mandap means any immovable property as defined in section 3 of the Transfer of Property Act, 1882 and includes any furniture, fixtures, light fittings and floor coverings, photography or video film facilities etc. therein let out for consideration for organizing any official, social or business function; As per Section 65(67) Mandap keeper means a person who allows temporary occupation of a Mandap for consideration for organizing any official, social or business function. Social function include marriage. 1. As per notification no. 1/2006, if the catering services are also provided, value for the purpose of service tax shall be 60% of the gross amount i.e. abatement shall be allowed for 40%. e.g. if a Mandap Keeper liable to pay service tax and is providing catering service has charged `30,00,000, amount of service tax to be charged by him shall be 30,00,000 x 60% x 10.3% = `1,85,400 Hotels and Restaurant which let out their banquets as well as rooms, gardens etc. for holding/organizing any marriage, party, conference shows are covered in the definition of Mandap keeper and are subject to service tax. Renting out of halls etc. for the purpose of holding a dance, drama or music programme or similar competition is also chargeable to Service Tax, in this category. Programmes of dance, drama and music are social functions. Renting out of premises by Art Gallery for exhibition will be exempt from service tax. Services provided by the religious centers as mandap keeper in their precincts (premises) have been exempted from service tax. E.g. Mr. X has arranged marriage of his son in Birla Temple and has paid `21,000, no service tax shall be charged in this case.

2.

3.

4. 5.

If any Tent has been erected on the open ground for the purpose of any such function etc., it will be covered in Pandal and Shamiana services and not in Mandap Keeper Services. Illustration 9: Maharaja Banquets is engaged in providing service in relation to letting out marriage hall and all facilities relating to marriage like catering service. DJ facilities, flower decoration etc. The hall was booked by Mr. A for the marriage of his son on 3rd July, 2011. Maharaja Banquets issued a bill of `20, 00,000(excluding service tax) on 15th July, 2011 for the marriage function and it includes the charges for all other facilities like catering services, DJ services, flower decoration etc. Maharaja Banquets purchased input for `3,00,000 and paid excise duty @ 10% plus EC plus SHEC and has also taken input services for `2,00,000 plus service tax plus EC plus SHEC.

Service Tax

210

Purchased a laptop of `1, 00,000/- plus excise duty@10% plus EC & SHEC for providing DJ services and paid the amount on the same date. Calculate the service tax payable by Maharaja Banquets. Service tax rate applicable is 10.3%. Cenvat credit on laptop purchased is allowed 50% in the year of purchase and the balance in the subsequent year. (b) Calculate the amount of service tax payable if assessee is availing abatement under N.N. 01/2006 and Advise the assessee which option is better for him. Solution (a): Output service tax [20,00,000 x 10.3%] Less: Input tax Credit On inputs (3,00,000 x 10.3%) On input service [2,00,000 x 10.3%] On Laptop [1,00,000 x 10.3% x 50%] Net Service tax payable 2,06,000 (30,900) (20,600) (5,150) 1,49,350

Solution (b): Assessee is availing abatement under N.N. 01/2006, hence assessee is not eligible to take cenvat credit of input, input service and capital goods and is liable to pay service tax on 60 percent of the gross amount charged. Total Value Charged 20,00,000 Less: Abatement N.N. 01/2006 (40% of gross amount charged) 8,00,000 Taxable value 12,00,000 Service tax @ 10.3% on 12,00,000 1,23,600 Option (b) is better for the assessee as the service tax payable in option (b) is less than the service tax in option (a) Illustration 10: Determine in the following cases whether service tax is payable or not: (i) Mr. A rendered a service which become taxable from 1 st June,2011 and issued invoice on 30th April, 2011 and received the amount on 20th May, 2011. Solution: No Service tax is payable on the above service as the invoice and payment are received before the service becomes taxable. (ii) Mr. A received payment of ` 2,00,000(excluding Service tax ) on 1st July,2011 and invoice is issued within 14 days from the date of receiving the payment and the service rendered has become taxable from 10th July, 2011. Solution: As per Rule 5 of Point of Taxation Rules 2011, No service tax shall be payable if the payment has been received before the service becomes taxable and invoice has been issued within a period of 14 days. In the given case, as payment is received before the service is become taxable and invoice is issued within 14 days , so no service tax shall be payable . Illustration 11: Mr. X is engaged in providing Mandap Keeper Services and also he has one Art Gallery and has given the following information for the quarter July to September 2011:

Service Tax

211

(i) Let out the Garden and hall for the purpose of marriage. Issued a bill of ` 10, 00,000/- including charges for catering services on 12th July, 2011. Half the amount is received on 21st July, 2011. (ii) Let out hall for the purpose of holding dance, drama and issued a bill of `3,00,000 on 15th July, 2011 but the amount is not yet received. (iii) Let out the Art Gallery to the artist for the exhibition of work of art. Issued a bill of `2,00,000 on 21st July, 2011. (iv) Let out hall for the purpose of holding dance for `5,00,000 on 25th September, 2011 and issued the bill on 05th October, 2011. All the figures are excluding service tax. Calculate the value of taxable service and service tax payable for the quarter July to September, 2011. Solution: Computation of Taxable value of service tax under Mandap keeper service for the quarter July to September, 2011 Particulars (i) Letting out garden including catering charges is taxable but only 60% of gross amount charged is chargeable as per Notification no 01/2006 dated 01.03.2006. [10,00,000 x 60%] (ii) Letting out hall for holding dance, drama is a social function and covered under Mandap Keeper Service. (iii) Exhibition of work of arts is not covered under Mandap Keeper Service as per circular no 42/05/2002-ST dated 29.04.2002 (iv) Letting out hall is taxable in the Month of October (Note ) Total taxable value Service tax @ 10.3% Amount in ` 6,00,000 3,00,000 Nil Nil 9,00,000 92,700

Note: Letting out hall for holding dance is covered under Mandap Keeper Service but as per Point of Taxation Rules if bill is issued within 14 days of completion of service, point of taxation shall be the date of issue of invoice i.e 05.10.2011 and it will be taxable in the month of October.

Service Tax

212

Question 71(V. Imp.): Explain provisions relating to Practising Chartered Accountant. Answer: Practising Chartered Accountant Section 65(105)(s) (w.e.f. 16th October 1998, Notification No. 53/1998-ST dated 7th Oct 1998) Section 65(105)(s) Practising Chartered Accountant service means any service provided or to be provided, to any person, by a practising chartered accountant in his professional capacity, in any manner. Section 65(83) - Practising Chartered Accountant means a person who is a member of the Institute of Chartered Accountants of India and is holding a certificate of practice granted under the provisions of the Chartered Accountants Act, 1949 and includes any concern engaged in rendering services in the field of chartered accountancy. Services rendered to any person, by a practising chartered accountant in his professional capacity, in any manner shall be chargeable to service tax. As per Notification No. 32/2011 dated 01.05.2011, if any Chartered Accountant has represented any person in the Court of Commissioner (Appeals) or before Income Tax Appellate Tribunal or before any Statutory Authority, in that case also, it will be subject to service tax. As per rule 7 of Point of Taxation Rule, 2011, service tax shall be payable on actual receipt basis i.e. rule 3 of POT Rules, 2011 shall not be applicable. MEMBERS WHO ARE DEEMED TO BE IN PRACTICE A member of the Institute shall be deemed to be in practice when individually or in partnership with Chartered Accountants in practice, he, in consideration of remuneration received or to be received(i) engages himself in the practice of accountancy; or (ii) offers to perform or performs service involving the auditing or verification of financial transactions, books, accounts or records, or the preparation, verification or certification of financial accounting and related statements or holds himself out to the public as an accountant; or (iii) renders professional services or assistance in or about matters of principle or detail relating to accounting procedure or the recording, presentation or certification of financial facts or data; or (iv) Management Consultancy and other Services and it will include (a) Financial management planning and financial policy determination. (b) Capital structure planning and advice regarding raising finance. (c) Working capital management. (d) Preparing project reports and feasibility studies. (e) Preparing cash budget, cash flow statements, profitability statements, statements of sources and application of funds etc. (f) Budgeting including capital budgets and revenue budgets.

Service Tax (g) Inventory management, material handling and storage. (h) Market research and demand studies. (i) Price-fixation and other management decision making. (j) Management accounting systems, cost control and value analysis. (k) Control methods and management information and reporting. (l) Personnel recruitment and selection. (m) Setting up executive incentive plans, wage incentive plans etc. (n) Management and operational audits. (o) Valuation of shares and business and advice regarding amalgamation, merger and acquisition. (p) Business Policy, corporate planning, organisation development, growth and diversification.

213

(q) Organisation structure and behaviour, development of human resources including design and conduct of training programmes, work study, job-description, job evaluation and evaluation of work loads. (r) Systems analysis and design, and computer related services including selection of hardware and development of software in all areas of services which can otherwise be rendered by a Chartered Accountant in practice and also to carry out any other professional services relating to EDP. (s) Acting as advisor or consultant to an issue. (t) Any other service as notified by ICAI. Illustration 12: Mr. X a Chartered Accountant, who is liable to pay service tax has submitted particulars as given below: (i) Rendered services in May 2011 and issued bill for `1,20,000 inclusive of service tax. (Out of which `75,000 was received by a cheque on 10th August 2011 and balance on 3rd March 2012.) (ii) Rendered services in the month of June 2011 in connection with scrutiny assessment and a bill of `75,000 was issued and payment was received on 10.10.2011 and amount is inclusive of service tax. (iii) Rendered free services in July 2011 (market value `20,000). (iv) Represented one client in the court of Commissioner (Appeals) in September 2011 and a bill of `1,00,000 was issued and payment was received on 3rd October 2011 and amount is inclusive of service tax. (v) Rendered services to different clients in the month of January 2012 and bills of `7,00,000 was issued inclusive of service tax but only `5,00,000 was received in the same month in full and final settlement.

Service Tax

214

(vi) Received `70,000 in advance inclusive of service tax in March 2012 for services to be rendered in April 2012. Gross receipt is not exceeding `60,00,000 during the year. In the last year he has paid service tax of `2,00,000. Compute amount of service tax payable for each quarter and also the last date upto which tax should be paid for the financial year 2011-12. (b) If in the above case there is a delay of 10 days in payment of tax in each quarter. Compute interest payable under section 75 and penalty payable under section 76. (c) If the return for the half year ending September 2011 was filed on 18th October 2011 return for half year ending March 2012 was filed on 27th April 2012, compute penalty payable for each of the return and also determine the last date upto which revised return can be filed. Solution: As per rule 7 of Point of Taxation Rule, 2011, service tax shall be payable on actual receipt basis i.e. rule 3 of POT Rules, 2011 shall not be applicable. April to June 2011 Nil July to September 2011 Since amount charged is inclusive of service tax hence amount of service tax shall be = 75,000 / 110.3 x 10.3 = 7,003.63 Rounded off under section 37D = 7,004 Last date for making payment shall be 5 th October 2011 and if payment is through internet banking, last date shall be 6th October 2011 October to December 2011 1,75,000 / 110.3 x 10.3 = 16,341.79 Rounded off under section 37D = 16,342 Last date for making payment shall be 5 th January 2012 and if payment is through internet banking, last date shall be 6th January 2012 January to March 2012 Since amount charged is inclusive of service tax hence amount of service tax shall be = 5,00,000 + 45,000 + 70,000 = 6,15,000 = 6,15,000 / 110.3 x 10.3 = 57,429.73 Rounded off under section 37D = 57,430 Last date for making payment shall be 31st March 2012. Solution (b): April to June 2011 Nil July to September 2011

Service Tax Interest Payable under section 75 7,004 x 15% x 10/366 = `28.70 = `29 Penalty payment under section 76 1% of the amount of default for 10 days = 1% x 7,004 x 10/31 = `22.59 Or Penalty calculated @ `100 per day for 10 days = `1,000 Penalty liable to be paid is `1,000 October to December 2011 Interest Payable under section 75 16,342 x 15% x 10/366 = `66.98 = `67 Penalty payment under section 76 1% of the amount of default for 10 days = 1% x 16,342 x 10/31 = `52.72 Or Penalty calculated @ `100 per day for 10 days = `1,000 Penalty liable to be paid is `1,000 January to March 2012 Interest Payable under section 75 57,430 x 15% x 10/365 = `236.01 = `236 Penalty payment under section 76 1% of the amount of default for 10 days = 1% x 57,430 x 10/30 = 191.43 Or Penalty calculated @ `100 per day for 10 days = `1,000 Penalty liable to be paid is `1,000

215

Solution (c): If the return for the half year ending September 2011 was filed on 18th October 2011, there is no penalty because return is filed within prescribed time period. Revised return can be submitted within a period of 90 days from the date of submission of the original return. 16th Jan 2012 If return for the half year ending March 2012 was filed on 27th April2012, the penalty shall be `500. Revised return can be submitted within a period of 90 days from the date of submission of the original return. 26th July 2012 Illustration 13: Ashish Agarwal is a Chartered Accountant. He acquired the certificate of practice from the ICAI in November, 2011. For the quarter ended on March, 2012, his receipts are as follows: (The amounts given below are exclusive of service tax). He is not availing SSP exemption. Particulars Certification of documents under Export and Import Policy of Government of India Preparation of the financial statement of ABC Ltd. Representation of the client before Income Tax Appellate Tribunal Receipts for tax consultancy provided in the month of December, 2011 Amount (`) 2,00,000 5,00,000 50,000 10,000

Service Tax

216

He has purchased certain capital goods like computers, laptops, printers, air-conditioners etc. on 01.01.2012 for ` 10,00,000 and has paid excise duty @ 10% plus EC plus Delhi VAT @ 12.5% Using the above information, calculate the value of taxable services for the quarter ended on March, 2012. Tax credit for capital goods shall be allowed to the extent of 100% in the first year itself. Solution: Calculation of the value of taxable services for the quarter ended on March, 2012:Particulars Amount (`) Certification of documents under Export and Import Policy of Government of India 2,00,000 Preparation of the financial statement of ABC Ltd. 5,00,000 Representation of the client before Income Tax Appellate Tribunal 50,000 Receipts for tax consultancy provided in the month of December, 2011 10,000 Total 7,60,000 Service Tax EC @ 2% SHEC @ 1% 7,60,000 x 10.3% 76,000 1,520 760 Less: Cenvat credit i.e. 10,00,000 x 10.3% 1,00,000 2,000 1,000 Balance Cenvat credit to be credit forward 24,000 480 240 If a Chartered Accountant is providing the following services, it will not be subject to service tax 1. Contributing articles to newspapers/magazines 2. Rendering services as author of a book 3. Delivering guest lecture 4. Editing any magazine etc. 5. Other services which are not considered to be services rendered in professional capacity

Service Tax

217

Question 72 (V. Imp.): Explain provisions relating to Scientific and Technical Consultancy Service. Answer: Scientific and Technical Consultancy Service Section 65(105)(za) th (w.e.f. 16 July, 2001, Notification No. 4/2001-ST dated 9th July 2001) Section 65(105)(za) Scientific And Technical Consultancy service means any service provided or to be provided, to any person, by a Scientist or a Technocrat, or any science or technology institution or organisation, in relation to scientific or technical consultancy. Section 65(92) - Scientific or technical consultancy means any advice, consultancy, or scientific or technical assistance, rendered in any manner, either directly or indirectly, by a scientist or a technocrat, or any science or technology institution or organisation, to any person, in one or more disciplines of science or technology. The following points have been clarified vide Letter No. F. NO. B-II/I/2000-TRU, Dated 9-7-2001 Annexure I. Point raised for clarification Clarification Whether services rendered by doctors, medical colleges, In common parlance, these categories of nursing homes, hospitals, diagnostic and pathological labs, service providers are not known as scientists etc. would come under the purview of the proposed levy. or technocrats or science or technology institutions or organisations. They will not be covered under service tax. Whether public funded research institutions like CSIR, Yes. Service tax is liable to be paid when any Indian Council for Agricultural Research, Defence scientific or technical consultancy service is Research and Development Organisation, Indian Institute rendered whether by public funded of Technology and IISC, Regional Engineering Colleges institutions or by private agencies. etc., which are exempt from payment of income-tax are covered under the service tax. Whether testing services will be covered under the Mere testing will not attract service tax. proposed levy? However, in case testing is an integral part of the consultancy, then such activity is part and parcel of the taxable service and no abatement of any kind admissible. Many public funded research institutions receive grants or In the facts of this case, no service is aids from the Government for conducting research/project rendered to any one. Hence the question of work. Whether such activities would be covered under the payment of service tax does not arise. levy? However, if they render service to anyone on payment basis, service tax will be payable on such services. Whether the service tax will be leviable on consultancy If scientific or technical consultancy is provided to Government departments, public sector provided to a Government department for undertakings? which consultation fees are received, then service tax would be applicable. As per rule 7 of Point of Taxation Rule, 2011, service tax shall be payable on actual receipt basis i.e. rule 3 of POT Rules, 2011 shall not be applicable.

Service Tax

218

Illustration 14: Mr. Mohit Agarwal , a technocrat, is providing scientific and technical consultancy services and his receipts are `20,00,000 through taxable services in the financial year 2010-11. He furnishes the following information relating to the services rendered , bills raised, amount received pertaining to this service, for the financial year ended on 31st March, 2012 as underParticulars Advice to Mr. Murali in regard to technical feasibility of a project for `2,00,000 Suggestion to Mrs. Neelu Bansal for improvement in existing technology for `3,00,000 Technical consultancy provided to Government departments for `6,00,000 Services provided to UNICEF, an international organisation in relation to adopting a new technology for `8,00,000 Service rendered on 22.12.2011 06.02.2012 05.01.2012 05.03.2012 Bill raised on 03.01.2012 06.02.2012 22.11.2011 05.03.2012 Amount received on 05.01.2012 15.12.2011 05.01.2012 22.03.2012

Service tax and education cess have been charged separately. 1. Compute service tax liability of Mr. Mohit Agarwal for the financial year 2011-12 and also determine the date upto which service tax should be paid. 2. If the return for the half year ending on September 2011 was filed on 27 th October 2011 compute penalty payable and also determine the last date upto which revised return can be filed. Solution: Since gross receipts in the earlier year has exceeded `10,00,000 hence service tax is payable. Further, since the service provider is an individual, service tax shall be paid on quarterly basis in the manner given below: As per rule 7 of Point of Taxation Rule, 2011, service tax shall be payable on actual receipt basis i.e. rule 3 of POT Rules, 2011 shall not be applicable. April to June, 2011 July to September, 2011 Nil Nil

October to December, 2011 Services provided to Neelu Bansal 3,00,000 Service tax @ 10% 30,000 Add: Education cess @ 3% 900 Total service tax liability 30,900 Last date for making payment shall be 5 th January 2012 and if payment is through internet banking, last date shall be 6th January 2012. January to March 2012 Advice to Mr. Murali Technical consultancy to government departments Services provided to UNICEF Nil Taxable value of services Service tax @ 10% Add: Education cess @ 3% 2,00,000 6,00,000 8,00,000 80,000 2,400

Service Tax Total service tax liability Last date for making payment shall be 31st March 2012.

219 82,400

2. If return for the half year ending September 2011 was filed on 27 th October 2011, the penalty shall be `500. Revised return can be submitted within a period of 90 days from the date of submission of the original return. 25th January 2012. Note: (1) As per Notification No.16/2002 ST dated 02.08.2002, the Central Government has exempted all the taxable services specified in section 65 of the Finance Act, 1994 provided by any person to the International organizations like UNICEF, from whole of the service tax leviable thereon under section 66 of the Act.

Service Tax

220

Question 73 (V. Imp.): Explain provisions relating to Commercial Training or Coaching. Answer: Commercial Training or Coaching Section 65(105)(zzc) (w.e.f. 1st July 2003, Notification No. 7/2003-ST dated 20th June 2003) Section 65(105)(zzc) Commercial Training or Coaching service means any service provided or to be provided, to any person, by a commercial training or coaching centre in relation to commercial training or coaching. Commercial Training or Coaching Centre shall include any centre or institute, by whatever name called, where training or coaching is imparted for consideration, whether or not such centre or institute is registered as a trust or a society or similar other organisation under any law for the time being in force and carrying on its activity with or without profit motive and the expression commercial training or coaching shall be construed accordingly. Section 65(26) Commercial training or coaching means any training or coaching provided by a commercial training or coaching centre. Section 65(27) Commercial training or coaching centre means any institute or establishment providing commercial training or coaching for imparting skill or knowledge or lessons on any subject or field other than the sports, with or without issuance of a certificate and includes coaching or tutorial classes. Services not liable to be taxed 1. As per N.N. 24/2004 dated 10.09.2004, Services rendered by Recreational Training Institute shall be exempt from service tax. Recreational training institute means a commercial training or coaching centre which provides training or coaching relating to recreational activities such as dance, singing, martial arts or hobbies. 2. Services rendered by Vocational Training Institute shall be exempt from service tax. As per Letter No. 334/1/2010 dated 26.02.2010, Vocational training institute means an Industrial Training Institute or an Industrial Training Centre affiliated to the National Council for Vocational Training, offering courses in designated trades as notified under the Apprentices Act, 1961. Some such trades are as given below: 1. Welder (Gas and Electric) 2. Electrician 3. Wireman 4. Auto Electrician 5. Carpenter 6. Plumber 7. Mason (Building Constructor) 8. Furniture and Cabinet Maker 9. Beautician 10. Hair Dresser 11. Health and slimming Assistant 12. Data Preparation & Computer Software 13. Desk Top Publishing Operator 14. Programming and systems Administration Assistant As per N.N. 33/2011 dated 25.04.2011, no service tax shall be charged in case of pre-school coaching or training. Also no service tax shall be charged in case of any coaching or training leading to grant of a certificate or diploma or degree or any educational qualification which is recognised by any law. As per Letter No. 334/3/2011 dated 28.02.2011, if any institute is providing recognised education as well as unrecognised education, in that case unrecognised education shall be taxable. As per N.N. 23/2010 dated 29.04.2010, if any vocational training provider registered under skill development initiative scheme with Director General of Employment and Training and is offering

3.

4. 5.

Service Tax

221

coaching in relation to Modular Employable Skill Courses approved by National Council of Vocational Training, it will be exempt from service tax. 6. 7. Individual imparting home coaching in their independent capacity i.e. not on behalf of a Commercial Training or Coaching Centre. In-house training provided to the employees by their employer in his independent capacity. However, if the employer avails the services of a Commercial Training or Coaching Centre for the purpose of imparting training to his employees, then employer (in his capacity as Service Recipient) will be liable to pay Service Tax to the concerned Commercial Training or Coaching Centre.

Question: ABC technical training institute is providing training for the course like welding, electrician, carpenter etc. it is not recognized by National Council for Vocational Training and has charged `11,00,000. Discuss tax consequences. Answer: The company is liable to pay service tax because it is not recognized by National Council for Vocational Training. If it is a recognized by National Council for Vocational Training, it will be exempt from service tax.

Service Tax

222

Question 74 (V. Imp.): Explain provisions relating to Technical Testing and Analysis Service. Answer: Technical Testing and Analysis Service Section 65(105)(zzh) (w.e.f. 1st July, 2003, Notification No. 7/2003-ST dated 20th June 2003) Section 65(105)(zzh) Technical Testing and Analysis service means any service provided or to be provided, to any person, by a Technical Testing and Analysis agency, in relation to technical testing and analysis. Section 65(106) - Technical testing and analysis means any service in relation to physical, chemical, biological or any other scientific testing or analysis of goods or material or information technology software or any immovable property, but does not include any testing or analysis service provided in relation to human beings or animals. Technical Testing and Analysis includes testing and analysis undertaken for the purpose of clinical testing of drugs and formulations; but does not include testing or analysis for the purpose of determination of the nature of diseased condition, identification of a disease, prevention of any disease or disorder in human beings or animals. Section 65(107) - Technical testing and analysis agency means any agency or person engaged in providing service in relation to technical testing and analysis. Exemption on services provided by Government owned State or District Level Laboratory As per Notification No.6/2006, services provided by Government owned State or District level laboratory in relation to testing and analysis of water quality shall be exempt from service tax. Exemption to approved clinical Research Organizations As per Notification No. 11/2007, dated: 1-3-2007, services provided by Clinical Research Organization approved to conduct clinical trials by the Drugs Controller General of India, in relation to testing and analysis of newly developed drugs, including vaccines and herbal remedies, on human participants so as to ascertain the safety and efficacy of such drugs on human participants shall be exempt from service tax. As per Circular/Letter D.O.F. NO. 334/1/2008-TRU, dated 29-2-2008, Testing and Analysis of IT software shall be included in technical testing and analysis services Exemption to Central and State Seed Testing Laboratories As per Notification No. 10/2010-ST, dated 27-2-2010, services provided by a Central or State Seed Testing Laboratory and Central or State Seed Certification Agency notified under the Seeds Act, 1966 to any person, in relation to technical testing and analysis shall be exempt from service tax. Illustration 15: Mr. X is running an agency of Technical Testing and Analysis service and submitted the information as (a) Provided services to ABC Ltd in relation to biological, chemical scientific testing services on 01.06.2011 for `12,00,000 (exclusive of Service Tax) and bill is issued on same date but ABC Ltd paid the same upto 01.09.2011.

Service Tax

223

(b) Input services taken `4,00,000 plus Service Tax plus EC on 15.06.2011 for providing output taxable service but the payment was not made till year end. (c) It provides services in regard to Testing and Analysis of Information Technology Software of `6,00,000 (exclusive of service tax) and billed on 10.01.2012 but amount is received on 30.04.2012. (d) It provides service for Testing and Analysis in regard to human beings or animals on 31.01.2012 and charged `14,00,000 for the same and payment was received on 15.02.2012. (e) Services provided to UNICEF on 01.02.2012 and charged `5,50,000 and issued bill on 10.02.2012 and amount received for the same on 26.02.2012 (f) It provide service for the purpose of clinical testing of drugs and formulation and received the advance for the same of `3,00,000 (exclusive of service tax) on 15.02.2012 the bill was issued on 01.04.2012 and service was completed on 15.04.2012. (g) Input service taken `2,00,000 plus Service Tax on 18.02.2012 for providing output taxable service and paid the amount on 18.04.2012. (h) Mr. X has made payment of Service Tax on 31stJuly, 2011 for the quarter April to June 2011 through cheque but the cheque was cleared on 4thAugust, 2011. (i) Mr. X has filed Service Tax return on 31st December, 2011 for the half year ended on 30th September, 2011. Presume gross receipt is not exceeding `60,00,000 and also assessee is not eligible for SSP exemption of `10,00,000. You are required to compute:(i) Service Tax Payable by Mr. X for the F.Y 2011-2012? (ii) Interest payable u/s 75 and penalty u/s 76? (iii) Whether it is mandatory to file Service Tax Return Electronically by Mr. X? (iv) Compute Penalty to be paid for late filing of Service Tax Return. (v) Upto what period the record of Service Tax should be maintained? Solution: April to June, 2011 Services provided to ABC Ltd Service tax @ 10% Add: Education Cess @ 3% Service Tax Liability Less Input Service Tax Credit (4,00,000 x 10.3%) Service Tax Payable ` 12,00,000 1,20,000 3,600 1,23,600 41,200 82,400

July to September, 2011 Service Tax Payable 41,200 Note: Since the Amount of input service is not paid on time i.e. within 3 month of input service taken hence Cenvat Credit taken earlier should be reversed back i.e. 41,200.( Rule 4(7) of Cenvat Credit Rules, 2004. October to December, 2011 NIL

Service Tax January to March, 2012 Services provided for IT software testing Services for testing and analysis on human or animal Services provided to UNICEF Service provided for Clinical testing and drugs Taxable value of services Service tax @ 10% Add: Education cess @ 3% Service Tax Liability Less: Input Service Tax Credit (2,00,000 x 10.3%) Service Tax Payable (ii) Interest u/s 75 82,400 x 15% x 26/366 = 878.03 Rounded off u/s 37D = 878 Penalty u/s 76 1% of the amount of default for 26 days 82,400 x 1% x 26/31 = 691.09 Rounded Off u/s 37D = 691 or Penalty calculated @ `100 per day for 26 days `2,600 Hence Penalty would be ` 2,600

224 6,00,000 NIL NIL 3,00,000 9,00,000 90,000 2,700 92,700 20,600 72,100

(iii) Notification No. 43/2011 Every assessee shall submit the half yearly return electronically. Hence Mr. X is required to file Service Tax Return electronically. (iv) Penalty for late filing of Service Tax Return Rule7C (Service Tax Rules, 1994) Upto 30 days `1,000 and after that `100 per day i.e. for 37 days (from 25th November, 2011 to 31st December, 2011) = `3,700 Hence penalty shall be `4,700. (v) According to Rule 5 (Service Tax Rules, 1994) All records should be preserved for at least 5 years immediately after the financial year to which such records pertain. Question: Max Hospital has rendered services for technical testing and analysis services with particulars as given below: 1. Conducted tests for determination of symptoms of various diseases and charged `6,00,000 2. Given vaccinations for prevention of diseases and charged `3,00,000 3. Conducted tests for determination of disorder in human being (blood test, echo-cardiograph, liver function test, lipid profile, TSH, Bone densitometry, kidney test, Treadmill test (TMT), Ultra-sound etc) and charged `25,00,000 They have not collected any service tax. Discuss tax consequences. Answer: As per section 65(106), all the above services are exempt from service tax. Question: Shri Ram Centre for water research has rendered technical testing and analysis services for testing and analysis of water and charged `5,00,000. Discuss tax consequences.

Service Tax

225

Answer: As per Notification No.6/2006, services provided by Government owned State or District level laboratory in relation to testing and analysis of water quality shall be exempt from service tax. Since exemption is given only to the Government laboratory, in the given case, service tax is payable. Question 75 (V. Imp.): Explain provisions relating to Business Exhibition Service. Answer: Business Exhibition Service Section 65(105)(zzo) th (w.e.f. 10 September 2004, vide Finance (No.2), Act, 2004.) Section 65(105)(zzo) Business Exhibition service means any service provided or to be provided, to an exhibitor, by the organiser of a business exhibition, in relation to business exhibition. Section 65(19a) - Business exhibition means an exhibition, (a) to market; or (b) to promote; or (c) to advertise; or (d) to showcase, any product or service, intended for the growth in business of the producer or provider of such product or service, as the case may be. As per Circular No. 80/10/2004-S.T., DATED 17-9-2004, business exhibition service is a service rendered to an exhibitor by an organizer of a business exhibition that intends to market, promote, advertise or showcase products or services for growth in business of the producers or providers of such products or services. Thus, organizers of events such as trade fairs, road shows, fashion shows, display show-cases kept in airports, railway stations, hotels etc. would be covered under Business Exhibition Service. A display of consumer goods in shops or shopping centres for customers to select and purchase would normally not attract any service tax, as normally no separate charges are collected by the shop-keepers for displaying such goods. However, in case an amount is collected for merely displaying an item, the same would be chargeable to service tax. While event management service (a currently taxable service) also relates to organizing such events, but in that case, the services are rendered to the organizer by an event manager in relation to planning, promoting, organizing etc. Thus, an organizer of a business exhibition is not covered under Event Management Services, but would be covered under Business Exhibition Services. Similarly, while services rendered in relation to a circular, label, documents, hoardings or any other audio visual representation of a product or service falls under advertisement services, the services relating to actual exhibition or display of the product or services would fall under the category of Business Exhibition Services. Illustration 16: ABC Ltd. is engaged in providing Business Exhibition services and the company has submitted the information as given below for the month of September 2011. 1. Services completed for X Ltd. for arranging one fashion show on 10.09.2011 and invoice issued under Rule 4A on 22.09.2011 and amount is `2,00,000 plus service tax and it was received on 10.10.2011. 2. Services completed for Y Ltd. for arranging road show on 01.10.2011 but bill issued on 30.09.2011 and payment was received in advance on 31.08.2011 and amount is `3,00,000 plus service tax.

Service Tax

226

3. Services completed for Z Ltd. for arranging trade fair on 10.09.2011 but bill issued on 10.10.2011 and payment was received on 03.10.2011 and amount is `4,00,000 plus service tax. Compute amount of service tax payable for the month of September 2011 and also the last date upto which payment should be made. Service tax paid by the company in the Financial year 2010-11 was `11,00,000. (b) If the payment was delayed by 10 days. Compute interest under section 75 and penalty under section 76. (presume gross receipt is exceeding `60,00,000) Solution: 1. In this case point of taxation shall be 22.09.2011 and service tax should be paid upto 6th October 2011. 2. In this case, point of taxation shall be 31.08.2011, hence service tax should be paid upto 6 th September 2011. 3. In this case, point of taxation shall be 10.09.2011 and service tax shall be paid upto 6th October 2011 Amount of service tax to be paid for September 2011 is as given below: Amount of taxable service 2,00,000 Amount of taxable service 4,00,000 Total amount of taxable services 6,00,000 Service Tax @ 10.3% 61,800 Last date for payment of service tax shall be 6th October 2011. (b) In case of delay for 10 days, interest under section 75 shall be i.e. 61,800 x 18% x 10/366 = `303.93 Rounded off under section 37D = `304 Penalty under section 76 i.e. 100 x 10 = `1,000 Or 61,800 x 1% x 10/31 = `199.35 Penalty shall be `1,000 Illustration 17: Abhi-Ash Services Ltd. Provides Business Exhibition Services to the exhibitors for organizing trade fairs, road shows, fashion shows etc. and submitted the information as follows:(a) Provided service to Bachan Ltd. on 01.10.2011 for ` 8,50,000 and bill was issued on the same date. (b) Input services taken of ` 12,00,000 on 02.11.2011 for providing output taxable services and paid the amount on the same date. (c) It has provided services of ` 12,50,000 to Hero Honda Ltd. For road show on 18.12.2011 and billed the same on 20.12.2011. No payment has been received till the end of the year. (d) Services provided to UNICEF on 05.10.2011 and charged ` 5,00,000 and issued bill on 10.02.2012 amount for the same was received on 26.02.2012.

Service Tax

227

(e) It has provided services for which advance was received of ` 30,000 on 14.11.2011. However, the bill was issued on 01.03.2012 and services were completed on 15.03.2012. (f) Input services taken of ` 10,000 on 10.12.2011 for providing output taxable services, but the payment was not made till the end of the year. (g) Company has made payment of Service Tax on 31.01.2012 for the month December 2011 through cheque although the cheque was cleared on 05.02.2012. (h) Company has filed Service Tax Return for the half year ended 31 st March, 2012 on 31st December, 2012. (All the above figures are excluding of Service Tax) You are required to compute:(i) Service Tax Payable by Abhi-Ash Services Ltd. for the months October2011 to December2011? (ii) Interest payable u/s 75 (presume rate to be 15%) and penalty us/ 76? (iii) Whether it is mandatory to file Service Tax Return electronically by Abhi-Ash Services Ltd.? (iv) Compute Penalty to be paid for late filing of Service Tax Return. (v) Upto what period the record of Service Tax should be maintained? Solution: (i) Computation of Tax Liability of Abhi-Ash Services Ltd. October, 2011 Services provided to Bachan Ltd. Service Tax @10% Add: Education Cess @3% Service Tax Payable November, 2011 Advance Received on 14.11.2011 Service Tax @10% Add: Education Cess @3% Service Tax Payable Less: Input Service Tax Credit (12,00,000 x 10.3%) Balance Cenvat credit carried forward December, 2011 Services provided to Hero Honda Ltd. Service Tax @10% Add: Education Cess @ 3% Service Tax Payable Less: Cenvat Credit brought forward Less: Cenvat Credit (10,000 x 10.3%) Balance Service Tax Payable ` 8,50,000 85,000 2,550 87,550 30,000 3,000 90 3,090 1,23,600 1,20,510 12,50,000 1,25,000 3,750 1,28,750 1,20,510 1,030 7,210

Note: (i) Services provided to UNICEF is exempt from Service Tax. (ii) Since the Amount of input service is not paid on time i.e. within 3 month of input service taken hence Cenvat Credit taken earlier should be reversed back i.e. `1,030 in the Month of March 2012.

Service Tax (ii) Interest u/s 75 7,210 x 15% x 26/366 = 76.83 = 77 Penalty u/s 76 Higher of the following:(a) @ 100 per day for 26 days = `2,600 Or (b) 7,210 x 1% x 26/31 = ` 60.47 = 60 Hence the penalty would be ` 2,600

228

(iii) Every assessee shall submit the half yearly return electronically. Hence, Abhi-ash services Ltd. required to file Service Tax return electronically. (iv) Penalty for late filing of Service Tax Return Rule7C (Service Tax Rules, 1994) Upto 15 days ` 500 Next 15 days ` 500 On Balance Days (i.e. for 220 days from 26.05.2012 to 31.12.2012 @100 per day) `22,000 Total Penalty Payable `23,000 However, Maximum penalty that can be imposed is ` 20,000. Hence the penalty would be ` 20,000. (v) According to Rule 5 of Service Tax Rules, 1994 All records should be preserved for atleast period of 5 years, immediately succeeding the financial year to which such records pertains.

Service Tax

229

Question 76 (V. Imp.): Explain provisions relating to Information Technology Software Service. Answer: Information Technology Software Service Section 65(105)(zzzze) th (w.e.f. 16 May 2008, Notification No. 18/2008-ST dated 10th May 2008) Section 65(105)(zzzze) - Taxable service means any service provided or to be provided, to any person, by any other person in relation to information technology software including, (i)development of information technology software, (ii) study, analysis, design and programming of information technology software, (iii) adaptation, upgradation, enhancement, implementation and other similar services related to information technology software, (iv) providing advice, consultancy and assistance on matters related to information technology software, including conducting feasibility studies on implementation of a system, specifications for a database design, guidance and assistance during the start-up phase of a new system, specifications to secure a database, advice on proprietary information technology software, (v) providing the right to use information technology software for commercial exploitation including right to reproduce, distribute and sell information technology software and right to use software components for the creation of and inclusion in other information technology software products, (vi) providing the right to use information technology software supplied electronically; Section 65(53) "information" has the meaning assigned to it in clause (v) of sub-section (1) of section 2 of the Information Technology Act, 2000 (21 of 2000). Section 65(53a) - "information technology software means any representation of instructions, data, sound or image, including source code and object code, recorded in a machine readable form, and capable of being manipulated or providing interactivity to a user, by means of a computer or an automatic data processing machine or any other device or equipment. Section 2(1)(v) of the Information Technology Act, 2000 - information includes data, text, images, sound, voice, codes, computer programmes, software and databases or micro film or computer generated micro fiche. Receipts for providing the right to use the packaged software is exempt under Notification No. 53/2010. Illustration 18: Swastik Services Ltd. is carrying on the business of providing Information Technology software services and has provided the following information:

Service Tax

230

It has provided software upgradation service to ABC Private Ltd. on 12.03.2012 electronically and has charged ` 23,00,000. However, the bill was issued on 20.03.2012. It has conducted feasibility study on implementation of a new IT system for SEZ which is completed on 15.01.2012 and received payment of ` 1,50,000 on the same date. It has also provided software development service to Mr. Amit for personal use on 18.01.2012 for `1,80,000 and received the payment on the same date. Swastik Services Ltd. has taken input services of ` 80,000 for providing taxable output services on 15.03.2012. Company has filed Service Tax return for the Half year ended September 2011 on 15.03.2012.

(All the above figures are exclusive of Service Tax) Ignore SSP exemption. You are required to:(i) Compute Service Tax payable by the Swastik Services Ltd. for the period from 01.01.2012 to 31.03.2012. (ii) Compute the amount of Penalty on late filing of service tax return by the company. What is the maximum amount of penalty that can be imposed? Solution: (i) Computation of Service Tax payable by Swastik Services Ltd. January 2012 Value of services provided to Mr. Amit Service Tax @ 10% Add: Education Cess @ 3% Total Service Tax payable February 2012 March 2012 Value of Service provided to ABC Pvt. Ltd. Service Tax @ 10% Add: Education Cess @ 3% Total Service Tax payable Less: CENVAT Credit Availed (80,000 x 10.3%) Net Service Tax payable (ii) Penalty for late filing of Service Tax Return Rule7C (Service Tax Rules, 1994) Upto 15 days Next 15 days On Balance Days (i.e. for 112 days from 25.11.2011 to 15.03.2012 @100 per day) Total Penalty Payable Maximum amount of penalty that can be imposed is ` 20,000 ` 1,80,000 18,000 540 18,540 NIL 23,00,000 2,30,000 6,900 2,36,900 8,240 2,28,660 ` 500 500 11,200 12,200

Service Tax

231

Question: ABC Ltd. is selling various softwares like Quick Heal Anti Virus, Tally, Windows, Office etc. and is not charging service tax. Discuss tax consequence. Answer: No service tax is payable in case of sale of such readymade softwares which is called packaged software rather it is subject to State VAT as per N.N. 53/2010. Question: INFOSYS Limited has rendered services for study, analysis, design and programming of information technology software, outside India and has charged `600 lakhs. They are also rendered services for providing advice and consultancy for the matters of IT Software in India and has charged `400 lakhs. They have not collected any service tax. They have taken input services for rendering such services and has paid `100 lakh plus service tax. Discuss tax consequences. (Ignore SSP exemption) Answer: Services rendered outside India are exempt but services rendered in India are taxable and `400 lakhs shall be considered to be inclusive of service tax and output service tax shall be 400,00,000 x 10.3 / 110.3 37,35,267.00 Company shall be allowed tax credit of `100,00,000 x 10.3% 10,30,000.00 Net Tax Payable shall be 27,05,267.00

Service Tax

232

PRACTICE PROBLEMS
TOTAL PROBLEMS 11
Problem 1: Mr. X is rendering services which are exempt from service tax but the services have become taxable w.e.f. 01.08.2011 and he has issued the bills and received amount of `4,00,000 upto 31.07.2011. He has crossed the limit of `14,00,000 on 01.01.2012. He has taken input services in September 2011 and has paid `3,00,000 plus service tax. He has again taken input services in March 2012 and has paid `2,00,000 plus service tax. His services are covered u/s 194C of Income Tax Act and are subject to TDS @ 1%. Other expenses incurred in connection with services are `6,00,000. Bills issued and amount received upto 31.03.2012 is `25,00,000 The bills issued are inclusive of service tax. Compute income tax liability, service tax liability and amount of TDS. (b) Presume he has rejected general exemption of `10,00,000 Problem 2: Mr. X is rendering services which are exempt from service tax but the services have become taxable w.e.f. 01.07.2011 and he has issued bills and received `2,00,000 upto 30.06.2011. He has crossed the limit of `12,00,000 on 27.01.2012. He has taken input services in October 2011 and paid `3,00,000 plus service tax. He has taken input services in February 2012 and has paid `4,00,000 plus service tax. His services are covered u/s 194J of Income Tax Act and are subject to TDS @ 10% Total receipt upto 31.03.2012 is `32,00,000. The bills issued are inclusive of service tax. Compute service tax, income tax, TDS. If registration was delayed by 10 days what will be the penalty under section 77. If service tax payment was delayed by 20 days, compute interest u/s 75 and penalty u/s 76.

Service Tax If return was delayed by 50 days, compute penalty under Rule 7C. (b) Presume bills issued were exclusive of service tax.

233

Problem 3: ABC Limited is a service provider and is registered with service tax department and is liable to pay service tax and the company has submitted the information as given below: Date of Completion 07/04/2011 30/05/2011 30/09/2011 07/02/2012 Date of Invoice Issued 20/04/2011 10/06/2011 07/10/2011 28/02/2012 Date of Payment Received 07/05/2011 18/06/2011 06/10/2011 10/02/2012 Value (exclusive of service tax) 3,00,000 2,00,000 4,00,000 5,00,000

Determine the amount of service tax payable by the company for each month and also the last day up to which amount should be paid. If every payment was delayed by 10 days, compute interest payable under section 75 (presume gross receipt to be ` 60 Lakh) Problem 4: M/s Alpha Multi Services Pvt. Ltd. is engaged in providing coaching as well as Dance classes and registered in Service Tax. Following value of service was provided during the financial year 2011-12. i) Value of Coaching service ` 40,00,000 (taxable @ 10.3%) ii) Value of Dance service ` 55,00,000 (Exempted service) During the year M/s Alpha multi service receives services in relation to their business and paid service tax of ` 4,00,000 plus education cess and SHEC. Compute Cenvat credit and Net Service Tax payable. Problem 5: Manorama Banquets Private Limited furnishes the following information for the months of April, May and June 2011. Particulars Date of Date of Date of Completion Payment Invoice of Service Received issued 1. Let out the Banquets for ` 5, 00,000 for the marriage of 24.05.2011 30.06.2011 06.06.2011 Mr. Ghanshyam. 2. Banquets is booked for ` 3,00,000 by United Nations 27.06.2011 30.08.2011 15.07.2011 th for conducting a seminar on 27 June, 2011 3. Banquets is booked for ` 10,00,000 for organizing a 15.04.2011 30.04.2011 04.05.2011 music programme by Zee India Private Limited 4. Let out the Banquets for the marriage of Mr. 21.04.2011 30.04.2011 03.05.2011 Radheshyam Sharma, Banquets booked for 1000 person and the rate per plate per person is ` 1500/- (inclusive of all

Service Tax the charges in relation to rent, electricity, catering etc.) 5. Banquets is booked for `5,00,000 by US Embassy for conducting a seminar 25.05.2011 05.07.2011

234 07.06.2011

(All the above figures are exclusive of service tax) 1. Calculate the Service Tax Payable for each of the three months and the last date of depositing the service tax. (service tax paid during the last year was `10,02,000 including the amount of `50,000 paid through cenvat credit.) 2. Whether it is mandatory to file return electronically and mention the last date of filing of service tax return for the half year ended on 30th September, 2011. 3. Whether Return can be revised. Problem 6: Krishna Private Ltd is providing Scientific and Technical Consultancy Service and has submitted the information as follows:(a) Provided consultancy on some scientific matter to ABC Ltd on 01.07.2011 for `32,00,000 (exclusive of Service Tax) and bill is issued on 01.08.2011 but ABC Ltd paid the same on 31.08.2011. (b) Input service taken of `9,00,000 plus Service Tax plus EC on 12.07.2011 for providing output taxable service and payment was made on same date. (c) It provides consultancy service to Government department on 10.07.2011 for `26,00,000 plus Service Tax plus EC and billed on 14.07.2011 but amount is received on 31.07.2011. (d) It provided testing service on 10.07.2011 for `14,00,000 and billed on the same date and payment was received on 15.10.2011. (e) Services provided to Chinese Embassy on 01.08.2011 and charged `5,50,000 and issued bill on 10.08.2011 and amount received for the same on 26.08.2011. (f) Received advance for providing service of `45,00,000 plus service tax plus EC from an individual on 25.08.2011, the bill was issued on 30.08.2011 and service was completed on 15.09.2011. (g) It provided services to partnership firm and charged `17,00,000 on 01.09.2011 and bill was issued on same date but the payment was received on 10.09.2011. (h) Input service taken of `6,00,000 plus Service Tax plus EC for providing output taxable services on 30.09.2011 and payment was made on the same date. (i) The payment of Service Tax was made on 31stJuly, 2011 of `7,29,600 (provisional basis). (j) Service Tax return was filed on 31st December, 2011 for the half year ended on 30th September, 2011. You are required to compute:(i) Service Tax Payable by Krishna Private Ltd for the months of July 2011 September 2011, if it has paid Service Tax on centralized billing system? (ii) Will your answer be different in case the company is adopting decentralized billing system? (iii) Whether it is mandatory to file Service Tax Return Electronically? (iv) Compute Penalty to be paid for late filing of Service Tax Return.

Service Tax

235

(v) Upto what period the record of Service Tax should be maintained. Is there any penalty for not maintaining of service tax record? (vi) If this is not Krishna Private Ltd and service was provided by an institution which is funded by Government then whether it is taxable? (vii) Mention the form in which Service Tax Return is to be submitted. Problem 7: M/s International exhibition service P. Ltd. has started providing Business Exhibition services to its clients and has submitted the following details of service provided during the FY 2011-12. i) Charged ` 11,00,000 for displaying product of M/s Healthy Goods at Indian trade fair. Bill raised on Healthy Goods on 05.04.2011 and service was completed on 07.05.2011 and payment was received on 20.05.2011. Charged ` 9,00,000 for organising fashion show for Allen Cooper. Payment received from Allen Cooper on 10.04.2011 and bill raised on 18.06.2011 and service completed on 20.06.2011. Charged ` 7,00,000 for exhibiting the product outside India. Service rendered outside India on 01.07.2011 and Bill raised and payment received on 01.08.2011 and 01.08.2011 respectively. It has organised Road shows and charged ` 10,00,000. Road show organised on 12.12.2011 and Bill raised and payment received on 12.01.2011. Charged ` 15,00,000 for displaying different products in show-cases at airport and Railway stations. Bill raised for displaying the product on 05.01.2012 and services completed on 07.01.2012 and payment received on 08.01.2012.

ii)

iii)

iv) v)

All the above figures are exclusive of Service Tax. Required: Determine the service tax liability of the company as per Point of Taxation Rules 2011 for each month. Problem 8: Sandhya Services Ltd. is registered in the state of Jammu & Kashmir. It is engaged in the business of providing services related to IT Software. Gross Value of taxable services provided by it during F.Y. 201011 was `60 Lacs. It has submitted following information:(a) It has provided services to a company registered in Jammu & Kashmir on 25.04.2011 and charged `12,25,000. Bill was issued on 05.05.2011. (b) It has provided services to ABC Ltd., a company registered in Delhi, and charged ` 5,00,000 which was completed on 16.05.2011. However, the bill was issued on 15.07.2011. (c) It has also provided services to the developer of SEZ on 11.05.2011 and charged ` 70,000 for the value of service. Bill was issued on 15.05.2011. However, payment was received in advance on 14.04.2011. (d) Input services taken of `1,50,000 for the output services provided in the state of Jammu & Kashmir on 26.04.2011. Payment was made on the same date.

Service Tax

236

(e) It has also provided services to R Ltd., registered in Agra, for `25,00,000 (excluding service tax) on 17.07.2011. However the payment was received in advance on 27.06.2011. (f) Payment of service tax for the month ended June 2011 was made electronically on 31st July 2011. (g) Return for the half year ended 30th September 2011 was filed on 18.12.2011. You are required to compute: (a) Service Tax liability of Sandhya Services Ltd. for the period April 2011 to June 2011. (b) Interest u/s75 and penalty u/s 76. (c) Amount of penalty on late filing of service tax return. What is the due date of filing of service tax return by a company? Problem 9: Compute amount of TDS in the following cases (a) ABC Ltd. has let out one commercial building to XYZ Ltd. and rent charged is `31,00,000 plus service tax. (b) Mr. X a property dealer has rendered services for getting one commercial building on rent to XYZ Ltd. and Commission charged is `8,00,000 plus service tax. (c) Haryana Government has given contract for sale of lottery Tickets to XYZ Ltd. and XYZ Ltd. has charged commission of `35,00,000 plus service tax from Haryana Government. (d) If LIC has to commission of `5,00,000 to their agent Mr. X in connection with insurance business. (e) Mr. X, a Chartered Accountant has given professional services to XYZ Ltd. and charged `25,00,000 plus service tax. (f) ABC Ltd. has rendered advertising services to XYZ Ltd. and charged `35,00,000 plus service tax. Problem 10: Amir Khan Pvt Ltd is a Bombay based company and it has one branch in Delhi and they are providing services which are chargeable to service tax and they have submitted the particulars for Jan, Feb and March 2012 as given below: i) Provided services by the Bombay office to XYZ Ltd, Registered in Delhi and charged `45,00,000 and services were completed on 10.01.2012 and bill was issued on 15.01.2012 but payment was received on 01.02.2012. Bombay office has taken input service of `90,00,000 plus service tax plus EC and amount is paid on 01.01.2012. Bombay office provided services in Indian territorial water to a partnership firm and charged `31,00,000 and services were completed on 01.02.2012 and bill is issued on 10.02.2012 and payment is not received till year end.

ii) iii)

Service Tax iv)

237

Delhi Office provided services to ABC Ltd in Lucknow and charged ` 6,00,000 and services were completed on 01.03.2012 and bill issued on 31.03.2012 but payment was not received till year end. Delhi Office provided services to individual in Jammu and Kashmir and charged ` 4,00,000.

v) vi)

Delhi Office provided services in Delhi on 10.03.2012 and bill was issued on 01.04.2012 and payment was received on 07.04.2012 and charged `12,00,000. (Presume all the amounts are excluding service tax) Each of the branch has been registered separately. Compute Service tax payable by each of the branch for the month of Jan, Feb and March 2012. They are not eligible for SSP exemption. Bombay office is registered as input service distributor and they have transferred the balance of cenvat credit to their Delhi branch (b) Presume there is centralized registration and Bombay Office is registered as head office. Problem 11: Mr. Sachin Bahl is a practicing Chartered Accountant and he has submitted the following information as:(a) Service of Income tax return preparation and filing etc is provided on 10 th July 2011 for `1,25,000 (inclusive of service tax) and issued the bill on 15th July, 2011 and payment received on 31st July, 2011. (b) He purchased a laptop of `40,000 on 10th January, 2012 and paid excise duty @ 10% plus EC @ 2% plus SHEC @ 1% and DVAT @ 12.5%. Payment was made on same date. Cenvat Credit of Excise duty paid on capital goods is allowed 50% in the current year and balance in the subsequent year. (c) He rendered services during February 2012 billed `4,00,000 (inclusive of service tax) and received the same in February and he has also received advance on 28thFebruary, 2012 `1,00,000 (inclusive of service tax) for which service is to be rendered in April 2012. (d) He provided services on 10thMarch, 2012 of `2,00,000 and bill is issued on 20thMarch, 2012 but payment for the same is not received till year end. (e) Provided services to jewellers in March, 2012 and received `1,50,000 in cash (inclusive of service tax) and also received two gold coins and market value of the same is `60,000. (f) He also edits a Journal published by Taxman and received payment of `1,00,000 on 25th March, 2012 regarding the same. No service tax has been collected. (g) One of the clients Mr. Rahul has not paid income tax. Income Tax Officer has issued a notice u/s 148 to Rahul for assessing income escaping assessment. Rahul has approached to Mr. Sachin Bahl to represent him before the Income Tax Officer and Mr. Sachin Bahl has received `80,000 (inclusive of service tax) for that on 28th March, 2012. (h) He has also provided services to a unit in SEZ on 15 thMarch, 2012 and has charged amount `2,50,000 and received on 31stMarch, 2012.

Service Tax (i) Service Tax payment is delayed by 10 days for the 2nd quarter.

238

(j) He has submitted Service Tax Return on 31 st October, 2011 for half year ended on 30 th September 2011 and on 20thApril, 2012 for half year ended on March 2012. Compute Service Tax Liability of Mr. Sachin Bahl. Also compute interest and penalty under section 75 and section 76 respectively for the financial year 2011-12. (Previous year receipt is upto 60 lakhs) (i) Upto what period the record of Service Tax should be maintained. Is there any penalty for not maintaining of service tax record? (ii) Whether it is mandatory to file Service Tax Return Electronically? (iii) Can the Service Tax Return be revised?

Service Tax

239

SOLUTIONS
TO

PRACTICE PROBLEMS
Solution 1(a): If any service provider has taken any services and has paid service tax, cenvat credit shall be allowed for the service tax paid on input services but if service provider is not liable to pay service tax, no cenvat credit is allowed. Similarly, if the service provider is claiming exemption for `10,00,000, no cenvat credit shall be allowed for service tax paid by him on input services unless he has achieved the limit of `10,00,000. In the given case, no cenvat credit is allowed for input services taken in the month of September 2011. He will be allowed cenvat credit for input services availed in March 2012 because at that time he is liable to pay service tax because the limit of `10,00,000 has already been crossed. ` Gross receipts 25,00,000.00 Less: Amount received when services were exempt 4,00,000.00 Less: General exemption 10,00,000.00 11,00,000.00 Output tax (11,00,000 / 110.3 x 10.3) Less: Cenvat credit (2,00,000 x 10.3%) Net tax Rounded off u/s 37D Computation of total income Gross receipts (excluding service tax) (25,00,000 1,02,719.85) Less: Input services in September 2011 (3,00,000 + 10.3%) Less: Input services in March 2012 Less: Expenses Total income Rounded off u/s 288A Computation of tax liability Tax on `12,66,380 at slab rate Add: EC @ 2% Add: SHEC @ 1% 1,02,719.85 20,600.00 82,119.85 82,120.00 23,97,280.15 3,30,900.00 2,00,000.00 6,00,000.00 12,66,380.15 12,66,380.00 2,31,914.00 4,638.28 2,319.14

Service Tax Tax liability Less: TDS Tax payable Rounded off u/s 288B

240 2,38,871.42 25,000.00 2,13,871.42 2,13,870.00

Solution 1(b): Since the service provider has rejected general exemption, service tax shall be charged from the beginning when the services became taxable. Cenvat credit shall be allowed for the input services from the beginning itself. ` Gross receipts 25,00,000.00 Less: Amount received when services were exempt 4,00,000.00 21,00,000.00 Output tax (21,00,000 / 110.3 x 10.3) Less: Cenvat credit (3,00,000 x 10.3%) Less: Cenvat credit (2,00,000 x 10.3%) Net tax Rounded off u/s 37D Computation of total income Gross receipts (excluding service tax) (25,00,000 1,96,101.54) Less: Input services in September 2011 Less: Input services in March 2012 Less: Expenses Total income Rounded off u/s 288A Computation of tax liability Tax on `12,03,900 at slab rate Add: EC @ 2% Add: SHEC @ 1% Tax liability Less: TDS Tax payable Rounded off u/s 288B 1,96,101.54 30,900.00 20,600.00 1,44,601.54 1,44,602.00 23,03,898.46 3,00,000.00 2,00,000.00 6,00,000.00 12,03,898.46 12,03,900.00 2,13,170.00 4,263.40 2,131.70 2,19,565.10 25,000.00 1,94,565.10 1,94,570.00

Solution 2(a): If any service provider has taken any services and has paid service tax, cenvat credit shall be allowed for the service tax paid on input services but if service provider is not liable to pay service tax, no cenvat credit is allowed. Similarly, if the service provider is claiming exemption for `10,00,000, no cenvat credit shall be allowed for service tax paid by him on input services unless he has achieved the limit of `10,00,000. In the given case no cenvat credit is allowed for input services taken in the month of October 2011. He will be allowed cenvat credit for input services availed in February 2012 because at that time he is liable to pay service tax because the limit of `10,00,000 has already crossed. `

Service Tax Gross receipts Less: Amount received when services were exempt Less: General exemption Balance Output tax (20,00,000 / 110.3 x 10.3) Less: Cenvat credit (4,00,000 x 10.3%) Net tax Rounded off u/s 37D Computation of total income Gross receipts (excluding service tax) (32,00,000 1,86,763.37) Less: Input services in October 2011 (3,00,000 + 10.3%) Less: Input services in February 2012 Less: Interest paid u/s 75 Total income Rounded off u/s 288A Computation of tax liability Tax on `22,81,140 at slab rate Add: EC @ 2% Add: SHEC @ 1% Tax liability Less: TDS Tax payable Rounded off u/s 288B Penalty for registration In case of delay, penalty shall be = 200 x 10 = 2,000 or 10,000 whichever is higher i.e. `10,000. Last date of payment of service tax = 31.03.2012 Interest payable under section 75 1,45,563 x 15% x 20/365 = `1,196.41 = 1,196 Penalty payable under section 76 1,45,563 x 1% x 20/30 = `970.42 Penalty calculated @ `100 per day for 20 days = `2,000 Penalty liable to be paid is `2,000 In case of delay in furnishing the return, penalty shall be `1,000 + `2,000 = `3,000 under Rule 7C.

241 32,00,000.00 2,00,000.00 10,00,000.00 20,00,000.00 1,86,763.37 (41,200.00) 1,45,563.37 1,45,563.00 30,13,236.63 3,30,900.00 4,00,000.00 1,196.00 22,81,140.63 22,81,140.00 5,36,342.00 10,726.84 5,363.42 5,52,432.26 3,20,000.00 2,32,432.26 2,32,430.00

Solution 2(b): If any service provider has taken any services and has paid service tax, cenvat credit shall be allowed for the service tax paid on input services but if service provider is not liable to pay service tax, no cenvat credit is allowed. Similarly, if the service provider is claiming exemption for `10,00,000, no cenvat credit shall be allowed for service tax paid by him on input services unless he has achieved the limit of `10,00,000. In the given case no cenvat credit is allowed for input services taken in the month of October 2011.

Service Tax

242

He will be allowed cenvat credit for input services availed in February 2012 because at that time he is liable to pay service tax because the limit of `10,00,000 has already crossed. ` Gross receipts 32,00,000.00 Less: Amount received when services were exempt 2,00,000.00 Less: General exemption 10,00,000.00 Balance 20,00,000.00 Output tax (20,00,000 x 10.3%) Less: Cenvat credit (4,00,000 x 10.3%) Net tax Computation of total income Gross receipts (excluding service tax) Less: Input services in October 2011 (3,00,000 + 10.3%) Less: Input services in February 2012 Less: Interest paid u/s 75 Total income Rounded off u/s 288A Computation of tax liability Tax on `24,67,750 at slab rate Add: EC @ 2% Add: SHEC @ 1% Tax liability Less: TDS (12,00,000 + 22,06,000) x 10% Tax payable Rounded off u/s 288B Penalty for registration In case of delay, penalty shall be = 200 x 10 = 2,000 or 10,000 whichever is higher i.e. `10,000. Last date of payment of service tax = 31.03.2012 Interest payable under section 75 1,64,800 x 15% x 20/365 = `1,354.52 = `1,355 Penalty payable under section 76 1,64,800 x 1% x 20/30 = `1,098.66 Penalty calculated @ `100 per day for 20 days = `2,000 Penalty liable to be paid is `2,000 In case of delay in furnishing the return, penalty shall be `1,000 + `2,000 = `3,000 under Rule 7C. Solution 3: April 2011 3,00,000 x 10.3% (POT 20.04.2011) Last date of payment Interest payable u/s 75 [30,900 x 10/366 x 15%] Interest rounded off u/s 37D May 2011 ` 30,900 5th May, 2011 126.64 127 NIL 2,06,000.00 (41,200.00) 1,64,800.00 32,00,000.00 3,30,900.00 4,00,000.00 1,355.00 24,67,745.00 24,67,750.00 5,92,325.00 11,846.50 5,923.25 6,10,094.75 3,40,600.00 2,69,494.75 2,69,490.00

Service Tax June 2011 2,00,000 x 10.3% (POT 10.06.2011) Last date of payment Interest payable u/s 75 [20,600 x 10/366 x 15%] Interest rounded off u/s 37D July 2011 to September 2011 October 2011 4,00,000 x 10.3% (POT 06.10.2011) Last date of payment Interest payable u/s 75 [41200 x 10/366 x 15%] Interest rounded off u/s 37D November 2011 to January 2012 February 2012 5,00,000 x 10.3% (POT 07.02.2012) Last date of payment Interest payable u/s 75 [51,500 x 10/366 x 15%] Interest rounded off u/s 37D March 2012

243

20,600 5th July, 2011 84.43 84 Nil 41,200 5 November, 2011 168.85 169
th

Nil 51,500 5th March, 2011 211.07 211 Nil

Solution 4: Since Service provider is engaged in providing taxable as well as exempted service and input service is being used for providing both the services, in such case, he has following option for availing Cenvat credit in accordance to Rule 6 of Cenvat credit Rule, 2004. i) Maintain Separate Accounts. ii) Pay an amount equal to 5% of value of exempted service i.e. Particulars Amount payable on taxable services Amount payable on exempt services under rule 6 i.e. 5% of `55,00,000 Less: CENVAT Credit Net tax Service tax 4,00,000 2,75,000 6,75,000 4,00,000 2,75,000 Education cess 8,000 8,000 8,000 Secondary and higher education cess 4,000 4,000 4,000 -

Solution 5: 1. Computation of service tax payable by Manorama Banquets Private Limited April 2011 Let out Banquets for conducting Music Programme is a taxable service (Note 3) Let out Banquets for marriage is a taxable service [1,500 x 1,000 x 60%] (Note 4) Value of taxable service Service Tax @ 10.3% Last date of payment is 6th May, 2011.

` 10,00,000 9,00,000 19,00,000 1,95,700

Service Tax May 2011 June 2011 Let out Banquets for marriage is a taxable service (Note 5) Service Tax @10.3% Last date of payment is 6th July, 2011.

244 Nil 5, 00,000 51,500

Notes: 1. Service provided to United Nations is exempt. 2. Service provided to Embassy is exempt from service tax. 3. Point of taxation is the date of completion of service as the invoice raised after 14 days from the date of completion of service. 4. Point of taxation is 30.04.2011 as the amount received before the issue of invoice. As per Notification No. 01/2006 dated 01.03.2006, service tax shall be payable on 60 percent of gross amount charged when catering service is provided along with Mandap keeper service. 5. Point of taxation is 06.06.2011 as the invoice issued within 14 days of completion of service so the point of taxation shall be the date of issue of invoice. 2. Last date for filing service tax return for the half year ended on 30 th Sep, 2011 is 25th October, 2011.Wef 01.10.2011 every assessee is required to file service tax return electronically as per Notification No.43/2011 dated 25.08.2011.Therefore the return shall be filed electronically. 3. Service tax return can be revised within 90 days from the submission of original return. Solution 6: As per rule 7 of Point of Taxation Rule, 2011, service tax shall be payable on actual receipt basis in case of Scientific and Technical Consultancy Services i.e. rule 3 of POT Rules, 2011 shall not be applicable but only when service provider is individual or partnership firm but in the given case the service provider is a company hence taxability shall be as per Rule 3. (i) Computation of Service Tax Liability July 2011 Services provided to ABC Ltd. Services provided to Government Department Total value of taxable services Service tax @ 10% Add: Education Cess @ 3% Service Tax Liability Less: Input Service Tax Credit (9,00,000 x 10.3%) Net Service Tax Payable Less: Service Tax paid Excess Service Tax paid August 2011 Services provided to individual Service Tax @ 10% Add: Education Cess @ 3% Service Tax Liability Less: Excess Service Tax paid in July 2011

32,00,000 26,00,000 58,00,000 5,80,000 17,400 5,97,400 92,700 5,04,700 7,29,600 2,24,900 45,00,000 4,50,000 13,500 4,63,500 2,24,900

Service Tax Net Service Tax Payable September 2011 Services provided to partnership firm Service Tax Liability 17,00,000 x 10.3/110.3 Less: Input Service Tax Credit (6,00,000 x 10.3%) Net Service Tax Payable

245 2,38,600 17,00,000 1,58,749 61,800 96,949

Note: Mere testing will not attract service tax. However, in case testing is an integral part of the consultancy, then such activity is part and parcel of the taxable service and no abatement of any kind admissible. Note: Services provided to Embassy is exempt from Service Tax. (ii) In case company is adopting decentralized billing system then self adjustment of Service Tax is limited to `2,00,000 only in a particular month. Hence, in this case Service Tax liability would be `2,63,500 (4,63,500-2,00,000) for the month of August 2011. Balance of `24,900 would be adjusted in Next Month and liability of Service Tax for the Next Month would be `72,049 (96,949-24,900) (iii) Notification No. 43/2011-ST, Every assessee shall submit the half yearly return electronically. Hence Krishna Private Ltd required to file Service Tax Return electronically. (iv) Penalty for late filing of Service Tax Return Rule7C (Service Tax Rules, 1994) Upto 15 days 500 Next 15 days 500 On Balance Days (i.e. for 37 days from 25.11.2011 to 31.12.2011@100 per day) Total Penalty Payable Service Tax return is to be filed in form ST-3. ` ` `3,700 `4,700

(v) According to Rule 5 (Service Tax Rules, 1994) All records should be preserved for at least 5 years immediately after the financial year to which such records pertain. As per section 77, any person who has failed to maintain the book of accounts shall pay penalty which may extent upto `10,000. (vi) Yes, Service Tax is liable to be paid when any scientific or technical consultancy service is rendered whether by public funded institutions or by private agencies. Solution 7: Computation of Service Tax liability as per Point of Taxation Rules 2011 for each month April 2011 Value of taxable services (11,00,000 + 9,00,000) Less: SSP Exemption Service Tax (10,00,000 x 10.3%) December 2011 Value of taxable services

20,00,000 10,00,000 1,03,000

10,00,000

Service Tax Service Tax (10,00,000 x 10.3%) January 2012 Value of taxable services Service Tax (15,00,000 x 10.3%)

246 1,03,000

15,00,000 1,54,500

Solution 8(a): Computation of Service Tax payable by Sandhya Services Ltd. April 2011

NIL

May2011 Value of Service provided to ABC Ltd. 5,00,000.00 Service Tax Payable 46,690.84 (5,00,000 x 10.3/110.3) Rounded off u/s 37D 46,691.00 Note: In case service tax is not charged separately in the bill then the amount charged shall be considered to be inclusive of service tax and calculations shall be done accordingly. June 2011 Value of Service provided to R Ltd. Service Tax @ 10% Add: Education Cess @3% Net Service Tax payable 25,00,000.00 2,50,000.00 7,500.00 2,57,500.00

Note: No CENVAT Credit shall be allowed for service tax paid on input services used for providing exempted services. Note: Since value of taxable service provided by Sandhya Services Ltd. is not exceeding `60 Lacs during F.Y. 2010-11, hence interest shall be charged for late payment of service tax u/s 75 @ 15% for the F.Y.2011.12. Solution 8(b): Interest u/s 75 2,57,500 x 15% x 25/366 = 2,638.32 Rounded Off u/s 37D `2,638.00 Penalty u/s 76 Higher of the following:(i) @ 100 per day for 25 days = `2,500 Or (ii) 2,57,500 x 1% x 25/31 = ` 2,076.61 = `2,077 However, it cannot exceed 50% of Service Tax Payable Hence the penalty would be `2,500. Solution 8(c): Penalty for late filing of Service Tax Return Rule7C (Service Tax Rules, 1994) 30 days On Balance Days

1,000

Service Tax (i.e. for 24 days from 25.11.2011 to 18.12.2011@ 100 per day) Total Penalty 3,400 Following are the Due dates of filing of service tax return by a company:Return period Due Date of filing 01st April to 30th September 25th October 01st October to 31st March 25th April

247 2,400 Payable

Solution 9(a): Rent Add: Service Tax @ 10.3% Total Less: TDS (31,00,000 x 10%) 3,10,000 Amount Payable Solution 9(b): Commission Add: Service Tax @ 10.3% Total Less: TDS (8,00,000 x 10%) Amount Payable Solution 9(c): Commission Add: Service Tax @ 10.3% Total Less: TDS (35,00,000 x 10%) 3,50,000 Amount Payable

` 31,00,000 3,19,300 34,19,300

31,09,300

8,00,000 82,400 8,82,400 80,000 8,02,400

35,00,000 3,60,500 38,60,500

35,10,500

Solution 9(d): Amount of TDS shall be `5,00,000 x 10% = `50,000 and service tax of `51,500 shall be paid directly by LIC to the Government under Reverse Charge under Rule 2(1)(d). Solution 9(e): Professional Charges Add: Service Tax @ 10.3% Total Less: TDS (27,57,500 x 10%) 2,75,750 Amount Payable Solution 9(f):

25,00,000 2,57,500 27,57,500

24,81,750

Service Tax Advertising Charges Add: Service Tax @ 10.3% Total Less: TDS (38,60,500 x 2%) Amount Payable Solution 10: Service tax payable by Bombay office Jan 2012 Amount of taxable service 45,00,000 Less: Tax credit 90,00,000 Balance Tax credit

248 35,00,000 3,60,500 38,60,500 77,210 37,83,290

Service Tax 4,50,000 9,00,000 4,50,000

EC 9,000 18,000 9,000

SHEC 4,500 9,000 4,500

Feb 2012 Amount of taxable service Service Tax EC SHEC 31,00,000 3,10,000 6,200 3,100 Less: Tax credit Brought forward 4,50,000 9,000 4,500 Balance Tax credit 1,40,000 2,800 1,400 Bombay office is registered as input service distributor and they have transferred the balance of cenvat credit to their Delhi branch Service tax payable by Delhi office March 2012 Amount of taxable service 6,00,000 12,00,000 Less: Tax credit received from input service distributor Tax Payable Solution 10(b): Service tax payable by Bombay office Jan 2012 Amount of taxable service 45,00,000 Less: Tax credit 90,00,000 Balance Tax credit Feb 2012 Amount of taxable service 31,00,000 Less: Tax credit Brought forward Balance Tax credit March 2012

Service Tax 60,000 1,20,000 1,40,000 40,000

EC 1,200 2,400 2,800 800

SHEC 600 1,200 1,400 400

Service Tax 4,50,000 9,00,000 4,50,000 Service Tax 3,10,000 4,50,000 1,40,000

EC 9,000 18,000 9,000 EC 6,200 9,000 2,800

SHEC 4,500 9,000 4,500 SHEC 3,100 4,500 1,400

Service Tax Amount of taxable service 6,00,000 12,00,000 Less: Tax credit Brought forward Tax Payable Service Tax 60,000 1,20,000 1,40,000 40,000 EC 1,200 2,400 2,800 800 SHEC 600 1,200 1,400 400

249

Solution 11: Computation of Service Tax Payable by Mr. Sachin Bahl April to June 2011 July to September 2011 Since amount charged is inclusive of service tax hence amount of service tax is:= 1,25,000 / 110.3 x 10.3 Rounded off under section 37D Interest payable under section 75 11,673 x 15% x 10/366 Rounded off under section 37D Penalty payment under section 76 1% per month of the amount of default for 10 days = 1% x 11,673 x 10/31 = `37.65 or Penalty calculated @ 100 per day for 10 days = `1,000 Penalty shall be higher of the two i.e. = `1,000

Nil

11,672.71 11,673.00 47.84 48.00

As the return for the half year ending on September 2011 was filed on 31stOct, 2011 There is a delay in filing by 6 days, as per Rule-7C if return is filed within 15 days the penalty shall be `500. October to December 2011 January to March 2012 Calculation of Service Tax Payable: = 4,00,000 / 110.3 x 10.3 = 1,00,000 / 110.3 x 10.3 = 80,000 / 110.3 x 10.3 = 2,10,000 / 110.3 x 10.3 Total Service Tax Payable Rounded off under section 37D Less: Input tax credit (40,000 x 10.3% / 2) Net service tax payable Nil

37,352.67 9,338.17 7,470.53 19,610.15 73,771.52 73,772.00 2,060.00 71,712.00

Note: Cenvat Credit of Excise duty paid on capital goods is allowed 50% in the current year and balance in the subsequent year. Note: Service of editing of Journal published by Taxman is not covered under Practicing Chartered Accountant service since it is not in professional capacity (i) According to Rule 5 (Service Tax Rules, 1994) all records should be preserved for at least 5 years immediately after the financial year to which such records pertain. As per section 77, any person who has failed to maintain the books of accounts shall pay penalty which may extend upto `10,000

Service Tax

250

(ii) Notification No. 43/2011-ST, Every assessee shall submit the half yearly return electronically. Hence Sachin Bahl required to file Service Tax Return electronically. Yes, Service Tax return can be revised within 90 days of filing of original return.

EXAMINATION QUESTIONS
IPCC NOV 2011
Question 1 (4 Marks) Vikas Coaching Centre engaged in commercial training and coaching service furnishes you the following information and the amount received by it for the half year ended on 30.09.2011. Particulars Coaching fee for Civil Service examination Postal coaching fees for university examination Sports coaching fee from a local college Fee for management diploma of a foreign university (not recognized by law in force in India) Coaching and training provided by sending staff to the residence of service receivers Determine the value of taxable services .Your answer must be with reasons. Solution Computation of value of taxable services Particulars Coaching fee for Civil Service examination Postal coaching fees for university examination Fee for management diploma of a foreign university (not recognized by law in force in India) Coaching and training provided by sending staff to the residence of service receivers Total Value Value of taxable services (16,70,000 / 110.3%) ` 3,50,000 2,40,000 1,10,000 4,40,000 6,40,000 (Modified) ` 3,50,000 2,40,000 4,40,000 6,40,000 16,70,000 15,140,53

Notes 1. Fee for management diploma of a foreign university not recognized by law in force in India is taxable under CTC. But if it was recognized by Indian law then it shall be exempt. 2. Postal coaching fees for university examination are taxable service. 3. Coaching and training provided by sending staff is taxable under commercial training and coaching service. If any individual is imparting home coaching in their independent capacity i.e. not on behalf of a Commercial Training or Coaching Centre, in that case it will be exempt from service tax. 4. Sports coaching fees received is not covered under commercial training and coaching service.

Service Tax Question 2 (i) When does e-payment of service tax become mandatory?

251 (4 Marks)

(ii) State the due date for e-payment of service tax by individuals and companies. Answer 2(i): The assessee who has paid service tax of `10,00,000/- or above including the amount paid by utilization of CENVAT credit in the preceding financial year has to compulsorily deposit the service tax liable to be paid by him electronically, through internet banking. Answer 2(ii): Rule 6(1) of Service Tax Rules, 1994 provides that service tax on the value of taxable services deemed to be provided shall be paid to the credit of the Central Government in the following manner:Assessee Individual, Proprietary concern or a partnership firm Duration of payment Quarterly Due date of payment (i) by the 6th day of the month, if the duty is deposited electronically through internet banking; and (ii) by the 5th day of the month, in any other case, immediately following the quarter in which the services are deemed to have been provided*. Any other Assessee Monthly (i) by the 6th day of the month, if the duty is deposited electronically through internet banking; and (ii) by the 5th day of the month, in any other case, immediately following the calendar month in which services are deemed to have been provided.* *Also, the service tax on the value of services deemed to have been provided during the month of March, or the quarter ending in March, as the case may be, shall be paid to the credit of the Central Government by the 31st day of March of the calendar year. Question 3 (4 Marks) ABC Private Ltd., is engaged in providing a taxable service. For the Financial Year 2011-12, it has issued bills and has received `18,00,000. The break-up is as given below: Month in which services are performed and bills are issued April, 2011 (includes `1,00,000 for the services rendered to a notified International Organization) May, 2011 (includes `1,25,000 for the services rendered within Indian Territorial Waters) January, 2012 (includes ` 1,75,000 for services rendered to its Associated Enterprises) February, 2012 (includes `1,50,000 for services rendered in the state of Jammu & Kashmir) The above amounts are exclusive of service tax. In the financial year 2010-11, ABC Private Ltd., had paid 2,06,000 as service tax @ 10.3%. State the amount of service tax payable for Financial Year 2011-12. Ignore SSP exemption. Answer: Receipt (`) 4,00,000 3,00,000 5,00,000 6,00,000

(Modified)

Service Tax Computation of service tax payable by ABC Private Ltd., for Financial Year 2011-12 Particulars April, 2011 May, 2011 January, 2012 February, 2012 Total value of taxable service Service tax @ 10.3%

252 (`) 3,00,000 3,00,000 5,00,000 4,50,000 15,50,000 1,59,650

Notes: 1. Services rendered to a notified International Organization is exempt. 2. Service rendered to an Associated Enterprise is a taxable service. 3. Service rendered in the state of Jammu & Kashmir is not covered under service tax and not taxable. Question 4 (4 Marks) Explain optional composition scheme under Service tax for distributor or selling agents of lotteries. Answer: Option to pay service tax in case of distributor of lottery tickets Rule 6(7C) As per section 65(105)(zzzzn) -Distribution of lottery ticket service means any service provided or to be provided to any person, by any other person, for promotion, marketing, organising or in any other manner assisting in organising games of chance, including lottery, Bingo or Lotto in whatever form or by whatever name called, whether or not conducted through internet or other electronic networks. The distributor or selling agent, liable to pay service tax for the taxable service of promotion, marketing, organising or in any other manner assisting in organising lottery, referred to in sub-clause ( zzzzn) of clause (105) of section 65 of the said Act, shall have the option to pay an amount at the rate mentioned below: Sl. No. (1) 1. Rate (2) ` 6,000 on every ` 10 lakh (or part of ` 10 lakh) of aggregate face value of lottery tickets printed by the organising State for a draw ` 9,000 on every ` 10 lakh (or part of ` 10 lakh) of aggregate face value of lottery tickets printed by the organising State for a draw Condition (3) If the lottery or lottery scheme is one where the guaranteed prize payout is more than 80% If the lottery or lottery scheme is one where the guaranteed prize payout is less than 80% :

2.

In case of online lottery, the aggregate face value of lottery tickets shall be taken as the aggregate value of tickets sold. The distributor or selling agent shall exercise such option within a period of one month of the beginning of each financial year and such option shall not be withdrawn during the remaining part of the financial year. Question 5 State the contents of Service tax return. (any eight points). Solution: Contents of the Return (4 Marks)

Service Tax 1. Half year for which return is being filed 2. Name of the assessee 3. STC Number 4. Address 5. Constitution of the assessee like individual or partnership firm etc. 6. Assessee liable to pay service tax as service provider or service receiver under reverse charge 7. Gross amount received 8. Amount of service tax and education cess 9. Tax credit allowed 10. Net tax payable 11. Details of services rendered 12. Verification of the return 13. Signature of service provider or service recipient under reverse charge

253

Question 6 (4 Marks) Write short notes on Service Tax Code Number and the objective sought to be achieved thereunder. Answer: A service provider can charge service tax only if he is registered with the Service Tax Department. As per Notification No. 26/2005 dated 07.06.2005, every service provider in whose case, gross receipt has exceeded `9,00,000 shall apply for registration to the Service Tax Department in Form No ST-1 within 30 days from the date of crossing the limit of `9,00,000, however, he will charge service tax after crossing the limit of `10,00,000. He will also submit a copy of Permanent Account Number and proof of residence. Department shall grant him a registration certificate in Form No ST-2 within 7 days from the receipt of application otherwise the service provider shall be deemed to be registered. The service provider shall be given a Registration Number by the Department which will be called STP code i.e. Service Tax Payer Code and it will be 15 digit PAN based number and first 10 digit shall be that of PAN and remaining 5 digit shall be allotted by Service Tax Department e.g. AAEPC1298D ST-001 The last three digit shall indicate total number of registration for the same permanent Account Number. The purpose of issuing PAN based registration number is to link income tax number and service tax number so that the department may do cross verification.

PCC NOV 2011


Question 2 (4 Marks) Chandra Limited was started on 01.04.2011 to provide taxable services. Details of bills issued and payment received is as given below: No service tax has been charged separately. S. No. 1 2 3 4 5 Particulars Amount received in respect of service rendered in July and August 2011 Services provided to the branch of World Health Organisation a unit of UN Services rendered to its Auditors against audit fee payable Advance received for services to be rendered in May,2012 Services provided to office staff and their relatives free of cost Amount (`)s 10,30,000 1,00,000 5,15,000 6,18,000 20,000

Compute the taxable services and service tax payable for Financial Year 2011-12 by Chandra Limited.

Service Tax Answer: Computation of Service Tax Payable for Financial Year 2011-12 S. No. Particulars 1 Amount received in respect of service rendered in July and August 2011 2 Services provided to the branch of World Health Organisation a unit of UN 3 Services rendered to its Auditors against audit fee payable 4 Advance received for services to be rendered in May,2012 5 Services provided to office staff and their relatives free of cost Value of Taxable Service Less: SSP exemption Balance value of taxable services Service Tax Payable 11,63,000 x 10.3 / 110.3 Rounded off u/s 37D Notes: 1. Services provided to the Branch of World Health Organisation a unit of UN is exempt. 2. Advance received is taxable. 3. Free services are not taxable.

254 (Modified) Amount (`)s 10,30,000 Exempt 5,15,000 6,18,000 Exempt 21,63,000.00 10,00,000.00 11,63,000.00 1,08,602.90 1,08,603.00

Question 3 (4 Marks) (i) Whether service tax is payable by independent directors who are part of the Board of Directors under Management Consultancy service. (ii) S. Ltd. paid service tax of `8 lacs during the preceding financial year and utilized CENVAT credit of `3 lacs. Whether he is required to deposit service tax electronically for the financial year 2011-12. Answer 3(i): Service tax is not payable by Independent Directors. The payments made by Companies to Directors cannot be termed as payments for providing Management Consultancy service and is, accordingly, not chargeable to service tax under the category Management Consultancy service. However, in case such directors provide any advice or consultancy to the company, for which they are being compensated separately, such service would become chargeable to service tax. Answer 3(ii): The assessee who has paid service tax of `10,00,000/- or above including the amount paid by utilization of CENVAT credit in the preceding financial year has to compulsorily deposit the service tax liable to be paid by him electronically, through internet banking. Since S Ltd. paid service tax of `8,00,000 in cash and `3,00,000 through cenvat credit, S Ltd. has to pay service tax electronically. Question 4 Briefly explain the following in reference to service tax: (i) Applicability of service tax liability in case of Job work (ii) Exemption available to small scale service providers. (iii) Liability of service tax in case of sponsorship services (4 Marks)

Service Tax

255

(iv) Liability of service tax in case of export of services. Answer 4(i): Job work It is subject to service tax. If, however, job work done amounts to manufacture of excisable goods, it is not taxable under service tax. Solution 4(ii): As per Notification No.6/2005, dated 01.03.2005, exemption is allowed for receipt upto `10,00,000 provided in the preceding year, value of taxable services rendered is not exceeding `10,00,000. While computing amount of `10,00,000, value of services which are exempt from service tax shall not be considered. If any person is rendering services from more than one premises, value of services rendered from all the premises shall be taken into consideration. If any person is rendering more than one services, value of all the services shall be taken into consideration. Solution 4(iii): As per section 65(99a) - Sponsorship includes naming an event after the sponsor, displaying the sponsors company logo or trading name, giving the sponsor exclusive or priority booking rights, sponsoring prizes or trophies for competition; but does not include any financial or other support in the form of donations or gifts, given by the donors subject to the condition that the service provider is under no obligation to provide anything in return to such donors. In relation to sponsorship service provided to any body corporate or firm located in India, the person liable to pay service tax is the body corporate or firm, as the case may be who receives such sponsorship service. If the person sponsoring the event is out of India, in that case, reverse charge shall not be applicable rather the person organizing the event i.e. the service provider shall be liable to pay service tax. Answer 4(iv): Section 93A/ Export of services, Rules 2005 If any person has exported services from India, no service tax is payable however the payment should be received in India in foreign currency within the time allowed by Reserve Bank of India otherwise exemption is not allowed. If any person has taken input services for the purpose of export of services, service tax paid on such input services shall also be refunded. Question 6 State the liability of service tax in respect of the following services: (4 Marks)

(i) Services provided for personal use or use by the family members of a foreign diplomatic agent in a foreign diplomatic mission. (ii) Services provided for issuing fitness certificate of vehicles by Regional transport office. (iii) Services received from outside India and consumed in India. (iv) Service rendered to a friend free of cost. Answer 6(i):

Service Tax

256

Exemption to services provided for personal use or for use of Family Members of Diplomatic Agents or Career Consular Officers posted in Foreign Diplomatic Mission / Consular Post in India as per Notification No. 34/2007 dated 23.05.2007 Answer 6(ii): Service tax shall not be leviable on fee collected by public authorities while performing statutory functions/duties. Answer 6(iii): Where any taxable service have been provided by any person from outside India to any person in India in such cases the person who has taken services from outside India shall be liable to pay service tax as if he is the service provider and he should apply for registration even if the gross amount is less than `9 lakh and also he should pay service tax even if the gross amount is not exceeding `10 lakhs and it is called Reverse Charge Mechanism. Answer 6(iv): No, service tax is not payable on free service. Service tax is payable only when there is consideration. No service tax is payable when value of service is 'zero' as the charging section-section 66 provides that service tax is chargeable on the value of taxable service. Question 7 (4 Marks) State the Service Tax provisions regarding adjustment of Service tax paid when service was not provided either wholly or partly. Answer: Where an assessee has issued an invoice, or received any payment, against a service to be provided which is not so provided by him either wholly or partially for any reason or where the amount of invoice is renegotiated due to deficient provision of service, or any terms contained in a contract, the assessee may take the credit of such excess service tax paid by him, if the assessee, (a) has refunded the payment or part thereof, so received for the service provided to the person from whom it was received; or (b) has issued a credit note for the value of the service not so provided to the person to whom such an invoice had been issued. OR Briefly explain the important points to be kept in mind while paying Service Tax. Answer: Payment of Service Tax Section 68/ Rule 6 of STR, 1994 An individual or proprietary firm or partnership firm shall be required to pay service tax on quarterly basis and should be paid upto 5th of the month subsequent to the quarter in which the service is deemed to be provided (i.e. the quarter in which point of taxation falls) but if the payment is being made electronically through internet banking, payment can be made upto 6 th of such month. However, payment for the last quarter should be made upto 31st March of that year. Any other service provider like Company or HUF etc. shall be required to pay service tax on monthly basis and it should be paid upto 5th of the month succeeding the month in which service is deemed to be provided but if payment is being made electronically through internet banking, payment can be made upto 6 th of the month however, payment for last month should be made upto 31st March of that year

Service Tax

257

If an assessee has paid a total service tax of rupees ten lakh or more including the amount paid by utilization of Cenvat Credit, in the preceding financial year, he shall deposit the service tax liable to be paid by him electronically, through internet banking. As per section 77, if any person has failed to pay service tax electronically, he shall pay penalty which may extend upto `10,000. The assessee shall deposit the service tax liable to be paid by him in Form G.A.R-7 with the bank designated by the Central Board of Excise and Customs for this purpose. If the assessee deposits the service tax by cheque, the date of presentation of cheque to the bank designated by the Central Board of Excise and Customs for this purpose shall be deemed to be the date on which service tax has been paid subject to realization of that cheque.

IPCC MAY 2011


Question 2 (4 Marks) During the year ended 31.03.2011, Kohli & Co., running a coaching centre, has collected a sum of `10.2 lacs as service tax. `70,000 was met through Cenvat credit and the balance was paid by cheque on various dates. The details pertaining to the quarter ended 30.06.2011 are as under: Particulars Value of free coaching rendered Coaching fees collected from students (Service tax collected separately) Advance received from a college for coaching their students, on 30.06.2011. However, no coaching was conducted and the money was returned on 12.04.2012 Amount (`) 20,000 14,50,000 3,00,000

Determine the service tax liability for the quarter and indicate the date by which the service tax has to be remitted by the assessee. (Modified) Answer. Computation of service tax liability of Kohli & Co. for the quarter ended 30.06.2011:Particulars Free coaching rendered Coaching fees collected from students
14,50,000 10.3 100

Amount of Service Tax ` Nil

1,49,350 Advance received from a college for coaching their students Total service tax liability for the quarter ended 30.06.2011 Notes: 1. Free coaching is not exigible (liable) to service tax 2. Coaching fees collected from students will be liable to service tax @ 10.3%.

28,015 1,77,365

Service Tax

258

3. Advance receipt is chargeable to service tax. It is immaterial that no coaching was conducted and the money was returned on 12.4.2012. Advance would be chargeable to service tax in the quarter ended 30.06.2011. Advance has been assumed to be inclusive of service tax
3,00,000 10.3 110.3

Rule 6(3) Where an assessee has issued an invoice, or received any payment, against a service to be provided which is not so provided by him either wholly or partially for any reason or where the amount of invoice is renegotiated due to deficient provision of service, or any terms contained in a contract, the assessee may take the credit of such excess service tax paid by him, if the assessee, (a) has refunded the payment or part thereof, so received for the service provided to the person from whom it was received; or (b) has issued a credit note for the value of the service not so provided to the person to whom such an invoice had been issued. 4. During the preceding financial year, the service tax liability met by the assessee, inclusive of CENVAT credit availed was more than ` 10 lakh. Hence, during the current financial year, for all quarters, payment of service tax will have to be made electronically. Therefore, the last date for making the payment of service tax by Kohli & Co. (non-corporate assessee) for the quarter ended 30.06.2011 is 6th July, 2011. Question 3 (2 Marks Each) (i) Where any transaction of taxable service is entered into with an associated enterprises, receipt of service tax is not material for levy of service tax. Explain with reasons, whether you agree or disagree with this statement. (ii) Briefly discuss about the adjustment of excess amount of service tax paid in case of renting of immovable property service, owing to property tax payment. Answer 3(i): The term associated enterprises is not defined under service tax provision rather it is defined under section 92A of Income Tax Act and it means an enterprise (a) which participates, directly or indirectly, or through one or more intermediaries, in the management or control or capital of the other enterprise; or (b) in respect of which one or more persons who participate, directly or indirectly, or through one or more intermediaries, in its management or control or capital, are the same persons who participate, directly or indirectly, or through one or more intermediaries, in the management or control or capital of the other enterprise. Point of Taxation As per Rule 7 of Point of Taxation Rule, in case of associated enterprises, where the person providing the service is located outside India, the point of taxation shall be the date of credit in the books of account of the person receiving the service or date of making the payment whichever is earlier. Answer 3(ii): In case of renting of immovable property service, a deduction of property taxes paid in respect of the immovable property is allowed from the gross amount charged for renting of the said immovable property vide Notification No.24/2007 ST dated 22.05.2007. However, where any amount in excess of the amount required to be paid towards service tax liability has been paid on account of non-availment of such deduction, such excess amount may be adjusted against the service tax liability within one year from the date of payment of such property tax.

Service Tax

259

The details of such adjustment have to be intimated to the Superintendent of Central Excise having jurisdiction over the service provider within a period of 15 days from the date of such adjustment. Question 4 (4 Marks) State the provisions which enable the Central Government to make rules for administering service tax. For what purposes are such rules made? Name any four such rules issued by the Central Government so far. Answer. Section 94 of Chapter V and section 96-I of Chapter VA of the Finance Act, 1994 grant powers to Central Government to make rules for carrying out the provisions of these Chapters. Rules should be read with the statutory provisions contained in the Act. Rules can never override (dominate) the Act and cannot be in conflict with the same. So far, the Central Government has issued the following rules for administering service tax (a) Service Tax Rules, 1994 (b) Point of Taxation Rules, 2011 (c) CENVAT Credit Rules, 2004 (d) Export of Service Rules, 2005 (e) Service Tax (Registration of Special Category of Persons) Rules, 2005 (f) Service Tax (Determination of Value) Rules, 2006 (g) Taxation of Services (Provided from Outside India and Received in India) Rules, 2006 Note: Any four rules may be mentioned. Question 5 (4 Marks) Briefly explain the provisions relating to advance payment of service tax. Answer: Service tax law provides the assessee a facility to make advance payment of service tax on his own volition (voluntary) and adjust the amount so paid against the service tax which he is liable to pay for the subsequent period. Such facility is available when the assessee: (i) intimates the details of the amount of service tax paid in advance, to the Jurisdictional Superintendent of Central Excise within a period of 15 days from the date of such payment, and (ii) indicates the details of the advance payment made, and its adjustment, if any in the subsequent return to be filed under section 70. Question 7 (4 Marks) State the due dates for filing of service tax returns. Will the delayed filing of service tax return result in payment of any late fee? If so, how much? Answer: The service tax return (in Form ST-3) should be filed on half yearly basis by the 25 th of the month following the particular half-year. The due dates on this basis are as under: Half year 1 April to 30th September 1st October to 31st March
st

Due date 25th October 25th April

In case the due date of filing of return falls on a public holiday, the assessee can file the return on the immediately succeeding working day. Yes, late fee will be levied for delay in furnishing of the service tax return. The prescribed late fee is given hereunder: S. No. Period of delay Late fee

Service Tax (a) (b) (c) Particulars 15 days from the date prescribed for submission of the return Beyond 15 days but not later than 30 days from the date prescribed for submission of the return. Beyond 30 days from the date prescribed for submission of the return ` 500 1,000

260

An amount of ` 1,000 plus ` 100 for every day from the 31st day till the date of furnishing the said return However, the total late fee for delayed submission should not exceed ` 20,000.

PCC MAY 2011


Question 2 (4 Marks) Ahmed & Co. of Srinagar rendered taxable services both within and outside the State of Jammu & Kashmir. It received `26,12,000 for the services rendered inside the State of Jammu & Kashmir and `18,00,000 for the services rendered outside the State of Jammu & Kashmir. Compute its taxable service value and service tax liability. In case, Ahmed & Co. was situated in Mumbai what would be value of its taxable service and service tax liability? Answer: Exigibility to service tax for services rendered in Jammu & Kashmir and other places:The services rendered inside the State of Jammu and Kashmir are not liable to service tax and hence, `26,12,000 is not chargeable to service tax. The services rendered outside the State of Jammu and Kashmir are liable to service tax. Hence, amount of service tax shall be The service tax liability would be `18,00,000 10.30 = ` 1,68,087 110.30 . In case Ahmed & Co. was in Mumbai, the services rendered in the State of Jammu and Kashmir would not be liable to service tax levy and the services rendered at any place other than Jammu and Kashmir would be liable to service tax. In effect, the value of taxable service and service tax liability would be the same. There wont be any change in value of taxable service and service tax liability Question 3 (4 Marks) State with reasons whether services rendered to associated enterprises are liable for service tax: Answer: Services rendered to associated enterprise are liable to service tax. No special treatment from exigibility to service tax or exemption has been provided for services rendered to an associated enterprise. Question 6 (4 Marks) How can an assessee adjust the excess payment of service tax against his liability of service tax for subsequent periods? What is the basic condition for it? Answer: If the assessee has paid to the credit of Central Government any amount in excess of the payment required to be paid towards service tax liability for a month or quarter, as the case may be, he can adjust such excess amount paid by him against his service tax liability for the succeeding month or quarter, as the case may be, subject to following conditions:-

Service Tax

261

(i) Self-adjustment of excess credit would not be allowed in case of reasons involving interpretation of law, taxability, classification, valuation or applicability of any exemption notification. (ii) Excess amount paid and proposed to be adjusted should not exceed `2 lakh for the relevant month or quarter. If the assessee has opted for centralized registration, excess amount can be adjusted without monetary limit, provided the excess payment is on account of delayed receipt of details of payments from branch offices. (iii) Adjustment can be made only in the succeeding month or quarter. (iv) The details of self-adjustment should be intimated to the Superintendent of Central Excise within a period of 15 days from the date of such adjustment. Question 3 State with reasons whether the following are liable for service tax: (i) (ii) Services rendered to Reserve Bank of India Services rendered by a sub-contractor (4 Marks)

(iii) Services provided to developer of Special Economic Zone Answer: (i) Following taxable services are exempted from the whole of the service tax leviable thereon: (a) taxable services provided to any person, by the Reserve Bank of India; (b) taxable services provided by any person, to the Reserve Bank of India when the service tax for such is liable to be paid by the Reserve Bank of India i.e. if reverse charge is not applicable, service tax shall be charged by the service provider from Reserve Bank and shall be paid to the Government. (c) taxable services received in India from outside India by the Reserve Bank of India. (ii) Services rendered by sub-contractor are liable to service tax. A sub-contractor is essentially a taxable service provider. The fact that services provided by such subcontractors are used by the main service provider for completion of his work does not in any way alter the fact of provision of taxable service by the sub-contractor. (iii) Taxable service provided to a developer of special economic zone or a unit (including a unit under construction) of special economic zone by any service provider, for consumption of the services within such special economic zone, are exempt from the whole of service tax. If the services are consumed outside SEZ, the service provider should charge service tax however, the recipient can claim refund.

IPCC NOV 2010


Question 1 (5 Marks) Smart & Express Co., is providing taxable information technology software services and is liable to pay service tax in financial year 2010-11. The firm furnishes the following information relating to the services rendered, bills raised, amount received pertaining to this service, for the financial year ended on 31 st March, 2012 as under: (i) (ii) (iii) Amount received being 10% of the assignment fees on 31st March, 2012 for upgradation and enhancement of software services to be rendered during the financial year 2012-13. Services provided to UNICEF, an International Organisation in Gandhinagar, for analysis, design and programming of latest information technology software. Services billed to clients on 07.07.2011 and invoice was issued on 31.07.2011 and

` 6,00,000 5,00,000

3,00,00,000

Service Tax amount received is `2 crore upto 31.03.2012 All amounts are inclusive of service tax.

262

Compute the value of total taxable services and service tax payable thereon for the year ended 31.03.2012, assigning reason in brief to the treatment of all items. (Modified) Answer. Computation of value of taxable services and service tax payable by Smart & Express Co. ` (i) Amount received in advance for upgradation and enhancement of software services is also liable though the services were rendered during the financial 6,00,000 year 2012-13 (ii) Services rendered to UNICEF not liable to service tax Nil (iii) Entire amount of `3 crore shall be taken into consideration because as per 3,00,00,000 Point of Taxation Rules, point of taxation in this case shall be 07.07.2011 Total 3,06,00,000 Amount of Service Tax (3,06,00,000 x 10.3) / 110.3 28,57,480

Question 2 (4 Marks) How can an assessee adjust the excess payment of service tax against his liability of service tax for subsequent periods? What is the basic condition for it? Answer: Refer to Answer given in PCC MAY 2011 Question No.6 Question 3 (4 Marks) Write a note in brief on provisional payment of service tax. Answer: Provisional payment of Service Tax Rule 6 of STR, 1994 In case the assessee is unable to correctly estimate, at the time of the deposit, the actual amount of service tax for any month or quarter, he may make a written request to Assistant/ Deputy Commissioner of Central Excise for making payment of service tax on provisional basis. The concerned officer may allow payment of service tax on provisional basis on such value of taxable service as may be specified by him. For the purpose of provisional assessment at the time of filing the return, the assessee is required to file a statement in form ST - 3A giving detail of difference between service tax deposited and the service tax liable to be paid for each month. The Assistant/Deputy Commissioner of Central Excise, on the basis of memorandum in form ST - 3A may complete the assessment after calling for necessary documents or records, if need be. Question 4 (4 Marks) How will a taxable service be valued when the consideration thereof is not wholly or partly in terms of money? Answer: As per section 67(2) of the Finance Act, 1994, if the consideration for a taxable service is not wholly or partly in terms of money, then the value of such service shall be such amount in money, with the addition of service tax charged, is equivalent to the consideration. In other words, where the service rendered is for a consideration not wholly or partly consisting of money the value of the taxable service is equivalent to the total value of the consideration. However, the total of such money and non-money values of the consideration has to be treated as inclusive of the service tax payable thereon. Question 5 (4 Marks) What do you mean by e-filing of returns? Is there any facility of e-filing of service tax returns? If yes, then which of the services are eligible for this facility?

Service Tax

263

Answer: With effect from 01.10.2011, e-filing of service tax returns has been made mandatory for all the assesses (Notification No. 43/2011 dated: 25.08.2011). The assessee can e-file the return through software ACES i.e. AUTOMATION OF CENTRAL EXCISE AND SERVICE TAX. The assessee should login the relevant site www.aces.gov.in. To transact business on ACES a user has to first register himself with ACES through a process called Registration with ACES. The assessee should have STP code i.e. PAN based 15 digit registration number allotted by Service Tax Department. The assessee should fill in self-chosen user-ID and his e-mail ID and system will check the availability of the chosen user ID and then generate a password and will send it to the assessee on the e-mail ID mentioned by him and the assessee can submit the return in the prescribed form. Question 6 (2 Marks) State with reasons in brief whether the following statements are correct or incorrect with reference to the provisions of Service Tax. Service tax provisions are not applicable in Jammu and Kashmir because State Government concurrence was not obtained in respect of Finance Act, 1944. Answer: Correct Reason: As per Article 370 of the Constitution, any Act of Parliament applies to Jammu and Kashmir only with the concurrence (permission) of the State Government. Since, no such concurrence has been obtained in respect of Finance Act, 1994, service tax provisions are not applicable in Jammu and Kashmir. Note The words Finance Act, 1944 mentioned in the question should be read as Finance Act, 1994.

PCC NOV 2010


Question 2 Prasad & Co. seeks your advise for the following in the context of service tax: (i) It wants to file revised service tax return even though the original return was filed belatedly. (ii) It will pay service tax only on actual receipt of money from the customers though it maintained its books of account on mercantile basis. Your answer must be with reasons. Answer 2(i): Revised service tax returns may be filed within 90 days from the date of filing original return. Even if the original return is filed belatedly, the return could be revised by filing a revised return. Answer 2(ii): After introduction of Point of Taxation Rules, 2011, service tax shall be payable on the basis of point of taxation and not on the basis of actual receipt. Service tax should be paid in the month succeeding the month or quarter in which services were deemed to have been provided. However in the following cases, service tax shall be paid on actual receipt basis in case of individual or partnership firm. (i) (ii) (iii) (iv) (v) (vi) Practising Chartered Accountant Practising Cost Accountant Practising Company Secretary Architect Legal Consultancy Services Interior Decorator (4 Marks)

Service Tax (vii) Consulting Engineer (viii) Scientist or a Technocrat Similarly, in case of Reverse Charge, service tax shall be payable on actual receipt basis.

264

Question 5 (4 Marks) State how the service tax paid would be adjusted when the service is not provided either wholly or partially? Answer: Refer to Answer given in PCC NOV 2011 Question No.7

IPCC MAY 2010


Question 5 (4 x 2 = 8 Marks) (a) Is Service tax payable in respect of services provided in the Indian Territorial Waters? (b) Is Service tax leviable on fee collected by Public authorities while performing statutory functions under the provisions of law? (c) Can an assessee file a revised Service tax return? (d) Explain the term Commercial training or Coaching centre. Answer 5(a): Yes, services provided within Indian territorial waters are liable to service tax, as the levy of service tax extends to the whole of India except Jammu and Kashmir and India includes Indian territorial waters. Indian territorial waters extend upto 12 nautical miles from the Indian land mass. Services provided in exclusive Economic Zone shall also be taxable. The provisions are applicable even in the Exclusive Economic Zone of India i.e. the provisions are applicable upto 200 nautical miles from the baseline (Notification no. 1/2002 dated 01.03.2002) Answer 5(b): Circular No. 96/7/2007 ST dated 23.08.2007 has clarified that service tax shall not be leviable on fee collected by public authorities while performing statutory functions under the provisions of law. However, if the service is not in the course of statutory function but is undertaken for a consideration, service tax may be leviable if it is a taxable service. Answer 5(c): An assessee can submit a revised return, in Form ST-3 in triplicate, to correct a mistake or omission in the original return, within a period of 90 days from the date of submission of the original return. Answer 5(d): Commercial training or coaching centre means any institute or establishment providing commercial training or coaching for imparting skill or knowledge or lessons on any subject or field other than the sports, with or without issuance of a certificate and includes coaching or tutorial classes. [Section 65(27)]. Question 6 (3 Marks Each) (i) Does a service provider have an option to pay Service tax at a rate different from the general rate applicable on gross value of taxable services, in the case of purchase and sale of foreign currency? (ii) What is the late fee payable for delay in furnishing the Service tax return? Can the same be waived? Answer 6(i): The person liable to pay service tax in relation to purchase or sale of foreign currency, including money changing, provided by a foreign exchange broker, including an authorised dealer in foreign exchange or an

Service Tax

265

authorized money changer, referred to in sub-clauses (zm) and (zzk) of clause (105) of section 65 of the Act, shall have the option to pay an amount calculated at the following rate towards discharge of his service tax liability instead of paying service tax at the rate specified in section 66 of Chapter V of the Act, namely : (a) 0.1% of the gross amount of currency exchanged upto ` 100,000, but minimum ` 25; (b) on next 9,00,000 @ 0.05% (c) on balance @ 0.01% but maximum `5,000. Provided that the person providing the service shall exercise such option for a financial year and such option shall not be withdrawn during the remaining part of that financial year. Answer 6(ii): The late fee payable for delay in submitting the service tax return is furnished below: S. No. (a) (b) (c) Period of delay (No. of days from the due date of filing the return) 15 days 16 30 days Beyond 30 days Late fee (`) 500 1000 `1000 plus `100 for every day of delay beyond 30 days.

However, the total late fee payable shall not exceed `20,000. Provided also that where the gross amount of service tax payable is nil, the Central Excise Officer may, on being satisfied that there is sufficient reason for not filing the return, reduce or waive the penalty.

PCC MAY 2010


Question 6 (i) Is service tax payable on free service? (4 x 2 = 8 Marks)

(ii) State the due dates for payment of service tax in the case of an individual rendering taxable service. (iii) A company located in the State of Jammu & Kashmir rendered service in Delhi. Is the service provided by the company liable for service tax? (iv) Mr. Raj rendered taxable service in February, 2012. The amount was however realized on 18.04.2012. What is the due date for payment of service tax? Answer 6(i): Refer to Answer given in PCC NOV 2011 Question No.6 Answer 6(ii): Refer to Answer given in IPCC NOV 2011 Question No.2 Answer 6(iii): Yes, when the company located in the State of Jammu & Kashmir had rendered services outside the State, the service tax would be attracted as the location where service is provided is relevant. Answer 6(iv): After introduction of Point of Taxation Rules, 2011, service tax has to paid on the basis of point of taxation and in the given case, point of taxation falls in Feb 2012 hence Mr. Raj should pay service tax upto 31.03.2012.

IPCC NOV 2009

Service Tax

266

Question 5 (2 x 3 = 6 Marks) (a) Should service tax be paid even, if it is not collected from the client or service receiver? (b) Mr. Raju is a multiple service provider and files only a single return. State with reasons whether he can do so? (c) Explain the term Vocational Training Institute under the provisions of service tax. Answer 5(a): Section 68 of the Finance Act, 1994 casts the liability to pay service tax upon the service provider or upon the person liable to pay service tax as per Rule 2(1)(d) of the Service Tax Rules, 1994. This liability is not contingent upon the service provider realizing or charging the service tax at the prevailing rate. If any service provider has not charged service tax, amount received by him shall be considered to be inclusive of service tax e.g. a service provider has charged `22,06,000 and has not charged service tax 10.3%, in this case, service tax payable by him shall be 22,06,000 / 110.3% x 10.3% = 2,06,000 Answer 5(b): Yes, Mr. Raju can file a single return though he is a multiple service provider. He has to furnish the details in each of the columns of the Form No.ST-3 separately for each of the taxable services rendered by him. Thus, instead of showing a lump sum figure for all the services together, service-wise details should be provided in the return. Answer 5(c): Letter No. 334/1/2010 dated 26.02.2010, defines a vocational training institute means an Industrial Training Institute or an Industrial Training Centre affiliated to the National Council for Vocational Training, offering courses in designated trades as notified under the Apprentices Act, 1961. Question 6 (3 x 3 = 9 Marks) (i) Whether export service provided by service provider is excluded for the purpose of payment of service tax? (ii) List the documents to be submitted alongwith the first service tax return. (iii) What is the due date for payment in case of e-payment of service tax ? Answer 6(i): Yes. Export of services is exempt from payment of service tax if services are exported in accordance with the Export of Services Rules, 2005. However, service provided to an exporter is not excluded for the purpose of payment of service tax. The person liable to pay service tax under sub-section (1) or sub-section (2) of section 68 shall pay service tax as applicable on the specified services provided to the exporter and used for export of such goods, and such person shall not be eligible to claim exemption for the specified services. Answer 6(ii): Every assessee shall furnish to the Superintendent of Central Excise at the time of filing the return for the first time a list of following documents in duplicate: (a) all the records prepared or maintained by the assessee for accounting of transactions in regard to (i) providing of any service, whether taxable or exempted; (ii) receipt or procurement of input services and payment for them; (iii) receipt, purchase, manufacture, storage, sale or delivery, as the case may be, in regard to inputs and capital goods; (iv) other activities such as manufacture and sale of goods, if any. (b) all other financial records maintained by him in the normal course of business. Answer 6(iii):

Service Tax Refer to Answer given in IPCC NOV 2011 Question No.2

267

PCC NOV 2009


Question 6 (5 Marks) Ms. Priyanka, a proprietress of Royal Security Agency received `1,00,000 by an account payee cheque, as advance while signing a contract from providing taxable service; she received `5,00,000 by credit card while providing the service and another `5,00,000 by a pay order after completion of service on January 31, 2012. All three transactions took place during financial year 2011-12. She seek your advice about her liability towards value of taxable services and the service tax payable by her. (Modified) Answer: Computation of taxable service of Ms. Priyanka for financial year 2011-12 Particulars ` Advance received by an account payee cheque 1,00,000 Amount received while providing service through credit card 5,00,000 Amount received on completion of service by a pay order 5,00,000 Value of taxable service 11,00,000 Calculation of Service Tax Liability Particulars ` Service tax @10% on `11,00,000 1,10,000 Add: (i) Education cess @ 2% on service tax 2,200 Add: (ii) Higher education cess @ 1% on service tax 1,100 Total service tax payable 1,13,300 Notes: (i) Value of taxable services provided is assumed to be exclusive of service tax (ii) Money includes any cheque, pay order, currency, promissory note, letter of credit, draft, travellers cheque, money order, postal remittance and other similar instruments but does not include currency that is held for its numismatic value. (iii) Gross amount charged includes payment by credit card, cheque, deduction from account and any form of payment by issue of credit notes/debit notes and book adjustment. Question 8 (4 x 3 = 12 Marks) (a) Ms. Amrapali, a registered Service Provider did not render any services during the financial year 201112. Whether she is required to file service tax return? (b) Mr. Bharat is a registered Service Provider. He transfers his business to Mr. Rakesh on 31 st July, 2011. Explain the requirement to be complied with by Mr. Bharat and Mr. Rakesh on such transfer under the provisions of Service tax. (c) Write the provisions on liability for payment of Service tax on services provided abroad. (d) Which Act and Rule governs the levy of Service tax in India? Answer 8(a): Every assessee shall file a half yearly return in Form ST-3. Even if there is no service provided during a half year, a Nil return has to be filed. Therefore, Ms. Amrapali is required to file a service tax return. Answer 8(b): When a registered assessee transfers his business to another person, the transferee (Mr. Rakesh) should obtain a fresh certificate of registration within 30 days.

Service Tax

268

When a registered assessee ceases to carry on the taxable service for which he is registered, he should surrender his registration certificate to the Central Excise authorities. Thus, Mr. Bharat should surrender his registration certificate. Answer 8(c): The services rendered abroad will not attract service tax as the provisions of service tax extend to whole of India except Jammu & Kashmir and, India includes territorial waters. Answer 8(d): Service tax provisions are governed by:(i) Finance Act, 1994 (ii) Service Tax Rules, 1994. (iii) Point of Taxation Rules, 2011 (iv) Service Tax (Advance Rulings) Rules, 2003 (v) CENVAT Credit Rules, 2004 (vi) Export of Service Rules, 2005 (vii) Service Tax (Registration of Special Category of Persons) Rules, 2005 (viii) Service Tax (Determination of Value) Rules, 2006 (ix) Taxation of Services (Provided from Outside India and Received in India) Rules, 2006.

PCC JUNE 2009


Question 6 (4 x 2 = 8 Marks) (a) Mr. X, a service provider who pays service tax regularly, was of the opinion that a particular service was not liable for service tax. He, therefore, did not charge service tax in his bill. He received the bill amount without service tax. How will service tax liability of Mr. X be determined in such case? (b) Whether service tax return can be furnished after the due date? (c) How can the excess payment of service tax be adjusted? (d) When does a small service provider require to register under the Finance Act, 1994, but not liable to collect and pay service tax? Answer 6(a): The liability of a service provider or person liable to pay service tax under rule 2(1)(d) of the Service Tax Rules, 1994, to pay service tax is not contingent upon the service provider realizing or charging the service tax at the prevailing rate. The statutory liability does not get extinguished if the service provider fails to charge or realize the service tax from the service receiver. In this case, the amount received from the service receiver will be taken to be inclusive of service tax. Accordingly, service tax payable by the service provider shall be ascertained by making back calculations in the following manner:Service tax payable = Amount received Service tax rate (100 + Service tax rate )

Answer 6(b): A delayed return can be furnished by paying the prescribed late fee. Section 70(1) of the Finance Act, 1994 as amended inter alia provides for filing of periodical return after the due date with the prescribed late fee of not more than ` 20,000/-. Answer 6(c): Refer to Answer given in PCC MAY 2011 Question No.6

Service Tax

269

Answer 6(d): A service provider can charge service tax only if he is registered with the Service Tax Department. As per Notification No. 26/2005 dated 07.06.2005, every service provider in whose case, gross receipt has exceeded `9,00,000 shall apply for registration to the Service Tax Department in Form No ST-1 within 30 days from the date of crossing the limit of `9,00,000, however, he will charge service tax after crossing the limit of `10,00,000. He will also submit a copy of Permanent Account Number and proof of residence. Department shall grant him a registration certificate in Form No ST-2 within 7 days from the receipt of application otherwise the service provider shall be deemed to be registered. Question 8 (3 Marks) (a) How is the value of taxable services is determined when the consideration against taxable services is received in other than monetary terms? (b) What are the sources of Service Tax Law? Answer 8(a): Section 67 of the Finance Act, 1994 as amended provides that if the consideration for a taxable service is not wholly or partly in terms of money, then the value of such service shall be such amount in money, with the addition of service tax charged, is equivalent to the consideration. In other words, where the service rendered is for a consideration not wholly or partly consisting of money, the value of the taxable service is equivalent to the total value of the consideration. However, the total of such money and non-money value of the consideration has to be treated as inclusive of the service tax payable thereon. For instance, Mr. A, a Chartered Accountant provided taxable professional service to one of his clients. For rendering services, Mr. A charges ` 20,000 from his client and also asks his client to give him a law book worth ` 2,472. The total consideration in this case will be ` 22,472 and the value of the taxable service shall be ` 20,373.52 (`22,472 100/100+10.3) and ` 2,098.48 shall be the service tax payable. Answer 8(b): There is no independent statute on service tax as yet. However, the sources of service tax law are:i. Finance Act, 1994 ii. Rules on service tax iii. Notifications on service tax iv. Circulars or Office Letters (Instructions) on service tax v. Trade notices on service tax

PCC NOV 2008


Question 6 (2 x 5 = 10 Marks) (a) Explain as to how and when the amendments made in Finance Bill, in respect service tax matters come into force? (b) Who is responsible to pay service-tax when the recipient of sponsorship services is located outside India? (c) Whether free services after sale of motor vehicles, given by the authorized dealers, for which they are reimbursed by the vehicle manufacturers, are subject to service tax? (d) Whether life insurer carrying on life insurance business has option to calculate service tax at different rate?

Service Tax

270

(e) Who is liable to make E-payment of service tax? Answer 6(a): Amendments made by the Finance Bill, in respect of service tax matters, come into force from the date of enactment (the process of a law becoming official) of the Finance Bill i.e., the date on which the Finance Bill receives the assent of the President of India. However, wherever it is specifically provided so in the Finance Bill, certain amendments like new taxable services introduced vide the Bill and changes made in the scope of existing taxable services become effective from a date to be notified after the enactment of the Finance Bill. Answer 6(b) : As per rule 2(1)(d)(vii) of the Service Tax Rules, 1994, amended vide Notification No. 1/2007 ST dated 01.03.2007, in relation to sponsorship service provided to any body corporate or firm located in India, the person liable to pay service tax is the body corporate or firm, as the case may be, who receives such sponsorship service. However, where the recipient of sponsorship services (assuming the recipient of service to be a body corporate or firm) is located outside India, the service provider will be responsible to pay service tax. Answer 6(c): Yes, Free Services of motor vehicles (where the customer does not pay any charges), the service charges of which are reimbursed by the vehicle manufacturer are subject to service tax. Answer 6(d): An insurer carrying on life insurance business who is liable for paying service tax has the option to pay an amount calculated @ 1.5% of the gross amount of premium charged by him towards the discharge of his service-tax liability instead of paying service-tax @ 10%. However, such option is not available in cases where: (i) the entire premium paid by the policy holder is only towards risk cover in life insurance; or (ii) the part of the premium payable towards risk cover in life insurance is shown separately in any of the documents issued by the insurer to the policy holder. Answer 6(e): Refer to Answer given in IPCC NOV 2011 Question No.2 Question 7 (3 Marks Each) (a) Mr. X, the owner of a property had entered into an agreement with a bank. The agreement was entered into on 1st April, 2010 to give ground floor of the property on rent on monthly rent of `1,00,000 p.m. (inclusive of service tax) The bank had taken the property for commercial purpose. Explain whether Mr. X is liable to pay service tax for the financial year 2011-12 on the transaction with bank? (b) Mr. Y, consulting engineer raised a bill of `2,24,720 (including service tax) on his client for consulting services rendered by him in the month June, 2011. A partial payment of `1,68,540 was received by Mr. Y in the month March, 2012. Compute the service tax amount payable by Mr. Y and the due date which service tax can be deposited. Answer 7(a): In this case, Mr. X is liable to pay service tax because renting of immovable property for commercial purpose is taxable service as per section 65(105) and service tax payable shall be 12,00,000 / 110.3 x 10.3 = `1,12,058.02 Rounded off u/s 37D = `1,12,058 Since property was let out in the Financial Year 2010-11, general exemption of `10,00,000 has not been allowed in Financial Year 2011-12.

Service Tax Answer 7(b): Total bills raised Amount received in the month of March, 2012 Value of taxable services = 1,68,540 /110.3 x 100 = 1,52,801.45 Service tax payable = 1,52,801.45 x 10.3% = 15,738.54 Rounded off 37D = 15,739 The due date for payment of service tax will be 31st March, 2012. Question 8 (a) Can service-tax return be revised by a person?

271 ` 2,24,720 1,68,540

(3 Marks Each)

(b) What are the due dates for payment of Service tax? Answer 8(a): An assessee can submit a revised return, in Form ST-3, in triplicate, to correct a mistake or omission, within a period of 90 days from the date of submission of the original return. Answer 8(b): Refer to Answer given in IPCC NOV 2011 Question No.2

PCC MAY 2008


Question 6 (a) Briefly explain the nature of Service tax. (3 x 2 = 6 Marks)

(b) Mr. Saravanan has collected a sum of ` 15,000 as service tax from a client mistakenly, even though no service tax is chargeable for such service. Should the amount so collected be remitted to the credit of the Central Government? (c) Who are the persons liable to File Service tax returns? Answer 6(a): Nature of Service Tax Service tax is a tax on services. It is an indirect tax. It is charged by the Service Provider from the Service Recipient on the gross amount. This is not a tax on profession, trade, calling or employment but is in respect of service rendered. If there is no service, there is no tax. Basically, service is a value addition that can be perceived (to feel) but cannot be seen, as it is intangible; however, usage of some goods during the course of rendering the service would not mean that there is no service. Answer 6(b): Service Tax collected by mistake Section 73A of the Finance Act, 1994 casts an obligation on every person who has collected service tax from any recipient of service in any manner as service tax, to remit the same to the credit of the Central Government. On account of this provision, where any person has collected any amount, which is not required to be collected from any other person, in any manner as representing service tax, he should also immediately pay the amount so collected to the credit of the Central Government. Hence, Mr. Saravanan has to remit the service tax collected by him on the non taxable services to the credit of the Central Government before the due date. Answer 6(c): Persons liable to file Service Tax Returns

Service Tax

272

Section 70 of the Finance Act, 1994 enjoins that every person liable to pay service tax shall himself assess the tax due on the services provided by him and shall furnish a return to the Superintendent of Central Excise Question 7 (2 + 4 Marks) Ms. Priya rendered a taxable service to a client. A bill for ` 40,000 was raised on 29.04.2011; ` 15,000 was received from the client on 01.05.2011 and the balance on 31.08.2011. No service tax was charged in the bill. The questions are: (a) Is Ms. Priya liable to pay service tax, even though the same has not been charged by her? (b) In case she is liable, what is the value of taxable service and the service tax payable? (Modified) Answer: Section 68 of the Finance Act, 1994 casts the liability to pay service tax upon the service provider or upon the person liable to pay service tax as per Rule 2(1)(d). This liability is not contingent upon the service provider realizing or charging the service tax at the prevailing rate. The statutory liability does not get extinguished if the service provider fails to realize or charge the service tax from the service receiver. Hence, Ms. Priya is liable to pay service tax. In these cases, the amount charged from the client in lieu of having rendered the service will be taken to be inclusive of service tax and accordingly tax payable will be calculated by making back calculations. The rates of service tax payable are: Basic rate Education cess (2% + 1% of 10%) Service tax is payable on receipt basis Value of taxable service Value of Taxable Service Service tax payable Question 8 (a) Briefly explain about the charge of service tax. 10% = 10.3%
Gross amount charged 100 (100 + Effective rate) 40,000 100 = 36,265 = 110.3 40,000 10.3 = 3,735 = 110.3

(3 Marks each)

(b) Mr. Vasudevan has rendered free service to a client which is taxable, but has not charged or received any fee from the client. Is service tax payable on such free service? Answer 8(a): Charge of Service Tax [Section 66] Section 66 is the charging section of the Finance Act, 1994 which deals with the levy and collection of service tax. It prescribes the applicable rate of service tax which is to be levied on the value of various taxable services. For collection of service tax, it provides that the 'prescribed manner' needs to be followed. Applicable rate is provided in the section itself whereas the prescribed manner for collection and payment of tax is provided in the Service Tax Rules, 1994. With effect from 24.02.2009, the rate of service tax prescribed by section 66 is 10% of the value of taxable services referred to in section 65(105) of the Act. Section 65(105) provides that taxable service shall not only include service provided but also the "service to be provided". Thus, (a) the charge is on the services provided or to be provided; (b) the services provided or to be provided must be the one which is covered in section 65(105); (c) the rate of tax is 10%; (d) the measure of tax is on "value of taxable services" provided which is defined in section 67.

Service Tax

273

Answer 8(b): Service Tax not payable on Free Services Section 67 of the Finance Act, 1994 ensures payment of service tax based on valuation even when consideration is not ascertainable (to find out the true or correct information about something). However, these provisions apply only when there is consideration. If there is no consideration i.e., in case of free service, section 67 cannot apply. Thus, no service tax is payable when value of services is zero, as the charging section 66 provides that service tax is chargeable on the value of taxable service. Hence, if the value is zero the tax will also be zero even though the service may be taxable. However, this principle applies only when there is really a 'free service' and not when its cost is recovered through different means.

PCC NOV 2007


Question 6 (4 x 2 = 8 Marks) (a) Where a service provider maintains books of accounts on mercantile basis relating to taxable services provided by him, will service tax be payable on accrual basis? (b) Which are the documents to be submitted along with service tax return? (c) What are the due dates for filing of service tax returns? (d) Is a service provider allowed to pay service tax on a provisional basis? Answer 6(a): After introduction of Point of Taxation Rules, 2011, service tax shall be payable on the basis of point of taxation and not on the basis of actual receipt. Service tax should be paid in the month succeeding the month or quarter in which services were deemed to have been provided and further maintaining of books of accounts on the basis of mercantile system of accounting shall not matter. Answer 6(b): Along with service tax (ST-3) return, the following documents should be attached: (i) copies of GAR-7 challans which indicate the payment of service tax for the months/quarter covered in the half-yearly return. (ii) memorandum in Form ST-3A giving full details of the difference between the amount of provisional amount of tax deposited and the actual amount payable for each month. This memorandum (Form ST-3A) is to be attached only when the assessee opts for provisional payment of service tax. Answer 6(c): The service tax return (in Form ST-3) should be filed on half yearly basis by the 25 th of the month following the particular half-year. The due dates on this basis are tabulated as under: Half year Due date st th 1 April to 30 September 25th October 1st October to 31st March 25th April th In case the due date of the filing of return i.e. either 25 October or 25th April falls on a public holiday, the assessee can file the return on the immediately succeeding working day. Answer 6(d): In case the assessee is unable to correctly estimate, at the time of the deposit, the actual amount of service tax for any month or quarter, he may make a written request to Assistant/Deputy Commissioner of Central

Service Tax

274

Excise for making payment of service tax on provisional basis. The concerned officer may allow payment of service tax on provisional basis on such value of taxable service as may be specified by him. Question 7 (3 Marks) J.C. Brothers, a partnership firm, (liable to pay service tax) gives the following particulars relating to the services provided to various clients by them for the half-year ended on 30.09.2011: (i) Total bills raised for ` 8,75,000 (exclusive of service tax) out of which bill for ` 75,000 was raised on an approved International Organisation and payments of bills for ` 1,00,000 were not received till 30.09.2011. (ii) Amount of ` 50,000 (exclusive of service tax) was received as an advance from XYZ Ltd. on 25.09.2011 to whom the services were to be provided in October, 2011. You are required to work out the: (a) taxable value of services (b) amount of service tax payable. (Modified) Answer: Computation of taxable value of services provided by the J.C. Brothers for the half year ending on 30.09.2011: Particulars ` Total bills raised 8,75,000 Less: Bill raised on an approved International Organisation (Note (i)) 75,000 8,00,000 Add: Advance received for the services to be provided in October 2011 (Note (ii)) 50,000 Taxable value of services 8,50,000 Computation of service tax payable Taxable value of services Service tax @ 10% Add: Education cess @ 2% Add: SHEC @ 1% Total Service tax payable 8,50,000 85,000 1,700 850 87,550

Notes: (i) Services provided to approved International Organisation are exempt from the service tax. (ii) Service tax is payable on advance received for taxable services. (iii) Service tax is payable even on the value of the taxable services which are deemed to be provided even if payment has not been received [Rule 6(1) of Service Tax Rules, 1994]. Question 8 (3 Marks Each) (i) Who is liable to pay service tax in relation to services provided by a goods transport agency? (ii) What are the due dates for payment of service tax? Answer 8(i): In relation to taxable service provided by a goods transport agency, where the consignor or consignee of goods is(a) any factory registered under or governed by the Factories Act, 1948, (b) any company formed or registered under the Companies Act, 1956, (c) any corporation established by or under any law,

Service Tax

275

(d) any society registered under Societies Registration Act, 1860 or under any law corresponding to that Act in force in any part of India, (e) any co-operative society established by or under any law, (f) any dealer of excisable goods, who is registered under the Central Excise Act, 1944 or the rules made thereunder, or (g) any body corporate established, or a partnership firm registered, by or under any law. the person liable for paying service tax is any person who pays or is liable to pay freight either himself or through his agent for the transportation of such goods by road in a goods carriage. Answer 8(ii): Refer to Answer given in IPCC NOV 2011 Question No.2

PCC MAY 2007


Question 6 (i) Is e-filing of service tax return permitted? (ii) Should service tax be paid even if not collected from the client or service receiver? (iii) Will the payment to a hotelier of `10,000 on behalf of an architect by a service receiver be included in the value of taxable services? Answer 6(i): Refer to Answer given in IPCC NOV 2010 Question No.5 Answer 6(ii): Section 68 of the Finance Act, 1994 casts the liability to pay service tax upon the service provider or upon the person liable to pay service tax as per rule 2(1)(d) of the Service Tax Rules, 1994. This liability is not contingent upon the service provider realizing or charging the service tax at the prevailing rate. The statutory liability does not get extinguished if the service provider fails to realize or charge the service tax from the service receiver. Answer 6(iii): Service tax chargeable on any taxable service is on the basis of gross amount charged by service provider for such service provided or to be provided by him. It is not necessary that the service receiver should pay the consideration only to the service provider; any money paid to the third party is also includible. Hence, the hotel bill met by the client would be includible in the value of taxable services. Question 8 (3 x 4 = 12 Marks) (a) An assessee who has collected service tax from a client is unable to perform the service. Briefly explain the situations in which and the conditions subject to which he can adjust the service tax relating to above, against his forthcoming service tax liability. (b) Can Vignesh & Co., providing services from different locations and billing the clients from each location, opt for centralized service tax registration? (c) Can it be said that if the taxable service is not capable of ascertainment, the same cannot form part of value of taxable services from May, 2006 onwards? (d) In case of import of services, is a recipient of such services in India liable to pay service tax? Answer 8(a): Refer to Answer given in PCC NOV 2011 Question No.7 (3 x 2 = 6 Marks)

Service Tax

276

Answer 8(b): If a service provider is providing services from more than one premises, in such cases he can apply for separate registration for each of such premises. He may apply for a single registration called Centralized Registration provided he has either centralized accounting or centralized billing system. In such cases one of the places shall be considered to be Head Office and all other premises shall be considered to be branches. A single registration certificate shall be issued. If the service provider do not have centralized accounting and also there is no centralized billing system, he will not be allowed to apply for centralized registration. In the given situation, centralized billing is not adopted and further presuming that centralized accounting is not been done, Vignesh & Co. cannot opt for centralized service tax registration. Answer 8(c): No, it cannot be said so since with effect from 18.04.2006, the Finance Act, 2006 has introduced detailed provisions for valuation of taxable services. The provisions of section 67, as amended, state clearly that if the consideration for a taxable service is not ascertainable, the value of such service shall be the amount as may be determined in the prescribed manner. Answer 8(d): Clause (iii) of rule 2(1)(d) of the Service Tax Rules, 1994 provides that in relation to any taxable service provided or to be provided by any person from a country other than India and received by any person in India, the person liable to pay service tax is the recipient of such service. Thus, in case of import of services, recipient of such services in India shall be liable to pay service tax. FORM ST-1 Application form for registration under section 69 of the Finance Act, 1994 (32 of 1994) (Please tick appropriate box below) New Registration Amendments to information declared by the existing Registrant. Registration Number in case of existing Registrant seeking Amendment ............. 1. (a) Name of applicant

(b) Address of the applicant

2. Details of Permanent Account Number (PAN) of the applicant (a) Whether PAN has been issued by the Income Tax Department

Yes No (b) If Yes, the PAN

Service Tax (c) Name of the applicant (as appearing in PAN)

277

3. (a) Constitution of applicant (Tick as applicable) (i) Proprietorship

(ii) Partnership

(iii) Registered Public Limited Company

(iv) Registered Private Limited Company

(v) Registered Trust

(vi) Society/Cooperative Society

(vii) Others

(b) Name, Address and Phone Number of Proprietor/Partner/Director (i) Name

(ii) Address

(iii) Phone Number

4. Category of Registrant (Please tick appropriate box) (a) Person liable to pay service tax (i) Service provider (ii) Service recipient (b) Other person/class of persons (i) Input service distributor (ii) Any provider of taxable service whose aggregate value of taxable service in a financial year exceeds nine lakh rupees 5. (a) Nature of Registration (Tick as applicable)

Service Tax (i) Registration of a single premise (ii) Centralised Registration for more than one premises (b) Address of premises for which Registration is sought (i) Name of Premises/Building

278

(ii) Flat/Door/Block No.

(iii) Road/Street/Lane

(iv) Village/Area/Lane

(v) Block/Taluk/Sub-Division/Town

(vi) Post office

(vii) City/District

(viii)State/Union Territory

(ix) PIN

(x) Telephone Nos.

(xi) Fax No.

(xii) E-mail Address

(c) In case of application for Centralised Registration, furnish address of all the premises from where taxable services are provided or intended to be provided [Format as per 5(b) above] (d) In case of application for Input Service Distributor, furnish address of all the premises to which credit of input services is distributed or intended to be distributed [Format as per 5(b) above] 6. Address of the premises or office paying service tax under centralised billing or centralised accounting under sub-rules (2) and (3A) of rule 4 of the Service Tax Rules, 1994. Address

Service Tax

279

7. Description of taxable services provided or to be provided by applicant Sl. No. (1) Description of service (2) Relevant clause of section 65 of the Finance Act, 1994, to be indicated, if possible (3)

8. Name, Designation and Address of the Authorized Signatory/Signatories:

DECLARATION I, ...........................................hereby declare that the information given in this application form is true, correct and complete in every respect and that I am authorized to sign on behalf of the Registrant. (a)For new registration: I would like to receive the Registration Certificate by mail/by hand/e-mail (b)For amendments to information pertaining to existing Registrant: ................................................ Date from which amendments are made: ............... (Self certified photocopy of Registration Certificate is required to be enclosed) (Signature of the applicant/authorized person with stamp) Date: .............................. Place: ............................. ACKNOWLEDGEMENT (To be given in the event Registration Certificate is not issued at the time of receipt of application for Registration) I hereby acknowledge the receipt of your Application Form (a) For new Registration (As desired, the New Registration Certificate will be sent by e-mail/mail/handed over to you in person on..............) (b) For amendments to information in existing Registration Signature of the Officer of Central Excise (with Name and Official Seal) Date: .............................. FORM ST-2 Certificate of registration under section 69 of the Finance Act, 1994 (32 of 1994)

Service Tax

280

Shri/Ms. ...................................................................... (name with complete address of premises) having undertaken to comply with the conditions prescribed in Chapter V of the Finance Act, 1994 read with the Service Tax Rules, 1994, and any orders issued thereunder is hereby certified to have been registered with the Central Excise Department. The Service Tax Code and other details are mentioned hereunder. 1. PAN No. 2. Service Tax Code (Registration Number) 3. Taxable Services 4. Address of business premises: (i) Name of Premises/Building

(ii) Flat/Door/Block No.

(iii) Road/Street/Lane

(iv) Village/Area/Lane

(v) Block/Taluk/Sub-Division/Town

(vi) Post office

(vii) City/District

(viii) State/Union Territory

(ix) PIN

Telephone Nos.

(x) E-mail Address

5. Premises code

6. This certificate is issued incorporating the changes intimated by the applicant and the previous certificate of registration bearing Registration Number ................ issued on ........ stand cancelled.

Service Tax

281

[Sl. Nos. 2, 3, 4 to be repeated for every premises being granted a registration under this certificate. Premises code is given by the department based on the Commissionerate + Division+ Range + Sl. No.] Notes: 1. In case the registrant starts providing any other taxable service (other than those mentioned above), he shall intimate the department. 2. In case the registrant starts billing from other premises (other than those mentioned above), he shall intimate the department. 3. These intimations and any other information which registrant wishes to bring to the notice of the department can be submitted on-line by the registrant after logging on to website. 4. This registration certificate is not transferable. 5. List of Accounting Codes is enclosed. These may invariably be furnished in the challan at the time of making payment of service tax. Place: ....................... Name and signature of the Central Excise Date: ....................... Officer with official seal CC: (by e-mail) To (1) The Pay and Accounts Officer (Commissionerate Name) (2) The Superintendent of Central Excise (Where premises are located).

Service Tax

282

FORM ST-3 (IN TRIPLICATE) (Return under section 70 of the Finance Act, 1994) (Please see the instructions carefully before filling the Form) [ORIGINAL/REVISED RETURN (Strike whichever is NOT applicable)] For the period (Please tick the appropriate period) AprilSeptember October-March Financial Year

1A. Has the assessee opted to operate as Large Taxpayer [Y/N] (As defined under rule 2(ea) of the Central Excise Rules, 2002 read with rule 2(1)(cccc) of the Service Tax Rules, 1994) 1B. If reply to column 1A is yes, name of Large Taxpayer Unit (LTU) opted for (name of city) 2A. Name of the assessee

2B. STC No. 2C. Premises Code No. 2D. Constitution of assessee (Please tick the appropriate category) (i) Individual/Proprietar y (iii) Registered Public Ltd. Company (v) Registered Trust (vii) Other 3. Computation of Service Tax (To be filled by a person liable to pay service tax/Not to be filled by input service distributor) (To be repeated for every category of taxable service on which service tax is payable by the assessee) A1. Name of taxable service (ii) Partnership (iv) Registered Private Ltd. Company (vi) Society/Co-op. Society

A2. Assessee is liable to pay service tax on this taxable service as, (Please tick the appropriate category)

Service Tax (i) a service provider; or (ii) a service receiver liable to make payment of service tax B. Sub-clause No. of clause (105) of section 65 (Please see instructions) C1 Has the assessee availed benefit of any exemption notification (Y/N) . C2. If reply to column C1 is yes, please furnish Notification Nos. D . -

283

If abatement is claimed as per Notification No. 1/2006-ST, please furnish Sr. No. in the notification under which such abatement is claimed E2. Prov. assessment order No. (if any)

E1. Whether provisionally assessed (Y/N)

F. Value of taxable service, service tax payable and gross amount charged Month/Quarter** Apr. / Oct. (2) May / Nov. (3) June / Dec. (4) July / Jan. (5) Aug. / Feb. (6) Sept. / Mar. (7)

(1) (I) Service tax payable (a) Gross amount received/(paid#) in money (i) against service provided (ii) in advance for service to be provided (b) Money equivalent of considerations received/(paid#) in a form other than money (c) Value on which service tax is exempt/not payable (i) Amount received against export of service^ (ii) Amount received/(paid#) towards exempted service (other than export of service, i.e., (i) above) (iii) Amount received as/(paid to#) pure agent (Please see instructions) (d) Abatement amount claimed (e) Taxable value = (a + b) minus (c + d) (f) Service tax rate-wise break-up of taxable value = (e) (i) Value on which service tax is payable @ 5% (ii) Value on which service tax is payable @ 8% (iii) Value on which service tax is payable @ 10% (iv) Value on which service tax is payable @ 12% (v) Other rate, if any (please specify) (g) Service tax payable =

Service Tax [5% of f(i) + 8% of f(ii) + 10% of f(iii) +12% of f(iv) + f(v) other rate] (h) Education cess payable (@ 2% of Service Tax) (i) Secondary and higher education cess payable (@ 1% of Service Tax) (Please see instructions) (II) Taxable amount charged (j) Gross amount for which bills/invoices/ challans are issued relating to service provided/to be provided (including export of service and exempted service) (k) Money equivalent of other consideration charged, if any, in a form other than money (l) Amount charged for exported service provided/to be provided ^ (m) Amount charged for exempted service provided/to be provided (other than export of service given at (l) above) (n) Amount charged as pure agent (Please see instructions) (o) Amount claimed as abatement (p) Net taxable amount charged=(j+k)minus (l + m + n + o)

284

** Assessee liable to pay service tax on quarterly basis may furnish details quarter-wise i.e. Apr.-Jun., Jul.-Sep., Oct.-Dec., Jan.-Mar. # Applicable when service receiver is liable to pay service tax; ^ Not applicable to service receiver liable to pay service tax 4. Amount of service tax paid in advance under sub-rule (1A) of rule 6 Month/Quarter** Apr. May June July Aug. Sept. / / / / / / Oct. Nov. Dec. Jan. Feb. Mar. (1) (a) Amount deposited in advance (b) Challan Nos. (c) Challan dates ** Assessee liable to pay service tax on quarterly basis may furnish details quarter-wise i.e. Apr.-Jun., Jul.-Sep., Oct.-Dec., Jan.-Mar. 4A. Service tax, education cess and other amounts paid (To be filled by a person liable to pay service tax/Not to be filled by input service distributor) Apr./ May/ June/ July/ Aug./ Sept./ Month/Quarter** Oct. Nov. Dec. Jan. Feb. Mar. (1) (I) Service tax, education cess, secondary and higher education cess paid (a) Service tax paid (i) in cash (ii) by CENVAT credit^ (iia) by adjustment of amount earlier paid in advance and adjusted in this period under rule (2) (3) (4) (5) (6) (7) (2) (3) (4) (5) (6) (7)

Service Tax 6(1A)] (iii) by adjustment of excess amount paid earlier and adjusted in this period under rule 6(3) of ST Rules (iv) by adjustment of excess amount paid earlier and adjusted in this period under rule 6(4A) of ST Rules (b) Education cess paid (i) in cash (ii) by CENVAT credit^ (iia) by adjustment of amount earlier paid in advance and adjusted in this period under rule 6(1A) (iii) by adjustment of excess amount paid earlier and adjusted in this period under rule 6(3) of ST Rules (iv) by adjustment of excess amount paid earlier and adjusted in this period under rule 6(4A) of ST Rules (c) Secondary and higher education cess paid (i) in cash (ii) by CENVAT credit^ (iia) by adjustment of amount earlier paid in advance and adjusted in this period under rule 6(1A) (iii) by adjustment of excess amount paid earlier and adjusted in this period under rule 6(3) of ST Rules (iv) by adjustment of excess amount paid earlier and adjusted in this period under rule 6(4A) of ST Rules (d) Other amounts paid (i) Arrears of revenue paid in cash (ii) Arrears of revenue paid by credit^ (iii) Arrears of education cess paid in cash (iv) Arrears of education cess paid by credit^ (v) Arrears of Sec. and higher edu. cess paid by cash (vi) Arrears of Sec. and higher edu. cess paid by credit (vii) Interest paid (viii) Penalty paid (ix) Section 73A amount paid^ (x) Any other amount (please specify) (II) Details of Challan (vide which service tax, education cess, secondary and higher education cess and

285

Service Tax other amounts paid in cash) (a) Challan Nos.

286

(i) (ii) (iii) (iv) (i) (ii) (iii) (iv)

(b) Challans date (May please be furnished in the order of Challan Nos. furnished above)

**Assessee liable to pay service tax on quarterly basis may give detail quarter-wise i.e. Apr.-Jun., Jul.Sep., Oct.-Dec. and Jan.-Mar. ^Not applicable to service receiver liable to pay service tax. 4B. Source documents details for entries at columns 4A(I)(a)(iii), 4A(I)(a)(iv), 4A(I)(b)(iii), 4A(I)(b)(iv), 4A(I)(c)(iii), 4A(I)(c)(iv), 4A(I)(d)(i) to (vii) [To be filled only if any entry is made against columns 4A(I)( a)(iii), 4A(I)(a)(iv), 4A(I)(b)(iii), 4A(I)(b) (iv), 4A(I)(c)(iii), 4A(I)(c)(iv), 4A(I)(d)(i) to (vii)] Entry in table 4A above Source documents No./Period Source documents date S. No. Month/Quarter

4C. Details of amount of service tax payable but not paid as on the last day of the period for which return is filed...... 5. Details of input stage CENVAT credit (To be filled by a taxable service provider only/not to be filled by service receiver liable to pay service tax or input service distributor) 5A. Whether the assessee providing exempted/non-taxable service or exempted goods (1) (2) (a)Whether providing any exempted or non-taxable service (Y/N) (b)Whether manufacturing any exempted goods (Y/N) (c)If any one of the above is yes, whether maintaining separate account for receipt or consumption of input service and input goods [refer to rule 6(2) of CENVAT Credit Rules, 2004] (d)If any one of the (a) and (b) is yes, and (c) is no, which option is being availed under rule 6(3) of the CENVAT Credit Rules, 2004 (i)Opted to pay an amount equal to 10% of the value of exempted goods and 8% of the value of exempted service (Y/N); or (ii)Opted to pay an amount equivalent to CENVAT Credit attributable to inputs and input services used in or in relation to manufacture of exempted goods or provision of exempted service (Y/N).

Service Tax 5AA.Amount payable under rule 6(3) of the Cenvat Credit Rules, 2004 Apr. May Month / / Oct. Nov. (1) (a) Value of exempted goods cleared (b) Value of exempted services provided (c) Amount paid under rule 6(3) of Cenvat Credit Rules, 2004, by CENVAT Credit (d) Amount paid under rule 6(3) of Cenvat Credit Rules, 2004, by cash (e) Total amount paid = (c) + (d) (f) Challan Nos., vide which amount mentioned in (d) is paid (g) Challan dates 5B. CENVAT Credit taken and utilized
Month/Quarter**

287

June / Dec. (4)

July / Jan. (5)

Aug. / Feb. (6)

Sept. / Mar. (7)

(2)

(3)

Apr. / Oct. (2)

May / Nov. (3)

June / Dec. (4)

July / Jan. (5)

Aug. / Feb. (6)

Sept. / Mar. (7)

(1) (I)CENVAT Credit of Service Tax and Central Excise duty (a)Opening balance (b)Credit taken (i) On inputs (ii)On capital goods (iii)On input services received directly (iv)As received from input service distributor (v)From inter-unit transfer by an LTU* Total credit taken = (i + ii + iii + iv + v) (c)Credit utilized (i)For payment of service tax (ii) For payment of education cess on taxable service (iii)For payment of excise or any other duty# (iv) Towards clearance of input goods and capital goods removed as such (v) Towards inter-unit transfer of LTU* (vi) For payment under rule 6(3) of the Cenvat Credit Rules, 2004 Total credit utilized = (i + ii + iii + iv + v + vi)] (d)Closing Balance of CENVAT credit = (a + b - c) (II)CENVAT credit of Education Cess and Secondary and Higher Education Cess (a)Opening balance

Service Tax

288

(b)Credit of education cess and secondary and higher education cess taken, (i) On inputs (ii)On capital goods (iii)On input services received directly (iv)As received from input service distributor (v)From inter-unit transfer by an LTU* Total credit of education cess and secondary and higher education cess taken = (i + ii + iii + iv + v) (c)Credit of education cess and secondary and higher education cess utilized (i)For payment of education cess and secondary and higher education cess on services (ii)For payment of education cess and secondary and higher education cess on goods# (iii)Towards payment of education cess and secondary and higher education cess on clearance of input goods and capital goods removed as such (iv)Towards inter-unit transfer of LTU* Total credit of education cess and secondary and higher education cess utilised = (i + ii + iii + iv) (d)Closing balance of education cess and secondary and higher education cess = (a + b - c) # Relevant only if assessee providing taxable service is also engaged in manufacture and clearance of excisable goods. This would also include excise duty paid on capital goods and inputs removed as waste and scrap, in terms of sub-rule (5A) of rule 3 of the Cenvat Credit Rules, 2004. *To be filled only by Large Taxpayer as defined under rule 2(ea) of the Central Excise Rules, 2002 and who has opted to operate as LTU. In case LTU has centralized registration for service tax, this information is not applicable in respect of service tax credit. ** Assessees liable to pay service tax on quarterly basis may give detail quarter-wise i.e. Apr.-Jun., Jul.Sep., Oct.-Dec. and Jan.-Mar. 6. Credit details for input service distributor (To be filled only by an input service distributor) Apr./ May/ June/ July/ Aug./ Sept./ Oct. Nov. Dec. Jan. Feb. Mar. (1) (I) CENVAT Credit of Service Tax and Central Excise duty (2) (3) (4) (5) (6) (7)

(a) Opening balance of CENVAT Credit (b) Credit taken (for distribution) on input service (c) Credit distributed (d) Credit not eligible for distribution (rule 7(b) of CENVAT Credit Rules, 2004) (e) Closing balance (II) CENVAT Credit of education cess and secondary and higher education cess credit (a) Opening balance of Education Cess and

Service Tax secondary and higher education cess credit (b) Credit of education cess and secondary and higher education cess taken (for distribution) on input service (c) Credit of education cess and secondary and higher education cess distributed (d) Credit of education cess and secondary and higher education cess not eligible for distribution [rule 7(b) of CENVAT Credit Rules, 2004] (e) Closing balance

289

7. Self-assessment memorandum (a) I/We declare that the above particulars are in accordance with the records and books maintained by me/us and are correctly stated. (b) I/We have assessed and paid the service tax and/or availed and distributed CENVAT credit correctly as per the provisions of the Finance Act, 1994 and the rules made thereunder. (c) I/We have paid duty within the specified time limit and in case of delay, I/we have deposited the interest leviable thereon. 8. If the return has been prepared by a Service Tax Return Preparer (STRP), furnish further details as below : (a) Identification No. of STRP (b) Name of STRP

(Signatures of Service Tax Return Preparer)] Place: Date: (Name and Signature of Assessee or Authorized Signatory)

ACKNOWLEDGEMENT I hereby acknowledge the receipt of your ST-3 return for the period ...................... Date: Place: (Signature of the Officer of Central Excise & Service Tax) (With Name & Official Seal)

Service Tax

290

S.No. Name of Division 1. Service Tax Division-I HQ Delhi

2.

Service Tax Division-II HQ Delhi.

Jurisdiction Area comprising of part of National Capital Territory of Delhi bound by the Delhi Haryana border on the North and West up to West Juan Drain or Mungaspur Drain intersection and along Mungaspur drain and Najafgarh Drain till Trinagar or Zakhira; Najafgarh Road from Zakhira upto Raja Garden Chowk, Ring Road or Mahatma Gandhi Road via Dhaula Kuan, Lajpat Nagar upto Maharani Bagh and thereafter Yamuna River upto Delhi Haryana Border. Area comprising of the part of National Capital Territory of Delhi (excluding the area included in Delhi-I) and including Trans Yamuna Area (East Delhi) of National Capital Territory of Delhi and areas bound by West Juan Drain or Mungspur Drain on Delhi Haryana Border upto Zakhira, Najafgarh Road upto Raja Garden Chowk, Ring Road or Mahatma Gandhi Road via Dhaula Kuan, Lajpat Nagar (Sourthern Part), upto Maharani Bagh and Delhi Haryana Border on the south and south west of Delhi.

Office Address 37, IInd Floor, Nehru Place Tel. 26451449 Fax. 26451448

Block No. 11, C.G.O Complex, 7th Floor, Lodhi Road, New Delhi - 110003. Tel. 24366212 Fax. 24366211

Service Tax 3. Service Tax, Division, Gurgaon District of Gurgaon, Rewari Mahendargarh Udyog Meenar, in the State of Haryana Vanijya Nikunj, Udyog Vihar, Phase-V, Gurgaon -122016. Tel. 0124-2342086 Fax 0124-2346331 4. Service Tax, Division, Faridabad Faridabad district in the State of Harvana New CGO Complex, NH.IV, Faridabad -121001. Tel. 0129-2431086 Fax 0129-2431478

291

You might also like